You are on page 1of 70

Excelling at Chess Calculation− Jacob Aagard 1

B71 Nb5 17.Kd2 Nxa3 Bd8 30.Rf4 Rxf4 31.Nxf4 XABCDEFGHY


Krajina,D 18.Rxa7 Nb1+ 19.Ke3 Bb6 32.Nd5 Bc5 33.Rf1 8r+-+-+-+(
Kozul,Z Nxc3 20.Re7 b5 21.Rxe6 Ke6 34.Nxf6 d5 35.Nxh7 7zPk+N+-+-' 4
Vinkovci 1989 b4 22.Rb6 Na2 23.Kd2 Be7 36.g4 d4 37.g5 d3 6-+-+-+-+&
[Jacob Aagaard] Kf7 24.Ra6 b3 38.g6 d2 39.g7 Rc1+ 5mKP+-+-+-%
25.cxb3?? 40.Rxc1 dxc1Q+ 4L+-+-+-vl$
1.e4 c5 2.Nf3 d6 3.d4 [ 25.Ra7+! 41.Kxc1 Kf7 42.Nf8 3+-+-+p+-#
cxd4 4.Nxd4 Nf6 5.Nc3 wins immediately ] Kxg7 43.Nd7 Kf7 44.Kc2 2-+-+-+-+"
Nc6 6.f4 g6 7.Nxc6 [ and 25.c3 would also Ke6 45.Nb6 Bc5 46.Na4 1+-+-+-+-![
bxc6 8.e5 Ng8 9.Qf3 d5 win in the long run. Now Bf2 47.Kd3 Kd5 48.Nb2 xabcdefghy
10.Be3 h5 11.h3 Bf5 it goes the other way. ] . 6.Nd5+ Kd6 7.Ne3
12.0-0-0 e6 13.Ba6 Ba3!! 25...c3+! What White had All very convincing. But in and Black cannot win. ]
What makes this move imagined would happen fact White could have 2.bxa7 Be1+ 3.Kb5 f2
most fantastic is that there here is hard to guess, but done better at the 4.a8Q+! Kxa8 5.Ka6!
is no immediate threat. this game was played with beginning! To allow a promotion with
Black simply undermines FIDE-time (where both . check is always attractive.
White's defence on the players get an extra 30 (press F10 to continue) [ If instead 5.Kb6?
queenside. 14.bxa3 seconds after each move) 1-0 Ba5+!! and Black wins. ]
[ Stronger perhaps was and White still had about 5...f1Q+ 6.Bb5 Qf3
14.Qe2 Qa5 15.Nb5! a minute left, so should 7.Bc6+! Qxc6+ 8.Nb6+
with unclear play. ] have smelt the trap. analysis Nijboer-Acs Kb8 Stalemate.
14...Qa5 15.g4 hxg4 Possibly the stress of [Jacob Aagaard] All of this is extremely
16.hxg4 Rxh1 17.Rxh1 prolonged time trouble nice, but there is a small
Bxg4 18.Qxg4 Qxc3 was getting to him. XABCDEFGHY problem in one of the
19.Bg1 26.Kd3?? 8-+r+k+-tr( lines...
[ 19.Qh3! would have [ 26.Ke3 c2 27.Rc6 7+-+-vl-+p' 3 .
kept White alive. ] c1Q+ 28.Rxc1 Nxc1 6-+-zpNzp-+& (press F10 to continue)
19...Qxa3+ 20.Kd1 Qxa6 29.b4 would still draw 5+-+-wq-+-% ½-½
21.Rh8 0-0-0 22.Rh7 d4 since the knight cannot 4pzpL+l+-+$
23.Rxf7 Qc4 24.Rxa7 escape the attentions of 3+-+-+-+-#
Kb8 25.Ra3 Ne7 26.Qh3 the white king; e.g. Na2 2PzPP+-+PzP" analysis Afek #1 pt.1
Nd5 ( or 29...Nb3 30.Kd3 ) 1+K+RwQR+-![ [Jacob Aagaard]
0-1 30.b5 Ke7 ( 30...Nc3?? xabcdefghy
31.b6 wins ) 31.Kd2 If he had played 22.Bb5+! XABCDEFGHY
etc. ] [ instead of 22.Qxe4!? 8r+-+-+-+(
Resika,N 26...Nb4+ 27.Kxc3 Nxa6 , then Black would not 7zPk+N+-+-' 5
Lund,E 28.b4 Nc7 29.Kd3 Kf6 have been able to reach 6-+-+-+-+&
Budapest 2002 30.Ke3 Kf5 an endgame in which he 5mKP+-+-+-%
[Jacob Aagaard] 0-1 had some chances of 4L+-+-+-vl$
survival. ] 3+-+-+p+-#
XABCDEFGHY After something like 2-+-+-+-+"
8-+-+-+r+( B67 22...Bc6 23.Bxc6+ Rxc6 1+-+-+-+-![
7zp-+l+nmk-' 1 Nijboer,F 24.Rd4! Black is xabcdefghy
6-zp-vLp+r+& Acs,P completely outplayed and After 1.b6 Bf2 2.Bd1
5+-+p+-+-% Wijk aan Zee 2003 it is not easy to see how Bxb6+ 3.Nxb6
4-+pzP-tR-+$ [Jacob Aagaard] he should improve on this. Black can play f2!
3zP-zP-+L+-# The method which could , changing the move order,
2-+PmK-zP-+" XABCDEFGHY have helped Nijboer to [ instead of 3...Rxa7+ ]
1+-+-+-+R![ 8-+r+k+-tr( find this is changing the 4.Be2
xabcdefghy 7+-+-vl-+p' 2 move order of the [ No better is 4.Bf3+
1.Bh5! A nice tactical 6-+-zpNzp-+& combination, as explained Kxa7 5.Bg2 Rg8 6.Bh3
move that wins the 5+-+-wq-+-% in Chapter Four. Rg5+ and Black wins
exchange, 4pzpL+l+-+$ first the knight and then
[ because of 1.Bh5 Nxd6 3+-+-+-+-# the bishop. ]
2.Bxg6 Kxg6 3.Rg1+ 2PzPP+-+PzP" Afek #1 4...Rxa7+ 5.Kb5 Ra2
Kh7 4.Rh4# . 1+K+RwQR+-![ Chess in Israel 1999 6.Bf3+ Ka7!! 7.Bg2
Black could resign here, xabcdefghy [Jacob Aagaard] Rb2+ and Black wins.
but instead he played... ] 22.Qxe4!? Qxe4 23.Bb5+ One of the points of the
1...Rf6 2.Rg1+? Qc6 Now White obtains a study was meant to be
This is sufficient to win, better endgame. (Diagram 4) that White could force the
[ but the computer comes [ If instead 23...Kf7 black king away from the
up with 2.Rg4+! 24.Ng5+ wins ] The following analysis knight, and thereby be in
which wins a lot more [ or 23...Rc6 24.Nd4! was given in the Study time to stop the f-pawn.
than a rook. ] Kf7 ( 24...0-0 25.Bxc6 Database 2000 and After 2 Bd1 this is no
2...Kh7 3.Rxf6 Nxd6 d5 26.Nf5 wins ) 25.Bxc6 probably comes from Afek. longer the case.
4.Rfg6 Rxg6 5.Bxg6+ Qe5 26.Rfe1 Qh5 1.b6! Rxa7+ So what would we do if
Kh6 6.f3 Nb5 7.Bf7 Nd6 27.Bf3! with a strong, [ 1...Bd8? If 2.Nc5+ Kc8 we reached this position in
8.Bg8 Nf5 9.Ke2 Nh4 probably winning 3.Bd7# ] a game?
10.Bf7 Nf5 11.Rg6+ Kh7 attack. ] [ or 1...Bf2 2.Bd1 Bxb6+ .
12.Rg1 Nd6 13.Bg8+ 24.Bxc6+ Rxc6 25.Nf4 3.Nxb6 Rxa7+ 4.Kb5 f2 (press F10 to continue)
Kh6 14.Rh1+ Kg7 Kf7 26.Nd5 Rg8 27.g3 5.Bf3+ ( 5.Be2? Ra2
15.Rh7+ Kxg8 16.Rxd7 Rg4 28.b3 axb3 29.cxb3 6.Bf3+ Ka7! ) 5...Kc7
Excelling at Chess Calculation− Jacob Aagard 2

analysis Afek #1 pt.2 C) 3...-- ] leaving the accurate get his finger out and
[Jacob Aagaard] [ 2.Bc2 Bxb6+ 3.Nxb6 calculation to the get the queenside into
A) 3...Rxa7+ 4.Kb5 f2 opponent.; play! This is what we
XABCDEFGHY ( or 4...Ra2 5.Be4+ C) 2...-- ] would expect a
8r+-+-+-+( Ka7 6.Bxf3 ) 5.Be4+ ½-½ grandmaster would most
7zPk+N+-+-' 6 Kc7 6.Nd5+ Kd6 likely try to achieve.
6-+-+-+-+& 7.Ne3 Ke5 8.Bg2 Rg7 The first of the two
5mKP+-+-+-% 9.Bf1 Rg1 10.Kc5!! Nohr,F relevant candidates is
4L+-+-+-vl$ Ke4 11.Nc4! Nielsen,M the following:
3+-+-+p+-# and it appears that Taastrup 2002 19.dxe7!? Qxe7
2-+-+-+-+" Black cannot win.; [Jacob Aagaard] 20.Bg5! . This is the
1+-+-+-+-![ B) 3...f2! 4.Be4+ most logical and simple
xabcdefghy Kxa7 5.Bd3 Rd8! XABCDEFGHY way of mobilizing the
Well, 1.b6 is the only move 6.Be2 Re8 and Black 8r+l+-trk+( queenside: f6 is under
that does not lose very wins.; 7zpp+qzpnvlp' 7 attack and White
quickly. C) 3...-- ] 6-sn-zP-+p+& develops the bishop
[ For instance 1.Nc5+ [ 2.Bb3! 5+-+-+-+-% with tempo. Soon the
Kxa7 2.b6+ Kb8+ A) 2...Bxb6+ 3.Nxb6 4-+-+N+-+$ rook will enter the action
3.Na6+ Kb7 4.Bb5 Rxa7+ ( this time 3...f2 3+L+-+Q+-# and White will have a
Be1+ and Black wins 4.Bc4 Rxa7+ 5.Kb5 2PzP-+-zPPzP" move or two to start a
without a fight is easy is a little different! ) 1tR-vL-tR-mK-![ direct assault on the
to calculate. With 1 b6 4.Kb5 f2 5.Bc4 Kc7 xabcdefghy black kingside, before
we would at least have 6.Nd5+ Kd6 7.Ne3 19.-- Black is able to develop
a little counterplay. ] Rb7+ 8.Ka6 Rb1 [ In this position White his own pieces from the
So after 1...Bf2! 9.Bd3 Re1 10.Nf1 was soon lost after queenside. Quick
we would again stand at a Kc6 11.Ka5 Kc5 19.Ng5? exd6 20.Ne6? analysis shows that
crossroads. 12.Ka4 Kd4 13.Bb5 , since Black, had he Black should probably
Here we would try to set Rb1 14.Ng3 Ke3 been trained in looking respond with 20...Qb4!.
up our candidates. 15.Nf1+ Kf4 for candidates, could A) . After 20...Qd7
Besides 2 Bd1 we also and it does not seem have used the 21.Bf6!! Black cannot
have 2 Bc2 and 2 Bb3, all possible for Black to unprotected rook on the defend himself. Qf5
of which send the bishop make progress.; first rank to eliminate the ( or 21...Kh8 22.Qc3
in the direction of the B) 2...Kc6 is an extra unpleasant knight by the Nh6 23.Qc5 Rg8
f1-square. From our option, but after odd-looking Qe8! 24.Rad1 Bxf6 25.Nxf6
calculation of 2 Bd1 3.Nb8+ Kc5 4.Na6+ , pinning the knight and and White wins )
(which is the most forcing Kc6 5.Bd5+! preparing ...Bxe6, thus 22.Qc3 Bh6 23.Nd6
move and therefore the (we do not even need securing the king and Qh5 24.Nxf7 Rxf7
first one we should to see this since we his material advantage. 25.Re8+ Bf8 26.Rxf8+
analyse) we discovered are happy with a draw) ( The more natural Kxf8 27.Qb4+
that the undefended Kxd5 6.b7 and White 20...Qc6! , forcing an and Black is mated.;
bishop became a problem. even wins. endgame with two minor B) 20...Qc7 21.Rac1
Therefore it is better to (Thanks to John Shaw pieces against a rook, Qd7 22.Bf6
keep the bishop closer to for his help with should also have won is similar. It is not easy
the king. Of course this is analysing this study.) quite easily. )] to imagine a line where
an abstract conclusion Of course it is difficult [ Instead of 19 Ng5?, the rook is worse on c1
which it would be to calculate all this, White has two equally than it was on a1.;
impossible to reach and I imagine that an interesting opportunities, C) . After 20...Qb4
without calculation. In average grandmaster both leading to sufficient , if he has nothing
such a complex position between 2500 and compensation for the better, White has
as this experience will 2600 would take 10-15 piece. But before we turn 21.Bd2 Qe7 22.Bg5
only help so far, while minutes to make the to the concrete options, with repetition,
general rules and right choice, and let us linger a second to ( though 22.Ng5!?
guidelines will not help not occasionally fail. But is consider the abstract looks far more
at all. What we need to do it impossible to learn objectives in the position. tempting now than it
is to analyse the moves how to calculate it all? White has sacrificed a did earlier. Sometimes
carefully. Actually, when No. First of all, let us piece in order to attack. it is enough to know
we get to the bottom of it, look at what really He needs either to you have a definite
there are not that many needs to be seen: regain his material, or draw in order to go for
variations: 2.-- The difference equivalent material, or a line, especially if
[ 2.Bd1 Bxb6+ 3.Nxb6 between 3...f2 and 3... deliver mate in order to that line includes the
A) 3...Rxa7+ 4.Kb5 f2 Rxa7+ is important. In justify his actions. There sacrifice of a minor
5.Bf3+! ( 5.Be2? Ra2 two out of three lines is no doubt that he has piece, a rook or even
6.Bf3+ Ka7! ) 5...Kc7 everything becomes the initiative, but in order more.
6.Nd5+ Kd6 7.Ne3 clear very quickly, so to take advantage of that All this is not so
and Black cannot win.; they can be he needs to bring all his surprising. That White
B) 3...f2! 4.Be2 ( or disregarded altogether pieces into play as went for 19 Ng5 in the
4.Bf3+ Kxa7 5.Bg2 as invalid. Just as quickly as possible, to game should be
Rg8 6.Bh3 Rg5+ easily we would not be gain a majority at the accredited to a lack
and wins ) 4...Rxa7+ able to refute 2 Bb3! scene of action, in order of attacking technique
5.Kb5 Ra2 6.Bf3+ which, in a game, to fulfil his objectives. (Finn Nøhr is mainly
Ka7!! 7.Bg2 Rb2+ would alone be enough Less abstractly, this a positional player)
and Black wins.; to inspire us to play it, means that he needs to rather than an inability
Excelling at Chess Calculation− Jacob Aagard 3

to calculate play as described in discovered this If you tried to solve the


variations. ); the abstract objectives possibility. Strong study, this was the main
D) 20...-- ] above. As yet we have chess-playing thing you should have
[ However, some not managed to find a programs have spotted, and what can be
unimpressed youngsters defence here for Black, transformed the way done about it. 5.Kc3!
and the equally and most likely there we think by showing Retaining the pawn is non-
unimpressed Fritz 7 isn't one. One attempt us corridors between negotiable. a3 6.Nd5 a2
came up with: starts Qd8 but then our logic and our 7.Ne3+ Kc1
19.Bh6!! . 22.Qc3+ e5 eyes that previously [ Or 7...Ke1 8.Kb2
A) . The main idea is ( Black has no choice were dark and hazy, and wins. ]
not so hard to spot. If here as after 22...Kh6 if not completely 8.Ne2+ Kb1 9.Kd2!
Black plays 19...Bxh6 23.dxe7 Qxe7 24.Nd6! closed to us. Though Setting up the cage where
then 20.dxe7 he is under heavy a computer makes a Black will be smothered to
and White wins a lot of artillery fire, unable lousy chess teacher death. a1Q 10.Nc3+ Kb2
material due to Qxe7 to find cover it can be a useful tool 11.Nc4#
21.Nf6+ and Black has anywhere ) , White has in the hands of the 1-0
to give up his queen time and active pieces serious chess player
for insufficient enough for launch a and chess trainer,
compensation. But deadly attack on the by pointing out O'Donovan,J
Black of course needs dark squares, different alleyways British Chess Magazine
to respond.; weakened by the and possibilities. But [Jacob Aagaard]
B) . He cannot play exchange of the more about that
19...exd6 because of bishops. Best is 23.f4! later.; XABCDEFGHY
20.Nf6+; and Black seems to be E3) 23...--; 8-+-+-+nmk(
C) ; nor can he play quite lost. The F) 19...-- ] 7+-+-wq-tR-' 9
19...Qf5 (an attempt to variations go 6-+p+-snP+&
free the queenside something like this: 5+-zPQ+-zP-%
and relieve the E1) 23...Bg4 24.fxe5 Liburkin,M 4-+-+-mK-+$
pressure on the king Bxd1 25.e6+ Kh6 64 magazine 1939 3+-+-+-+-#
by exchanging queens) 26.e7 Qd7 27.Qd2+ [Jacob Aagaard] 2-+-+-+-+"
since after 20.dxe7 Kg7 28.Qd4+ Kh6 1+-+-+-+-![
Qxf3 21.exf8Q+ Bxf8 29.Rxd1!; XABCDEFGHY xabcdefghy
22.gxf3 Bxh6 23.Nd6 E2) 23...Nd7 24.Bxf7 8-+-+-+-+( 1.--
the pin costs Black a Kxf7 ( or 24...Rxf7 7+-sN-+-+-' 8 [ White cannot win after
piece and he ends up 25.fxe5 Qb6+ 6-+-+-+-sN& the tempting zugzwang
the exchange down.; 26.Kh1 Qc6 27.e6+ 5zp-+-+-+-% operation 1.Rxg8+ Nxg8
D) . But what is White Qxc3 28.Nxc3 Rf5 4-+-+-+-+$ 2.Qf7 as Black has the
really threatening? 29.e7 Nf6 30.e8Q 3+K+-+-zp-# stalemate defence Qf8!!
Nothing in particular; Nxe8 31.Rxe8 Rf8 2-+P+-+-+" . So White needs to
his next move would 32.d7 and White has 1+-+-mk-+-![ find another way to
be 20 Rad1, after not yet finished xabcdefghy proceed. There is hardly
which he does start to cashing in ) 25.Ng5+ 1.Nf5! any methodology that
have a lot of threats, Kg8 26.Qc4+ Kh8 [ Using the other knight, e. can be used for solving
but mainly 19 Bh6! is 27.Nf7+ Rxf7 g. by 1.Ne6 g2 2.Ng5 this study that you
about domination. 28.Qxf7 and White fails to a4+! 3.Ka3 ( if could take with you into
Black cannot get out of has a winning attack 3.Kc3 a3 etc ) 3...g1Q a practical game. The
this with 19...e6 without having 4.Nf3+ Kd1 5.Nxg1 only thing is open eyes
since after 20.Bxg7 invested any serious Kxc2 with a book draw. ] to additional
Kxg7 21.Qf6+ material in the 1...g2 2.Nh4 g1Q opportunities, i.e. the
D1) 21...Kh6 process. [ 2...g1N is of course conscious incompetence
22.Re3 ( or 22.Ng5 So the conclusion to also possible, but it will as a chess player that
illustrates the need to this exercise is that not make a draw in the gives you the right to
see little other than White wins after 19 long run, so we stop look at all possibilities
candidates ); Bh6!! – truly a there after a few afresh.
D2) 21...Kg8 22.Qd4 stunning move that general observations, Here that might assist
White has a very neither of the such as that the knight you to find: ]
strong, probably players considered cannot immediately [ 1.Qxg8+! Nxg8 2.Rf7!
decisive attack.; during the game, nor catch the c-pawn and and White wins, as the
E) . Black's only move any of the spectators. force the undesired stalemate trick is gone, e.
is to force White to The tactics following endgame. ] g. Nf6 3.gxf6 Qxc5
exchange on g7 this move are not so 3.Nf3+ Kd1! This is again 4.Rh7+ Kg8 5.f7+ Kf8
without relinquishing hard for a strong the idea. 6.Rh8+ Kg7 7.Rg8+
his weak, but still player with [ After 3...Kf1 4.Nxg1 Kh6 8.f8Q+ and so on. ]
existing control of f6. sufficient knowledge Kxg1 the c-pawn cruises 1-0
19...Kh8! is therefore of attacking down to the eighth
the only defensive technique to find, but rank. ]
option that gives Black simply seeing the 4.Nxg1 a4+! An important
any hope of surviving. first move is difficult. thing to remember when
White should now It is probably no one calculates is that the
proceed with 20.Bxg7+ surprise to the reader opponent is playing
Kxg7 21.Rad1! that it was Fritz 7 according to his plans,
bringing the rook into which originally and not according to yours.
Excelling at Chess Calculation− Jacob Aagard 4

Karpov,A 38.Ra8 g4 39.fxg4 fxg4 calculation. No matter Rxc2 31.Bd4 Rbc6


Serper,G 40.Rxa7 Nd5 41.Ra4 whether White is winning 32.f4 and the attack is
Dortmund 1993 Ne3+ 42.Kg1 Kf6 43.Rf4+ or not, it is obvious that repelled, though some
[Jacob Aagaard] 1-0 the second line is play still exists.
preferable to the first, Actually this might be
XABCDEFGHY and for the practical what you would end up
8-+r+-+k+( Breder,D player that is all the playing no matter
7zpltr-wqpzpp' 10 Aagaard,J information required. whether you picked all
6-zp-+psn-+& Budapest 2002 Actually White is the candidates or not.
5+-+-wQ-+-% [Jacob Aagaard] probably going to win But Nunn is and always
4-zPPtR-+-sN$ without too much has been a very
3zP-+-+-zP-# XABCDEFGHY difficulty. aggressive player and
2-+-+-zPLzP" 8-+-+-+k+( But, as said, in the here he came up with a
1+-+R+-mK-![ 7zp-+q+-+p' 11 game White played 1 nice combination. ]
xabcdefghy 6-+-+-+p+& Qa8+. ] 28...Qb8 29.Rxf7!
Many candidates can be 5+-+-+R+-% 1...Qd8 2.Qxd8+ Rxd8 Nunn gives this an '?' in
selected for White in this 4-zpptr-+-+$ 3.Rf3 b3! I am not sure if his book, unjustly, as we
position, but the move 23 3+-+P+Q+-# White had overlooked this shall see below. Here we
Nf5 would not make 2PzPP+-zP-zP" move. He probably did, are really talking about
sense unless you see the 1+-+-+-mK-![ which is just another candidate ideas. White
idea behind the knight xabcdefghy reason why he should could either have come up
sacrifice. Most 1.Qa8+? have focused on the with this highly original
experienced players [ There are of course starting position and found rook sacrifice,
would spot the idea of many ways to solve this the solution to the [ or he could have
sacrificing the knight first position, but the simplest problem. 4.axb3 cxb3 foreseen the opportunity
and only afterwards find is to list candidates. In 5.cxb3? to exploit the back rank
the reasoning behind it. the game the critical [ 5.c3!? was worth with the following
Abstract ideas like position is that after considering, although sequence: 29.Rda1!
overloading and the 1.Qa8+ Qd8 2.Qxd8+ after a5 6.d4 a4 7.Re3 Rxc4 30.Nxc4
weakness of the back Rxd8 3.Rf3 , when the Rc8 Black should have and White is winning.
rank would not matter, but opponent's counter- sufficient counterplay. Black cannot defend his
a quick scan over all chances should be taken Now the next move back rank so he is simply
possible meaningful into consideration. But secures an active black a piece down in the
moves would pick up on first we should do our job rook and thereby a resulting endgame after
the knight sacrifice of selecting candidates draw. ] he loses his own queen.
immediately, and the for calculation, as this 5...Rd4! 6.Re3 Kf7 7.Kf1 One line could go d5
consequences of it would often saves us time Rb4 8.Re2 Rxb3 9.Rd2 31.Nxb6 Qxb6 32.R1a6
soon become apparent – compared with analysing a5 10.Ke2 a4 Black has a and wins.
and once these one line deeply first. ] very active rook vs. a It is not really fair to
consequences are clear, it [ The second (and better) passive rook. The extra talk about 29 Rxf7 and
is hard to imagine that idea in the position is to pawn does not matter 29 Rda1 as candidate
White should play any play 1.Rf8+ Kg7 2.Rb8! much anymore. 11.Ke3 moves. They both
other move, so probably . This move escaped my Ke6 12.Kd4 g5 13.Ke4 include certain ideas
Karpov played it rather opponent's attention h6 14.f3 Rb4+ 15.d4 and just seeing the
quickly and focused his completely. Rb3 16.Rf2 Kd6 17.Rc2 moves themselves
energy on the resulting ( He only calculated at Rb8 18.h3 Re8+ 19.Kd3 proves little. If you do
endgame. the tempting 2.Qf6+ Rf8 20.Ke3 Re8+ 21.Kd3 not spot the idea behind
23.Nf5! Black cannot , but did not see any Rf8 22.Ke3 Re8+ 23.Kf2 29 Rda1, which is the
allow the knight to come purpose to it. Had he Rb8 24.Ke3 simplest win in the
to d6, so the following taken the time to look ½-½ position, then you have
endgame cannot be for other possibilities I no chance of actually
avoided. exf5 24.Qxe7 am sure he would have including it in your list
Rxe7 25.Bxb7 Rf8 found 2 Rb8! ) Nunn,J of candidates. You
26.Ba6! The tactics are . The deadly threat of 3 Meyer,E simply see that 29...
now over and White has a Rb7 is so terrible that World Student Team Ch., Teesside Rxc4 wins your queen
clear advantage. With his Black will have to hurry [Jacob Aagaard] and you move on.
last move White keeps into a rook endgame with Actually, you probably
control over the c-file and 2...Qg4+ 3.Qxg4 Rxg4+ XABCDEFGHY would not see 29 Rda1
the promoting square for 4.Kf1 . 8-+r+-+k+( at all, unless you are
his c-pawn. So now, with We now have two 7+q+-+pzpp' 12 well trained in finding
a strong bishop vs. knight, candidates which can 6-tr-zpp+-+& surprising candidate
with more active rooks and be calculated, but 5+-+-+-vl-% moves. ]
an extra pawn on the actually we do not need 4-zpQ+-sN-+$ 29...Ne5 This seems
queenside, White had no to. It is obvious that 3+P+-sNnzP-# forced, even though it is
problems converting his White is worse off with 2-vLP+-zP-zP" objectively no better than
advantage into a full point. a passive rook on f3 in 1tR-+R+-+K![ alternatives;
g6 27.c5 bxc5 28.bxc5 the 1 Qa8+ line, than xabcdefghy [ e.g. 29...Rxc4
Rc7 29.Rc1 Re8 30.c6 with an active rook on b8 28.Ra7! 30.Rxg7+ Kh8 ( if
Kg7 31.Kf1 Re5 32.Bb7 in the second line. This [ It is tempting to start 30...Kf8 31.Nxe6+ Ke8
Ra5 33.Rcd1 Re5 34.Rd7 kind of comparison can calculating the different 32.Nxc4 and White's
Re7 35.Rxc7 Rxc7 often save us time in lines after 28.Qf1 Ne1+ position is completely
36.Rd8 Re7 37.f3 g5 evaluation and 29.Nfg2 Nxc2 30.Nxc2 overwhelming – the
Excelling at Chess Calculation− Jacob Aagard 5

threats are so many that the computer's choice in XABCDEFGHY is best refuted by
it seems they can hardly the position). Instead, 8-+-+-+k+({ 30.Rxf7 Qxf7 31.Qxd5
be counted! ) 31.Rb7+ the text move is so 7+-+l+-zpp' 13 when it is all about the
Rc3 32.Rxb8+ Rxb8 obvious that it is easy 6p+-+-zp-+& three extra pawns )
33.Nxe6 and White is a to play it without thinking. 5+p+P+-+-% 29.Qe5! and White will
few pawns up. ] It is a check, a fork even. 4-+-+-+-+$ enter the endgame a full
30.Rxg7+! White of course By comparison the 3+LwqpwQ-+-# pawn up with no doubt
had this in mind. Kxg7 queen is not doing 2P+P+-mKPzP" about the result, e.g.
[ 30...Kh8 31.Rxg5 Rxc4 anything directly on e6, 1+-+-+-+-! Qxe5 30.Nxe5 Rf5
32.Nxc4 and Black's there is no immediate xabcdefghy 31.Nd7 and Black is out
position is collapsing. ] threat to the black king combination at all. 30.d6+ of ideas.
31.Nxe6+? Don't think, or queen. But if you do Kh8 Here White used a What is the only defence
feel! – Bruce Lee. stop to think for a good deal of time before to this kind of
It is easy to see the knight moment, then you will realizing that he had aggression? ]
fork and then start to quickly see all these nothing better than to take 28...Rae8! This is the
calculate the complex great threats and start the queen and hope for a chess equivalent to
lines after 31 Nxe6 Kg8!. to calculate. And before swindle in the endgame a turning the other cheek. It
But if Nunn had stopped we can calculate, we piece down, but with an is clearly the only move,
for a moment and actually must realize whether we active queen. 31.Qxc3 as the alternatives all lose
just looked at the position, have a choice or not. ] [ 31.Qe7 Qd4+ 32.Kg3 directly. For a practical
he would undoubtedly 31...Kf6? This is just Qg4+ 33.Kf2 Qf4+ player this is a nice move
have spotted a far reckless. 34.Ke2 Bg4+ to find. Under pressure,
stronger continuation. [ Black needs to get his and mates is one line losing another pawn and
[ 31.Qxe6! is more or king into safety and this that Heini was forced clearly close to complete
less winning on the spot. can only be done with to analyse. ] failure, he finds a move
There is no way for 31...Kg8! , when the king [ 31.Qb6 d1N+ 32.Kf1 that at least keeps the
Black to defend his king and queen might be on Ne3+ 33.Kf2 Qd2+ engine running for a while.
against all the intruding the same line, but White 34.Kf3 Qxg2+ 35.Kxe3 Now White simply
white pieces. In the cannot exploit it. After Qg1+ was another panicked and was not able
position White is 32.Qe2 Bxe3 33.fxe3 possible line. All in all to use his limited time to
threatening Nf5+, Qg4, Qb7+ 34.e4 Qd7 nothing looks alright for find a playable
Bxe5+ and Rxd6. Black is only slightly White. ] continuation. But still,
Against so many threats worse. The reason for 31...d1N+ The rest is giving up the rook gives
there can be no defence, Black's mistake is likely simple. 32.Ke2 Nxc3+ no chance at all of a
as analysis of the to be time trouble, 33.Kd3 b4 34.Kd4 g5 playable position, and the
position proves: Re8 ( or though Nunn does not 35.Kc5 Ne4+ 36.Kb6 f5 practical chances are also
31...Qb7+ 32.Nfg2 Kf8 mention anything thereof 37.Bd5 Nxd6 38.Kc7 close to nil. So it would
33.Rxd6 Rxd6 in his book. ] Ne8+ 39.Kxd7 Nf6+ have been better to
34.Qxd6+ Qe7 35.Bxe5 32.Bxe5+! It seems that 40.Ke6 Nxd5 41.Kxd5 choose one of the
and wins; actually it is Black somehow missed Kg7 42.Kc4 a5 43.a3 alternatives and then see
a more a slaughter this, bxa3 44.Kb3 f4 45.c4 if anything turned up.
house than a chess [ but even so 32.Nc7!? g4 46.c5 Kf7 29.Rxd5?? Qxd5+
game here ) 32.Bxe5+ is also a winning move. 0-1 From here on the game is
( the most human The attack is simply over. 30.Qxd5 exd5
approach; 32.Qf5; , and too strong. ] 31.Ng5 Rxe2 32.Nf3
32.Qg4 also win easily ) 32...dxe5 33.Nd5+ Webb,S Rxa2 33.Rd7 Rb2
32...dxe5 33.Rd7+ [ 33.Ng4+ Ke7 34.Qd5 Aagaard,J 34.Rxd5 Rxb3 35.Ng5
and White wins, as after Rxe6 35.Qd7+ Swedish League 2002 Ra8 36.Ne6 Rb6 37.Nc7
Kh8 there is a simple would also have won. ] [Jacob Aagaard] Rc8 38.Nb5 a4 39.Nd4
combination with 33...Kxe6 34.Qg4+ Kf7 Ra8 40.Rd7 a3 41.Nf5
34.Ng6+ hxg6 35.Qh3+ 35.Nxb6 Rd8 36.Qf5+ XABCDEFGHY a2 42.Rxg7+ Kh8 .
and mate follows. Ke8 37.Qe6+ Be7 8r+-+-trk+( How should White have
Why did Nunn overlook 38.Rxd8+ Qxd8 39.Nd5 7+R+-+pzpp' 14 continued after 18...Rae8 -
this both in the game 1-0 6-+-wQp+-+& ?
and in his later analysis? 5zp-tRnsN-+-% .
In the game there could 4-+-+-+-+$ (press F10 to continue)
have been all kinds of Danielsen,Hei 3+P+-+-zP-# 0-1
reasons: time pressure, Aagaard,J 2P+-wqPzPKzP"
assumptions, bad form, Taastrup 1999 1+-+-+-+-![
or whatever. In the [Jacob Aagaard] xabcdefghy analysis Webb-Aagaard
preparation of the book it White should start with [Jacob Aagaard]
is another matter. No 28.Nxf7!! This is the only
computer program would (Diagram 13) serious candidate. The
miss this move for the method of elimination (Diagram 15)
world, and yet Nunn 29...d2! A neat idea. It should make this clear.
still does not include it always feels good to Webb played this with This is a truly complicated
in his book (and Nunn is sacrifice the queen, and only five minutes left on situation and it takes
very famous for his use even better to under- his clock, some time to calculate all
of computers in his promote a pawn in the [ seeing lines like: the possibilities. I have
writing). The reason is process. But to be honest 28.Nxf7 Qxe2 ( or personally listed three
that he did not stop to the combination is so 28...Rxf7 29.Qxe6 candidates beyond the
think here (or check simple that it is really no just wins; the tricky Qf4!? move Simon played in the
Excelling at Chess Calculation− Jacob Aagard 6

XABCDEFGHY 32.Nh6+ Kh8 fantastic route to , which was brutally met by


8-+-+rtrk+( . Afterwards, when I was equality: Rd8 34.Qc6 44...Rxf1+! 45.Kxf1 Bb5+
7+R+-+Nzpp' 15 showing the game to a h5 35.Nf2 Qf5+ 36.Kg2 46.Re2
6-+-wQp+-+& few friends, GM Ne3+ 37.Kg1 and now [ There is nothing better;
5zp-tRn+-+-% Djurhuus suggested that Qh3!! 38.Nxh3 Rd1+ after 46.Kf2 Qc2+
4-+-+-+-+$ White would be able to 39.Kf2 Ng4+ Black makes a draw. ]
3+P+-+-zP-# make a draw like this, etc. An amazing and 46...Bxe2+ 47.Qxe2 Qf7+
2P+-wqPzPKzP" but actually he is more original drawing 48.Qf2 Kg7
1+-+-+-+-![ or less winning instantly combination! ] . This endgame is still
xabcdefghy with the defensive/ 31...Qb2 nominally better for White
game: 29 Qe5, 29 Rcc7 aggressive move 33.Rf7! [ After the forcing with the extra pawn, but it
and 29 Qd7. The right which leaves Black in 31...Ne3+ 32.Kh3 Qf6 is not that easy to make
thing, I believe, is to serious trouble. Black 33.Ng4 Nxg4 then progress, and Lasker
calculate them one at a will be able to offer 34.Kxg4! and White eventually drew it without
time and come to resistance after this wins. ] too many problems.
conclusions as quickly as move, but it should not 32.Kh3! Qa1! 49.Ne3 Bf4 50.Ke2 Qc7
possible. Remember, we be too difficult to work [ If 32...Qf6 33.Ng4 51.Qg2+ Kf8 52.Nxd5
calculate not to indulge out that he will be wins. ] Qc2+ 53.Kf3 Bd2 54.Qf1
ourselves in variations, running around like a 33.Nf7+! Qxb3+ 55.Ke2+ Ke8
but to find the best move wounded animal for the [ If 33.Rc1 Qf6 34.Ng4 56.Qf5 Qc4+ 57.Kxd2
as quickly and as reliably rest of the game, if Qg5 35.Kg2 Rd8 Qxd4+ 58.Ke2 Qc4+
as possible. Pseudo- White plays correctly. ] 36.Qc6 h5 37.Nf2 Qd2 ] 59.Kf2 Qc5+ 60.Kg2
thinking with concepts and After 29...Qd4 White needs [ while 33.Rxd5 exd5 Qd6 61.Kf3 Kd8 62.Ke4
ideas will bring you little to find his candidates 34.Nf7+ Kg8 35.Qxd5 Qe6+
satisfaction here. You again. But first things first. Qf6 36.Ng5+ ½-½
need to be concrete and Should White have only leaves White with
to find the relevant predicted this move? I am a clear advantage, but
variations. not sure. He should have at least now we have analysis Marshall-Lasker pt.1
It should not take too long seen it, but also that he choices. ] [Jacob Aagaard]
to find the right first move. would have a wide choice 33...Kg8 34.Rxa5 Qf1+
29.Qd7! afterwards and that Black 35.Kh4 Qa1 36.Raa7 XABCDEFGHY
[ Not too relevant is the has not solved his Qf6+ 37.Kh3 Ne3 8-+-+l+-mk(
piece sacrifice 29.Qe5? problems immediately. If 38.Qd3 Rxf7 39.Qxe3 7+-wq-+-+-' 17
Rxf7 30.Rxf7 Kxf7 we compare this with the and White will win. 6-+-vlp+-+&
31.e4 when it is hard to previous variations it is 5+-+p+-+-%
believe in the white obvious that only after 29 4-+-zP-trN+$
attack, and for this Qd7 can White play for a Marshall,F 3+P+-wQ-+-#
reason alone it is win, and once we have Lasker,Em 2-+-+-+R+"
possible to disregard it assured ourselves that New York 1924 1+-+-+LmK-![
as a serious candidate. Black does not have an [Jacob Aagaard] xabcdefghy
The combination also easy way to neutralize the 44.Bd3! This move has a
has an easy tactical initiative, we can play the XABCDEFGHY double function. First of all
refutation in Qe2! move without wasting time. 8-+-+l+-mk( it pre-empts the tactics
32.Qd4 Nf6 and Black is Here I have analysed two 7+-wq-+-+-' 16 with 44...Rxf1+! which the
close to winning. ] continuations: 6-+-vlp+-+& kids got really tired of
[ More persistent is 30.Nh6+! 5+-+p+-+-% looking at. Secondly, it
29.Rcc7!? Qxe2 [ Stronger than 30.Rc4 4-+-zP-trN+$ sets up a serious threat of
30.Nh6+ ( but not Qf6 , when it is all about 3+P+-wQ-+-# 45 Qh3+, which Black
30.Qe5 Qxe5 31.Nxe5 finding the right path out 2-+-+-+R+" simply cannot allow. So
Nxc7 32.Rxc7 Rc8 of the jungle! 31.Qb5 1+-+-+LmK-![ his response is forced.
and Black is winning ) ( if instead 31.Qa4 Ra8 xabcdefghy Bh5 45.Nh6 The natural
30...Kh8 . This position 32.e4 Ne7 33.Ne5 Here Marshall did not see way to proceed. Black has
could easily have been Qxf2+ 34.Kh3 Qf1+ Black's threat and went for no counterplay, so White
foreseen as well. A 35.Kh4 Qg2 36.Rxe7 44.Qxe6? threatens with 46 Rg8
closer look would have Qxh2+ 37.Kg4 h5+ [ There are other mate and thereby forces
revealed that White is 38.Kg5 Qxg3+ 39.Kxh5 continuations that prove Black's next reply. Rf8
actually in trouble! Qxe5+ and Black wins ) insufficient for White on . Here it is important to
However, there is a nice 31...Rd8 32.Nxd8 Qxf2+ similar accounts; e.g. calculate slowly in order to
resource in 31.Qxf8+!! 33.Kh3 Rxd8 34.Rg4 44.Rh2+? Kg7 grasp the finesses in the
( if he plays 31.Nf7+? Qf1+ 35.Kh4 Qf6+ 45.Qxe6?! Rxf1+ position. Training in
then Rxf7 wins on the 36.Kh3 with a draw. So 46.Kxf1 Bb5+ finding candidate moves
spot ) 31...Rxf8 32.Nf7+ 30 Rc4 does not provide and Black escapes with is also very important.
and White has perpetual us with any advantage. ] a draw once again. ] [ Byron continues with
check because of the 30...Kh8 31.f3! [ Even worse is 44.Nh6? 45...Rf8 46.Qg5
various rook mates. ] [ If 31.Nf5 Qe4+ 32.Kg1 , when Rxf1+ 45.Kxf1 , writing "White suddenly
29...Qd4!? Black's best Qb1+ with a draw. ] Bb5+ is terrible for has the attractive threats
defence. [ After 31.Kh3!? Qf6 White: after 46.Kg1 of 47 Qg8+ and 47 Qf6+
[ He has to be careful, as 32.Ng4 Qg6 33.f3 ( and 46.Ke1 Bf4 as well as the more
after 29...Re7 White remains a pawn 47.Qxe6 Qc1+ 48.Kf2 mundane 47 Qxh5.
White will surely answer up with good chances. Qf1# is not exactly an Black has no good reply.
30.Rxd5!! Rxd7 Or so it seems, but in improvement ) 46...Bf4 Not a difficult
31.Rdxd7 Qxe2 fact Black has a Black is winning. ] continuation to find, but
Excelling at Chess Calculation− Jacob Aagard 7

tricky enough to cause XABCDEFGHY [ 2...Qxd5 is the best square (f6), as Black
Marshall to lose his way. 8-+-+-tr-mk( move here according to slowly completes his
" And then he continues 7+-wq-+-+-' 19 theory and also development. But in fact
with the next position. 6-+-vlp+-sN& according to my own 9...N8d7 is the more or
I would like to mention 5+-+p+-+l% practical experience. ] less the decisive mistake.
that I have not been able 4-+-zP-+-+$ 3.d4 Nxd5 4.Nf3 The knight has no real
to find any other 3+P+LwQ-+-# This is rather harmless function on f6; the only
mistakes in Byron's book 2-+-+-+R+" but still gives White a plan available is
from the examples I 1+-+-+-mK-![ good chance of getting a connected to ...Bf5 and ...
have been working with. xabcdefghy better game. When Ne4 in order to relieve
But here we shall see How should White confronted with a sideline some of the pressure
what 12-year old Alex continue here? 47.-- (such as the Scandinavian through exchanges. But
Hansen decided to play [ 47.Bxg6 Bf4 48.Nf7+ Defence) even strong White can easily prevent
before we do anything Kg7 49.Qh4! ( 49.Qh5 players often play too this, and does so in the
else, as it impressed allows Black an cautiously and therefore game. Instead it was
me quite a bit. ] immediate draw with do not get all the necessary to find real
46.Nf7+! This is certainly Qc1+ 50.Kf2 Qe3+ ) advantage they could; but work for the black pieces,
more forceful, and Black 49...Qc1+ 50.Kf2 Qd2+ on the other hand they and not just let the fingers
cannot do anything to 51.Kf3 ( the only way still avoid walking into nasty do the talking. Black
defend himself. Rxf7 to play for the full point; traps and snowstorms of needed to perform a
47.Qh6+ Rh7 48.Bxh7 51.Kf1 Qd1+ 52.Qe1 blurring tactics, while still correct evaluation of the
Qxh7 49.Qf6+ and mates. Qf3+ 53.Kg1 Be3+ securing just as good position and a discovery
This was done in a is another draw ) chances for an advantage of White's plan (though
training session where the 51...Qe3+ 52.Kg4 Kxg6 as they would have in a not so easy to predict, this
two other boys found 46 53.Nh8+ Rxh8 54.Qxh8 normal opening system. was the task Black had
Qg5 and were praised Kf7 55.Qh7+ Kf8 g6 5.Be2 put before himself by
therefore. But when I went 56.Qg6 Bd6 57.Qf6+ [ 5.c4 was objectively choosing a passive
home and had a second Ke8 58.Rf2 Be7 59.Qf4 stronger. ] formation with 8...c6).
look at the position, I Qxb3 and despite the 5...Bg7 6.0-0 0-0 7.c4 [ The correct move was
began to have doubts extra exchange it is not Nb6 8.Nc3 c6!? 9...Be6! in order to force
about the correctness of obvious that White has This move is actually not White to decide upon
Byron's explanation above. serious winning chances. as bad as I initially his queenside set-up
Maybe it was not as simple The position is thought when Jens played straight away. White has
as I thought, and maybe obviously better for it, but Black should utilize two possible ways to
Alex's win was not just an White, but whether the its advantages creatively. meet this:
alternative way to do it, as advantage is decisive or [ Theory recommends A) 10.Qb3!?
I had explained him, but not hard to estimate. ] 8...Nc6 9.d5 Ne5 is tempting when one
actually the only safe way [ 47.Rc2 (I like this move 10.Nxe5 Bxe5 11.Bh6 sees how the game
to decide the game in more, but White is still Re8 12.Qd2 e6 13.Rfe1 continues, but here
White's favour. not winning as easily as exd5 14.cxd5 Black has a strong
before) Qg7! 48.Qxg6 when White is a little reaction in a5! 11.a4
( 48.Bxg6 Bf4 49.Nf7+ bit better, though it is Na6! , when the knight
Korchnoi,V Rxf7 50.Qh5+ Kg8 nothing special. ] finds an active square
Vaganian,R 51.Qg4 is only slightly [ During the game I also on b4 from which it is
Skelleftea 1989 better for White ) considered 8...Bg4 truly restricting
[Jacob Aagaard] 48...Qxg6+ 49.Bxg6 Kg7 , but afterwards quickly White's options, as well
50.Nf7 Bf4 51.Rg2 found what looks to be a as preventing any
XABCDEFGHY Be3+ 52.Kf1 Bxd4 positional refutation: 9.h3 problems down the b-
8-+-+-+-+({ 53.Bh5+ Kf6 54.Nh6 Bxf3 10.Bxf3 Nc6 file. I find it hard to
7zpp+-+-+-' 18 and White has a piece (the critical moment) believe that White has
6k+l+R+-+& more in the endgame. 11.c5! Nd7 12.Be3 e5 a real advantage here.
5+-wQp+p+-% But with only one pawn 13.Bxc6! (a surprise, but His co-ordination is
4-zPnzP-zP-+$ left, who knows? There the black queenside is simply not good
3zP-+-+-zP-# are many drawn completely ruined and ... enough.;
2-+P+-+KzP" endgames which can Nxd4 is prevented) bxc6 B) 10.b3 is the most
1+-+q+-+-! arise out of the blue. ] 14.d5 cxd5 15.Qxd5 natural move. Now
xabcdefghy ½-½ with a clear advantage Black can either
35...b6!! 36.Qxc6 Ne3+ to White, e.g. Qe7 develop slowly and be
37.Rxe3 Qf1+ 38.Kxf1 16.Qc6 Nf6 17.Bg5 . ] somewhat worse, or
and stalemate. B01 9.h3 With his last move enter the more
½-½ Aagaard,J Black defended the complicated variation
Ostergaard,J diagonal f3-b7 and after ... c5 11.Be3 Nc6 12.d5!
Helsingor 2003 Bg4 would be attacking Bxc3 13.Rc1!
analysis Marshall-Lasker pt.2 [Jacob Aagaard] both c4 and d4. Therefore ( Black looks fine after
[Jacob Aagaard] 9 h3 is a natural reaction. 13.dxe6 Bxa1
1.e4 d5 Jens said after Now comes the first 14.Qxa1 f6 15.Bxc5
the game that he suddenly critical moment in the Qc8 16.Re1 Qxe6
(Diagram 19) got the impulse to try game. What should Black – White has the two
something new. I can play here? bishops, but I doubt
46.Qg5?! is met by a very understand that – but the N8d7? A specious move. they are sufficient for
unexpected counter-strike: Scandinavian Defence...? A badly placed piece is en an advantage or even
Bg6!! . 2.exd5 Nf6 route to a more active for equality ) 13...Bxd5
Excelling at Chess Calculation− Jacob Aagard 8

14.Rxc3 Bxf3 15.Bxf3 but Black has no real my potentially best piece, intuition nor calculation
Qc7 16.Bxc5 Qe5 weaknesses. Instead, what and at the same time can help much in solving
17.Bxc6 Qxc3 I did was to approach the solves some of the such a problem. The
18.Bxb7 Rab8 19.Qf3 position prophylactically – problems with his queen. longest variation was four
and White is better as as Mark Dvoretsky has Note that after 14.Qxe7 moves long and hardly
here the bishops and advised in all his books Bxd3 15.Rxd3 Qf5 anything more than an
the pawn are more and told me to do on White suddenly has illustration of why White
than enough for the every occasion we have three unprotected pieces, was moving in the wrong
exchange. Of course met. So I started to look and all of them are direction. With this kind of
Black has other for Black's plans and threatened. So it positional thinking a
choices, but White found two. appears there is no way strong player can outplay
seems to be able to The first involves 11...Bf5 to prevent ...Be6. a weaker player virtually
prove an opening with the idea of 12...Ne4. However, this is not every time. Unfortunately
advantage.; The knight is a bit stupid the point of prophylaxis. for me it is seldom that I
C) 10.-- ] on f6 and the bishop has The point is to ensure can muster that kind of
[ A pupil of mine no good square either. that the opponent's concentration and clarity.
suggested 9...Bf5 The exchange of knights every desirable But if we go through the
, but as the idea of ... on c3 would give some operation comes at a games of great players,
Ne4 will not happen, breathing room and high cost. So I started to we will find this depth of
there is no function for maybe indicate ...Bf5-e4, go through my shortlist though all the time if we
the bishop on f5, and which White cannot really of positional factors to look for it – especially if
the move is hardly an allow as the pawn on d4 check when in doubt we go through the games
improvement on the would come under attack. about my future of Capablanca, Alekhine,
game. ] [ So my first thought was operations. One of Lasker and all the other
10.Bf4 The natural post for to play 10...Nf6 11.Re1 these factors is old guys, who were clearly
the bishop in any case, in order to meet Bf5 weaknesses. My better than their opponents.
and while it might look as with the natural and weakness is obviously So when they found a
if the bishop is shooting desirable 12.Bf1 c4; Black's potential strong plan the opponents
into thin air, it actually . Not a great plan, but weaknesses are e7 and, usually did not see them
has some important at least something that to my surprise, b7. After and hence did not prevent
prophylactic functions. develops my pieces, as the light-squared them.
The black queen has only well as preventing ...Ne4. bishop moves away b7 is [ Incidentally, in case it
two available squares on Nevertheless, I was not easy to guard. So seems seem that the
the 7th rank (c7, d7) and not filled with a feeling the move played in the first move that came into
none on the 6th. White of ease by this. It game entered my mind my head was the right
can severely disturb seemed that I was not with a strong sense of one, I should add that in
Black's development by exploiting the full clarity and I quickly fact it was not so. I did
not allowing the queen potential of my position, decided upon it. ] not want to waste your
access to these two but rather making a 11.a4!? It was a great time with 11.Qd2?! Bf5
squares. The bishop takes specious operation of my surprise to my opponent and Black is more or less
control over c7, as well as own. ] that his position after 11 OK, even though White
preventing the only active [ Also, Black has a a4 is close to being has 12.Qe3! preventing
possibility behind 9...N8d7, different plan: he can strategically lost. But to ...Nf6-e4. ]
i.e. 10...e5. The knight play 10...Nf6 11.Re1 me it felt logical. He had 11...Be6 Black has no
can always jump to e5 Be6 12.b3 a5 played a passive opening good moves anymore.
later and disturb the queen, , when after 13.a4 system and wasted two [ After 11...a5 12.Re1!
so Black has no easy way I have more space and moves on an operation of Bf5 13.Bf1 White will
to develop his pieces, as some problems persist cosmetic character. continue with 14 Qb3
the queen will always be for Black, but he has Beyond the aesthetic and 15 Rad1. It is not
in the way of the rooks. forced me to fix my nature of the knight easy to see how Black
Nf6 The problem with a queenside pawns and manoeuvre to f6 there is will continue. My team-
move like this is that therefore lose flexibility. nothing: no function of the mate Michael
White now has complete I did not like that much move, no nothing. As I Jørgensen noticed that
control over e5, and Black either. I evaluated the wrote earlier, some 15 Be3 with the idea 16
cannot do anything position after h6 people say that the move d5 was also possible.
against the white pawns to be better for White, you think of first is the one But I would not try to
on c4 an d4. but besides completing you play in 75% of cases capitalize on my
This is the first critical my development I did not – which may be true, but opponent's problems in
moment for White in the see how to improve my how many of these moves such a fashion. Not
game. He has three pieces position. So perhaps are theoretical moves, because I am a nice guy,
yet to be developed. It is White's advantage is not recaptures, exchanges or but because I do not see
natural to think that White so large after all? ] mistakes? Chess is a how he can improve
will play Qd2, Rad1, Rfe1 [ What I needed was a game with great potential his position. When I have
and Bf1 when his pieces way to combat both 11... for depth, both in the chance to improve
are in harmony and he Be6 and 11...Bf5. I calculation and in my position more than
has more space. But quickly came up with strategic thinking. The my opponent has the
although this gives White 10...Nf6 11.Bd3 Be6 solution to this position chance to improve his,
a slight advantage there is 12.Qe2 , avoiding 12 b3. was a move that White at I would rather complete
nowhere particular to go But after Qc8 13.Rad1 first thought Black wanted my mobilization and only
afterwards. White could Bf5! Black succeeds in to provoke, but actually it then strike. We shall
play Bh6 and exchange getting rid of his most turns out to be the other return to this
the right set of bishops, troublesome piece for way round. Neither interesting idea at move
Excelling at Chess Calculation− Jacob Aagard 9

16. ] difference between the right moment for White to do so; instead I saw a
[ 11...Bf5 was perhaps two kinds of advantage take action. When the short line and believed
the objectively strongest is not so great and game changes in this way, that I would not have to
move. White would then Mark thinks this kind of from the positional build- think at all.
continue 12.a5 Nc8 problem is not that up where calculation is [ Correct was 19.Qb4!
( not 12...Nbd7? 13.g4! relevant for training or not really needed, to a full and now we have:
and the bishop is writing, and completely fledged frontal attack, the A) 19...Na6
trapped ) , and either clear examples create way of thinking also needs – I did not like the look
13.Ne5 ( or 13.a6 a stronger impression. to change. Both players of this, but had I gone
with a very large Actually I disagree a little. will have to calculate as far as 20.Qxe7
positional advantage in I believe that clearly accurately and play the it would have been
both cases. explained examples best moves, or the quite obvious that I
Nevertheless, White is create the best evaluation of the position was just being silly.
forced to undertake impression, examples can alter very quickly. White will advance his
active operations before that give the core of a What I personally find extra pawn to d6 and
he has finished his position and the most difficult is to change enjoy life, having a
development and Black reasons behind from feeling the right way clear advantage as
should therefore have different claims. But intuitively and evaluating it well.;
more chances of survival. everybody has their by logic, to working with B) 19...Nd3
Actually this is why I own opinions. )] accuracy and calculation. does not look good
have come to the 12.Qb3! This was of I often mess up my because of 20.Nxf5
conclusion that, although course the idea. Qc8 position in exactly this Nxb4 ( 20...Qxf5
I was enormously proud 13.Rfe1! kind of situation because I 21.Bxd3 Qxd3
about my decision in the [ This is another am not playing with the 22.Qxb7 wins a pawn,
game, and had the important moment in this right frame of mind since if cxd5 23.cxd5
feeling that I penetrated game. It is tempting for (although my logic often Nxd5? 24.Rcd1
to the heart of the White to play 13.a5 proves correct and I and White wins even
problem completely, this Nbd7 14.d5 maintain my playing level more ) 21.Nxe7+ Kf8
was not in fact the case. with an initiative in the despite this obvious flaw 22.Nxc8 Raxc8 23.d6
I was so close and yet so centre. The question is in my thinking technique). Nd7 24.Red1
far. whether this is too soon. In this game I make such and White will win with
The correct move order It did not take me a mistake which I present his extra pawn.;
was 11 Re1! Be6 12 many seconds to decide to you as a pertinent C) 19...Nfe4 20.Nxf5
Qb3 followed by 13 a4 that it was. Black has example, one that should Qxf5 21.Nxe4 Nxe4
and White is no convincing way to not be ignored just 22.Bf3 is the line I had
strategically winning prevent this advance because I win the game prepared in the game.
beyond any doubt. There later, and meanwhile all the same (see the Only here there is a
is nothing to regret about White can improve his Nataf game later on for a big difference: Black
11 a4!? as White still has position more than Black true failure). 16.a5 does not have ...Nd2
a clear advantage; but can improve his. This There is no reason now to with counterplay.
the absolutely special kind of hesitate in playing this Instead the game
prophylactic 11 Re1 consideration often move. I am clearly better might continue cxd5
would not have allowed exists when you have and have no obvious way 23.cxd5 Nf6 24.Qxb7
even the feeble try for an advantage. ] of improving my position Nxd5 25.Bg4 Qf6
counterplay with 11...Bf5. 13...h6 14.Bh2 anymore. It is time to fight. 26.Bc8! and Black
I must say that I am The bishop was Nbd7 17.d5 Again this loses the exchange.
unsure whether unguarded on f4 and a just has to be right. While This is a simple matter
anybody (except a true possible prey to tactics this 17 d5 was not played of comparison. Note
chess genius on a good when White eventually blindfold, I did not see the that it was not that
day) could solve this takes decisive action. need to calculate the position was too
problem in all its details Therefore it is better everything here, as Black difficult for me to
at the board. Sometimes placed on h2. Also, White has only one move and calculate, but that I
true depth is only now has a wide range of White's play follows had problems
possible through long possible answers to 14... naturally. Bf5 18.Nd4 changing my frame of
and hard analysis g5. Rd8 Black cannot do Nc5 mind from logical
repeated many times. much with his position; [ 18...Bg6 was not an thinking and positional
Because honestly, who [ trying to mess things up alternative: 19.a6! play to calculation.
would refuse a position with 14...g5!? and the black queenside Luckily I still managed
such as that after 13 a6 was possibly the best falls apart. ] to win the game.;
when he saw the choice. ] 19.Qa3?! So here it is – D) 19...-- ]
possibility of it arising 15.Rac1 g5!? This is less the miscalculation. 19...Nfe4
with his opponent's best strong now that White has Actually this position [ White is clearly better
play? completed development. should not be too difficult after 19...Nd3 20.Nxf5
I discussed these The only move White for an International Master ( 20.Bxd3 Bxd3 21.a6
considerations with Mark. might still want to play is such as myself. White has Bxc4 22.Rxe7 b6
At first I showed him 11 Be2-f1 when all pieces two obvious moves and 23.Nxc6 Nxd5 24.Rc7
a4 as if it was the are ideally placed, but this one of them could very Nxc7 25.Ne7+ Kh7
solution to the position, is not so important, well be better than the 26.Nxc8 Raxc8
but when I found the especially now Black is other. So the thing to do is is probably good for
refinement and told him, considering some to calculate both and find White, but
he was not too counterplay with ...g5-g4. the difference between unnecessarily
impressed. The Therefore it is logically the them. Here I did not really complicated ) 20...Qxf5
Excelling at Chess Calculation− Jacob Aagard 10

21.Bxd3 Qxd3 22.dxc6 27.Bxe4 Bxc1 28.Bxg6 indeed. White is thinking Positional evaluation, or
bxc6 23.Qxe7 Qxc4 the win is even easier. ] about Qd4+, but mainly he a player with a
24.Ne4 Qe6 25.Qxe6 26...Nxc3 27.Qxg6 Na4? is reorganizing his pieces stronger intuition, would
fxe6 26.Nc5 [ After 27...Rd6 for entering the c-file. not go so far down the
and the endgame looks White wins by 28.Qh5! White's chances for an wrong road but notice
rather sad for Black. Na2 29.Rxe7 Qf8 advantage are still very the rook invading the
Now I had to really think 30.Rce1 and Black much based on his lead in seventh rank, indirectly
for the first time since cannot defend himself. development. attacking the king, the
my 11th move. I had If nothing else then 31 Peter said in the weakness of f6 and e6,
clearly done my best to h4 will free the h2- commentary room that, and the continued X-
throw the advantage bishop and White will although it appears that ray threat on the long
away, but at least I had enjoy his material there are no other diagonal. All these
had my cold shower and advantage. ] constructive moves in the danger signs are of a
now found it no problem 28.Rxe7 There is no position, it took him 30 positional nature and,
to concentrate on playing defence against mate. minutes before he even added together, are
the best moves. After A lucky escape, even started to consider this decisive. Had no safer
around 35 minutes though it might not seem move. I think this is a path existed for Black it
thought I came up with so superficially. great example of his two would be hard to argue
the right way to 1-0 greatest strengths in action. like this, but it does. He
continue. ] The first is thorough does not have to weaken
20.Nxe4 Nxe4 21.Qe3! preparation; 17...Rd8 is a his position or create a
It turns out that White can D86 novelty and Peter was route for the white rooks
still put Black under a lot Nielsen,PH aware of that. The second and knight to enter the
of pressure, who has no McShane,L is his great feel for the position.
easy way of getting out of Copenhagen 2003 pieces. Being an intuitive McShane, on the other
this alive. [Jacob Aagaard] player he mixes hand, who knows what
[ Instead 21.Nxf5? Qxf5 calculation with evaluation he is talking about, saw
22.Bf3 cxd5 23.cxd5 1.d4 Nf6 2.c4 g6 3.Nc3 and intuition constantly, 19...f6 as a purely
Nd2! is not clear at all. ] d5 4.cxd5 Nxd5 5.e4 and this way finds the tactical mistake. He said
21...Bg6 Nxc3 6.bxc3 Bg7 7.Bc4 right move using many he had not overlooked
[ If 21...Nd6 22.Be5! c5 8.Ne2 Nc6 9.Be3 criteria, as he did here. e5 anything like this for
and the black kingside cxd4 10.cxd4 Qa5+ This was the move Black the last year or two,
is enormously fragile. I This was the third main wanted to play. 19.Qa1 and still saw nothing
would consider White's line Grünfeld between the f6? This weakens too wrong with his
position to be two players in just two many squares. approach.
strategically winning, months. For the second [ It was much better to So we disagree. It would
though it might require a time McShane deviates keep the control over e5 be easy for the reader
lot of work to prove it. ] from the previous game – with 19...Re8 to take the point of
[ 21...e6 loses more or here with a speciality of when "it would take a view that McShane is
less by force to 22.Bf3 Svidler's. 11.Bd2 Qd8 long time to come to right, simply because he
c5 23.Nxe6 fxe6 12.d5 Ne5 13.Bc3 0-0 any definite conclusions. is a much better player
24.Bxe4 Bxb2 25.Rb1 14.Bb3 Qb6 15.f4 Nd7 But I would rather show than me. But if this was
Bd4 26.Bxf5 exf5 [ In V.Kramnik-P.Svidler, my tricks." – Nielsen in true then Dorfman's two
27.Qe6+ Qxe6 28.Rxe6 Dortmund 1998, White the commentary room. books should be
Rd7 29.Rxh6 soon had a winning Actually Luke McShane masterpieces, simply
with an extra pawn in positional advantage simply overlooked because he is very
the endgame. ] after 15...Ng4 16.Bd4 something later on (26 strong. Life is not that
22.Bf3! An accurate move. Qb4+ 17.Qd2 Qxd2+ Rf8+!) and therefore simple.
White wants to answer a 18.Kxd2 e5 19.h3 exd4 saw nothing wrong with McShane explains how
later ...f5-f4 with Qd3 in 20.hxg4 19...f6. So was it a he plays chess and I
order to target the g6- . The forthcoming positional or a tactical explain how I would
bishop. Suddenly he is opposite-coloured mistake? I would argue make a decision in this
playing chess again! c5 bishops are of little both. A tactical mistake position. That McShane
[ 22...f5 loses to several importance, compared is an oversight and the does better with his
moves, but most clearly with the strong d-pawn game was decided method than I do with
to 23.Ne6 Bxb2 and the weakness of f7. ] partly because of a mine is irrelevant, since
24.Bxe4 fxe4 25.Rb1 16.Bxg7 Kxg7 17.Rc1 blind spot in I am sure he would do
Bf6 26.Nxd8 Qxd8 Rd8! McShane's calculation better with mine as well.
27.a6 bxa6 28.dxc6 [ 17...Nf6 18.e5 Qb4+ (the first of its kind in I think the right thing is
with an extra exchange 19.Kf2 Ne4+ 20.Ke3 three years according to to look at his opponent,
and the plan c6-c7 and Nc5 21.Qd4 Qxd4+ the man himself). My Peter Heine Nielsen
Rb8. ] 22.Nxd4 b6 23.Bd1 Bb7 argument for it being a (who is of the same
23.Nb5 f5 24.g4! 24.Bf3 gave White a positional mistake, and strength as McShane),
Undermining e4. I want clear edge in the primarily a positional and realize that Peter
that e7-pawn! f4? endgame in S.Atalik-S. mistake, is that does not solve his
A result of time trouble Berndt, European Club McShane allows an problems through
and a bad position. Cup, Chalkidiki 2002. ] awful lot, and lets the calculation alone, but
25.Qd3 a6 26.Nc3?! White to move. The game go in a direction also through a strong
This works out OK, but opening is over. How where his weakened intuition. Obviously there
again actual calculation should the pieces be position depends on a is the matter of style
would have been helpful. organized? minor finesse eight and personality here. I
[ After 26.Na3! Bxb2 18.Rc4!! A very fine move moves further on. am sceptical of relying
Excelling at Chess Calculation− Jacob Aagard 11

too much on calculation, overlooked this. Now he is [ Previously the feeble 26.Qxg7+ Ke8
and fear that this mated. Rxf8 27.Qxe5+ 15.Nd2 was played in S. 27.Bd6! and Black is
practice might be a 1-0 Kishnev-S.Smagin, mated.;
roadblock in the German League 1999. B) 20...Bxe1 21.Be5!
development of Luke With the text move the ( if 21.Rxe1 gxf6
McShane's enormous A21 Norwegian junior 22.Nxf6+ Kg7
chess talent. But I Murshed,N develops his rook to 23.Ne8+ Rxe8
believe that, for the Dorfman,J take over the only fully 24.Rxe8 Be6 25.Rxe6
reader, the lesson Palma de Mallorca 1989 open file in the position fxe6 26.Qxe6 Rf8 )
should be obvious. If you [Jacob Aagaard] and prepares for the 21...Qb6 (the only
can calculate very well, middlegame. Now Black move) 22.Qd3!!
you should of course use 1.d4 d6 2.c4 e5 3.Nc3 has some serious ( a cunning move with
this ability to your exd4 4.Qxd4 Nc6 5.Qd2 problems with continuing the twin threats of 23
advantage; if you cannot, g6 6.b3 Bg7 7.Bb2 Nf6 his development. He Qg3 and 23 Rxe1;
do not feel any pressure 8.g3 0-0 9.Nh3 cannot easily mobilize after 22.Rxe1 Be6
to play as if you can, "Here on White's part one the queenside without Black is OK ) 22...Bb4
but use your intuition can contemplate evolution removing the protection 23.Qg3 ( the attack
and other facets of by Nf4, Bg2 and 0-0. of the knight on f6. ] with 23.Nf6+ gxf6
your chess ability in Nothing similar exists for 15...h6 16.Bh4 Bd6 24.Bxf6 does not really
order to find your own Black. This means that 17.h3! Nb6?! work after Qc7 25.Qe3
balance. ] White has a static [ 17...b6 was the quiet Be7 , though White
20.Qc3 exf4 advantage. Therefore move here. After should still make a
Black executes his plan. Black went in for vigorous something natural like draw ) 23...f6 24.Nxf6+
Now it is also virtually measures, and a double- 18.Re2 White would be Kf7 25.Ne4
impossible to turn back. edged situation after:" a5!? slightly better. Instead and White has a
21.Nxf4 Ne5 22.Rc7+ [ However, it is difficult to Kasparov, in winning attack; e.g. g5
White calculated the understand why Black characteristic fashion, ( or 25...Rg8 26.Qf3+
position accurately in does not have every tries to solve his Ke7 27.Nbc5 Qd8
order to allow his rook to chance to equalize with problems tactically, 28.Bxg7 and Black's
enter the lion's den. Kh8 9...Bf5 10.Bg2 Qd7 though this seems to position collapses )
23.Rf1! Bringing the last 11.Nf4 Rae8 completing violate the inherent 26.f4 g4 27.Nbc5
piece into play. Bd7?! his development. ] principles of the position. Bxc5 28.dxc5 Qb4
After this the win is 10.Nf4 a4 11.Nxa4 Ne4 Black is in no way ready 29.Nd6+ Kg6 30.Nxc8
straightforward as Black 12.Qc1 Nd4 13.Bg2 Re8 for a full frontal and White wins.;
loses a tempo. 14.0-0 Bg4 15.f3 g5 confrontation, and he C) 20...-- ]
[ Nielsen gave the Here Dorfman turns to his comes out of it in a very 20.Kh1 Nd6? Kasparov
following winning line in next example. The game bad way. ] still tries to solve his
the commentary room: finished: 16.e3 Nxf3+ 18.Bxf6! The critical test. problems tactically, but
23...Re8 24.h3!! 17.Bxf3 Bxf3 18.Rxf3 White is obviously ready again his position is not
(a fine waiting move) gxf4 19.Bxg7 Kxg7 for a fight. Nxc4 19.Ne4! ready for it.
Bd7 25.Ne6 Bxe6 20.Qb2+ Qf6 21.Qxf6+ Hitting the knight and [ Necessary was
26.Rxf6 Qg1+ 27.Rf1 Kxf6 22.Re1 d5 23.cxd5 indirectly protecting the 20...Nb6! after which
Qh2 28.dxe6 b5 24.Nb2 Kg7 25.Rxf4 bishop; White can respond in
followed by g2-g3 and Rxa2 26.Nd3 Rd2 [ the point was to avoid two ways:
Black's control over e5 27.Rg4+ Kh8 28.Nf4 f5 playing 19.Bh4?! . ] A) 21.f4!? Bxf4 22.g3
is broken. Here it is 29.Rh4 Ng5 30.Rf1 Rxe3 19...Bh2+ Bg5 ( 22...Bxg3?
very useful that the h- 31.Rh5 Nf3+ 32.Kh1 [ 19...Nb6?? 20.Nxd6 23.Qg2
pawn has advanced Rxb3 33.Ra1 Rbb2 gives White a great is not advisable )
since there is no check 34.Ra8+ Kg7 35.Ne6+ advantage as Black's 23.Bxg5 hxg5
on h4. Kf6 36.Rf8+ Ke7 only move is gxf6 24.Nxg5 f5
Now White calculates 0-1 with a ruin. ] and Black is OK.;
the position sufficiently [ 19...gxf6?? leads to B) 21.g3 Bxh3
to make the correct immediate mate by 22.Be5! ( compare this
decision. ] D52 20.Nxf6+ Kg7 21.Qh7+ with 22.Kxh2 Bxf1
24.Ne6! Rdc8 Carlsen,M Kxf6 22.Qxh6+ Kf5 23.Be5 when Black
[ When he got home after Kasparov,G 23.g4# . ] has an extra option in
the game Nielsen found Reykjavik (rapid) 2004 [ 19...Bb4? Qd7! 24.Rxf1 Nxa4
an email sent by an [Jacob Aagaard] is met strongly by and the position is less
internet spectator, with 20.Qxc4! clear ) 22...Qd8
the following winning 1.d4 d5 2.c4 c6 3.Nf3 with a branching of the 23.Kxh2 Bxf1 24.Rxf1
line: 24...Bxe6 25.Rxf6 Nf6 4.Nc3 e6 5.Bg5 ways: Nd5 ( 24...Nxa4?!
Qg1+ 26.Rf1 Qxh2 Nbd7 6.e3 Qa5 7.Nd2 A) 20...Be6 21.Qd3 25.Ra1 Nb6 26.Nd6
27.g3 Qh5 28.Bd1 Qg5 Bb4 8.Qc2 0-0 9.Be2 e5 Bxe1 ( or 21...Bxb3 is particularly
29.Rf5! gxf5 30.Qxe5+ 10.0-0 exd4 11.Nb3 Qb6 22.Be5 Qe7 23.Qg3 unpleasant for Black )
Kg8 31.Qxe6+ Kh8 I don't much like this move, f6 24.Nxf6+ Rxf6 25.Nbc5 b6 26.Nd3
32.Qe5+ Kg8 33.d6 but that is a question of 25.Bxf6 Qf7 26.Bxg7 Qd7 and Black is only
Qe3+ 34.Kf1 Qd3+ opening theory and not a Kh7 27.Bxh6 Bxe1 a little bit worse.
35.Kf2 Qd2+ 36.Be2 subject for this book. 28.Rxe1 Kxh6 29.Re4 White's best now is
and Black will be 12.exd4 dxc4 13.Bxc4 and White wins ) 27.Nc3! to eliminate
mated. ] a5 14.a4 Qc7 15.Rae1! 22.Be5 Qb6 23.Qg3 the strong knight on
25.Rxf6 Rxc7 26.Rf8+! Amazingly this natural f6 24.Nxf6+ Kf7 d5.;
McShane had simply move is a novelty. 25.Nd7! Bxd7 C) 21.-- ]
Excelling at Chess Calculation− Jacob Aagard 12

21.Kxh2 Nxe4+ 22.Be5 32.axb5 Ra7 33.Rc1 34...Qxh3+ to depart from theory as
Nd6 23.Qc5 Nd4 34.Bb6 Rxa5 with a strong attack.; soon as possible. My
[ 23.d5!? Rd8 24.Nd4 35.Bxa5 Nb3 36.Bc7 B) . However, White opponent is well known
is also strong, but after Nxc1 37.Rxc1 could reply in similar for his hard work on
Qd7 White does not and wins ) 31.Rc1 Qd2 fashion: 31.Bxg7! the opening, so this
have anything better 32.Nb8 Qe2 33.Qxe2 (this would need to be approach made good
than 25.Qc5 transposing Rxe2 34.Rfe1 Rxe1 found before playing practical sense. But
below. ] 35.Rxe1 Be6 36.Re5 30 Nxa5) Qg6 usually I do not like to
23...Rd8 24.d5 Qd7 and White wins.; 32.Rxe8+ Rxe8 deviate from my
25.Nd4!! This move is B) 29...Bb7 30.Nxa5 33.Re1 ( 33.Qb5? opening repertoire and
particularly tasty. Black is wins. Here Bxg2 Nxg7 34.Rg1 Qe4! play inferior moves, as
not given a chance to does not work: gives Black the this often means ending
develop. Nf5 31.Kxg2 Nh4+ 32.Kg3 initiative, as 35.Rxg2? up in a bad position. I
[ Black is in a terrible Qe2 ( after 32...Qg6+ loses to Qf4+ 36.Kh1 would rather risk meeting
state. If 25...b6 26.Qxc6 33.Kxh4 the white Re1+ 37.Rg1 Qf3+ my opponent's opening
Ba6 27.Rg1 and White queen controls g5 ) 38.Kh2 Qxf2+ 39.Rg2 repertoire head on, as
has an extra pawn ] 33.Qd5 and Black's Qf4+ 40.Rg3 Re3 ) experience tells me that
[ or 25...cxd5 26.Qxd6 attack has been 33...Rd8 34.Rd1 they are not perfectly
Qxd6 27.Bxd6 Rxd6 refuted ( but not here with a draw by prepared either, and I
28.Re8+ Kh7 29.Rc1 33.Kxh4?? Qf3! repetition. These lines might improve on their
and the resulting and Black wins. ); are not conclusive, but analysis or on other
endgame with an extra C) 29...Qf6 30.b3 Ba6 at least show that people's play in similar
piece should win easily 31.Rfe1 and Black is a Black has good positions. ]
for White, although pawn down without counterplay.; 12.f4 This move is not
Kasparov would counterplay.; C) 31.-- ] really very good. Nataf
probably make hell hot D) 29...Bd7 30.Nb8!! 30...Nxd4 31.Qxd4 Qg6 decided upon it within five
for the little boy and Bxa4 ( if 30...Raxb8 32.Qg4!? White is minutes, and at first I
fight to the absolute 31.Bxb8 Rxb8 32.Qc7 nominally better in this thought it was very strong
end. ] and White ends the endgame but has no real as the white knight gains
26.dxc6 bxc6 27.Nxc6 exchange up ) 31.Rfe1! winning chances. the d4 square. Later I
Re8 ( the right rook; after [ 32.f3 was a bit more decided that this square
[ The queen is exposed 31.Rde1 Qc8! ambitious, though was not so important.
on d7, so in principle it Black survives, at least Black should be able to [ After the game, when I
was better to play for the time being ) make it by now. ] was falling apart with
27...Qd5!? , although 31...Bxd1 ( 31...Qc8 32...Qxg4 33.hxg4 Bc6 fatigue and Nataf had
28.Qxd5! ( keeping it 32.Rxe8+ Bxe8 34.b3 f6 35.Bc3 Rxe1 awoken fully, he found
simple; 28.Qb6!? Re8 33.Rd8 leads to a 36.Rxe1 Bd5 37.Rb1 Kf7 the following convincing
29.Qb5 is also strong ) winning endgame for 38.Kg3 Rb8 39.b4 axb4 way to gain an
28...Rxd5 29.Rd1 Rc5 White, since if Qe6 40.Bxb4 Bc4 41.a5 Ba6 advantage: 12.c4! a6
30.Rc1 Rxc1 31.Rxc1 34.Qb5! Nd6 35.Qc6! 42.f3 Kg6 43.Kf4 h5 13.Nc3
Bd7 32.Bc7! Rc8 wins the house ) 44.gxh5+ Kxh5 45.Rh1+ A) 13...Nd7 14.a5
33.Nxa5 Bxa4 34.b4 32.Rxe6 fxe6 33.Qc6! Kg6 46.Bc5 Rb2 47.Kg3 and White is better.;
gives White a winning (a nice fork) Raxb8 Ra2 48.Bb6 Kf7 49.Rc1 B) . My attempt to
endgame. ] 34.Bxb8 Rxb8 g5 50.Rc7+ Kg6 51.Rc6 create a blockade on
28.Rd1 Qe6 35.Qxe6+ Kh8 Bf1 52.Bf2 the queenside dark
[ If 28...Qb7 29.Nxa5 36.Qxf5 a4 ( if ½-½ squares with 13...a5
Qe4 30.Rfe1 Qxa4 36...Rxb2 37.Qc8+ was eliminated by
31.b4 and White has an Kh7 38.Qc1 14.c5! dxc5 15.f4! e4!
extra pawn for nothing. ] wins; Black does not B33 ( Black has to play this
29.Rfe1? The critical have a fortress since Nataf,I since 15...exf4?
moment. White makes a White can return the Aagaard,J 16.Bxf4 Bg6 17.Qb3
natural move, but after queen at the right Stockholm 2004 gives White a
this Black is able to moment and enter a [Jacob Aagaard] decisive positional
develop some counterplay. winning pawn advantage ) 16.g4 Bd7
[ The correct move was endgame ) 37.Qe5 1.e4 c5 2.Nf3 Nc6 3.d4 17.Nxe4 f5 18.gxf5
29.Bc7! which is pure and White has a cxd4 4.Nxd4 Nf6 5.Nc3 Bxf5 19.Ng3
prophylaxis against the winning endgame, e5 6.Ndb5 d6 7.Nd5 and Black is in
Black's coming although it would still Nxd5 8.exd5 Nb8 9.a4 trouble.;
development. The take some converting.; Be7 10.Be2 0-0 11.0-0 C) 13...-- ]
bishop is aiming at a5 E) 29...-- ] Bf5?! 12...exf4 13.Bxf4 Bg6
and is no longer in 29...Bb7 30.Nd4!? [ I had only prepared for 9 14.a5?! As I said, White
trouble on e5. But most Putting on the brakes. c4 and did not know the was not at his best.
importantly, Black no [ After 30.Nxa5 theory of 11...f5 [ Later we quickly agreed
longer controls the g5- Black has a nice tactical (which I was and am that White is better after
square, which is very idea in Bxg2! convinced is the best 14.Qd2! a6 15.Na3 Nd7
important in the lines A) , and if 31.Kxg2? move). I had seen that 16.Nc4 with a standard
after ...Bb7 (as can be Nh4+ 32.Kh2 Illescas Cordoba had position for this variation.
seen by comparison). ( 32.Kg3?? loses here played 11...Bf5 against 9 With the text move he
After 29 Bc7! we have: to Qg6+! ) 32...Nf3+ c4 (although that could loses the chance of
A) 29...Qb3 30.Qb5 33.Kg3 Ng5! 34.f4 have been a mistake in going to a3. ]
Qc2 ( or 30...Be6 ( 34.Rh1?? Ne4+ the database!), so I 14...a6 15.Nd4 Nd7
31.Nxa5 Qxb5 wins the queen ) decided to play like this 16.c4 Re8 Another critical
Excelling at Chess Calculation− Jacob Aagard 13

moment. On a normal day greatest flexibility and his White's position simply candidates, White played
Nataf would have realized opponent the greatest cannot be saved with best the text move. ]
which way the tide was inconvenience, if only from play by Black. 27...Re2 This looks logical.
moving and used a lot of a practical viewpoint. If [ 24.Qd3 was necessary [ An interesting
time at this critical the opponent has to attend in order to bring the rook alternative was
moment to find the right to a lot of tactical threats quickly to d1, and after 27...Re4!? (found by
way. Instead he played a all the time, if nothing else Nc5 25.Qd2 Black is Alexander Rosenkilde),
very suspect move. he will have to invest time clearly better, but when White is more or
17.Bh5?! In principle this in avoiding them. nothing immediately less forced to give up
should be a good 20.Bg3?! This logical conclusive strikes the the exchange on f6, as
exchange for White, as it move is actually rather eye. ] 28.Bxc5 ( and 28.Qf2
is his 'bad' bishop for my bad as from here on it is 24...Qb8! Neither to me loses to Bxd4 29.Rxd4
'good' bishop. But actually hard for White to organize nor my opponent was this Re2 30.Qh4 Nxb3
this is a naive his pieces effectively. move anything special. 31.Rxb3 g5 32.Qg4
understanding of the [ The natural move The queen is the worst R2e4 and White's
position. The important 20.Qd2!? was better. placed piece and position collapses )
question is where the Black has a pleasant activating it via a7 is a 28...Qxc5 is horrible. ]
pieces are going and game with Re4 standard manoeuvre in 28.Qc3 "Sometimes, now
where they would like to ( 20...Nc5!? ) 21.Be3 these kind of structures. is not tomorrow." – Esben
go: to give them a definite Rae8 22.Bf2 Nc5 25.Qd3 Lund. Bxd4+!!
function and avoid 23.Ra3 R4e7 [ At first my computer Quite a surprising and
specious moves, as , but White can still program believes that paradoxical move. Black
described above in defend with 24.Qc2! White should try 25.Ne2 gives up his powerful
Aagaard-Østergaard. , and if Ne4? , but quickly realizes its bishop for the weak and
Here the exchange on g6 ( instead Black should mistake. Suddenly Black pinned knight. All in the
is completely in Black's probably continue with has an extra option in name of tactics of course.
favour. The knight 24...Qc8! , bringing the b5! which is very [ I also calculated a long
manoeuvre ...Nd7-c5-e4 is least active piece into uncomfortable for White. variation with 28...Nxb3
now much more pleasant, the game ) 25.Ne6! He is completely 29.Qxb3 Bxd4+ 30.Kh1
while the d4-knight White is definitely back disorganized and Black Bc5 ( 30...Qb8!
becomes somewhat lost in business. Attention is ready to increase the was an important move
in space with no prospects, to such small tricks is pressure. One possible I missed, but I would
despite its central position; an important part of the line to show what White still have rejected the
its only dream was of the defence. ] can expect is 26.Rc2 line since Black is not
f5-square, which will now 20...Nc5 Again an easy bxc4 27.Rxc4 Rb7! well coordinated and
evaporate. Black's dark- move to play, as the 28.Nd4!? ( if 28.Be1 White has counterplay
squared bishop is in no natural move is assisted Ng5 29.Rd3 Rbe7 against d6 and b7 )
way bad either: after ... by the threat of winning 30.Rc2 Ne4 31.Bxc5 Qxc5
Be7-f6 the bishop will the exchange. 21.Rf3 with enormous when, compared with 27
become a monster with Things are no fun for pressure ) 28...Bxd4+! Qc3, White has an extra
strong influence on the White. 29.Qxd4 Nf6 30.Bc3 option in 32.Qxb7 Re1+
centre; the white bishop [ If 21.Bf2 Ne4 22.Be3 Rxb3 31.Rxf6 gxf6 ( 32...f5 leaves Black
on f4 has nothing similar ( 22.Qd3!? is probably 32.Qxf6 Rxc3 33.Qxc3 with a structural
in its crystal ball. Finally, necessary and Black is Qb1+ 34.Kf2 ( 34.Qc1?? advantage, but this is
White no longer guards the clearly better ) 22...Nc3 Re1+! ) 34...Qf5+ 35.Qf3 less than I was
d3-square very well and 23.Qd2 Ne2+! 24.Nxe2 Qe5 and White has bargaining for ) 33.Rf1
will have some problems Bxa1 25.Rxa1 Qe7 enormous problems with . At first I thought I could
with his pawns on the light 26.Bf4 Qxe2 27.Qxe2 his king and pawns, win with Qf2!?
squares, which were Rxe2 28.Bxd6 Rae8 which cannot all be , but instead of ending
better off with the and Black should win saved. ] my analysis here, I
protection of his light- the endgame, though 25...Qa7 26.Rd1 looked for the
squared bishop. White still has some Now Black can no longer opponent's move at the
[ White's position was not counterplay. ] improve his position and end of the variation and
easy, but moves like 21...Ne4 22.Be1 must strike before White found 34.Rdxe1! Rxe1
17.Kh1 ] Allowing the exchange of has a chance to get out of 35.Qc8+ Kh7 36.Qh3+
[ or even 17.Bd3 knight for bishop would trouble. The first move is with perpetual check. ]
would have served him leave White permanently not difficult. Nc5 27.Qd2 29.Qxd4 R8e4
better. ] and unpleasantly worse. This looks strange, but it Winning the queen.
17...Qc7! I used some 10 Re7! Making room for the is understandable if we try 30.Bxc5 Rxd4 31.Bxd4
minutes to picture the other rook to enter the to follow the Qb8 32.Re3! White needs
organization of the black game. Everything is still grandmaster's line of to exchange the rook in
pieces. To be honest, it easy for Black. thought. order to protect his
was not very difficult and I [ 22...b5!? was also [ 27.Qc3 loses to Nxb3 second rank.
could have done it faster, possible, but with 22... 28.Qxb3 Bxd4+ 29.Kh1 [ 32.Rdf1 Qe8 33.Rxf7
but I had enough time. Re7 I had an simple way Bc5 and White has no Qxf7 34.Rxf7 Kxf7
18.Bxg6 hxg6 19.b3 Bf6 of improving my position compensation for the is not an endgame White
It is easy for Black to find and bringing all the pawn ] can survive. ]
the right move order once pieces into play before [ while after 27.Bxc5 32...Rxe3 There is no
he has decided where his seeking a direct Qxc5 he has no defence choice.
pieces belong. He just confrontation. ] on the dark squares. [ If 32...Ra2? 33.Rde1
needs to find the move 23.Bb4 Rae8 24.Rc1?! Having eliminated Kf8 34.Re8+! Qxe8
order that grants him the After this move I am sure these possible 35.Bxg7+ Kxg7 36.Rxe8
Excelling at Chess Calculation− Jacob Aagard 14

and Black has made completely finished. undermine the c4-pawn In the game White must
trouble for himself. ] So Black has at least and then Black will win a have felt completely
33.Bxe3 Qe8 34.Bf4! three directly winning pawn. Later he will be exposed to the public in
My opponent was fully lines, which could bring his king to h5 and all his stupidity, just as we
awake now and started to easily have been push the g-pawn and in all have over and over
display some of his calculated accurately the end White will not be again. He resigned.
enormous strength. At this had I invested some able to resist. But Nataf 0-1
point I had 25 minutes energy in doing so. would probably not have
remaining to reach move The problem was that I allowed me to do all of
40 and should have been was getting tired and, this without a fight... Ziegler,A
cruising. My simple chess facing a little resistance ½-½ Aschauer,G
had won the queen for in the most promising Aschach 1999
insufficient material and line, chose not to make [Jacob Aagaard]
with just a little accuracy I a real effort calculating it. A27
should win. However, I Not so strange, then, Doroshkievich,V XABCDEFGHY
was now getting tired and that things went as they Tukmakov,V 8r+-wqkvl-tr(
losing focus. So instead of did. ] USSR Championship, Riga 7zplzpp+-+-' 20
sensing the critical 35.h4! I was still under the [Jacob Aagaard] 6nzp-+psn-+&
moment, I decided on a impression that I just had 5+-+P+psN-%
long-term plan. This was a to bring the king to d7, 1.c4 e5 2.Nc3 Nc6 3.Nf3 4-+P+-+-zp$
great psychological and then the black queen f5 4.d4 e4 5.Bg5 Nf6 3+-+-zP-zPP#
practical mistake, no would enter the white Up to this point everything 2PzP-+-zPL+"
matter the objective value position and some pawn looks normal and had 1tRNvLQmK-+R![
of my long-term planning. would drop off. This kind White now played 6 Nd2 xabcdefghy
If I want to make things of thinking will be dealt probably no one would Sometimes surprises can
easier for myself, it is with under the heading remember this game. throw us off balance and
better to expend some "assumptions" in the next However, White decided then it is important to stay
energy in calculating the chapter. Kf8 36.g3 Ke8 to play actively and enter alert. It is easy to lose
win at the appropriate [ Even now after a mass exchange, which balance when something
moment, rather than 36...Qe2!? 37.Bxd6+ he expected to lead to a surprising happens on the
delaying it for a later Ke8 38.Rb1 Qe4 superiority in the centre. board. It seems fairly
occasion. Qe7?! 39.Rf1 Qe3+ 40.Kg2 Unfortunately, what he did definite that Black had not
Thus I played a horrible, Qxb3 Black is doing was to calculate only the anticipated... 10.Nf7?!
unambitious move. There well. ] most obvious moves in his This move is not very good,
are no real excuses 37.Kf2 Qc7 38.Re1+ Kd7 head, forgetting that the but it wins the game
though one can be 39.Bg5 f6 40.Bd2 Qc5+ opponent is not a passive almost immediately. If
produced. 41.Re3 Qd4 collaborator but the Black had been able to
[ After the game Nataf Having reached the time antagonist, who will try to keep his cool he would
showed me the control, I suddenly crush us if we give him have seen through
following winning line: realized that things are the chance. Here White White's bluff and not taken
34...Qe2 not as easy as I had completely forgot that his him on his word.
A) 35.Rf1 Qa2 36.Rf3 expected. 42.Bc3 Qg4 opponent would look [ Correct was 10.0-0!
( 36.Bxd6 Qxa5 43.Bd2 Qd1 44.Bb4 seriously at every move , after which Black would
also wins ) 36...Qxa5 Qc2+ 45.Ke1 Qb1+ and be more than happy not have enough for his
37.Bxd6 Qe1+ 38.Rf1 46.Ke2 Qc2+ . to find a mistake in his pawn. ]
Qe3+ 39.Kh1 Qxb3 The position is still calculation. 6.d5? exf3 10...Qe7?
and Black wins.; winning, but I could not 7.dxc6 fxg2 8.cxd7+ [ Black should of course
B) . I had in fact seen see how and, with no Nxd7! It is so easy to think play 10...Kxf7! 11.dxe6+
all of this, but simply fighting energy left, that Black is forced to dxe6 12.Bxb7 ( not
overlooked that 35.Rd2 decided to end the game recapture with the queen 12.Qxd8?? Bxg2!!
is met strongly by here. Nataf was very or bishop, as the knight is and Black ends with a lot
Qe1# .; surprised when I offered pinned. But once we start of material for the
C) 35.-- ] him a draw, as he really to look at the queen ) 12...Qxd1+
[ I did not spend much considered his position lost, position and think about 13.Kxd1 Rd8+ 14.Nd2
additional time on this but previous experiences the moves, instead of ( 14.Ke2 Nc5 15.Bc6
position before writing have taught me not to assuming anything, we will hxg3 16.fxg3 Nfe4
this book, and as I partly burn the bridge I am be able to avoid these is also very tasty for
expected it is actually crossing when I am tired, kind of mistakes. This is Black ) 14...Nc5 15.Bg2
much worse for White as everything is happening why I say, first of all, hxg3 16.fxg3 Nfe4
than it looks. Besides in a very slow tempo and I calculate wide, not deep. and Black has taken
34...Qe2 Black has other might not get off the The point is that whatever over the initiative. ]
ways to terminate the bridge before it falls apart. happens in the long run is 11.Nxh8 Qg7 Black had
game to his advantage; After the game Nataf and I not important, if you drop probably counted on
for instance 34...Qd8 did not find it easy to find something here and now. winning back the knight
35.Ra1 Qf6 36.Rf1 Qc3 a winning plan, which First be sure that you now and thereby gaining
37.Bxd6 Qxb3 states something about know what is going on compensation, but he was
and Black wins. ] how tired we both were. right in front of your nose, in for a cruel shock.
[ But best of all I like The next day I did not find before you run away in 12.Nf7! If it works once, it
34...g5!? 35.Bxd6 Qe3+ it difficult to see that long variations. This is can work again! Nc5
36.Kh1 Qxb3 37.Rc1 Black needs to play ...b7- best achieved by lowering 13.Ne5 and White
Qb2 38.Rg1 Qa2 b6, take it with the queen, the tempo at which we managed to win the game
and White is get his pawn to a4 to calculate.
Excelling at Chess Calculation− Jacob Aagard 15

a rook up. Miezis,N Nc6! (again an unforcing


1-0 Manievich,V [ Another interesting way move) when White
Zuerich 1999 to play the position was cannot play 16.Qxg4?!
[Jacob Aagaard] with the more forcing Qxc3+ 17.Kf2 d4
B63 13...d4!? 14.Qxd4 Ng4 as his position is torn
Anand,V XABCDEFGHY 15.Qxe4 Nc6 16.0-0-0 to pieces. ]
Benjamin,J 8r+-+-trk+({ ( not 16.Be2?! Rfe8! 14...Bxf6 15.Qd2
Wijk aan Zee 1989 7zppzpqsnpvlp' 21 17.Bxg4 Bf6 18.Qd3 White tries to unpin the
[Jacob Aagaard] 6-+-zp-snp+& Nb4 19.Qd2 Bxc3 knight. Now 15...d4 is
5+-+-zpPvL-% 20.bxc3 Nd5 and White perfectly playable but
1.e4 c5 2.Nf3 d6 3.d4 4-+P+P+-+$ is torn apart ) 16...Rae8 Black was looking for
cxd4 4.Nxd4 Nf6 5.Nc3 3+-sNP+-zP-# which gives Black something more.
Nc6 6.Bg5 e6 7.Qd2 2PzP-wQN+KzP" good counterplay as [ My computer is so fed
Be7 8.0-0-0 0-0 9.Nb3 1tR-+-+R+-! well: while White has up with the white
Qb6 10.f3 Rd8 11.Kb1 xabcdefghy gained two pawns Black position that it suggests
d5 12.Bxf6 dxe4?? 13...gxf5 14.Bxf6 Bxf6 has mobilized all his 15.Ke2!? , but this is not
This mistake is due to 15.Nd5! This move might forces. ] a real option as after d4
assumptions. If Black had surprise some people, but [ 13...Ba3?! 16.Nxe4 Rae8
investigated the lines it is very natural indeed. is also thinking outside the white king is wide
properly he would have White exchanges the the box, with the idea open and Black wins
realized his mistake in black knight and in this 14.bxa3? ( but after easily; e.g. 17.Bg2 Qxf5
advance. way achieves superiority 14.Bc1! there is no 18.Bd2 Rxe4+ 19.Bxe4
[ Theory seems otherwise on the light squares. The obvious way to proceed ) Qxe4+ 20.Kf2 Bh4+
to offer Black next move is of course the 14...Qxc3+ 15.Bd2 21.Kg1 Re8 22.Qh5
reasonable chances real point. Nxd5 Qxa3 16.gxf6 e3 Re6 23.f5 g6
here. For example, 16.Rxf5!? 17.fxg7 Re8 and Black and it is about time to
12...Bxf6 13.exd5 Bxc3 [ 16.cxd5 would give gets a very strong resign. ]
14.Qxc3 exd5 15.Qc5 Black the chance to attack against the 15...Bh4+ 16.Kd1
d4! gave Black an contemplate f4!? uncastled white king. ] [ If 16.Bf2 e3! 17.Qxe3
acceptable game in F. , which Miezis decided 14.gxf6? Rfe8 and Black wins the
Handke-A.Kveinys, there was no reason to [ 14.Bd2 is met by the queen ]
German League 2001. ] allow. White will win archetypal e3!! [ while 16.Ke2 d4
13.Bxe7! Rxd2 14.Nxd2! his piece back next and White cannot keep 17.Nxe4 Qxf5
It is not hard to guess that move and keep his the centre closed. After looks no better than the
this move was somehow strong pressure down 15.Bxe3 ( White cannot game. ]
overlooked by Benjamin, the f-file. ] be unforcing himself as 16...d4! This now comes
or at least the 16...Bg7 Now White has a Black will not buy it: if with greater force.
consequences of it were. good knight vs. a bad 15.gxf6 Bxf6! 16.Bxe3 17.Nxe4
Or it might have been that bishop as well, Rfe8 17.Qd2 d4 18.Ne2 [ 17.Bxd4 should also be
he did not analyse the line [ so Black should have Nb4! and White is analysed. Then Black
all the way to the end – a considered 16...Ne7 outplayed ) 15...Rfe8! has a very clear
form of mistake we will to keep his own knight, (all Black cares about is advantage after Rad8
discuss in the next though his position development at this 18.Nxe4 Rxd4 19.Bd3
chapter. exf3 remains untenable stage) 16.Bd2 ( 16.gxf6 Qxf5 20.Qe3 Rfd8
[ If 14...Nxe7 15.Nc4 nevertheless. ] Bxf6 transposes to the and it is a miracle that
Qc7 16.Nb5 and White 17.cxd5 Qa4 18.Nc3 previous bracket ) White has not lost any
wins. ] Qb4 19.Raf1 f6 20.R1f2 16...Bc5+ 17.Be2 Ng4 material yet. But do not
15.gxf3?! c6 21.Qe2 Rf7 22.Qg4 18.Ne4 ( or 18.Rf1 Qb6 worry – he soon will! ]
[ Here White should have Raf8 23.Qh5 Qb6 and it will soon be all 17...Qxf5 18.Ng3
played 15.Nc4! Qc7 ( if 24.R5f3 Qd8 25.g4 Qd7 over ) 18...Bb4! 19.c3 [ After 18.Bd3 dxe3
15...Qf2 16.Ne4! ) 26.Rg3 a6 27.Rf5 Qe8 Rxe4 20.cxb4 Qb6 19.Qxe3 Nb4
16.Bd6! after which there 28.Qh4 Qd7 29.Kh1 Bh8 and Black wins. ] White will have similar
is no defence to be 30.Qh3 Bg7 31.Qf1 c5 [ White would do better problems to those in
found for Black. Instead 32.Rff3 Bh8 33.h3 Re8 to develop some pieces the previous note. ]
the rest of the game was 34.Kh2 of his own and try to 18...Bxg3 19.hxg3 dxe3
a collection of errors. ] 1-0 combat Black's plan 20.Bd3 Rad8! Black is
15...e5 16.Bh4? Be6 instead of fighting his still awake and does not
17.Nde4 Nd4 18.Bg2? individual pieces. feel at all forced to
Rc8 19.Bf2 f5! 20.f4! B82 14.Bb5! was the best exchange queens on
fxe4 21.fxe5 Rc4 De Firmian,N chance, even though White's terms. 21.Qc3
22.Rhe1? Rb4? 23.Kc1! Ehlvest,J White still appears to be [ If 21.Qxe3 Nb4 22.Kd2
Bg4? 24.Nd5 Qc5 Reggio Emilia 1989 in trouble. Fritz comes Qd5 and White will not
25.Nxb4 Bxd1 26.Kxd1! [Jacob Aagaard] up with the wonderful survive the attack. ]
e3 27.Rxe3 Nf5 28.Bd5+ Ba3!? as a possible 21...Qe4
Kf8 29.Rf3 Qxb4 1.e4 c5 2.Nf3 e6 3.d4 refutation. The main idea [ Black has many ways to
30.Rxf5+ Ke8 31.e6 cxd4 4.Nxd4 Nf6 5.Nc3 is 15.bxa3 win here; e.g. 21...Ne5!?
1-0 d6 6.f4 Be7 7.Qf3 0-0 ( White should probably 22.Qxe5 Rxd3+ 23.Ke1
8.Be3 e5 9.Nf5 Bxf5 try something calm like Qxe5 24.fxe5 Rd2
10.exf5 Qa5 11.g4? 15.Bc1!? and there might with a winning rook
This does not look sound. still be hope; suddenly endgame ]
e4! 12.Qd1 d5 13.g5 it will be possible to [ or 21...Rxd3+ 22.cxd3
A threat to the knight?! take on f6! ) 15...Ng4! Rd8 23.Kc1 Rxd3
Excelling at Chess Calculation− Jacob Aagard 16

24.Qc4 Nd4 25.Re1 26.axb3 and White has a XABCDEFGHY D20


Rd2 and White loses the strong attack for nothing. 8r+-tr-snk+( Miles,A
queen. ] One line could go Nh7 7+-wq-vlpzpp' 24 Deep Thought,COMP
22.Rg1 Rxd3+! 27.f4 Qe7 28.f5+ Kh6 6-+lzpp+-+& Long Beach 1989
Time to wrap up the game. 29.Qh3 g4 30.Qh1 Rh8 5+-zp-+-+-% [Jacob Aagaard]
23.cxd3 Rd8 24.Kc2 31.Nf4+ Kg7 32.e5 4-+P+PsN-zP$
Rxd3 25.Qxd3 Nb4+ Bxg2 33.f6+ Nxf6 3+p+-+-zP-# 1.d4 d5 2.c4 dxc4 3.e4
26.Kc3 Nxd3 34.exf6+ Qxf6 35.Qxg2 ] 2PvL-wQ-zPL+" Nf6 4.Nc3 e5 5.Nf3
0-1 24.Qxf7+ Kh6 25.g4! 1+-+RtR-mK-![ exd4 6.Qxd4 Bd6
It looks as if Black should xabcdefghy 7.Bxc4 0-0 8.Bg5 Nc6
be able to survive, but Kxg7? still loses to 9.Qd2 h6 10.Bh4 Bg4
Rinck,H surprisingly it is not so. 23.Nh5+ Kg6 24.Qf4 11.0-0-0 Bxf3 12.gxf3
Deutsche Schachzeitung Ra7 26.g5+ Bxg5 Kxh5 25.Qxf7+ Kh6 Nxe4! Exploiting the
[Jacob Aagaard] 27.hxg5+ Kxg5 28.f4+ 26.g4 Qd7 27.g5+ Bxg5 unprotected bishop on h4.
Kg4 28.hxg5+ Kxg5 29.f4+ 13.Bxd8
XABCDEFGHY [ Or 28...Kh6 29.Qf6+ Kh6 30.Qf6+ Ng6 [ Actually White could
8k+-sn-+-+( Ng6 30.Re3 with mate 31.Re3 e5 32.f5 still have saved the
7zp-vLK+-+-' 22 next move. ] and wins. ] position after 13.Qxh6!!
6-+-+-+-+& 29.Bh3+! But Fritz wants to try the , when it appears that
5+p+-+-+-% [ Black resigned because subtle 22...Rb8!! White keeps the skin on
4-+-+-+-+$ of 29.Bh3+ Kxh3 and there is no easy way his teeth: gxh6 14.Bxd8
3+P+L+-+-# 30.Qh5+ Kg3 31.Qg5+ for White to create a quick Nxf2 ( or 14...Rfxd8
2q+-+-+-+" Kf3 32.Rd3# . mate. For example: 15.fxe4 with a roughly
1+-+-+-+-![ At first sight this looks 23.Nh5 Black needs to equal position. )
xabcdefghy like a nice combination defend the king, but how? 15.Rhg1+ Kh7 16.Bf6
1.Be4+ Nb7 2.Kc8 by White, which wins at f6! Bf4+ 17.Kc2 Bg5
The first two moves had to least a piece. But [ If 23...Nd7 24.Qh6 Qd8 18.Nd5 Nxd1 19.Bd3+
be played, otherwise closer inspection shows 25.Bh3 b2 26.Bxe6 Kg8 20.Bxg5 hxg5
White had no counterplay, that Black should not and Black will soon find 21.Rxg5+ Kh8 22.Rh5+
but what happens after... follow these natural the attack irresistible. ] Kg8 with a draw. ]
Qa6 3.Bd5! Again the only and seemingly forced 24.Qh6 Ng6 13...Nxd2 14.Bxc7
move, but after... b4 lines. When defending it [ Not 24...b2? 25.e5! Bxh2!! A wonderful
...it is easy to stop here is especially important Bxg2 26.exd6 Bxd6 desperado – the bishop
and search in another to step out from the 27.Nxf6+ Kf7 28.Bxf8 sells itself for a pawn, as
direction. But White still most obvious variations, Rxf8 29.Kxg2 the white bishop will
has one possible move. as these are usually and Black is in deep continue to hang on c4.
4.Bc6! Now it is zugzwang. the justification for the trouble. ] [ Instead if 14...Bxc7
Black has no alternative to attacker's aggression. 25.Bh3 Qd7 26.Bh8 15.Kxd2 and White is
taking the bishop. Qxc6 Of course these lines Incidentally, you cannot only very slightly worse
stalemate. should be checked and expect a computer and will probably escape
½-½ mistakes in them instantly to give you the with a draw. ]
exploited, but often we right answers. It is just 15.Bxh2 Nxc4
need to think out of the another tool to help you The resulting endgame
Velicka,P box. improve your thinking. holds no real hope for
Jansa,V In the example above, Here it takes the computer White. 16.Rd7 b6 17.f4
Czech Championship, Zlin had he decided not to fall a long time to understand Rad8 18.Rhd1 Rfe8
[Jacob Aagaard] in with White's logic of that White is not cruising 19.b3 Rxd7 20.Rxd7
hunter and prey, but towards a full point, but is Re1+ 21.Nd1 N4a5
XABCDEFGHY instead taken up the actually on his way into 22.Kd2 Rh1 23.Bg3 h5
8r+-tr-snk+( challenge fully, Black trouble. Bf8! 27.Nxf6+ 24.f5 h4 25.Bf4 Nb4
7+-wq-vlpzpp' 23 might have spotted some Kxh8 28.Nxd7 Bxh6 26.a3 Nbc6 27.Rd3 Nd4
6-+lzpp+-+& extra resources which 29.Nxb8 Ba4 30.Rxd6! 28.b4 Nac6 29.f6 gxf6
5+-zp-+-+-% would have kept the White needs to fight for a 30.Nc3 Ne6 31.Nd5 Kg7
4-+P+PsN-zP$ game very unclear draw here; 32.Bd6 Ng5 33.Ke2 h3
3+p+-+-zP-# indeed: [ if 30.Rb1?! Ne5 34.Rd1 Rxd1 35.Kxd1
2PvL-wQ-zPL+" . 31.Kg2 Nxc4 Ne5 36.Ne3 h2 37.Nf5+
1+-+RtR-mK-![ (press F10 to continue) ] and Black has a strong Kg6 38.Ng3 Ne4 .
xabcdefghy 1-0 initiative. ] A very impressive win
White uses the three piece 30...b2 31.Rb6 Bc2 from the computer
rule to crack Black's 32.Nd7 Ra1 33.Rf1 program, whose
defence: one piece to B80 Rxf1+ 34.Kxf1 b1Q+ descendent later went on
sacrifice, one to protect analysis Velicka-Jansa 35.Rxb1 Bxb1 36.Bxe6 to beat Garry Kasparov.
and one to deliver mate. [Jacob Aagaard] Bxe4 37.Nxc5 0-1
21.Bxg7!? Kxg7 and the endgame should
22.Nh5+ Kg6! be a draw.
[ Not 22...Kh8? 23.Qh6 (Diagram 24) I do not present this as in Liberzon,V
and mates. ] any way conclusive Dlugy,M
23.Qf4! With the idea of 21.Bxg7!? Rxa2! analysis to the position, Palma de Mallorca 1989
Qg4+. Kxh5?! Maybe not [ instead of 21...Kxg7 ] but at least this approach [Jacob Aagaard]
the best defence, 22.Qe3 would have offered Black
[ but if 23...h6 24.Qg4+ [ This seems to make a much better chance of 35.c4?
Bg5 25.hxg5 hxg5 little difference. 22.Qe3 survival. [ Here White could have
Excelling at Chess Calculation− Jacob Aagard 17

XABCDEFGHY develop the bishop. ] XABCDEFGHY when he remains a piece


8-+-+-+k+( 10...Rb8 11.b3 Ng4? 8-+-mk-+-+( up. ]
7+-+-+pzpp' 25 Spot the desperado! 7+-+-+-sN-' 26 39.g3 Nd5 40.Nc4 Nb6
6-+-sn-vl-+& 12.Nxc6!! Only like this. 6-mK-+-+-tR& 41.Nd2 Rb4 42.Bc3
5sNqvL-+Q+-% The idea is to remove the 5+-zp-+-+-% Rxb1+ 43.Nxb1 a4
4-zP-+p+-+$ protection of the knight on 4-+-+-+-+$ 44.Na3 Nd5 45.Bb2 g5
3+-zPn+-+-# g4 and simply grab a 3+-+-tr-+-# 46.Nc4 h5 47.Ne5 Nb4
2-+-+-+PzP" pawn. 2-+-+-+-+" 48.Kf1 Nc2 49.Ke2 Kh6
1+-+L+-+K![ [ If White had changed 1+-+-+-+-![ [ If 49...a3 50.Kd2 axb2
xabcdefghy the move order, it would xabcdefghy 51.Kxc2 Kg7 52.Kxb2
kept his position alive have been Black who 2.Ne6 Rb3+ Again Black Kf6 53.Nf3 Kf5 54.Nh2!
with 35.Qd5! Nxc5 stepped out of the has little choice. Notice Ke4 55.Kc2 and White
36.c4!! when Black has desperado scenario: that the threat of mate or wins. ]
no way to win the game 12.Bxe7 Qxe7 a threat to the queen is 50.Kd3 Nb4+ 51.Kc4
straight away, albeit 13.Nxc6?? often as useful as a check Nd5 52.Bc1 Nf6 53.Kd3!
several ways to keep is met strongly by Qf6! when calculating variations, Leko is well known for his
the advantage, one of winning material, since as the opponent is no less superb endgame
which is Qb6 ( 36...Qe8 after 14.Nxb8 Qxf2+ limited in his options. technique.
37.bxc5!? 15.Kh1 Qh4 White has 3.Kc6 Kb8 4.Rd8+ Ka7 [ Here, instead of winning
is not completely clear ) to give up the queen by 5.Rd7+! White keeps the a pawn with 53.Nf7+
37.Qa8+ Qd8 38.Qxd8+ 16.Qxg4 ( as 16.Kg1 black king under control. Kg6 54.Nxg5
Bxd8 39.bxc5 Ne8 Qxh2+ 17.Kf1 Qh1+ Kb8 , he removes all hints
and Black looks better, 18.Ke2 Qxg2+ 19.Kd3 [ If 5...Ka6 6.Nxc5+ of counterplay. ]
but how much? Qg3+ 20.Kc4 Ne5+ wins the rook ] 53...Nh7 54.Ke2 Kg7
Instead of this the leads to mate. )] [ or 5...Ka8 6.Nc7+ Ka7 55.Nd7 Kg6 56.Nc5 Kf5
game ended quickly. ] 12...Bxc6 13.Bxe7 Nxf2 7.Nb5+ and mate next 57.Kf3 Nf6 58.Nxa4 h4
35...Qxb4!! Otherwise White just won move. ] 59.gxh4 gxh4 60.Nc3
This desperado wins the a pawn. This is a true 6.Nc7! Threatening 7 Ng4 61.Ne2 Ne5+
game instantly. desperado scenario. Both Rd8+ leading to mate. 62.Kg2 Nd3 63.Be3 Kg4
[ The alternatives cannot players have pieces Black again has only one 64.f3+ Kf5 65.Nc3 e5
compete: hanging and they are move. Kc8 7.Re7! 66.Ne4 Ne1+ 67.Kf2 Nc2
35...Qxa5? 36.bxa5 trying to sell themselves [ Not 7.Rh7? Kd8 68.Ke2 Nd4+ 69.Bxd4
Nxf5 37.a6 and White as dearly as possible. 8.Rh8+? allowing the exd4 70.Kf2 Ke5 71.Nc5
wins. ] 14.Bxd8 Nxd1 king to escape to e7; Kd5 72.Nd3 Kc4
[ 35...Qxc5 36.Qxc5 So what was the point of ( instead 8.Rd7+ 73.Ne5+ Kc5
Nxc5 37.bxc5 all of this? 15.Nd5!! would repeat moves and [ If 73...Kd5 74.Ng6
with a slight edge for The white knight removes still win the game. )] wins. ]
Black. ] itself from the possibility 7...Rd3 Again forced. 74.Kg2 Kd5 75.Ng6 d3
[ 35...Qxc4 36.Bxd6 Qc1 of being captured and 8.Nd5 76.Kf2 d2 77.Ke2 Kd4
37.Qf1 Qxd1 38.Qxd1 indicates a check on e7. [ White could also 78.Nxh4 .
Nf2+ 39.Kg1 Nxd1 The idea behind the move, prevent ...Rd8 by 8.Ne6 All this is very well, but as
with unclear and the entire combination, Kb8 9.Re8+ Ka7 Leko writes in his
consequences. ] is that the knight is better 10.Nc7 which wins as annotations, Bareev had a
36.Bxb4 Nxf5! lost on d5, where the well. ] good chance of keeping
White is lost. The twin bishop will be hanging 8...Kd8 9.Rd7+ White in the unpleasant
threats of ...Nxb4 and ... later, than on c3, where White wins the black rook pin by dominating the
Nf2+ decide the game. the knight can capture it with a knight check on the white knight with:
37.g4 Nd4 on its way out. Rbxd8? next move. .
0-1 After this White wins the 1-0 (press F10 to continue)
exchange. 1-0
[ The only move was
C66 15...Bxd5 16.Bxc7 Rbc8 Leko,P
Skovgaard,T 17.Bxd6 Rfe8 18.Raxd1 Bareev,E B19
Haubro,S Bxe4 19.c4 and White Dortmund Candidates analysis Leko-Bareev
Copenhagen (rapid) 1998 has excellent winning [Jacob Aagaard] [Jacob Aagaard]
[Jacob Aagaard] chances with his extra
pawn in the endgame. ] XABCDEFGHY XABCDEFGHY
1.e4 e5 2.Nf3 d6 3.d4 16.Ne7+ Kh8 17.Nxc6 8-+N+-+k+({ 8-+N+-+k+({
exd4 4.Nxd4 Nc6 Nc3 18.Nxd8 Rxd8 7zp-+-+-zp-' 27 7zp-+-+-zp-' 28
Not a very theoretical 19.Re3 Nb5 20.e5 Nd4 6-+-+p+-zp& 6-+-+p+-zp&
opening, but with the next 21.exd6 cxd6 and White 5+-+-+-+n% 5+-+-+-+n%
move we return to normal won. 4-tr-+-+-+$ 4-tr-+-+-+$
lines. 5.Bb5 1-0 3+-+-+-+-# 3+-+-+-+-#
[ 5.Nc3 was probably 2-vL-+-zPP+" 2-vL-+-zPP+"
better, in order to 1+R+-+-mK-! 1+R+-+-mK-!
organize queenside Topko,L xabcdefghy xabcdefghy
castling after Be3, f2-f3 Ceskoslovensky Sach 35...Nf4 36.Ne7+ Kh7 35...Rb7!! Now Black has
and Qd2. ] [Jacob Aagaard] 37.Nc6 Rb3 38.Ne5! a5 serious threats like ...a7-
5...Bd7 6.Bxc6 bxc6 [ 38...Ne2+ 39.Kf1 Nc3 a5-a4-a3 and ...Nh5-f4-d3,
7.Nc3 Nf6 8.0-0 Be7 1.Rd6+ Kc8 There is no does not win the in both cases regaining
9.Re1 0-0 10.Bg5 choice. exchange, since White the piece. If White wants
[ 10.b3 is another way to [ If 1...Ke7 2.Nf5+ wins. ] has 40.Rc1! to avoid this, he can try to
Excelling at Chess Calculation− Jacob Aagard 18

escape with the knight. with solid control over is no improvement for XABCDEFGHY
36.Nd6 Now 37 Nc4 is the c5-square). Black. After Qf6 8-+r+r+k+({
coming, after which White Therefore Black decided 30.Qxf6+ Kxf6 31.Rxe8 7+-+-+pzPp' 32
will be able to untangle to force White to take on he has no chance of 6p+N+-+p+&
himself and proceed to e6 by recapturing with saving the endgame. ] 5+pwqN+-+-%
exploit the extra piece. the knight. )] 29.d5 This double threat 4-+-+-+-+$
But Black can continue his 22...Nxb6 Now the game decides the game. Qxe8 3+-+-+QzPP#
domination of the knight faded to a draw. But why 30.Rxe8+ Rxe8 31.d6 2P+-+-zPK+"
with the amazing: Rb4!! don't you reverse the Rd8 32.Qf6 Rd7 33.a4 1+-+R+-+-!
The knight has nowhere to move order? Be6 34.Qd4 Rb7 35.h3 xabcdefghy
go, and the pin is still very [ 22...Nb2!! was Esben's a5 36.Kh2 h5 37.Qe5 Qe7 27.Qxa6 Qe4+
much a reality, so the improvement. By Rd7 38.Kg3 Kh7 39.f3 28.Kg1 Qf3 29.Rf1 Qf5
outcome would probably reversing the move Bb3 40.Qb5 Be6 41.Qe5 , I would never have
be a draw by repetition of order Black gets his Bb3 42.Qxa5 Rxd6 been able to lose the
moves: 37.Nc8 Rb7 desired position with the 43.Qb4 Rd3 44.Qb5 h4+ game. Instead I swam
38.Nd6 Rb4 etc. advantage after 23.Bxe6 45.Kh2 Re3 46.a5 Be6 like an axe, as the
axb6 . ] 47.a6 Ra3 48.Qb7 Rb3 Russians say. ]
23.Bxe6 Nb2 24.Rfc1 49.Qe7 Rb5 50.a7 Ra5 25.Nf6+ Kxg7 26.Qxc6!
Timman,J Bd5 25.Bxf7+ Bxf7 51.Qxh4+ This simple move is easy
Unzicker,W 26.Nd4 N2a4 27.h3 a6 1-0 to overlook. When you
Haifa Olympiad 1976 28.bxa6 Rxa6 29.Rab1 can take a rook with
[Jacob Aagaard] Ra7 30.Rb4 Kh7 31.Nb5 check why would you want
Ra5 32.Rc7 Be8 33.Nd6 Leko,P to exchange queens?!
XABCDEFGHY Bg6 34.Rc6 Nd7 35.Rc7 Kasparov,G Rxc6 27.Nxe8+ Kf8
8r+-+-+k+({ Nab6 36.Nc4 Nxc4 Bled Olympiad 2002 28.Nd6 Rc2 29.a4 bxa4
7zpl+n+rzp-' 29 37.Rbxc4 Nf6 38.Rc3 [Jacob Aagaard] 30.Ra1 Ke7 31.Ne4 Rc4
6-wq-+p+-zp& Be8 39.Rb3 h5 40.Rbb7 32.Kf3 f5 33.Nd2 Rc3+
5+P+n+-+-% Bd7 41.Ra7 Rd5 42.a4 XABCDEFGHY 34.Ke2 a3 35.Ra2
4-+L+-+-+$ Rd1+ 43.Kh2 Rd2 44.f3 8-+-+-+-+( 1-0
3zP-vL-+-+-# Ra2 45.h4 Kg8 46.a5 7+-+-+-+-' 31
2-+-+NzPPzP" Kf8 47.Rc5 Kf7 48.Re5 6-+P+k+N+&
1tR-+Q+RmK-! Kf8 49.Kg3 Be8 50.Ra8 5+-+-+-vl-% Blundell,D
xabcdefghy Kf7 51.Ra7+ Kf8 52.Rb7 4-+-+-zp-+$ Diagrammes 1994
Black has a combination, Nd7 53.Rg5 g6 54.Rb4 3zp-+-+-+-# [Jacob Aagaard]
but its consequences are Bf7 55.Kf4 Nf6 56.Rd4 2P+-+K+-+"
not completely clear. Ke7 57.g3 Nd7 58.Rb5 1+-+-+-+-![ XABCDEFGHY
20...Ne3! 21.Bd4! Nf8 59.Re4+ Ne6+ xabcdefghy 8-+-+-+-+(
Candidates! Did you 60.Ke3 Ra3+ 61.Kf2 With little time left on his 7+-+-+-+-' 33
anticipate this response? Ra2+ 62.Ke3 Ra3+ clock Peter Leko failed in 6-+-+-+-+&
[ If instead 21.fxe3 63.Ke2 Ra2+ 64.Kd3 slaying the giant 5+-+-zp-+k%
Qxe3+ 22.Kh1 Qe4! Kd6 65.Rc4 Be8 (physically Kasparov is 4-+-+Pzp-+$
23.Rg1 Qxc4 66.Rb6+ Kd5 67.Rc8 small of course, but 3+-+-+-+-#
and the pressure down Bd7 68.Rg8 mentally...!). The game 2-+N+-+-+"
the long diagonal is ½-½ concluded: 53.Ne5?? 1+K+-+-+-![
very uncomfortable for [ The line 53.c7 Kd7 xabcdefghy
White, besides which 54.Nf8+ only leads to a 1.Na1! The white king
there is the small Hansen,LB draw after Kc8! . ] needs the c4-square in
problem of the pawn. I Campora,D [ But reversing the order to win the game,
think it is dubious Biel 1991 moves prevents this [ as can be seen in the
whether or not White can [Jacob Aagaard] defensive manoeuvre analysis of the
keep his position and White wins by alternative: 1.Na3? f3
together. ] XABCDEFGHY 53.Nf8+! Kd6 54.c7 2.Nc4 Kh4!
[ But notice how this little 8-+r+-+-+( Kxc7 55.Ne6+ ( Black needs only to
line is again a matter of 7zp-+-+pmk-' 30 and the bishop is lost. ] avoid the mined square
candidates, e.g. 21.fxe3 6-+q+-sNpzp& 53...Bd8 54.Nc4 Kd5 . g4 in order to make the
Qxe3+ 22.Kh1 Qh3? 5+-+-+-+-% Later Leko was not too draw; 2...Kg5!? 3.Kc2
23.Nf4! (Jonathan Faydi) 4-+lzP-+-+$ bothered about missing Kg4 4.Kd3 f2 5.Nd2
Rxf4 24.Bxe6+! 3+-+-+-+-# the win, as he felt that it Kf4 is also sufficient,
and White is better. So 2P+-+-zPPzP" was just luck that he had transposing below )
once more, by 1wQ-+-tR-mK-![ the chance in the first A) 3.Kc1 Kg3 4.Kd2
calculating more slowly xabcdefghy place. f2 5.Ne3 Kf3 6.Kd3
and seeing 22...Qe4 28.Ne8+! ½-½ Kg3 and White cannot
we actually save time! ] [ Here the natural win.;
21...Nxd1 22.Bxb6 continuation is 28.d5 B) 3.Kc2 Kg4 4.Kd3
[ Here Black would like to followed by 29 Ne8+ Tetenkina,I Kf4 5.Nd2 f2 6.Ke2
play 22.Bxb6 axb6 winning, but Black can Aagaard,J f1Q+ 7.Kxf1 Ke3
. Unfortunately White defend against this with Stockholm 2003 and White cannot
would simply respond Qxf6 and wins. So White [Jacob Aagaard] win.;
23.Rfxd1 ( rather than decides to change the C) 3.Kb2 Kh3 4.Kc3
23.Bxe6 Nb2 move order. ] 24...Qxc6?? ( or 4.Kb3 Kg2 5.Kc2
and Black ends up with 28...Kg8 [ Had I played 24...Rxc6 f2 6.Ne3+ Kf3 7.Kd3
two pieces for a rook [ 28...Rxe8 29.d5+ 25.Nf6+ Rxf6 26.Qxf6 Kg3 and White cannot
Excelling at Chess Calculation− Jacob Aagard 19

win ) 4...Kg3! Karpov,A pawn, and keeping the XABCDEFGHY


and White is in Kasparov,G position afloat for now. ] 8-+-+-+rmk(
zugzwang. Had the Skelleftea 1989 31.Bh4 f3+! 32.Bxf3 7zP-+-+-+-' 36
knight been on b3 here, [Jacob Aagaard] Nxf3 33.Bxf6+ Qxf6 6-+-+-mKP+&
then 5 Kc4! would win 34.Rxh8 Kxh8 35.Kg3 5+-+-+-zP-%
without any resistance, XABCDEFGHY g5? 4-+-+-+-+$
as can be seen after 1 8-+lwq-tr-+( [ As Kasparov showed 3+-+-+-+-#
Na1.; 7+pzp-+-mk-' 35 later, 35...Kg8! 36.Qe3 2-+-+-+-+"
D) , and now: 3.-- ] 6-+-zp-vlp+& Nd4 was the correct 1+-+-+-+-![
1...f3 2.Nb3 Kg4 3.Kc2 5+-+Pzp-snp% winning sequence, when xabcdefghy
Kg3 4-zPP+PzpP+$ White has: 1.Kf7 Rg7+ ]
[ If 3...Kf4 4.Kd3 Kg3 3+NsN-+P+R# A) 37.g5 Qh8!! [ 1.Ke6 Kg7 ]
5.Ke3 Kg2 6.Nd2 f2 2-+-+L+K+" and Black wins [ 1.a8Q Rxa8 2.g7+
7.Ke2 Kg3 8.Kf1 wins. ] 1+-wQ-vL-+-![ (Kasparov).; Kg8! and in all cases
4.Kc3 Kg4 5.Kc4 Kg3 xabcdefghy B) 37.Ne2 Nxe2+ Black wins without a fight.
6.Kd5 Kf4 7.Nd2 f2 29.Rh2! 38.Qxe2 Qf4+ 1 g7+ at least needs a
8.Nf1 and White wins. [ No matter which White and Black has a very few moves of
1-0 plays, Black will consider serious advantage. calculation. ]
29.-- hxg4 30.fxg4 f3+ One idea is 39.Kh4 1...Kh7 2.g6+
31.Bxf3 Nxf3 32.Qh6+ Qh6+! 40.Kg3 g5! [ Same story. If 2.a8Q
Ponomariov,R Kf7 33.Kxf3 ( 33.Qh7+ and Black will win the Rxa8 3.g6+ Kg8!
Al Modiahki,M Bg7 makes no g-pawn and the and Black wins. This
FIDE World Ch., Las Vegas (analysis) difference ) 33...Rh8 game.; could be eliminated
[Jacob Aagaard] . When you see this plan C) 37.Nd2!? simply because it gives
it should be possible to was suggested by the opponent more
XABCDEFGHY find the difference someone at opportunities. ]
8-+l+-+k+({ between the two white ChessBase as OK for 2...Kh6 3.a8Q Again the
7+pvl-+-+p' 34 rook moves and avoid White, but this is not only move. Rxa8 4.Kf7
6-+n+p+-+& losing instantly – the case: Nc2 38.Qf3 And again. Ra7+
5zP-+-+-+-% White's primary ambition Qg5! ( an improvement You should have made it
4-+-+P+ptr$ here. ] from Fritz on this far just by eliminating
3+-zP-+-zp-# [ If 29.Rh1? hxg4 38...Qxf3+ 39.Nxf3 the alternatives. 5.Kg8!
2-+Q+-+LzP" 30.fxg4 ( 30.Bh4 gxf3+ Nxb4 ) 39.Nf1 Ne1 This idea needs to be seen.
1+-+-+R+K! 31.Bxf3 Nxf3 32.Bxf6+ 40.Qe2 Qf4+ 41.Kh3 But as you could easily
xabcdefghy Qxf6 33.Kxf3 g5 Nf3 42.Qg2 Ng5+ calculate this far without
31...Bb8! The right square, gives Black a strong 43.Kh4 Nh7 44.Ne2 any chance to find any
as the following line attack according to Qg5+ and Black wins a alternatives on the way,
shows: 32.Rd1 Ba7 Kasparov ) 30...f3+! pawn and presumably this is no longer so hard to
33.h3 Forced. gxh3 31.Bxf3 Nxf3 32.Qh6+ also the game.; find. And here again
34.Qd3 Kf7 33.Kxf3 Rh8! D) 37.-- ] elimination would be
[ If 34.Bf3 Ne5 35.Rd8+ skewering queen and 36.Ne2 Kg7 sufficient to solve the
Kg7 and White has no rook and winning the [ Not 36...Kh7?? 37.Qf1! problem, even if you used
defence against ...Nxf3 game. ] and White wins. ] elimination from all legal
followed by ...g2+. ] 29...hxg4 30.fxg4 Rh8 37.Qh1 moves. Rxg7+
34...hxg2+ 35.Kxg2 Rh2+ [ Now the line is less [ Now 37.Qf1 [ 5...Kxg6 6.Kh8
36.Kxg3 Bb8+! appealing: 30...f3+ is answered by Bxg4! is little different. ]
Had the bishop been at c5, 31.Bxf3 Nxf3 32.Qh6+ 38.Qh1 Nh2! 6.Kh8 Ra7 7.g7 Rxg7
there would be no check Kf7 33.Kxf3 Rh8 and Black wins stalemate.
on d6. These kind of 34.Qd2! is the subtle (Kasparov). ] ½-½
things cannot be predicted difference. With the rook 37...Nd4?
easily, but can be on h2 White can defend [ 37...Nh4! would still
understood using the it with this retreat. Now have kept White under Afek #3
method of comparison he quite possibly stands pressure, according to Schach-Herold 1983
simultaneously with better, e.g. Bg5 35.Qg2 Kasparov. ] [Jacob Aagaard]
accurate calculation. Now Qf6+ 36.Ke2 Rxh2 38.Nbxd4 exd4 39.Qd1!
Black wins after: 37.e5 37.Qxh2 Bxg4+ 38.Kd3 Qe5+ 40.Kf3 Qf6+ XABCDEFGHY
Bxe5+ 38.Kg4 Bxc3! Qf1+ 39.Kc2 Qxe1 41.Kg3 Qe5+ 42.Kf3 8-+-+-+N+(
And White has no defence; 40.Qh7+ and the best Qf6+ 7+-+-mK-mk-' 37
e.g. 39.Qxc3 e5+ 40.Kg5 Black can hope to ½-½ 6-+-+-+-+&
Kg7! 41.Rd6 h6+ achieve is the perpetual 5+-+-+Pzp-%
42.Rxh6 Rxh6 check after Kf8 41.Qxc7 4-+-+-+P+$
and Black should win the Qf2+ 42.Kb1 Qf1+ Afek #2 3+-+-+-zPr#
endgame without too 43.Kb2 Qf2+ , though Pfeiter JT 1981 2-+-+-+-+"
much trouble. White can still go for a [Jacob Aagaard] 1+-+-+-+-![
1-0 full point with 44.Ka3!? xabcdefghy
Qa7+ 45.Na4! 1.f6+ No other moves
– here we see why the (Diagram 36) make much sense,
king needed to go to f8 [ although 1.Nf6
and not f6; an act of 1.g7+ should be calculated:
comparison again – [ All other legal moves Rh2 2.Ne8+ Kh6 3.f6
Black's only move is can quickly be Re2+ 4.Kf8 Kh7 5.f7
Be7! defending the d- disregarded: Re3 and there is no way
Excelling at Chess Calculation− Jacob Aagard 20

for White to make Rxe3 ( 31...Kg8 XABCDEFGHY [ However, in the game


progress. If he ever gets looks better for White, 8r+-mklvl-tr( Black was oblivious to
his knight to e7 and king but is not completely 7zpp+-+p+p' 39 White's main threat:
to e8, then ...Kg7 will clear, so we remember 6-wq-+-+-+& 23...h6?? 24.Qd7+!
easily stop all the fun. ] this for later ) 5+-+psN-zP-% 1-0 ]
1...Kg6 2.f7 Again White , when White needs to 4-+-zp-+-+$ 1-0
does not have a serious find 32.b4! (and so does 3wQ-+L+-zP-#
alternative. Black of course) and 2PzP-+-mK-zP"
[ If 2.Ke6 Rh1 3.Ne7+ now after Qb6 33.Qxb6 1+-tR-tR-+-![ Alexander
Kh7 4.Kf5!? ( or 4.f7 Rxb6 34.b5 White has xabcdefghy Marshall
Kg7 5.Nf5+ Kf8 the advantage. ] [ If 23...Bxa4? 24.Nxf7+ Cambridge 1928
and White can never [ 28...Bd7 is met strongly Kd7 25.Bf5+ wins. [Jacob Aagaard]
hope to win ) 4...Kh6! by 29.f4! when Black Instead Black has a
and Black makes a draw. has only Rxe3 large number of moves. XABCDEFGHY
So this line can be 30.Qxg5+ Qxg5 31.fxg5 Many of them can be 8r+lsn-trk+(
eliminated. ] Kg6 (otherwise Nf5+) rejected immediately 7+-+-+p+p' 40
2...Rh7 3.Nh6 Again the 32.Bxb7 Bg4 33.Rd2 because they do not 6-wqp+-zp-wQ&
only move that makes any Kxg5 34.Bg2 pay attention to White's 5zppvl-zpP+-%
sense, and since it is not main threat (supposing 4-+-zpP+-sN$
[ as 3.Ke6 Rxf7 4.Ne7+ possible to find Black sees what that is!), 3zP-sNP+-zP-#
Kg7 5.Nf5+ Kg6 something clear for but there are some that 2LzPP+K+-zP"
is only a draw. ] Black here, probably need to be calculated. ] 1tR-+-+R+-![
3...Kxh6 4.Ke8 White simply has three [ 23...Qxb2+? 24.Rc2! xabcdefghy
Again no alternatives. pawns for the exchange Bxa4 ( 24...Qxc2+ 1.--
Rh8+ and hence excellent 25.Qxc2! ) 25.Nxf7+ Kd7 [ The natural first try is
[ If 4...Kg6 5.f8N+! chances of winning the 26.Bf5# . ] 1.Rf4 exf4 2.gxf4
and White wins ( but not game. ] [ 23...Qe6? 24.Qb3 , but Black can defend
5.f8Q? Rh8!! [ 28...Rxe3 looks bad ( 24.Bb5!? himself with dxc3
with a draw: 6.Ke7 Rxf8 because of 29.Nf5+ is also strong ) 24...Rb8 . This line can now be
7.Kxf8 Kf6 . )] Qxf5 ( Black can quickly 25.Nc6+! and White is improved upon. ]
5.f8R! Once again a eliminate 29...Kf6 clearly better. ] [ One try is 1.Nd5
question of elimination. 30.Qh6+ Kxf5 31.Bh3+ [ But Black has two good , but after cxd5 2.Rf4
[ Not 5.f8Q+? Kg6! Ke5 32.Qxg5# ) 30.Rxd6 moves at hand. Both are exf4 3.gxf4 Kh8
and White cannot win. ] Re1+ 31.Kf2 . This does very complicated, so it the black queen
5...Rh7 not look too good for makes sense to try to suddenly protects f6. ]
[ If 5...Rxf8+ 6.Kxf8 Kg6 Black, but he can come refute them, when it will [ So the right move is:
7.Ke7 Kg7 8.Ke6 Kg6 up with Qc2+!! 32.Kxe1 quickly become apparent 1.Na4! and White wins
9.Ke5 and White wins Qc1+ 33.Kf2 ( 33.Rd1? which one offers Black because of bxa4 2.Rf4
the pawn ending. ] Qe3+ 34.Kf1 Bb5+ the best chances: exf4 3.gxf4 and Black
6.Rf6+ Forcing the winning and mates ) 33...Qc5+ 23...Bc5!? 24.Nxf7+! has no defence; e.g. Ne6
pawn ending. Kg7 7.Rf7+ and Black has perpetual (this combination is the 4.Rg1+ Ng5 5.Rxg5+
and White wins. check. While this is not reason for eliminating fxg5 6.f6 etc.
1-0 so easy to determine, it this line) Bxf7 25.Bf5 That White should also
is clear that of the Qc6! ( Black needs to win after the more
three this is the line return the piece; if modest 1 Nd1 or 1 Qxf6
Gurevich,M Black should go for. ] 25...d3+? 26.Kg2 Qxb2+ is not important. ]
Mikhalevski,V [ However, there is a 27.Kh1 b5 28.Qa5+ 1-0
Antwerp 1999 fourth line which could Bb6 29.Qa6 and Black
[Jacob Aagaard] quickly have been will have to part with
eliminated from the list his queen to prevent Naes,F
XABCDEFGHY of candidates. mate on c8 ) 26.Qxc6 Raetsky,A
8-+-+r+-+({ 28...Rh6? This loses in bxc6 27.Rxc5 Bg6 Taastrup 2002
7zpp+-+-mk-' 38 one move: 29.Qg4 28.g4 Rf8 29.Re5 Bxf5 [Jacob Aagaard]
6-+-tr-+-+& (the bishop on a4 is 30.gxf5 Kd7
5+-wq-+-zpQ% hanging, and White and the endgame is XABCDEFGHY
4l+-sN-+-+$ threatens 30 Nf5+ as roughly even. ] 8-+-+-+-+({
3+P+-zPPzP-# well) Bd7?! 30.Qxd7+ [ 23...Be7! forces White 7+-+-mk-+-' 41
2P+-+-+LzP" 1-0 ] to retreat with 24.Qd1 6-+-+p+-+&
1+-+R+-mK-! 1-0 , after which the position 5zp-sn-+p+-%
xabcdefghy is at least no worse for 4PzpLtR-+p+$
Black was in time trouble Black. ( It is important 3+P+-zP-zP-#
and lost the game Solak,D that 24.Nxf7+ Bxf7 2-+-+KzP-+"
immediately. What he Kozamernik,J 25.Bf5 no longer works 1+-+-+-tr-!
should have done, had he Ljubljana 2003 due to Be8! . This is the xabcdefghy
had the time, was to [Jacob Aagaard] major difference to 23... 38...--
perform an elimination of Bc5, and why 23...Be7 [ On 38...Ne4!?
the different possibilities. is stronger. We do not White has prepared a
Here Black could select (Diagram 39) need to see anything fortress with 39.Rxe4
from a long line of further to appreciate fxe4 40.Kd2 . The idea
possibilities: 28...-- 23.Qa4!? This queen that 23...Be7 is clearly is that Black cannot
[ 28...Rh8 29.Qg4 Bd7 sacrifice obviously cannot the better of the options easily get the king into
30.Qe4 Re8 31.Qxb7 be accepted. -- at hand. )] play, but if he is able to
Excelling at Chess Calculation− Jacob Aagard 21

sacrifice the exchange and he was unsure we cannot pick up on their Beliavsky,A
on c4, then he will have whether or not he would ideas, it is because we Brondum,E
a won pawn ending, be able to break it down. ourselves are left in the Copenhagen 2002
because of the extra Remember it is one thing dark. Black was surprised [Jacob Aagaard]
move ...e6-e5. White to work this out in by this move, which he
would therefore try to analysis, it is quite himself would have found XABCDEFGHY
get his bishop to d1 in another to do so over instantly, had he been 8r+q+l+k+(
order to secure the draw. the board. given the chance. 7+-+-+pvl-' 42
Black could try to We can argue that he [ Black seems to have 6-+p+-+p+&
calculate the lines to the should have eliminated prepared for something 5+pzPp+-vL-%
end to see if it is 38...Rb1 quickly because like: 4-zP-zP-wQN+$
possible for White to of 39 Rd1 (which he 13.a3 Qb6 14.Na4 Qa7 3+-+-+-+-#
establish a fortress or obviously missed) and 15.Bc7?! ( 15.Rc1!? e5 2-+-+-zPP+"
not – and actually it isn't: instead played 38...Ne4, 16.Rxc6 bxc6 17.Bxe5 1+-+-tR-mK-![
Rb1 41.Kc2 ( if 41.Ke2 because then at least with compensation is xabcdefghy
Rc1! wins, because Bc4- there was a chance of possible ) 15...b5 16.Nb6 30.Re5!! By preventing
e2 is not possible ) winning (at no risk), Qxc7 17.Nxa8 Qb7 Black's defensive idea
41...Ra1 42.Kd2 ( if while after 38...Rb1 the with a clear advantage White's attack
42.Be2 Ra2+ 43.Kd1 game is quite obviously for Black. ] immediately becomes
Rb2 wins ) 42...Ra2+ going to be a draw. [ 13.e5 dxc3 14.bxc3 decisive. Bd7 There is
43.Ke1 Rb2 44.Kf1 Kf6 However, as Raetsky Qb6 15.exf6 nothing Black can do to
(the black king moves to himself pointed out after with unclear play. ] defend himself.
protect all the pawns the game, Black had a 13...exd5 [ After 30...Bxe5 31.dxe5
before the final much stronger move, [ If 13...Nxd5 14.exd5 Ra1+ 32.Kh2 f5
manoeuvring) 45.Ke1 which can probably only exd5 15.Bxd5 Be6 33.Nf6+ Kf7 34.Qh4
Kf5 46.Kf1 Rb1+ be found after seeing 16.Bxe6 fxe6 17.Qh5+ Qa8 35.Qh7+ Ke6
47.Kg2 ( if 47.Ke2 Rc1! the faults in 38...Rb1 g6 18.Qg4 and White 36.Nh5 the attack on the
wins ) 47...Rc1 48.Be2 and 38...Ne4; that is would be a little better. ] dark squares will be
Kg5 and White is now in 38...Rc1! which will win 14.exd5 Nd8? Black is decisive. ]
zugzwang. If he moves the game without too drifting. This is actually a 31.Nf6+ Kf8 32.Re1!
the bishop then 49...Rc3 much counterplay. Had form of forcing thinking: Black cannot answer the
wins more or less on the Raetsky looked for new because Black intuitively threat of Qd6 mate in a
spot, or if 49.Kh2 then ideas in the position wants to hang on to his satisfactory manner, and
Rc2 and it is all over. before making his move, extra piece he analyses therefore decided to call it
However, as Raetsky and not after the the wrong moves. a day.
correctly pointed out opponent's reply, he [ Instead after 14...Bg4 1-0
after the game, this is would have found this. 15.Qd3 Nxd5 16.Bxd5
too much calculation for On the other hand, he Be6 ( if 16...Rd8 17.Qe4
any human to do in the would not have played with an attack ) 17.Bxe6 test position 8
five minutes he had left 38...Rb1 had he not fxe6 18.Rxe6 0-0 Inside the Chess Mind 2004
on the clock. So to overlooked 39 Rd1, but 19.Rc1 White is only a [Jacob Aagaard]
avoid that he made a instead used more of little bit better. ]
simple move which he his precious remaining 15.d6 XABCDEFGHY
was still very upset minutes on finding the [ Even the weird 15.Qxd4 8rsnlwqkvl-tr({
with after the game: ] best move in the was possible: after 0-0 7zpp+-+pzpp' 43
[ 38...Rb1?? 39.Rd1! position. ] 16.d6 Bxd6 17.Bxd6 6-+p+-sn-+&
(because of this Black's ½-½ Qxd4 18.Nxd4 Re8 5+-+-zP-+-%
last move could have 19.Be7 White has a clear 4P+Lzp-+-sN$
been eliminated from advantage. ] 3+-sN-+-+-#
the calculation) Rb2+ A32 15...Ne6 16.dxe7 Qb6 2-zP-+-zPPzP"
40.Rd2 Rb1 41.Rd1 Raetsky,A [ If 16...Qxe7 17.Nxd4 1tR-vLQmK-+R!
Rb2+ 42.Rd2 Rxd2+ Jakobsen,O 0-0 18.Nxe6 Bxe6 xabcdefghy
43.Kxd2 Kd6 44.Ke2 Taastrup 2002 19.Bd6 wins ] 1...-- .
Ke5 45.Kd2 Kf6 46.Ke2 [Jacob Aagaard] [ or 16...Nxf4 17.gxf4 "The last moves must
Kg5 47.Kd2 Ne4+ Ng8 18.Nxd4 Nxe7? have been ...d4 and e5,
48.Ke1 e5 49.Bd5 Nc5 1.d4 Nf6 2.c4 c5 3.Nf3 19.Nc6! and White so let me see. White is
50.Ke2 f4 51.e4 Kf6 cxd4 4.Nxd4 a6 5.Nc3 wins. ] one pawn down. He has
52.Kd2 Ke7 53.Ke2 e6 6.g3 Qc7 7.Bg2 17.Rc1 three minor pieces in play.
Kd6 54.Kd2 Nd7 Qxc4 8.Bf4 Nc6 9.Nb3 [ 17.Be5!? Kxe7 Black has just one, the
55.Kd3 Nf6 56.Bf7 Kc5 Qb4 10.0-0 d5 11.Re1!? 18.Bxd4 also wins. ] knight on f6.
57.Bg6 fxg3 58.fxg3 Be7 12.e4 d4 17...Kxe7 18.Bc7 Qa7 What does Black want?
Ng8 59.Bf5 Nf6 60.Ke3 [ 12...dxe4 13.Bc7!? e3 19.Nxd4 Kf8 20.Nxe6+ There are some problems
Kd6 61.Kd3 Nh5 14.Rxe3 0-0 15.h3 Na7 Bxe6 21.Qd6+ Kg8 in the position. For
62.Bxg4 Nxg3 63.Bf3 16.Rc1 Nb5 17.a3 Nxc3 22.Rxe6 instance: If the d-file
Kc5 64.Ke3 Nf1+ 18.Rcxc3 with an attack 1-0 opens, then Bxf7+ could
65.Kd3 Ng3 66.Ke3 in G.Kasparov-V. be a problem at some
½-½ ] Kramnik, Moscow (blitz point. If the knight on f6
[ Raetsky's approach match) 2001. ] moves, then Nh4 is
was correct, but wrongly 13.Nd5! This was White's hanging.
executed. He eliminated intention. Usually when White is close to castling,
38...Ne4 because of the we think our opponents while Black needs one
possibility of the fortress, are playing bad moves, but more move. So it should
Excelling at Chess Calculation− Jacob Aagard 22

probably be one of Black's say that you play 2.0-0 is also interesting. Ne3 and tried to make the
objectives: to get the king and I play 0-0 3.exf6 Missed that one. ] most of it.
into safety. But let me first . Okay not very... ] [ 1...Be7 with the idea But let us turn to the
check the candidate [ 1...Bb4 2.0-0 dxc3 2.exf6 Bxf6 and both theme in focus here:
moves. Obviously both threatening to exchange knights are hanging. stepping-stones. In the
knights are hanging. queens. 3.Qxd8+ Kxd8 If you play 3.Nf3 then following there are a lot of
[ 1...dxc3 is one move. ] 4.exf6 . And then I can dxc3 . ] moves that are obviously
[ Another candidate is play... Hmm. I will just [ If 1...Be7 2.exf6 Bxf6 forced, which can be
1...Bb4 . ] go back again. ] 3.Qe2+ then Kf8 found quickly by going
[ One more is 1...Qa5 [ 1...Bb4 2.0-0 dxc3 you can always escape through the variations and
, but I am not sure if 3.Qxd8+ Kxd8 4.exf6 via g8 and h7. ] identifying the correct
this is any good. ] . I might play gxf6 [ If 1...Be7 2.Ne4 Nxe4 ideas at each stop.
[ 1...Qe7 seems a bit odd in this line. ( Or 4...Be6 .] Whether or not you tried
after 2.0-0 . Then dxc3 , but White is still [ If 1...Be7 2.0-0 then to analyse my move (16...
3.exf6 leaves White fully better in this line. )] 0-0 as well without Bg7), try now to follow the
developed. ] [ So what I would like to ruining your pawn chain. rest of the game in your
[ Let us start by looking at do is to play something Yes, I like that move. ] head and create clear
1...dxc3 . ] more active like 1...Ng4 [ Let me just check if it focus at the stepping-
[ There might be other , possibly. Okay, the does not fail tactically. stone positions. Bg7!?
candidates like 1...Qb6 knight is hanging on h4. 1...Be7 2.exf6 Bxf6 [ The easiest way to play
, but let us first look at So what do you do after 3.Ne4 Bxh4 then 4.Bf4 for Black was something
the normal moves. ] 1...Ng4? You save the is coming, maybe with like 16...Bc8 17.Nc3
[ 1...dxc3 2.Bxf7+ Kxf7 knight on h4, not by Nd6+, but you simply Bg7 18.Qe4 Bxc3+
3.Qxd8 cxb2 playing in to f5. You can play 0-0 and you are out 19.bxc3 Qd6
, threatening ...Bb4+. play 2.Qf3 of harm's way. Let me which would have won
Okay... ] , threatening... Because just check. ] the game without too
[ 1...dxc3 2.Bxf7+ Ke7 then Qxh4 [ 1...Be7 2.exf6 Bxf6 much difficulty, but the
3.exf6+ and something is not possible – 3.Qxf7+ . Another idea is to play text move is also very
nasty, I suppose. Okay. ] .] 3.Qf3 because f7 is strong. Black gives up
[ 1...dxc3 2.Bxf7+ Kxf7 [ 1...Ng4 2.Qf3 Qxh4 hanging, but you play a piece and in return
3.Qxd8 cxb2 – ( oh 3.Qxf7+ Kd8 . Okay, try 0-0 . Okay, 4.Ne4 Bxh4 wins the b2-pawn and,
3...Bb4 straight away ) again. ] 5.Qh5 . It is possible that I thought, the a1-rook. ]
. No, I think I will take on [ 1...Ng4 . Okay, if he it gives some initiative. 17.exd7+ Qxd7
b2. Take, take, take. If plays 2.Nf3 But I can always play... Time for the first stepping-
4.Bxb2 Bb4+ then or something, I will play Okay – time up. ] stone position. By fixing
5.Qd2 and Bxd2+ Bb4 and I castle, without [ 1...Be7 is my first this position in your mind
and it is ok. This is problems. ( Or maybe I choice ] you will be able to
probably not what you will play 2...Bc5 . )] [ and my second choice continue your calculation
want to do. White will [ 1...Ng4 2.Ne4 Bb4+ is 1...Ng4 ." from here in several
take on f6 and Black has , and you got a pretty .] directions, and return to
a horrible pawn structure, normal position. Black this position instead of the
so it is probably not good. is still a pawn up, so... ] original position with 16...
(JA: It is not clear to [ 1...Ng4 . The critical B03 Bg7. Sometimes,
me exactly what line Ivo line is maybe 2.Qf3 Lilja,R especially when you first
was looking at here. Qxh4 3.Qxf7+ Kd8 Aagaard,J start to use this technique,
Maybe 2 Qxd8+ – see and maybe 4.h3 ( or Copenhagen 1989 you will find it necessary
the following.) 4.e6; , or 4.Ne4 [Jacob Aagaard] to update the stepping-
So White has the . Black should be able stone position from time to
advantage in both 2 to hold this position. If 1.e4 Nf6 2.e5 Nd5 3.d4 time, because your
Bxf7+ and the 2 Qxd8+ the knight goes away d6 4.f4 dxe5 5.fxe5 Nc6 visualization of the
line. ] from c3 you got a check 6.Bb5 Bd7 7.Bxc6?! position is not very clear.
[ So do you want to play on b4. And ...Rf8 is Bxc6 8.c4? Here you have to look
1...Bb4 ? This is coming up. )] White's operation looked around in order to find a
interesting. At least [ So 1...Ng4 . ] promising: after the black response to Black's piece
there is no more threat [ Okay, so 1...dxc3 knight moves White is sacrifice. Only one move
of Bxf7+ if I take on c3. is not working. ] looking forward to 9 d5. gives White any chance of
So Black cannot escape [ 1...Bb4 gives a bad Unfortunately Black had resistance. 18.Qe4!
the pin. Now he can take position. ] an active defence. Ne3! Only a threat to the b7-
on f6, he can castle, he [ 1...Ng4 seems ok, very 9.Bxe3 Bxg2 10.Ne2 pawn gives White any
can play Qb3. If 2.Qb3 sharp, but you can Bxh1 11.Ng3 Bc6 12.d5 hope of not just being two
then even Nd5 3.Bxd5 probably survive the Bd7 13.e6 fxe6 14.Qh5+ exchanges down. Of
Bxc3+ and then both d5 attack. As White g6 15.Qe5 Rg8 16.dxe6 course 18 Qe4 would have
and h4 are hanging, and cannot castle directly, White's compensation for to be found, and being a
I can castle. So 2 Qb3 is you can always sacrifice the exchange is coming to move for the opponent,
not so dangerous. ] a piece back with ... an end. I could pretend this is not so easy. Bxb2!
[ What you probably do is Qxf2+ if you do not like it. that both players in this Without this Black's
1...Bb4 2.0-0 dxc3 Black can probably game were playing really combination starting with
3.exf6 0-0 and then you survive this position. deep chess, and that 16...Bg7!? does not make
cannot... Well, you can Are there other White had overestimated any sense. So this is not
play 4.fxg7 , well, it is a candidates that I have his compensation here. really a stepping-stone
bad position. ] missed? ] But that was not the case: position as such. To look
[ 1...Bb4 . Okay, let us [ 1...Be7 he simply overlooked 8... for a strong move on
Excelling at Chess Calculation− Jacob Aagard 23

every move does not in White tries to get his rook Now White can escape be able to look for
itself require a stepping- into play. the worst by means of a possibilities as easily as if
stone position to be [ The alternatives also small combination you were looking at a
visualized in our heads. needed to be checked to involving an exchange diagram, you'll have a
But once we begin to make sure they did not sacrifice. ] good idea about how
calculate variations they save White: 17.e5 dxe5 18.fxe5 Nd5 useful stepping-stones are.
are necessary. 19.Qxb7 27.Qe2 Re5! 28.Qxe3 19.Bxh7+ Kxh7 20.Rxd5 f5! The pawn ending is lost.
Rd8 Again the only moves Rxe3 and White cannot I had seen this position White cannot prevent the
that make sense from develop his queenside but failed to evaluate it h-pawn from queening.
both players. 20.Qxb2 without losing a piece, correctly. Bxd5 53.f4!? A nice try.
Qd1+ 21.Kf2 Rf8+ after which Black will [ Had I been in [ After 53.Kxe5 g4
22.Kg2 Qf3+ 23.Kh3 win easily with his reasonable form and 54.fxg4 fxg4 55.Kf4
[ Not 23.Kg1?? Rd1+ material advantage. ] able to fix the position in gxh3 56.Kf3 Kc7
and mates. ] [ 27.Qe4 Re5! 28.Qa8+ my head while searching White will soon run out
[ Here there would be no Kd7 29.Qh1 Rf8 for candidates here, I of moves. ]
need to calculate much and Black has a winning would have seen that 53...exf4 54.Ke5 g4
further: after 23.Kh3 attack. ] my intended 20...Bxg5 55.hxg4 h3! The pawn
Qxe3 Black regains his 27...Rd4 28.Qe2 is useless because of cannot be stopped.
piece with a very clear [ Not 28.Ra3 Qh6+ . ] 21.Qd3+ and White wins. [ White was pinning his
advantage. This could 28...Rh4+!! So I had to accept a last hopes on 55...fxg4
be enough to decide A nice sacrifice which position with opposite- 56.Kxf4 h3 57.Kg3 Kc7
upon this line rather drags the white king coloured bishops and 58.f3 and draws.
than 16...Bc8 – further into the open. roughly even chances, So in fact White could
subjectively it depends 29.Kxh4 where my grandmaster not improve his position
whether Black prefers to [ If 29.Kg2 Rf2+ 30.Qxf2 opponent outplayed me and should have been
keep the initiative or Rxh2+ 31.Kxh2 Qxf2+ in mutual time trouble. ] content with a draw.
defend with more 32.Kh3 Qb2 and Black 21.Bxe7 Re8 22.Bb4 Using stepping-stones
material. But objectively, wins decisive material. ] Kg8 23.Qe3 f6 24.Bd6 was one way of finding
it should be a matter of 29...Qh6+ 30.Nh5 Qc4 25.exf6 Nxf6 26.h3 that out. ]
which lines are most [ Or 30.Kg4 Rf4# . ] a5 27.g4 Rc8 28.Be5 0-1
reliable. As 16...Bc8! 30...Rxh5+ 31.Kg4 Rh4+ Nd7 29.Bd4 Qc7 30.Qg5
does not take much 32.Kg3 Qg5+ 33.Kf2 e5 31.Nc1 exd4
calculation, it should Rxh2+ 34.Ke1 Rxe2+ 32.Qxd5+ Kf8 33.Qf5+ D27
therefore be preferred. If [ More precise was Nf6 34.g5 d3 35.gxf6 Riazantsev,A
it was less clear, then 34...Qc1+ 35.Qd1 Qe3+ 1-0 Arbakov,V
this would be where we with mate next move. Voronezh 2003
would end our But Black won easily [Jacob Aagaard]
calculation of 16...Bg7. anyway. ] Brynell,S
Now, however, we have 35.Kxe2 Qe5+ Hellborg,T 1.d4 d5 2.c4 dxc4 3.Nf3
a new position for and White resigned after a Swedish League 2002 Nf6 4.e3 e6 5.Bxc4 c5
calculation, where we few more moves. [Jacob Aagaard] 6.0-0 a6 7.Bd3 Nc6
would not necessarily go 0-1 8.Nc3 cxd4 9.exd4 Be7
for the simple 23...Qxe3, XABCDEFGHY 10.Bg5 0-0 11.Rc1 b6
even though we had 8-+r+-+-+( 12.Re1 Bb7 13.Bb1 Rc8
previously ended our Hebden,M 7+-+nmkp+-' 45 14.a3 Re8 15.Qd3 g6
calculation with this Aagaard,J 6p+-+-+-+& 16.Bh6 Rc7 17.Ba2 Rd7
move. Now it is time for Port Erin 2003 5+-zP-zp-zp-% 18.Rcd1 b5 19.b4 Bf8
an improvement to the [Jacob Aagaard] 4N+K+-+-zp$ 20.Bg5 Be7 21.d5!?
variation. ] 3zP-+-+P+P# A natural reaction.
23...Rf5! The immediate XABCDEFGHY 2-+-+-zP-+" However, even in his
threat of ...Rh5 mate 8-+-+k+-tr({ 1+-+R+-+-![ annotations in Informator,
forces White to look for a 7+lwqnvlpzpp' 44 xabcdefghy Riazantsev does not give
defence. Again this 6p+-zppsn-+& 49.Rxd7+? Kxd7 an accurate picture of the
position could be a useful 5+p+-+-vL-% 50.Nb6+ Kc7 51.Nxc8 position.
stepping-stone. White 4-+-+PzP-+$ Kxc8 52.Kd5 All of this [ 21.Qe3! , with the plan
only has one chance. 3zPNzPL+-+-# was indeed forced. of 22 Qf4, was better, as
24.Qh8+ Kd7 25.Qd4+ 2-+P+Q+PzP" White's move was the only it is not easy to see
Kc8 26.Qg4 Qxe3 1+K+RtR-+-! one that made sense and how Black can improve
Black now regains one of xabcdefghy now it looks as if he is just his position.
his sacrificed pieces and 16...0-0? This was simply winning. After the text Black
White cannot defend. This a matter of being unable By taking this position as should start by selecting
would be the moment for to maintain the stepping- a stepping-stone, White his candidates. There
another stepping-stone in stone position in my head. would have been able to are two that
our calculation at move 23. [ 16...Nb6 was much see that Black has one, immediately spring to
White now has several stronger and would and only one idea that mind: the two different
possibilities and it would have prevented White's gives him counterplay. captures on d5, with
be necessary to calculate subsequent counterplay. Looking at the diagram the knight and with the
them all in order to After something like you should probably be pawn. But we should
choose between 23...Qxe3 17.Na5 Ba8 18.Qd2 0-0 able to see the also look outside the box
or 23...Rf5. Here we shall 19.e5 dxe5 20.fxe5 breakthrough pretty quickly. for extra opportunities,
only concentrate on the Nfd5 21.Bxe7 Qxe7 So if you imagine that with to see the position as it
game continuation. 27.a4 Black is close to winning. stepping-stones you would really is and not through
Excelling at Chess Calculation− Jacob Aagard 24

the mirror of our learn from our can improve his only see simple lines like
preconceptions. mistakes. (Then we position in a lot of lines. 27.Rxd7!? ( though I
Arbakov failed to do so would discover that, This is by no means quite like Riazantsev's
during the game, and although Black is easy, but as we shall 27.Bd5! with a clear
Riazantsev does not uncomfortable, 26... see later, this was advantage, based on
seem to have paid Bb7 is by no means forced sooner or later Bxd5? 28.Qc3
attention to these extra forced and Black does in order to save the and White wins )
possibilities in his not have to sink like a game. 27...Nxd7 ( 27...Bxf3
annotations. ] stone.); However, the game is 28.Rxh7# ) 28.Qxb7
21...Nxd5?! It is easy to C) 24...-- ] still not completely lost Nxf6 29.Qf3 to convince
understand why [ With 21...exd5 out of for Black.; ourselves that this is
Riazantsev believes this the picture, it could be D) 22.-- ] bad for Black. ]
to be the right move. But easy to analyse 22.Nxd5 Bxg5 24.Rxe8+ Qxe8 25.Bxd5
now, let us look at the 21...Nxd5 a little and [ 22...exd5 23.Bxd5 We have reached another
position from the eyes of a play it as soon as we A) 23...Bxg5 critical position. Black
Black defender. see that it is at least no 24.Rxe8+ Qxe8 needs to list his
[ There are a few lines worse (elimination). If 25.Nxg5 Ne5 26.Qh3 candidates and think
that needs to be no other options existed, transposes to the outside the box as usual;
calculated: then yes. But we should game.; but first, why not just
21...exd5? take a fresh look at the B) . On the funny analyse the most obvious
can be quickly position and decide if we 23...Nb8 all we need to move in the position? Ne5
disregarded as White are really forced to see is 24.Bxf7+! Kxf7 This forces White's reply.
has a standard reaction, enter these lines 25.Ne5+ and it is hard 26.Qh3
well known to both initiated by White. For to believe that Black [ 26.Qb3? Bxd5
players: 22.Rxe7! Qxe7 there is a pin down the will survive. We just 27.Rxd5 Nc4! 28.Rd1
23.Nxd5 Rxd5 24.Bxd5 d-file to the white queen, have to look a little at Qe2 29.Rf1 Qxf1+
. This is forced, and we and although this should Black's options, before and Black wins should
have the first stepping- not be overestimated, it quickly realizing that not be difficult to
stone. It is possible to is still an important this is much more calculate. But actually,
evaluate this position as feature of the position. ] difficult than the game after 27...Nc4! Black
dangerous for Black and [ In his annotations continuation. For does not need to see
simply look for Riazantsev did not example: Kg8 any further: it is
alternatives and save mention a move I 26.Nxd7 Bxg5 obvious that he will not
time. But we could also consider to be a serious 27.Qb3+ Kg7 28.Qc3+ be worse off. ]
investigate this a bit alternative. Black could and White wins in a 26...h5 Forced.
further... have avoided a lot of large number of ways, [ If 26...Rxd5 27.Qxh7+
A) . Analysis of problems if he had such as Kh6 29.Ne5! Kf8 28.Rxd5 Nf3+
unforcing options (such stepped out of the threat Qc8 30.Nf7+ Kh5 29.Nxf3 Bxd5 30.Qh8+
as 24...Rd8 25.Qb1 of dxe6 and exf7+ with 31.Qg7! with a winning Ke7 31.Qe5+
Na5 26.Re1 Qd6 21...Kg7! when White attack. ( 31.-- ); and White is a clear
27.Qa1 and White is has no easy way to C) 23...-- ] pawn up and on his way
much better) quickly continue the attack. 23.Nxg5 to win the endgame.
convinces us that A) 22.Bxf6+ Bxf6 [ 23.Nf6+?! Bxf6 But before heading into
Black needs to look 23.Ne4 does not seem 24.Qxd7 Qb8! this variation it was
at something more to give White anything. would give Black important that we
direct.; After exd5 24.Nc5 decent compensation for stopped and really
B) . However, 24...Ne5 Rxe1+ 25.Rxe1 Rc7 the exchange. We looked for the
is answered by 26.Nxb7 Rxb7 needed to see this when opponent's opportunities
25.Nxe5 Bxd5 26.Re1 Black is equal, as we chose the 22...Bxg5 here. If we had done that
and it is easy to see 27.Bxd5?? Rd7! move order. ] we would have seen that
that White has the would lose at least a 23...exd5 Another easy White has a strong
initiative. Riazantsev pawn.; decision. continuation. ]
gives the following line B) 22.Qe3 exd5! [ We should of course 27.Qxd7!! Not really a
to show the dangers of ( but not 22...Nxd5? look for alternatives, but queen sacrifice, but rather
the position for Black: 23.Nxd5 exd5 it is easy to eliminate flashy all the same. Qxd7
Bb7 27.Qf1 Kg7 24.Bh6+ Kg8 25.Ng5! them: 28.Bxf7+ Qxf7 29.Nxf7
28.Re3 Qe6 29.Qa1 when the threat of 26 23...Qxg5 24.Nf6+ Nxf7
Qf5 30.Bh6+! Kxh6 Nxf7! forces Black to and Black does not [ If 29...Kxf7? 30.f4
( if 30...Kg8 31.h3! give up the exchange have enough for the and White wins. ]
with the irresistible with Rf8 ) 23.Bxf6+ exchange. ] 30.Rd7 Bc8 31.Rc7 Bf5
threat of 32 Ng4! ) Bxf6 24.Qxe8 Bxc3 [ 23...Nb8!? (again we 32.Rc6 White was able to
31.Nxf7+ Kg7 32.Nd6 25.Re2 d4 and here quickly find that Black win this endgame
and White wins. We Black has fine should avoid this) comfortably. This could
would not need to see compensation for the 24.Nxf7! Kxf7 ( if and should have been
all this during the game exchange.; 24...Rxf7 25.Nf4! calculated accurately by
to get the impression C) 22.Qd2 exd5 and White has a great Black. Unfortunately
that Black is under will transpose in most structural advantage ) Arbakov is not really
pressure. Rather it is lines. 25.Qf3+ Kg8 ( or interested in calculation.
something we should To see all of this we 25...Kg7 26.Qc3+ Kf7 His main strength is
analyse afterwards to would first have to 27.Ne3 and White is natural talent and,
improve our feeling for think up 21...Kg7 and clearly better ) 26.Nf6+ therefore, rapid chess, in
the position and to see that Black indeed Kh8 , and now we need which he does not need to
Excelling at Chess Calculation− Jacob Aagard 25

display any discipline. In even see this extra Rxd5 33.Rxd5 Bxd5 when calculated
normal chess he has been resource; but this is a . But at least it is carefully enough, it
able to reach, on talent good day, so we see 27... difficult for White to should be apparent
alone, the level of a Nb8! and, hopefully, the get the job done. As it after two minutes that
normal grandmaster, but opponent's chances too) was Black lost without there is nothing to
not beyond that. Here 28.Qxh7+ Kf8 29.Qh6+! much of a fight really.; search for here.
talent was not enough and Qg7 ( unfortunately D) 26.-- ] Because it is such an
his opponent simply out- forced, as after 29...Kg8 1-0 obvious option, it could
calculated him. Kg7 30.Bxf7+ Rxf7 31.Rd8+ be tempting to return to
33.Rxa6 Bc2 34.f4 Nh6 leads to mate; , or if it again and again.
35.Kf2 Ng4+ 36.Kg3 Ne3 29...Ke7 30.Nf3! Rxd5 D32 However, while it might
37.Rb6 Bd3 38.Kf2 Nd5 31.Re1 wins ) 30.Qxg7+ Aagaard,J make sense to return
39.Rd6 Bc4 40.Kf3 Nc3 Kxg7 31.Bb3 Andersen,Ib to it once later, it is
41.Ke3 Nb1 42.Ra6 Bb3 and White is a pawn Copenhagen 1991 now time to move on
43.Kd3 Kf7 44.Kd4 Ba4 up. [Jacob Aagaard] and calculate the other
45.Ke5 Kg7 46.Ra7+ So 25...Kg7 is very two options.
Kh6 47.Kf6 Nxa3 48.Ra8 tempting, but simple 1.d4 d5 2.c4 e6 3.Nc3 Depending on your
Kh7 49.Kg5 Nc2 calculation will quickly c5 4.cxd5 cxd4?! style, personality and
50.Ra7+ Kg8 51.Kxg6 determine that this move This gambit is not good, taste, you will choose
Kf8 52.f5 Bb3 53.Ra8+ is no good. However, it even though it can at either the exchange
Ke7 54.f6+ Kd7 55.Rb8 would probably have times lead to interesting sacrifice or the more
Kc6 56.Rc8+ Kd6 helped us to get a new games. 5.Qa4+ Bd7 passive line first.;
57.Rc3 . idea, a move far out of 6.Qxd4 exd5 7.Qxd5 B) 15...Nxd4 16.exd4
So 25...Ne5?! was a the box which would Nc6 8.e3 Nf6 9.Qb3 Bc5 and then:
mistake. Probably this is have saved the game: ] 10.Nf3 Qe7 11.Be2 0-0-0 B1) 16...Bxd4?
the first thing we would [ 25...Kf8! . This is in fact 12.Bd2 Bf5 13.0-0 g5 fails to a brilliant
see in our calculation, the only move. Black 14.Rfd1 g4 15.Nd4! combination:
even before we listed all avoids the two This is what White was 17.Nb5!! ( not
the candidates, and this uncomfortable checks (at aiming at. By returning the 17.Qc4+? Qc5!
should not have taken f7 or c3) and all the pawn he opens up the and Black survives )
more than a few minutes. forcing lines which lead black king's position and 17...Qxe2 ( if
But as explained above, it to trouble. should be able to prove 17...Bc5 18.Qc4
is not likely that Arbakov A) . Of course White an advantage. Now it is wins ) 18.Nxd4 Rxd4
used even this amount of can take the h-pawn, time to list the candidates. 19.Qc3+ Qc4
time on his move. but this is not as As 15...Bd7 does not 20.Rdc1!!
At some point Black would tempting as it seems. make much sense, there and White wins, as
have to list the candidates. After 26.Nxh7+ Kg7 are only the three ways of after 20...Qxc3 21
. 27.Ng5 Ne5 capturing the knight to Bxc3! the skewer
(press F10 to continue) Black has basically consider. All need to be wins at least a rook.
1-0 played both 25...Ne5 calculated accurately Qxc3 21.Bxc3!
and 25...Kg7 (at the once this has been the skewer wins at
cost of a meaningless established. least a rook.;
analysis Riazantsev-Arbakov pawn) and White is in The most important thing B2) 16...Rxd4!
[Jacob Aagaard] trouble, e.g. 28.Qe4 to do in such a critical 17.Rac1 Kb8 ( if
( or 28.Qh3 Bxd5 moment is to organize the 17...Rhd8? 18.Be3
XABCDEFGHY 29.Qh7+ Kf6 30.Qh6 moves that need to be Rxd1+ 19.Nxd1!
8-+-+q+k+( Ke7 31.Qh4 f6 ) calculated in the right wins ) 18.Nb5 Rxd2
7+l+r+p+p' 46 28...f6 29.f4 fxg5 order. It is logical to take 19.Qg3+ Bd6
6p+n+-+p+& 30.fxe5 Qd8 the most obvious move 20.Nxd6 Rxd1+
5+p+p+-sN-% wins a piece.; first, since if this works, 21.Rxd1 Ka8
4-zP-+-+-+$ B) . Therefore White it might be possible to and Black at least
3zP-+Q+-+-# might play something eliminate the alternatives does not lose
2L+-+-zPPzP" careful like 26.Qb1 quickly, without instantly. By
1+-+R+-mK-![ which can be calculating them too calculating all three
xabcdefghy answered with Qe7 deeply. candidates in order,
25.Bxd5 -- and Black should be [ 15.Nd4 you would probably
[ 25...Nxb4? does not OK.; A) 15...Bxd4? 16.exd4 come to a conclusion
work because of 26.Qh3 C) . Actually the most Nxd4 is probably the – and that should be
h5 ( 26...Kg7 27.Qxh7+ dangerous move is most obvious line to that this is the best
Kf6 28.Qh6 ) 27.Bxf7+! ] 26.f4! with strong look at first. But this one possible. In the
[ 25...Kg7? is another threats. In analysis it can be eliminated from game Black went for
idea, the point of which might eventually work the candidates rather the exchange
is to avoid Bxf7+ by out that Black's quickly, as after sacrifice.;
stepping onto a dark position cannot be 17.Qc4+ Qc7 ( or B3) 16...--;
square. However, it saved after all; for 17...Nc6 18.Nb5! C) 15...-- ]
should be easy to spot a instance there might and Black lacks a good 15...Rxd4?! 16.exd4
strong continuation for be a win in the response ) 18.Qxc7+ Nxd4 17.Qc4 b5!?
White: 26.Qc3+ Qe5 ( if endgame after Qd8 Kxc7 19.Bf4+ Kc8 This was Black's idea
26...Ne5 27.Nxf7! 27.Nxh7+ Kg7 28.Ng5 20.Rxd4! Rxd4 21.Be5 behind the sacrifice. White
wins a pawn ) 27.Qh3! Nxb4 29.Qc3+ Qf6 and White wins now has no choice.
Nb8! (often we would 30.Qxf6+ Kxf6 material. This line is 18.Nxb5 Nxe2+
just cave in here and not 31.Ne4+ Ke7 32.axb4 not too difficult, and [ 18...Be6 19.Qd3 Bf5
Excelling at Chess Calculation− Jacob Aagard 26

does not force a draw, and then: ] Aagaard,J matter of character


though it might not be [ Actually I rejected Hofmann,M which way you want to
objectively weaker than 20.hxg3 because of Budapest 2003 win, i.e. by long
the text move. White Nxg3+ 21.fxg3 Ng4 [Jacob Aagaard] calculation or in a
has either to sacrifice which I believed was clearly winning endgame.
his queen with 20.Qxd4 winning for Black, but XABCDEFGHY Usually I would go for
(which is quite strong) 22.Nd4!! (found by 8-+r+-+k+( calculation, were the
( or find the combination Thomas Ernst) wins 7+-wq-+pzPp' 47 endgame not so easy. ]
20.Nxa7+ Qxa7 easily for White. This 6p+-+p+-+& 34...Rxb3+!? Desperation;
21.Qa6+ Qxa6 kind of defensive tactic 5+-sN-vllzP-% [ but after 34...Bxe4
22.Bxa6+ Kb8 23.Bf4+ later came in handy, as 4-zp-+-+-+$ 35.Qxf7+ Kh8 36.Qf6+
Ka7 24.Be5 and wins. )] can be seen below in my 3+-+-+-+-# Qxf6 37.gxf6
19.Kf1? Always, when game against Hofmann. ] 2PzPPtR-wQ-zP" the endgame is easily
moves seem to be too 20...Nxf4 21.Nd6+ 1+-mK-+R+-![ won for White. ]
obvious, we should pause. [ 21.Qxf4 Bg4 22.f3 xabcdefghy 35.cxb3 Qxb3+ 36.Ka1
Here White throws a Re8! gives Black the 29.Nd7! Bxb2+!? Qa4+ 37.Ra2 Qxe4
winning position away initiative. It does not [ Since 29...Bxg7 38.Re1
without even pausing for a take much calculation to 30.Nf6+ is nice for White, 1-0
second. dislike White's position. Black decides to go for
[ Instead after 19.Kh1! One line goes 23.Re1 the sacrifice, but White
Kb7 20.Re1 Black is gxh2 24.Qxh2 Be6 is fully prepared. ] Botvinnik,M
completely lost, whereas 25.Rxe6 fxe6 26.Rc1 30.Kxb2 Qc3+ 31.Kb1 Sorokin,N
now he has strong Kb7 27.b4 Ng4! 28.Qg3 b3 Black obviously thought USSR Championship, Moscow
counterplay. Equally, this Ne3+ 29.Ke2 Nf5 that this muddy position [Jacob Aagaard]
is the variation Black 30.Qe5 Bd6 31.Nxd6+ would give him a better
would have needed to Nxd6 and Black should chance than being an XABCDEFGHY
see in order to discard win eventually. ] exchange down with 8r+lwq-trk+(
15...Rxd4. ] 21...Kb8? Black misses limited counterplay. 7+p+nvlpzp-' 48
19...g3! Without this move his chance by thinking 32.Nf6+! An important in- 6p+-+p+-zp&
Black has nothing, but now inside the box. It appears between move, otherwise 5+-+-+-+n%
it becomes apparent that that no other moves exist, ...Bg4+ is possible. 4P+L+-+-vL$
the white king is far from but as so often before it is [ Not 32.axb3? Qxb3+ 3+-sN-zPN+-#
comfortable on f1. So the seemingly simple 33.Kc1 Qa3+ 2-zP-+QzPPzP"
what now for White? moves that are the with perpetual check, 1tR-+R+-mK-![
20.Bf4 It appears that both mistakes. since 34.Kd1 Bg4+ xabcdefghy
20 Bf4 and 20 hxg3 [ During the game I saw 35.Re2 Bxe2+ 36.Qxe2 15.Bxe7!
should objectively lead to no way to win against Rd8 gives Black a clear [ As explanation for his
a draw, but in the game 21...Qxd6!! 22.Rxd6 advantage. By decision he gave the
Black needs to prove it, gxh2 23.Qxc5+ Kb8 interposing the check on following line: 15.Ne5?!
which makes White's and, indeed, perpetual f6, White avoids all this. ] Bxh4 16.Qxh5 Qe7
choice clearly preferable. check is all there is. ] 32...Kxg7 33.axb3 Rb8!? 17.Nxd7 Bxd7 18.Rxd7
[ Instead 20.hxg3 22.Nxf5! I am quite pleased at Bxf2+! 19.Kxf2 Qxd7
is strongly met by Ng4! [ 22.Qxf4? having anticipated this 20.Rd1 and White is only
21.Qxe2 Qxe2+ looks dangerous, but idea. slightly better. I am
22.Kxe2 Black has Bg4! 23.f3 [ If instead 33...Qxb3+ sure that many a young
A) 22...Re8+ 23.Kf1? ( or if 23.Nf5+ Qc7! 34.Kc1 Qa2 35.Qd4! man would be really
Bd3+ 24.Kg1 Bxf2+ and all the white pieces ( 35.Kd1 Bxc2+ 36.Ke2 proud of himself if he
25.Kh1 Re5 are hanging – remember Qc4+ 37.Ke1 Bg6 had performed such a
and Black wins.; the threat of ...gxh2 ) 38.Qe3 also wins ) tactical operation,
B) 22...Re8+ 23.Kf3 23...Nh5!! 24.Qxg4 Qe3 35...Rxc2+ 36.Kd1 gaining bishop and
Ne5+ 24.Kf4 Bd7! 25.Ne4 Nf4 and Black and White wins. ] knight for rook. But the
25.a4 a6 26.Nc3 wins. ] 34.Ne4!? This move, fact is that, although
Ng6+ 27.Kg5 Re5+ 22...Qe4 23.Qxe4 Nxe4 which I found easily White is better, his
28.Kf6 Bd4 29.Kg7 24.hxg3 Now White can because I remembered advantage is not greater
Rh5+ 30.Kg8 Ne7+ relax. The resulting the analysis to Aagaard- than after the simple
31.Kf8 ( not 31.Kxf7?? endgame is better for him. Andersen above, is exchange. ]
Rf5+ 32.Kxe7 Bc5# ) Ne6 25.f3 Nf6 26.b4 probably the simplest. 15...Qxe7 16.Rd2!
31...Ng6+ Bb6 27.Rac1 h5 28.a4 [ The computer came up Black has no easy way of
with perpetual check.; a6 29.a5 with the much more freeing himself while
C) 22...Re8+ 23.Be3!? [ 29.Nd6! was even complex 34.Ka2 Rb5 White adds pressure. Nb6
Nxe3 24.fxe3 Rxe3+ stronger. ] 35.Nh5+ ( 35.Rd4 17.Rad1 Qc5 White was
25.Kd2 Rxg3 29...Ba7 30.Ne7 should also win, but I planning Rd2-d6.
gives Black excellent . Here Black lost on time. did not like this – it [ If instead 17...Bd7
compensation for the His position is worse, but seemed as if my king 18.Ne5 Nf6 19.a5 Nxc4
exchange. There is no not yet without some would be a little exposed, 20.Rxd7 Nxd7 21.Rxd7
obvious reason why he chances of survival. and the win therefore and this time White is
should lose the 1-0 less certain ) 35...Kg6 very close to winning. ]
coming endgame. Still, 36.Nf4+ Kxg5 37.Rg1+ [ 17...Qb4 18.Rd4!
this is the only place Bg4 38.Nxe6+ fxe6 would not have brought
White can seek for 39.h4+ Kh5 40.Qf7+ Black relief either. ]
an advantage.; Kh6 41.Qf4+ and White 18.Ba2 Nf6 19.e4 e5
D) 22...Re8+ wins. It is probably a 20.Qe3! A famous
Excelling at Chess Calculation− Jacob Aagard 27

decision. White now has a 23.Be5+ Kb6 Ne2+ 30.Kb3 g5 and then we discover
very large positional 24.Qxf7 Rc8 25.Ne6 31.Bxc5 Ng3 32.h7 that 32.Qxf7+ Kd8
advantage. Qxe3 21.fxe3 and the black Bxh7 33.Rxh7 Rxf1 33.Ne6+ Kc8 34.Kc1
Bg4 22.a5 Nc8 23.Rc1 position is beyond with even chances. ] does not give White
Bxf3 24.gxf3 Ne7 salvation.; 21.Bd3 Bxd3 22.Kxd3 anything after another
25.Nd5 Nc6?! 26.Nxf6+ B3) 20...Qg2 Rg8 23.g3 Rf8 24.f4 e5 subtle king move: Kb8!!
gxf6 27.Rd7 Rab8 21.Qh4 Kc8 25.fxe5 Rf2 26.Rxa1 preventing 35.Qa7+
28.Kf2! Nxa5 29.Rcc7 ( 21...Rxh8 22.Qxh8 Kd7?! , when it is not clear
Rbc8 30.Rxf7 Rxc7 Qxf2+ 23.Ne2 [ More natural perhaps that White should have
31.Rxc7+ Kh8 32.Bd5 does not look was 26...Rxb2 27.Rf1 any advantage.
b5 33.b3 Rd8 34.Kg3 f5 promising either ) Rxh2 28.e6 Kd8 29.Rf7 Actually 29...Qe7 could
35.Kh4 fxe4 36.fxe4 Rd6 22.Bf6 Rg8 23.Nf5 Rxa2 30.Rxb7 h5 easily be eliminated and
37.Kh5 Rf6 38.h3 Rd6 Bxf5 24.Bxf5+ Kb8 31.Ke4 ( not 31.Kd4 29...Qc3 found on the
39.h4 Rb6 40.Kg4 Rf6 25.b3 and White is Re2! ) 31...Ra5 32.Kd4 basic grounds that Black
41.Ra7 Rb6 42.Re7 Rd6 close to winning.; and White is also close has no other possibility
43.Rc7 Rf6 44.Ra7 Rb6 B4) 20...Qg1 to winning, though that does not stink. ]
45.Rc7 Rf6 46.Kh5 Rd6 21.Qh4 Black might have 30.Nc6 Qe6
47.Bf7! Rf6 48.Bg6 B4a) or 21...Bg4 chances for a draw, plus [ 30...Qb7 was better,
Nxb3 49.Kxh6 Rf8 22.f3 Re8 this line is not forced of but after 31.Qf3!
50.Rh7+ Kg8 51.Rg7+ 23.Qxg4+ course. ] (threatening 32 Nxe5)
Kh8 52.Bf7 Rxf7 53.Rxf7 ( 23.Be5!? 27.Rd1 White has a very
Kg8 54.Kg6 Nd2 55.Rd7 is also possible ) [ 27.Kc3 Rxh2 28.Rf1! strong ongoing attack. ]
1-0 23...Qxg4 24.fxg4 Ke6 29.Rf6+ Kxe5 31.Nxe5 Qxe5 32.Qa8+
Rxh8 25.Nf3 Rh3 30.Rf7 was also Ke7 33.Qb7+ Kd8
26.Be4 and White interesting. ] 34.Rxf7 Qe1+ 35.Kb2
Tunik,G has the 27...Rxh2 28.Kc3+ Qe5+ 36.c3
Shumiakina,T advantage; [ Also strong was 1-0
Pardubice 1994 B4b) 21...Kc8 28.Ke4+!? Ke7 29.b4
[Jacob Aagaard] 22.Bf6 Rg8 h5 ( if 29...Rxa2 30.Rh1
23.Ne2! and White probably Kosikov,A
XABCDEFGHY would be less wins ) 30.a3 with a very Bezman,V
8r+-+k+-tr({ clear. ( 23.Nxe6 large advantage. ] USSR 1986
7zpp+-snp+p' 49 fxe6 ); 28...Ke7 29.Rd4! Rg2 [Jacob Aagaard]
6-+pwQlvL-+& B5) 20...-- ] 30.Rg4 Ke6 31.Rg7
5+-+-+-+-% 16.Nxf5 Bxf5 17.f3 Qe6 Kxe5 32.Rxb7 Rxg3+ XABCDEFGHY
4-+PsNq+-+$ [ Not 17...Qc2+? 18.Ke1 33.Kb4 Rg2 34.Kc5 h5 8-+-+-sn-mk(
3+-+-+-+-# Qc1+ 19.Kf2 Qc2+ 35.a3 a5 36.b4 axb4 7+r+-+P+p' 51
2PzP-mK-zPPzP" 20.Kg1 and Black has no 37.axb4 Rg5 38.Rh7 6p+-zp-+-zP&
1sn-+-+L+R! defence against mate. ] Ke4+ 39.Kxc6 Kd4 40.c5 5zP-+-+-+-%
xabcdefghy 18.Qxe6+ fxe6 19.Bxh8 Kc4 41.Kb7 Kxb4 42.c6 4-+-+-+-+$
15...Nf5! This is the first 0-0-0+ 20.Kc3 All this was Rb5+ 43.Kc8 Ka5 44.c7 3+N+-+-+-#
move in a longer forced forced. It is tempting to 1-0 2-tR-+-+-+"
sequence. continue with the forcing 1+-+-mK-+-![
[ Black could also try the lines, but first you should xabcdefghy
very original idea of make sure that they are Hermansson,E 50.Rf2!? This was the
15...Ng8?! absolutely forced, and that Nolsoe,E suggested solution;
, but unfortunately it they are in your favour. Taastrup 2002 [ but in fact the strongest
does not work out after Here that is not the case. [Jacob Aagaard] continuation was 50.Rc2!
16.Bxh8 Rd8 17.Bd3! Rxh8? This leads to an Rxf7 51.Rc6 and Black
( not 17.Qe5 Qc2+ inferior endgame. XABCDEFGHY is still under some
18.Ke3 Qc1+ 19.Kf3 [ 20...Nc2! was much 8-+-+k+-tr({ pressure. Now Black
Qd1+ 20.Ke4 Qe1+ stronger, when White 7+-wq-+p+-' 50 should find White's idea
with perpetual check ) has problems with his 6p+-zp-+pzp& and defend himself
17...Qxg2 18.Qg3 Qxh1 development; e.g. 5+-+-vl-+-% against it, which is
A) 19.Nxe6 21.Be5 Rd1 22.Bf4 4-+-sNQ+P+$ actually not so difficult. ]
is again only a draw ( otherwise ...Ne3; here 3+P+-+-+-# 50...Rxb3? This is not the
because of Rxd3+!! 22.g4?! Bg6 23.Bg2 2P+P+-+-+" right way! Now White wins
( but not 19...fxe6? does not work because 1+K+-+R+-! easily.
20.Qxg8+ Kd7 of Ne1! with a strong xabcdefghy [ Instead 50...Re7+!
21.Qxh7+ Kc8 22.Be5! attack against the white 29...Qe7? Now Black is in 51.Kd2 Re6
with a decisive attack ) king ) 22...c5 when, in serious difficulties. was the correct
20.Kxd3 Qd1+ etc; my opinion, Black has [ Instead 29...Qc3!! manoeuvre. Now ...Rxh6
B) 19.Qxg8+ Kd7 sufficient compensation was the correct move. is threatened, so White
20.Qxh7 and now: for the exchange. This Probably Black did not seems to have no
B1) 20...Kc8 would be the starting see that after 30.Qa8+ choice: 52.Rg2 ( 52.Rh2
21.Nxe6 fxe6 point for the game. White Ke7 31.Qb7+ Ke8!! Rf6 is no improvement )
22.Be5 and White has many moves at his White cannot improve 52...Rg6 53.Rxg6 hxg6
has a decisive disposal here. One his position. It is easy to 54.Nd4 Kh7 55.Nb5
attack.; fantasy line goes 23.g4 ignore a move like 31... Kxh6 56.Nc7 Kg7
B2) 20...Bg4 Bg6 24.h4 Nd4 25.h5 Ke8, since White has a 57.Nxa6 Ne6
21.Bf5+ Bxf5 Bb1 26.Be3 Nxf3 27.g5 tempting check on f7, but and Black is at least no
22.Qxf5+ Kc7 Nd4 28.g6 hxg6 29.h6 we need to be concrete worse. ]
Excelling at Chess Calculation− Jacob Aagard 28

51.Rg2 Rb1+ 52.Kf2 proceeds unhindered play, and while the countered it. This made
Rb2+ 53.Kg1 Rxg2+ with his own plans. For correct evaluation him resign prematurely.
54.Kxg2 Now White is instance: 16.Nd2!? h6! might be a draw, [ Of course he cannot
winning. It was not too ( the only move; not White is in the driving play 60...Kg8 61.h6!
difficult for Black to see 16...Bd6?? 17.Nxe4 seat ) 19.Nxd7 Qxg5 gxh6 62.Rb7
this, so why did he play and wins; , while 16...e3 20.Nxb6 cxb6 and White wins ]
like this? The best guess I 17.Ne4 exf2+ 18.Qxf2 21.Qxb7 Re8 [ but the natural 60...Ra4!
can give is that Black Bb6 19.Qh4 Bc6 22.Qxb6 Qd2 23.Rf1 should have been fine.
went along with the forced 20.Bxe7 Qd5 21.Re1 Re2 24.Qxa6 Qxb2 After 61.Ra8+ Kf7
line because it seemed leaves White with a and the best White 62.a7 Ra1 Black was
natural. Instead he should clear advantage ) can hope for is a evidently afraid of 63.h6!
have been looking out for 17.Nxe4 and now: draw. Even the A) , with the idea of
his own interests. Ne6 A) 17...hxg5?! endgames do not 63...gxh6 64.Rh8!
55.Kf2 Nf8 56.Ke3 Ne6 (I dislike this as now it seem comfortable for and White wins.
57.Kd3 d5 58.Ke3 is Black who has to him.; ( 64.-- );
. White wins first the d- prove something) B5) 18.--; B) . But Black has a
pawn and then the a- 18.Nxc5! ( not 18.Nxg5 C) 17...-- ] counter idea, which
pawn. Qe8; , or 18.Qh5+ Kg8 16...Be6! Was this move a makes use of the fact
1-0 19.Nxg5 Qe8 20.Qh7+ shock for White? that all moves have
Kf8 21.dxc5 Qg6 Apparently so, since he disadvantages. Here
and Black is better ) completely lost his the h-pawn has given
Neukirch,D 18...Be8 ( Black does marbles and then the up control of an
Radulov,I not have time for game without a hint of important square, and
European Team Ch., Sinaia 18...b6? because of defence. 17.Qh5 Qd5 after 63...Kg6!
[Jacob Aagaard] 19.Nxd7 Qxd7 20.Re1 18.Nd2? Here White Black cannot lose.;
Re8 21.Re3 simply blunders and Black C) 63...-- ]
XABCDEFGHY and there is no wins a piece. Was this a 1-0
8r+-wq-+-mk({ defence, e.g. g4 matter of lacking training
7vlpzplsnQzpp' 52 22.h3! g3 23.Re4 in finding candidates?
6p+-+-+-+& wins ) 19.Qf8+ Ng8 [ Clearly better was C11
5+-+-+-vL-% 20.Ne6 Qf6 21.Qxf6 18.c4! Qf5 19.Qh4 Ng6 Nijboer,F
4-+-zPp+-+$ gxf6 22.Nxc7 Rc8 20.Qg3 Bxc4 21.Be3 Stellwagen,D
3+-zP-+-+-# 23.Nxe8 Rxe8 24.Kf1 Qe5 22.Qxe5 Nxe5 Groningen 2002
2PzP-+-zPPzP" with an unclear with a level endgame. ] [Jacob Aagaard]
1tRN+-+-mK-! endgame ahead. But if 18...h6 19.Nxe4 Bf7
xabcdefghy anyone is better here, 20.Qg4 1.e4 e6 2.d4 d5 3.Nc3
15...Bc5!! it would probably be [ Or 20.Qh4 Ng6 21.Qg4 Nf6 4.e5 Nfd7 5.f4 c5
[ Not 15...h6? 16.Bxe7 White.; Re8 and Black gets the 6.Nf3 Nc6 7.Be3 cxd4
Qe8 17.Qxe8+ Rxe8 B) 17...Bb6!! piece all the same. It 8.Nxd4 Bc5 9.Qd2 0-0
A) Notice that is a fantastic move, still should not be an 10.0-0-0 a6 11.Nb3!?
18.Bh4?! is less strong which would be easy to easy win, but clearly Be7
because of the tactical find had we not just White was not on top [ 11...b6 12.Kb1!?
trick e3 19.fxe3 Rxe3 played 15...Bc5. Why form this day. ] ( Fogarasi correctly
20.Kf2? ( White is try to sacrifice the 20...hxg5 21.Nxg5 Rd8 points out that after
forced to play 20.Nd2 bishop and then, after 22.Qh4+ Kg8 23.Qh7+ 12.Ne2 a5! Black has no
Re2 21.Rd1 Bf5 White develops his Kf8 24.Qh8+ Bg8 25.h4 problems ) 12...a5
22.Rf1 Rxd2 23.Rxf5 knight, retreat it again? Ng6 26.Qh5 Qxg2+! 13.Bb5! gave White a
Rxb2 with a draw. ) The short answer is 0-1 slightly better game in
20...Re4 21.Bg3 tactics. J.Aagaard-J.Kraai,
Rxd4! and Black has a B1) 18.Bxe7? Qe8 Budapest 2003. ]
clear advantage.; and Black is clearly Polugaevsky,L 12.Bd3 b5 13.Qe2 Nb4
B) 18.Bb4 and Black better.; Parma,B 14.Bd4 Nxd3+ 15.Rxd3
does not have B2) 18.Bxh6 Qf8 USSR vs. Yugoslavia 1965 a5!?
serious compensation 19.Qxf8+ Rxf8 [Jacob Aagaard] [ Black does not want to
for his pawn. The 20.Bg5 Re8 allow 15...b4 16.Na4
main problem is that and Black is better XABCDEFGHY when a battle for the
the a7-bishop is out of again.; 8-+-+-mk-+({ queenside dark
play, and White will B3) 18.Re1?! Qe8 7tR-+-+-zp-' 53 squares commences. ]
have time to ( 18...hxg5 19.Nf6 ) 6P+-+-+-+& 16.Rg3 a4 17.Nd2 b4
consolidate before it 19.Qxe7 hxg5 5+-+-+-+P% Contrary to Tibor, I find
is activated via b8 with 20.Nxg5 Qxe7 4-+-+-+r+$ this idea wildly inspiring
...c7-c6. ] 21.Rxe7 Re8 3+-+-mK-+-# and very deep indeed.
16.dxc5 with a clear edge for 2-+-+-+-+" 18.Qh5!! This is it. Can
[ When you make this Black.; 1+-+-+-+-! Black take the piece?
sort of silent sacrifice, it B4) 18.Nf6 Ng8! xabcdefghy [ 18.Nd1 with unclear
is not enough to have an ( simplest; if 18...Bf5 60...-- Here Black resigned, play was the obvious
idea for their acceptance. 19.Nh5! Qg8! having overlooked his alternative. ]
It is just as important to 20.Qxe7 hxg5 defence on move 63. The 18...Kh8!
have a defence against 21.Qxg5 Bh7 problem was that he saw [ I do not believe that
moves where the 22.Re1 and I do not a natural way to play the Black could have played
opponent ignores our like this for Black as position, but then saw an 18...bxc3? 19.Qh6 Bg5
generous offer, and his pieces are out of idea for White which 20.Rxg5 cxd2+ 21.Kd1
Excelling at Chess Calculation− Jacob Aagard 29

g6 (when Fogarasi says 24.Nxh4 fails to g5! C11 [ Stellan said that he had
that Black wins) since and Black wins Aagaard,J had this position once
White's attack is not over. ( but not 24...exf5 Brynell,S before, where he played
In fact Black has no 25.Nf3+ Kg6 Stockholm 2004 the preferable 15...Qc5
defence on the dark 26.Rg3+ Kh5 [Jacob Aagaard] leaving White slightly
squares after 22.h4! a3 27.Rh3+ better. Black has hardly
( or 22...Qe7 23.h5 Ra6 with a draw. ); 1.e4 e6 2.d4 d5 3.Nc3 any winning chances in
24.f5! exf5 25.e6 f6 B2) 22.Rh3 Nf8 Nf6 4.e5 Nfd7 5.f4 c5 the resulting endgame.
26.hxg6 and wins ) 23.h5 23.f6 gxf6 24.exf6 6.Nf3 Nc6 7.Be3 cxd4 Whether or not White's
axb2 24.Bxb2 Qe7 Bd6 25.Ng5 Rxg5 8.Nxd4 Bc5 9.Qd2 0-0 chances are that great
25.f5! exf5 26.hxg6 26.Qxg5 Ng6 wins.; 10.0-0-0 a6 11.Qf2!? is hard to say, but I
fxg6 27.e6 Rf6 B3) 22.Qxf7 Qf8! Kramnik's idea, which had some strategic ideas
28.Rxg6+ Rxg6 wins.; lately has given Black from the games O.
29.Qxg6+ Kf8 30.Rxh7 B4) 22.f6! gxf6 some headaches. Korneev-J.Kraai,
and White wins. 23.Qxf7 Bc5 [ I have previously played Zalakaros 2003, and A.
It is often the case, 24.Bxc5 Nxc5 the interesting 11.Nb3 Grischuk-A.Iljushin,
when calculating a line, 25.Rxg8+! ( not here – a move Russian Ch.,
that you see a defence 25.exf6 Ne4 26.Ne5 championed by Nijboer – Krasnoyarsk 2003, both
against your primary Nd2+ 27.Kxb2 but I thought Brynell of which White won, so
idea and then abandon it. Nc4+! 28.Nxc4 dxc4 might have prepared for at least I was not
But we should not do so 29.Rg7 Qd4+! this, so I chose another running blind. ]
since, quite often, the 30.Kc1 Qe3+ sideline that I felt 16.f5! Having being
defence against one line and Black wins ) comfortable with. ] allowed to play this I
is far from being the 25...Qxg8 26.Qxf6+ [ The reason is that the hesitated for a few
conclusion. It is similar and White has main line 11.h4 Nxd4 minutes to make sure that
to boxing when fending perpetual check. 12.Bxd4 b5 13.Rh3 b4 I was not rushing it. I have
off one punch often There is another 14.Na4 Bxd4 15.Qxd4 noticed that if you start
opens up for another. important lesson in a5 has been analysed to playing real moves quickly
The same happens in these variations: death and gives Black an (not calculating recaptures
chess: every move will just because you even game. I like the or already worked-out
leave at least one cannot play a general play in these moves) you can suddenly
square less well particular move at positions, but only when find yourself moving too
protected than it was one moment, does it is between the fast in positions where
before. Therefore we not mean that you players present at the you had a definite choice
should always look to should not consider it board. ] and, afterwards, would
see if new openings again on the next. 11...Nxd4 12.Bxd4 Qc7 have preferred to spend
appear in our When we involve 13.Bd3 Bxd4 14.Qxd4 some time.
opponent's defences. ourselves in concrete b5 The move itself is obvious.
Now let us look at the tactics, we should do [ 14...Qc5 has also been White has a strong attack
rest of the game. ] so with a truly open played. I am not sure and also a good structure.
19.f5 exf5? mind.; that White is better after It is important to notice
[ Here Black was forced B5) 22.-- 15.Ne2 b6 16.Rhe1 that White does not need
to take on c3 in order to and now White has , but it is a position it to play fxe6, but will focus
stay alive. The defence many tries, but only would be hard for White on f5-f6, threatening the
will cost him material. one that works: ] to lose, and Black has black king and keeping
19...bxc3! 20.Nf3! ( not 20.Nxd5 f6 21.Nf3 Nxe5 very few winning Black's pawns on light
20.f6? cxd2+ 21.Kd1 22.Nxe5 fxe5 23.Bxe5 chances. Part of my pre- squares
Nxf6! 22.exf6 Bxf6 Bf6 24.Rh3?! match tactics was to Stellan now thought for
and wins ) 20...cxb2+ [ Quicker was 24.Bxf6 allow for Brynell to about 50 minutes looking
21.Kb1 gxf6 25.Nf4 Rg8 stray from these puzzled as to what had
A) Black needs to 26.Qf7! intending 27 Rh3 passive positions, as a happened to him. He later
defend his king; if Rg7 28 Ng6+ mates, or if draw would do him no explained that he had
21...exf5 22.e6! Nf6 Rg7 27.Ng6+ good either. ] mixed up the theoretical
23.Qg5 g6 24.Rh3 immediately. ] 15.Rhe1! I was out of book variations, and that if ...
Rg8 25.Ne5 Rg7 24...h6 here, but this move, Bc8-b7 and Kc1-b1 had
26.exf7 f4 27.g4 [ If 24...Bg5+ 25.Bf4 h6 preparing both Re1-e3-h3 been included, then it
and White wins after ( or 25...Bxf4+ 26.Nxf4 and overprotecting the e5- would have been a known
fxg3 ( or 27...Bxg4 h6 27.Rd1 Qe8 pawn in order to play f4-f5, position in which Black
28.Qxg4 Bc5 29.Qxf4 28.Ng6+ Kg8 29.Nxf8 ) seems logical and correct should be OK. Now,
g5 30.Qe3 Bxd4 26.Qxg5 Qxd5 to me. however, he is very far
31.Qxd4 ) 28.Rxh7+ 27.Rxh6+ Kg8 28.Qh5 [ Also possible was from being OK. exf5
Rxh7 29.Nxg6+ Kg7 gxh6 29.Qg6+ Kh8 15.Qf2 in order to keep There is really no choice.
30.Nxe7+ Kxf7 30.Qxh6+ Kg8 31.Qg6+ the queens on, but then Black cannot allow f5-f6
31.Qxf6+ Ke8 Kh8 32.Re1 wins. ] Black has b4 16.Ne2 a5 with an attack;
32.Qc6+ Qd7 25.Nxf6 Rxf6 26.Rd1 followed by ...Bc8-a6 [ even after 16...Nxd3+
33.Qg6+ Rf7 34.Nxc8 Qf8 27.Bxf6 Qxf6 exchanging his problem 17.Rxd3 Qc4 18.f6!
Rxc8 35.Re1+ Kd8 28.Qe8+ Kh7 29.Rd8 b3 bishop. Though White I evaluated White's
36.Bb6+ Rc7 30.Qg8+ cannot prevent this position as winning.
37.Qg8+; [ Here 30.Rg3! mates. ] idea forever, it is useful The rooks will do a fine
B) 21...Rg8! 30...Kg6 31.Rg3+ Kh5 for him to postpone it for job attacking the black
B1) 22.Qxh7+ Kxh7 32.Qe8+ as long as possible. ] king and, as if this was
23.Rh3+ Bh4 1-0 15...Nc5? not bad enough, White
Excelling at Chess Calculation− Jacob Aagard 30

also has a wonderful (the only move) was that Ne7+ Kh8 21 Qxh7+ most natural move to
endgame coming with the endgame was Kxh7 22 Rh3 mate. But calculate first in this
knight vs. bad bishop. hopeless. Most likely I of course my kind of critical moment
These things are less would have counted on grandmaster opponent is the most forcing
clear with two set of my limited endgame would not fall for such move. And the most
rooks on the board, as technique to do the job a cheap trick and forcing move here is:;
attacks against the after 19.Qxc4 would come up with C) 19.-- ]
king still play a part in ( though I considered some sort of defence. 19.b4 The point of such a
the game, but here it answering with 19.Qf2!? Kh8 was the first move move is not difficult to
seems that White's , as the attack is still I thought of, and then figure out.
attack will be much very strong ) 19...bxc4 after 20.Nf6 h6! [ Black has a very limited
stronger. ] 20.Rd4 Be6 and then: saves the day, at least choice now, as 19.b4
[ My little German friend A) . I actually rejected for the time being. Qa4 ( and 19...Qd8
thinks that desperate 21.Nb6?! because of Nothing decisive 20.Nf6+ ) 20.Nb6
measures are necessary Rab8 ( instead comes to me both mean that the game
here, and that Black 21...Rad8! 22.Red1 immediately, so before is over immediately. So,
should consider 16...b4 Rde8! is a fine line by I continue to calculate out of his double attack
, but after 17.Qxb4 Rb8 Fritz ) 22.Nxc4 Rb4 these lines, I choose to (against e1 and a2),
18.Qh4 Qb7 19.f6 when White is a pawn look for other moves. Black is forced to
it begins to understand up but has four ( 20...-- ); execute the least
the enormous power of isolated pawns two of B) . One move I did not disturbing of the two,
two heavy pieces and a which are doubled, consider at the time after which he no longer
far advanced pawn although White is still was 19.Ree3!? has any threats, and
against a naked king. better of course.; as suggested after the White can continue with
For those interested in B) 21.Nc7! (I had not game by Stellan. his war effort
theoretical issues, it seen this at the time, Instead I quite quickly undisturbed.
should be said that a but it is not hard to find, found a very tempting Though this logic seems
pawn this far advanced so I am sure I would move, which I decided sound, it should be
in attack can more or have spotted it) Ra7 to investigate first. If remembered that White
less be considered as ( if 21...Rac8 22.Nxe6 it did not work out I has just sacrificed a
worth a piece. This and White will win ) would always be able pawn and exposed his
even works with the 22.Nxe6 fxe6 23.Rxc4 to return to this king madly. So although
famous three piece rule, and White has position again. 19 b4 was found by
which says that you excellent winning The point was that I strong intuition, it is
often need three pieces chances in the rook had only used 20 necessary to calculate
to conclude a mating endgame. I would minutes so far, and as the consequences of
attack (to begin it you place on rook on c6 I deemed it very such a move accurately
usually need an army), and the other on the d- unlikely that the game before we execute it. In
one to sacrifice itself to file. Black might dream would last until move this position that
open lines, one to of exchanging the e5- 40, I had no reason to means calculating it all
protect and one to pawn for the a-pawn, be careful with my time the way to the end. ]
mate. ] but even then the white here. The time we use 19...Qxa2 20.Nf6+
[ Incidentally, as some king would be closer in calculation should As a human it is very
commentators observed, to Black's passed be reasonable difficult to play moves (or
it is simpler to play pawns and should according to where we rather, it is very easy, but
16...b4 17.f6! gxf6 therefore decide the think the game is going. not so easy to do it well).
18.Bxh7+ Kxh7 game easily.; We need to be able to We need to play ideas in
19.Qh4+ with a mating C) 21.-- ] spend a lot of time at the shape of moves.
attack. ] [ Returning to 18...Qa5 the critical moments. White's last move was an
17.Nxd5 Nxd3+ – When you have such Because I had idea – it is easy to see
[ Stellan refrained from an enormous lead in somehow learned this that something forceful
17...Qa7 because of development and four over the years, I did has to follow the pawn
18.Qf4! . Black would pieces ready to attack not spend much time sacrifice and Nf6+ is the
then have to take on the enemy king (I include thinking about either only one. But in practice it
d3 anyway, or lose a the e5-pawn rather than 15 Rhe1 or 16 f5. As actually works the other
pawn without solving any the e1-rook in this these moves were way around. First you
of his positional number), you begin to possible why should I notice the idea of Nf6+
problems. The threat of look for something you not play them? with an attack, and then
Nf6+ with a mating can calculate all the way (Although I did stop for you try to make it work. I
attack is also still to the end. In fact White a minute or two at 18 had seen the variation 19
present. ] would like to find a Rxd3, just to make Nf6+ gxf6 20 Rg3+ Kh8 21
18.Rxd3 Qa5?! forced win now, and sure I did not overlook Qh4 Qxe1 mate (Black's
Stellan had probably logic encouraged me to some additional last move is his only
already accepted his think it was possible. possibility.) defence, but it is quite a
defeat here. It must have Obviously I have to pay Anyway, I had spotted defence!), so I intuitively
been strange for him to attention to 19...Qxe1+ such a tempting move started to improve this line
play this game, in which so that limits my here, first of all in my head, as well as
only two results made possibilities. because it was forcing, looking for other
sense, and draw was not A) . One move that that I decided to candidates, and the result
one of them! springs to mind forget about all other was 19 b4.
[ However, his reasons immediately is 19.Qh4 moves for the time [ Now after 20.Nf6+
for not playing 18...Qc4 with the idea of 20 being. Remember: The Black has a choice. Here
Excelling at Chess Calculation− Jacob Aagard 31

I had to calculate two Rg3-h3 with an checked the the winning idea for
moves: the obvious 20... unstoppable threat of difference between one move. ( 28.-- );
gxf6 and the other mate (e.g. 22.Qh4 Qe6 20...gxf6 and 20...Kh8 D) 27.-- ]
remaining legal move, 23.Rh3 with mate). So and noticed that they 27.Qh6
20...Kh8. I chose to start the rook would be are not important here [ 27.Rh3 was also
with the latter as I felt better placed on h3 in and that 20...gxf6 still possible. ]
that I would certainly most lines. Of course I appeared to be the 27...Bc4+
have a winning attack have to keep my eyes toughest defence.); [ On the evening after the
after the former. After open for the one case C) . There are no other game (which was kind
Kh8 21.Qh4 h6 when it is not, but in squares from which the of important to me, as I
I could not see a way this game there are queen can solve both was playing for a GM
to proceed. ] none. tasks, so I started norm), I wrote the
[ Here the method of Now Black has no other looking for other following in my personal
candidates comes in way to prevent mate moves. One that came notes: "Here something
handy. I used a little than delivering a few up was 25...Qxg3 very interesting
time and found another checks. ] , but never mind that happened. I had played
idea which did not seem 22...Qa1+ 23.Kd2 Rd8+ White must be better if the last nine moves
to offer Black any 24.Ke2 When calculating I just captured the relatively quickly, but
defence: 20.Nf6+ Kh8 the lines I actually queen, I took the time here I stopped to think. I
21.Rh3 h6 22.Qf4 believed that this was the to see that I could also thought both king moves
and after Qa1+ 23.Kd2 winning move and that I win immediately with won on the spot, but
Rd8+ 24.Ke2 would now either win the 26.Qxf6+! and the rook when I came home and
I will be allowed to play queen or mate my poor is picked up for free as inserted the game into
the winning 25 Rxh6+ opponent. But having all well. (Here I also my computer I realized
next move, if Black the time in the world I assured myself for the that 27...Bc4+ 28.Kf2??
doesn't resign on me started looking, simply sixth time or so that the loses to Qe2+
first. ] searching the board for c8-bishop cannot with mate in four moves.
[ But experience has possibilities. After some deliver any checks.) This experience was
taught me that when time I managed to see So eventually I came quite surreal and made
positions become muddy Black's next move, which up with the only move me consider if it is
like this, you should is rather obvious if you that seemed at all such with chess and luck,
always calculate very have the position playable for Black. that if you play enough,
accurately. Just physically in front of you, ( 26.-- ); then you will eventually
because Black plays but as I did not, finding the D) 25...-- ] have such large amounts
20.Nf6+ Kh8 does not move took me more time. 25...Rd1 26.Rxd1 of it in a random
mean he will never take Qxe5+ 25.Kf1 Of course I need to look tournament, that more or
the knight, so after [ I needed to check if I for other moves as well, less everything works?
21.Rh3 then gxf6 had a choice here, but I but as White is still Still, that would hardly
needs to be calculated simply noted that 25.Kf2 threatening a lot of things have been this
as well. I realized that I was possible (25 Re3 this natural move should tournament. Anyway, out
had 22.Qh4 with threats, does not make sense). I be the first to be seriously of sheer luck and a
but here my lack of did not see Rd2+ investigated. I did briefly sense of danger,
training in calculation ( or that Black can play check for alternatives, but consciously developed
started to set in. The 25...Qd4+ if the rook is could not find any. over the last few years,
position was no longer on h3, but simply that 25 Now White is threatening I played 28 Kg1."
clear in my mind's eye Kf2 was possible, and mate on the back rank, as Later I came to the
and the consequences of that if anything was well as 27 Qh6 and 27 conclusion that, because
Qa1+ were not wrong with the most Rh3. Black should be lost. of a strange experience
immediately obvious to cautious move, 25 Kf1, But as the material when I was 18 where I
me. So, for practical I could always return to situation was still unclear had a basic mate in one
reasons I noted this line this position. )] and I had plenty of time in a rapid game, but
and decided to compare [ This position after on my clock, I checked his instead tried to play
it with the main line, as 25.Kf1 then becomes possibilities. In the game another mate in one
the only difference is another stepping-stone he played: Be6 which surprisingly was
the position of the to me. I stopped here to [ I had also seen 26...Bb7 not there (how does a
white rook after... ] search the board A) , which could be 2350 player miscalculate
20...gxf6 21.Rg3+ Kh8 carefully. I looked for tried as the last resort mate in one?!), I had
22.Qh4 This is a matter of checks and defensive trick of 27.Rh3? learned my lesson and
comparison. Instead of moves., the first being Bxg2+!; have since relied on the
calculating the two long something that solved B) , but I had seen most simple move in
lines independently (which both of Black's problems, 27.Qh6 , which wins on situations where I did
is of course possible, the trouble facing his the spot.; not feel perfectly in
although time consuming), queen and king. Black C) . Later it turned out control. Though there
I can simply check the line needs both to defend f6 that 27.Rd4!? were 30 spectators to
after 20...gxf6 and shift and move the queen. also wins easily: the this game I did not
the rook between h3 and A) 25...Qd4 is met by main threat is 28 seriously consider
g3 whenever it seems 26.Qxd4! Rxd4 Qxh7+ and 29 Rh4 showing off or anything
there is a difference 27.Re8#; mate, while after the like that. The problem
between the two. B) , and 25...Qd6 only defence Be4 then was that, this being the
[ I quickly came to the is met by 26.Rh3! 28.Qh6 wins again. But game for my first GM-
conclusion that in some and the black king is of course this is norm, I was
variations I would play mated. (Here again I nothing but delaying enormously excited and
Excelling at Chess Calculation− Jacob Aagard 32

had been from the choice according to there are some look for ideas. ]
moment I had finished ECO. ] arguments against 19...Bxe1? My opponent
calculating 19 b4. ] 15...exd4 16.Nxd4 Rfe8 this: firstly, that my used only 10 minutes on
28.Kg1 After I made this 17.Ng3 d5 Here I sank opponent might think in deciding take the rook. I
move Stellan offered his into thought. this position as well, was secretly hoping for
hand in defeat and [ 17...d5 and might be this move, which I
congratulation at the A) . When playing 17 considering other considered completely
same time. Ng3 a bit too quickly, I moves than 18 e5, worthless. He had played
I had seen the rest of the had overlooked a very while he has no his first 18 moves instantly
game when I played 19 b4, simply point: that after alternatives at all to 18 and should now have
as well as having checked 18.e5 Black would not e5 Bb4; secondly, it is fallen into deep thought.
it again once I had have to play the pawn- a safety mechanism. I [ I had investigated the
finished it. I had the time, losing Ne4 had decided on mainly move 19...Rcd8
so why not? The game ( but would instead general grounds that when White has two
would be over if I was right. reply with the vicious the solution to my ways to proceed:
And if I was wrong it 18...Bb4! winning my e- problem would start A) 20.Bg5 Rxe5
would be nice to know in pawn. ); with 18 e5, but these 21.Rxe5 Qxe5 22.Nh5
advance. B) . At first I thought I might not be correct. If Nxh5 23.Bxd8 Nf4
Nevertheless, I like the would have to play we could decide "with compensation"
alternative win found by 18.exd5 with equality, everything on general for Black is given in
Fritz more than my own. It but then I realized that grounds, we would not ECO. I think Black
goes like this: Nxd5 followed by 19... need to calculate at all. might have even more
. Bf6 would be by no Abstract thinking is than that; e.g. 24.Bxa5
(press F10 to continue) means equal. Black very useful in helping ( or 24.Bb6 Nc4
1-0 would have two us with what we need 25.Ba7 g6 and Black
wonderful bishops on to calculate, but it is looks better to me )
the long diagonals, not a method which 24...Bxa5 25.Nb3 Bb6
analysis Aagaard-Brynell while my pieces are can substitute for 26.Qe1 Ne2+ 27.Kf1
[Jacob Aagaard] packed into the corner calculation itself. Ng3+ 28.Kg1 Ne4
and I would surely Anyway, after some 48 and Black has an
XABCDEFGHY have trouble facing my minutes I discovered attack.;
8r+l+-trk+( team-mates after the the move I eventually B) . My idea, however,
7+-+-+pzpp' 54 game (as so often played in the game. I was just to play my
6p+-+-+-+& before, I started used the next 5 or 6 previous line, now
5wqp+NzPp+-% speculating about minutes to check the improved by the
4-+-wQ-+-+$ leaving the playing variations before clearing of the first
3+-+R+-+-# hall unseen but executing my moves.; rank, i.e. 20.Re2!?
2PzPP+-+PzP" decided, for the D) 18.-- ] Ne4 ( not 20...Rxe5?
1+-mK-tR-+-![ hundredth time or 18.e5 Played after 55 21.Bf4 ) 21.Bf4 Bc5
xabcdefghy more, that it would be minutes I now had 45 22.Nb3 Nxg3 23.Bxg3
19.b4 Qxa2 20.Nf6+ very suspicious if I put minutes for the rest of the Nxb3 24.Qxb3
gxf6 21.Qh4!? on my jacket and made game. Bb4 19.Bf5!! and the position is
[ instead of 21.Rg3+ ] my way to the door).; This move is not obvious level.
21...f4 22.exf6 Kh8 C) . So I started at all. If I started making a As you can see my
23.Qh6 Rg8 looking for alternatives. list of candidates on the solution to the
[ I had actually seen that I found lines like 18.e5 previous move, or even problems I was facing
such positions could Bb4 19.Re2 Rxe5 here, I am not sure I does not provide me
arise and that I had 20.Bf4 Rxe2 21.Bxc7 would be able to find it with an advantage. But
23...Rg8 24.Re8! Re1+ 22.Qxe1 Bxe1 within the first 10 minutes. it does look as if it is
winning, but much 23.Be5 Bb4 24.Bxf6 Firstly I needed to get the the only way to avoid
more beautiful are the gxf6 25.Bf5 Re8 idea of allowing ...Bxe1 in being worse in this
full festivities with... ] , but could not really order to prevent ...Rxe1+; position, so it can be
24.Qg7+!! Rxg7 25.Re8+ make them work. then I needed to find a called a definite
Rg8 26.Rxg8+ Kxg8 After perhaps thirty move, any move, which success. I did not
27.Rd8# minutes thinking I would work well as a worry about the 55
1-0 decided it would not waiting move. minutes I had spent
make any sense to [ Obviously it could not be finding this idea, as I
look for a playable 19.Bg5 because of Rxe5 was sure that my
B00 position after anything and I am just a pawn opponent now would
Aagaard,J but 18 e5, as all other down. have to face real
Ong,K moves would hand The main reason behind problems. When the
Swedish League 2003 over the initiative for the bishop move is difference between two
[Jacob Aagaard] free. This did not mean actually to threaten the moves are minimal, it
that I instantly played rook on c8 and nothing makes no sense to use
1.e4 e5 2.Nf3 Nc6 3.Bb5 the move, but rather else. It looks counter- a lot of time on
a6 4.Ba4 Nf6 5.0-0 Be7 that I only analysed 18 intuitive, as f5 is the deciding between them.
6.Re1 b5 7.Bb3 d6 8.c3 e5 Bb4 after this. natural square for the But in every game
0-0 9.h3 Na5 10.Bc2 c5 Some might argue that knights. But sometimes there are critical
11.d4 Qc7 12.Nbd2 cxd4 I should then instantly intuition and abstract moments where it is
13.cxd4 Bb7 14.Nf1 play 18 e5 and only thinking cannot solve necessary to calculate.
Rac8 15.Bb1 consider my position our problems. We need It is for those we store
[ 15.Re2! is the better after 18...Bb4, but to be very concrete and time, and when we
Excelling at Chess Calculation− Jacob Aagard 33

face them we should 22.Qh5+ Kg8 23.fxg7 f6 does not want to take time was spent in vain as
not be afraid of using ( if 23...Re1+ 24.Kh2 f6 the bishop on the first I simply did not realize
the time necessary to 25.Qh8+ Kf7 26.Qh7 move in no way means that the bishop on b7
make a sound Rg8 27.Ndf5 wins ) that he will be shy to was hanging, but once I
decision. 24.Qh8+ ( or 24.Ndf5 take it on the next. did, it was enough to win
Some players (such as Re1+ 25.Kh2 Rxc1 Awareness to such the game. ]
Nunn and Larsen) 26.Qh8+ Kf7 27.Qh7 details will certainly win 24.Qf2! The natural move.
have written that and wins ) 24...Kf7 you games. ] Black is now under heavy
whenever they have 25.Qh7 Rg8 26.Qh5+ 22...Bxg3 23.Qg4 Bf2+ attack and cannot save
used more than 20 Ke7 ( if 26...Kxg7 24.Kh1 Qxf6 25.Bxc8 the position. Qa5
minutes for a move, 27.Ngf5+ Kf8 28.Ne6# ECO gives this as winning This looked at first to be
usually they have and Black is mated ) for White in the game I. the most intimidating
wasted this time or 27.Ngf5+ Kd8 28.Ne6+ Kapic-S.Busemann, continuation, but the game
even committed Kd7 29.Qf7+ Kc6 correspondence 1990. The is pretty clear-cut.
blunders. Personally I 30.Nfd4+ Kb6 ( or next day I tried to [ No better were
have a different 30...Kd6 31.Bf4# ) convince my team-mates alternatives such as
experience and have 31.Nxc7 and White that it was all theory and 24...Ba3+ 25.Kd2 Ne4+
often seen absolute wins. that I had been in full 26.Bxe4 dxe4 27.Qf5
top players think for a It is not necessary in control, but they Bd5 28.e6 and White is
long time over a single practice to work out the remembered very well my close to winning ]
move. Probably it is a details of these lines painstaking face from the [ or 24...Rf8 25.Qxf8+!
matter of taste as before taking the day before. h5 26.Qd7 Bxf8 26.Bg6+
much as anything. But decision to refrain from Bxc8 This loses and White wins a piece –
note that the purpose regaining the exchange. immediately, but Black is if Kd7 27.e6+ Kd6
of using a long time on All I had to do was see lost anyway. 28.Bf4+ . ]
a move is not to gain that a clear majority of [ If 26...Bxe3 27.Bxb7 25.Qf7+ Kd8 26.Qxb7!
control over each and variations seemed Bxd4 28.Bxd5 Bxb2 ( or A move that can easily be
every variation, but to favourable for me. 28...Nc4 29.Bxc4 bxc4 missed. Qa1+
make a decision. Here Often complicated 30.Qc8+ Kh7 31.Qxc4 [ 26...Ne2+ 27.Kd1!
I did not need to spend calculation should be Bxb2 32.Rf1 ) 29.Re1 was the first point to
a long time once I replaced with an Ba3 30.Re8+ Bf8 find. ]
had discovered 19 Bf5, intuitive decision. If you 31.Qc7 Nc4 32.Bxc4 27.Kd2 Nb5+ 28.Ke2
but I still wanted to calculate everything in bxc4 33.Qb8 Nxd4+ 29.Bxd4 Qxd4
check a few lines in a game in order to stay and the game would be 30.Qxa8+ Ke7 31.Qb7+
order to make sure in control, the only over all the same. ] Ke8 32.Bf5
that everything was thing you can be certain 27.Qxc8+ Kh7 28.Qc2+ [ 32.Qc8+ Ke7 33.Qc7+
OK. of is that you will lose g6 29.Rf1 Ke8 34.Bg6#
My opponent taking on time. ] 1-0 was stronger, but being
just 10 minutes to 21.Be3! This brings the a rook up can make
capture on e1 is not so bishop into play and some people a little
easy to understand. I protects the first rank and B80 careless. ]
have a feeling that he f2 at the same time. There Aagaard,J 32...Qxe5+ 33.Kf3 Qd6
did not fully scan for are not really any Eriksson,J 34.Qc8+
candidates and alternatives. Rxe3 Swedish League 2000 1-0
perhaps only [ After 21...Rc4 22.Bxh7+ [Jacob Aagaard]
calculated for some Kxh7 23.Qh5+ Kg8
time on taking the rook 24.fxg7 White has a 1.e4 c5 2.Nf3 d6 3.d4 Sasikiran,K
and then felt obliged to winning attack. ] cxd4 4.Nxd4 Nf6 5.Nc3 Skytte,R
do so. Anyway, White 22.fxe3! This move might a6 6.Be3 e6 7.f3 b5 Copenhagen 2003
was now pleased to not look anything special, 8.Qd2 Bb7 9.g4 h6 [Jacob Aagaard]
have a pawn near the [ but in my opponent's 10.0-0-0 Nbd7 11.h4 b4
black king.; time I had quickly 12.Na4 Qa5 13.b3 Nc5 XABCDEFGHY
C) 20.-- ] worked out the following 14.a3 Nxa4 15.axb4 Qc7 8-tr-tr-+-+(
20.exf6! Qb6 This was line, which showed that 16.bxa4 d5 17.e5 Nd7 7zp-+P+k+-' 55
the move I expected, but I the tempting bishop 18.f4 Nb6 19.f5 Nxa4 6Q+-+l+pzp&
did manage to calculate sacrifice should be 20.fxe6 Nc3 21.exf7+ 5+-wq-zpp+-%
some of his other possible avoided: 22.Bxh7+? Kxf7 22.Bd3 Bxb4 4-+P+p+-+$
replies (though not in Kh8!! 23.fxg7+ 23.Rdf1+ Ke8? 3+-+-zP-+-#
detail). ( possible is 23.Qh5 This was the first new 2P+-+LzPPzP"
[ Here is an updated Qxf6 24.Ngf5 Rxh3! move of the game and, to 1+-+R+RmK-![
version of these 25.Qxh3 Bd2 my good fortune, it is a xabcdefghy
analyses: but Black is not worse ) bad one. Sasikiran now turns a
20...Rcd8 21.Bxh7+ 23...Kxh7!! ( not [ 23...Kg8 had been complicated, though not
Kxh7 22.Qh5+ Kg8 23...Kxg7 24.Ngf5+ played in some previous particularly interesting
23.Qg5 and White wins. ] Kxh7 25.Qh5+ Kg8 games I had analysed. middlegame into a
[ 20...Bb4 21.Bxh7+ 26.fxe3! and White has a My opponent, on the memorable game with a
( 21.Bd2!? Bxd2 winning attack ) 24.Qh5+ other hand, was in series of 'trick shots'.
22.Qxd2 Qe5 23.Nh5 Qh6 and Black is better. trouble as 19 f5 was new White has won a pawn,
h6 24.fxg7 should also This kind of variation is to him. but only at the cost of
win for White, though not difficult for the I now used 50 minutes advancing his d-pawn so
this line is a little experienced player to to calculate the position far that it cannot survive.
strange ) 21...Kxh7 work out. That Black to the end. A lot of this However, Black needs a
Excelling at Chess Calculation− Jacob Aagard 34

few moves to round it up, even if he materially was for some reason worse. ]
and White would still have looks alright for the obvious to him! ) 39.Qf8 21.Rad1 Bc5
a small edge with his moment. ] Rd8 40.Rxf5 Kc6+ [ Now 21...Bc5 22.Ng6+
passed c-pawn. White 33.c5 Threatening Qd6+ 41.Kh3 Rh1+ 42.Kg4 Kf7 23.Nxh8+ Rxh8
now tries to provoke some and the rook on e2. Re1 h5+ 43.Kg5 Rh2 44.g3 would give Black
weaknesses in the black 34.Rxe5!? Again far from and White has a decent compensation for
camp which can be used the obvious choice. Later winning endgame on the the exchange, so White
later. 29.h4!? The idea of Sasikiran explained that way. ] comes up with a truly
this move is far from his opponent was in time 36...Bg8 inventive blow! ]
obvious. Rb2?! trouble and he wanted to [ If 36...Bc4 (or 36...Bf7) 22.Bxe5!? !? What an
An ill-conceived plan. avoid giving him obvious 37.Qd6+ Kc8 38.Qc6+ imaginative move! The
[ Sasikiran felt that Black moves. Normally this Kb8 39.Re7 Rc8 main point is simply to
would have been better sounds like a sound 40.Qd7 and Black is trap the black king in the
off playing 29...Ke7 strategy, but when you mated. ( Sasikiran did centre and then take
to liquidate the d-pawn, have simple winning lines not see 40 Qd7 and was things as they come. Very
after which the at your disposal, then the instead planning 40.g8Q often in Vescovi's games
scenario would be rather logic needs some Bxg8 41.Qa6 we see a readiness to
quiet. ] discussion. Personally I which looks very much take wild chances, with
30.h5! If you want to set think that once you have like the game. )] the belief that he will out-
yourself a difficult task, this competitive mindset, it 37.Qd6+ Finally Sasikiran calculate his opponent in
then try to guess the is difficult to switch around finishes the game with the resulting complications.
moves to come from the easily and be completely obvious moves. Kc8 In this game the
Indian grandmaster. I was pragmatic. If you have this 38.Qc6+ Kb8 39.Re7 combination can be
following the game live, attitude towards playing in Rc8 40.Qa6 defined as correct, since
and though I was able to time trouble, you have it To have been aiming for White did not have any
win his position against no matter what. And I am this position is what we advantage otherwise, but
myself, I was only able to certain that Sasikiran understand about being the important thing is the
guess Sasikiran's most knew what he was doing, creative. Rxc5 difficulties Black is
obvious moves. Rxe2? even though he did not [ 40...Rc7 41.Re8+ suddenly facing in how to
[ Objectively better was feel completely sure mates. ] defend against all kinds of
30...g5 but Skytte did not afterwards. 41.Qxa7+ threats. It is safe to say
see White's 32nd move. [ White could also have 1-0 that Black did not foresee
After 30...g5 White would won with the more White's idea, so he must
be better but not straightforward 34.Qd6+ suddenly have felt a bit
decisively. ] Kf6 35.Qxe5+ Kg5 B17 uneasy, for what is next?
31.hxg6+ Ke7 ( 35...Ke7 36.Qg7+ ) Vescovi,G fxe5 23.Qf3+ Ke7
[ If 31...Kf6 32.Rd6 36.Qxe6 Rxf1+ 37.Kxf1 Miton,K 24.Rd3! This was the idea.
and White wins fairly Qb1+ 38.Ke2 Qxa2+ Aeroflot Open, Moscow Now 25 Ng6+ winning a
easily. ] 39.Ke1 and there is no [Jacob Aagaard] rook or two is threatened.
32.Rd5!! A move which is perpetual check. Rhd8! Black is walking a
easy to miss. There are Sasikiran demonstrated 1.e4 c6 2.d4 d5 3.Nc3 thin line. This – and only
many ways to create this this afterwards in the dxe4 4.Nxe4 Nd7 5.Ng5 this – move was necessary,
kind of imagination. One if commentary room, so he Ngf6 6.Bd3 e6 7.N1f3 as the alternatives show:
them is by solving studies, had seen it! Instead he Bd6 8.Qe2 h6 9.Ne4 [ 24...Raf8 25.Ng6+ Ke8
another is by analysing chose to be as creative Nxe4 10.Qxe4 Qc7 26.Qc6+ Kf7 27.Nxe5+
with Fritz from time to time. as possible. ] 11.0-0 b6 12.Qg4 Kf8 Kf6 28.b4! Rc8 29.Qf3+
Even though we should 34...Rxf1+ 35.Kh2! [ 12...g5 lost all credibility Bf5 30.bxc5 and White
be careful not to let the The point. Kxd7 after V.Anand-V.Bologan, has a winning attack. ]
computer think for us, we The only move. Now try to Dortmund 2003. ] [ 24...Bd4 25.Ng6+ Kd6
should still pay respect to see if you are able to 13.b3 c5 14.dxc5 Nxc5 26.Nxe5! Qc5 ( not
the tactical awareness it guess White's next move. 15.Bb2 e5 16.Bf5 h5 26...Kxe5? 27.Re1+ Kd6
can teach us. Qb4 [ If 35...Rxd7 36.Qxe6+ 17.Qh3 Ne6! This is the 28.Rxd4+ Ke7 29.Qb7+
White wins in all variations Kd8 37.g7 and White end of the opening and Kf6 30.Rxe6+ Kxe6
from here on. wins on the spot. ] Black has equalized. 31.Qd7+ Kf6 32.Rd6+
[ If 32...Qb6 33.Qa3+ 36.g7!! Simply brilliant! [ 17...Bxf5 18.Qxf5 Re8 Ke5 33.Qe6+ Kf4
Kf6 34.g7! ( 34.Rd6? This move leads more or is Fritz's choice, but 34.Rd4+ Kg5 35.h4# )
Rxa2 35.Rxe6+ Qxe6 less to forced mate. And White should be better 27.Qe3! ( simplest;
36.Qxa2 is less clear ) again it is not easy for here. ] 27.Ng6!? Rhd8
34...Kf7 ( or 34...Kxg7 Black to find his way. 18.Bxe6 Bxe6 19.Qg3 f6 28.Rxd4+ Qxd4 29.Rd1
35.Qe7+ followed by 36 [ In the commentary room [ Also interesting was Qxd1+ 30.Qxd1+ Kc7
Rd6 and wins ) 35.Qf8+! we went for the 19...h4!? 20.Nxh4 Qxc2 31.Qxh5 also looks
Rxf8 36.gxf8Q+ Kxf8 materialistic approach 21.Rad1! Qxb2 22.Rxd6 promising; , but 27.Rfd1?!
37.d8Q+ Qxd8 with 36.Qxe6+ Kc7 Re8 23.Nf3 Qxa2 Kxe5 28.Re1+ Kd6
38.Rxd8+ Ke7 39.Rh8 37.Qe7+ ( 37.g7 24.Nxe5 Kg8 29.Qe3 Rh6 is not clear
and wins. ] also wins – Sasikiran ) and Black does not look at all ) 27...Qxe5
[ The aesthetic point of 37...Rd7 38.g7! Qb8 worse. ] 28.Rxd4+ Bd5 29.Rfd1
the combination was ( this was the move that 20.Nh4 Qxc2!? Rad8 30.Re4! Qf6
32...Bxd5 33.cxd5 Sasikiran did not like; Definitely an adventure. 31.Qg3+ Kd7 32.Rxd5+
with the threat of Qe6+, instead 38...Rxe7 [ Also possible was Kc6 33.Rde5 Rd1+
or if Qxd5 34.Qxe2 39.Rxe7+ Kd8 40.Rf7! 20...Kg8 and it is hard to 34.Re1 Rhd8 35.h4
with Qh5 to follow. Qb8+ 41.g3 Rxf2+ see any reason why and White should win
Black cannot survive, 42.Kh3 and White wins Black should stand with his extra pawn. ]
Excelling at Chess Calculation− Jacob Aagard 35

[ 24...Rhc8? 25.Ng6+ Bf4?? A terrible blunder. 30.Rxh8+ Kxh8 a-pawn is very


Ke8 26.Nxe5 [ Black should play 31.dxc5 bxc5 32.Re1 dangerous, much more
and White wins as he something like Kg8 33.Re4 Kf7 dangerous than the d-
has managed to defend 15...Rd8!? ] 34.Rc4 Ke6 35.Rxc5 pawn. After the text
the rook and can now go [ or 15...Re8!? Kd6 and although move with accurate play
on a rampage with his , though White seems a White should win this White should still be able
queen, starting with 27 little better after endgame, Black has at to make a draw. ]
Qxh5+. ] 16.Nd2!? with the idea of least activated his king 30...Bxd5 31.Rd2
25.Ng6+ Ke8 26.Qc6+ 17 Nc4. ] and thereby created It looks as if Black is in
Bd7?? Black simply 16.Ne5 Bxc1 some chances for deep trouble, but he
blunders, which is very [ After 16...Bxe5 17.dxe5 White to go astray.; comes up with a fantastic
easy to do when exposed White is much better, C) 28.-- ] solution. Ka7!!
to such creativity. since if Nxe5 18.Bf4 f6 28.Rce1 cxd4 29.cxd4 The pin is resolved and
[ The correct line was 19.Rad1 Rd8 20.Bxe5 Bd5 30.Rd6 With the the king comes to help
26...Kf7! 27.Nxe5+ ( if fxe5 21.Qxe6 threat of Rd8+, so Black from the side. 32.Bxd6
27.b4 Bc4 28.Nxe5+ and Black's position is has to move the queen. [ If 32.Rcd1 Re6! 33.Bc7
Kg8 and Black made it horrible. Nevertheless Qc4 31.Qe5! Rde8 34.Nxe6 fxe6
home safe ) 27...Ke7!! Black should have tried Forcing Black's next move. 35.Bxa5 b5 and Black
( perhaps the move 16...Bxe5 and lived with Kf7 has a promising
Black overlooked; not his ruin of a position, [ If 31...b5 32.Rd8+ Kh7 endgame, as the bishops
27...Kf6? 28.Rf3+ Kxe5 since now there is no 33.Qf5+ g6 34.Rd7+! are very strong. ]
29.Re1+ Kd4 30.Qxe6 chance to save the Nxd7 35.Re7+ Kg8 32...Rxd6 33.Rcd1
and the black king game. ] 36.Qxg6+ Kf8 37.Qg7# ] White was planning this of
should not survive, 17.Nxf7! The move Black [ or 31...Bf7 32.Rd8+ course, but Black is
though after Bd6 must have overlooked. Kh7 33.Qf5+ and wins. ] prepared. Kb6 34.Rxd5
31.Qe3+ Kd5 32.Rc1 The sacrifice is completely 32.b3! Rxd5 35.Rxd5 Kc6
Bc5 33.Rxc2 Bxe3 decisive. Kxf7 Forced; [ The black queen was A highly unusual situation.
34.fxe3 White only has a [ as 17...Bxb2 18.Nxh8 defending the king by White is a rook up, but his
winning rook endgame ) Bxa1 19.Qxe6 indirectly guarding the position can only just be
. Now White has three wins for White. ] f7-square (i.e. after saved. 36.Rd7!
tries, but only one is OK: 18.Qxe6+ Kf8 19.Qe7+ 32.Rxf6+ gxf6 33.Qh5+ [ 36.Nd7 Kxd5 37.Nxf6+
28.Qc7+ ( the only way Kg8 20.Raxc1 Rc8 Kg7 34.Re7+ Bf7 Kd4! dominates the
to continue, when the [ Or 20...Bd5 21.Bf5 ). Now the attack cannot knight and the running a-
game ends in a draw; Rd8 22.dxc5 Rh7!? be stopped. ] pawn wins for Black. ]
instead 28.Qb7+ Kf6 23.Bg6! and Black has 32...Qb5 33.Qe7+ Kg6 36...Kxc5 37.Rxb7 a4
29.Rf3+ Kxe5 no defence. ] [ If 33...Kg8 34.Rxf6! 38.Kf1? White loses his
is no longer dangerous 21.Bc4+ Kh7 22.Bd3+ gxf6 35.Re3 grip in time trouble.
for Black; , while after Kg8 23.Bf5 Nf6 24.Be6+ and the absence of ... [ He could probably still
28.Rf3 Rac8 29.Qb7+ Kh7 25.Bf5+ Kg8 Qc1+ is also felt. ] have made a draw in
Ke8 30.Qxg7 Rd1 26.Bxc8 Qxc8 27.Re6! 34.Re3! Rc8 35.Rg3+ the following fashion:
31.Qh8+ Ke7 32.Qf6+ I like this move for Kh5 36.Qe5+ g5 38.Rxf7 Bc3!
Kd6 33.Nc4+ Kd7 reasons of principle, 37.Rh3+ Black is mated. ( with the idea of ...Ba5
the black king dances [ though Fritz prefers 1-0 is the most ambitious try;
away ) 28...Ke8 29.Qc6+ 27.dxc5!? bxc5 28.Re5 38...Bd4 does not work
Ke7 30.Qc7+ Nd5 29.Qh4 Qc6 since after 39.Rxh7 Kc4
with perpetual check. ] 30.Rce1 Nf4 31.f3 Ng6 Rublevsky,S 40.Rc7+ Bc5 41.Rc6 a3
27.Qd5! Suddenly it is all 32.Qc4+ Kh7 33.Rxc5 Filippov,V 42.Rxg6 a2 43.Ra6
over. Bf8 28.Nxe5 and White should win Aeroflot Open, Moscow White makes a draw )
1-0 without much effort. ] [Jacob Aagaard] 39.Rxh7 a3 40.Ra7
27...Ba8?! This passive Bb4 ( or 40...Kb4
move gives White the XABCDEFGHY 41.Ra6 Kb3 42.Rxg6
B17 extra tempo needed for 8-mk-tr-+-+({ Ka4 43.Ra6+ Ba5
Petrosian,TL his attack to achieve full 7zpp+-+p+p' 56 44.Rd6 draws ) 41.g4!
Galkin,A force. 6-+-tr-vlp+& (ensuring that enough
Aeroflot Open, Moscow [ The best chance was to 5+-sNP+p+-% pawns are exchanged on
[Jacob Aagaard] look for an endgame with 4-+-+-vL-+$ the kingside) Kb5!?
27...Nd5! when White 3+-+-+-+-# 42.gxf5 gxf5 43.Kg2!
1.e4 Incidentally, Tigran would have to be careful: 2l+-+-zPPzP" ( the f5-pawn is the
Petrosian is rated 2477 A) 28.Qd6? Nf4! 1+-tR-tR-mK-! target; not 43.h4? Ba5
and is not the former ( but not time wasting xabcdefghy 44.Rd7 a2 45.Rd1 Bc3
World Champion, though moves like 28...cxd4? 29...a5!! This pawn is 46.Kg2 a1Q 47.Rxa1
he may well have been 29.Rce1 Kh7 30.Qg3! definitely an asset. But Bxa1 48.Kf3 Be5 49.h5
named after him. c6 2.d4 when White has a this move has more to do Kc5 50.h6 Kd5 51.h7
d5 3.Nd2 dxe4 4.Nxe4 winning attack ) 29.Re7 with the coming Ke6 and wins ) 43...Ba5
Nd7 5.Ng5 Ngf6 6.Bd3 Qg4 30.Qb8+ Bc8 manoeuvre with the king. 44.Rd7 a2 45.Rd1 Bc3
e6 7.N1f3 Bd6 8.Qe2 h6 31.g3 Nh3+ 32.Kg2 30.Re2 White pushes his 46.Kf3 (just in time; now
9.Ne4 Nxe4 10.Qxe4 Kh7 33.f3 Qg5 luck, but it is not with him 47 Kf4 is threatened)
Qc7 11.Qg4 Kf8 12.0-0 when Black is in the today. Be5 47.h4 Kc4 48.h5
b6 13.Re1 Bb7 14.c3 c5 game again and even [ If instead 30.Ne6?! Ka7 Kb3 49.h6 Kb2 50.h7
15.Qh3 This is the end of has the initiative.; 31.Nxd8 Rxd8 32.d6 a4 a1Q 51.Rxa1 Kxa1
the opening. Now Black B) 28.Qe8+! Qxe8 and Black has a very 52.h8Q Bxh8 53.Kf4
has to think for himself. 29.Rxe8+ Kh7 promising endgame. The with a draw. ]
Excelling at Chess Calculation− Jacob Aagard 36

38...a3 Now it is all over as 12...Nf5 13.Bb2 Nb6 ( 27...Rxe3 28.fxe3 It seems that Lautier
White will lose the rook 14.Qb3 Nxc4 15.Qxc4 Bxe3+ 29.Rf2 Bd4 was badly prepared for
eventually. The black king Bd7 16.Nc3 30.Ne2 Bxf2+ 31.Kxf2 his experienced
can hide on b1 using the [ Here it was interesting Kf7 is better for Black, opponent and his
bishop as a shield. 39.Ra7 to try to restrict the black but winning would not so attempts at being
Kb4 40.Ke2 Kb3 41.Kd3 bishop by 16.b5! Qb6 easy ) 28.fxe3 Rxe3 creative come at a price:
a2 42.Rb7+ Ka3 43.Ra7+ 17.a4 a6 18.Nc3 29.Rc4 Re1+ 30.Kf2 he plays less than
Kb2 44.Rb7+ Kc1 with a complex Rc1 and Black should perfect moves. ]
45.Rc7+ Kd1 middlegame ahead. ] win without too much 7...Nd7 8.f3?!
0-1 16...Bc6 17.Rad1 Qe7 trouble. ] This creates a lot of
18.Nd4 White is clearly not [ 27.Bd2!? was possibly weaknesses and has no
happy with the threat of ... better, even though Bb8 long-term advantages.
D21 Bxf3; looks similar to the Moves like this certainly
Khenkin,I [ and 18.Ne4!? game. ( Instead 27...Rd8 need to be eliminated
Rublevsky,S seems uncomfortable as 28.Be3 Bxe3 29.fxe3 from Lautier's play, if he is
Aeroflot Open, Moscow Black can play a6! Rd3 30.Rc4 Rxe3 to break the 2700-barrier
[Jacob Aagaard] 19.Nd4 Nxd4 20.Bxd4 31.Kf2 gives White a fair and join the absolute
Qh4 21.Bc5 Bxe5! chance for a draw. )] chess elite – a stated
1.d4 d5 2.Nf3 e6 3.c4 22.g3 Qh5 23.Bxf8 27...Bb8 28.Rc4 ambition of his.
dxc4 4.Qa4+!? Rxf8 with excellent [ 28.Rg4 Be5 29.Bd2 [ 8.Ng3!? was a possible
This is a creative move, compensation for the Rd8 also gives Black improvement, and after
but I dislike this kind of exchange. ] excellent compensation Be6 ( though 8...Bg6
creativity which is not 18...Nxd4 19.Rxd4 a6! for the pawn, but White looks more natural )
based on sound principles. 20.Rg4? White is looking should perhaps have 9.Nh5 Bf8 10.Bg3
Of course this move is for active counterplay, but played this way, and now White would have an
playable, but now White his chances are not on the 30.Ke1 holding on. ] advantage. ]
continues very ambitiously, kingside. 28...Rc8! Threatening to 8...Bg6! A strong
which he simply cannot do. [ He could still have kept win the exchange, or even prophylactic move which
Nd7 5.e4 c5! With his the balance had he a whole rook. 29.Kg1 restricts White's ideas of
last move White weakened played 20.a4!? Ba7 [ 29.Nd5 loses to Bb5 advancing the pawns on
his central dark squares 21.Rd3 Rad8 22.b5 30.Ne7+ Kf7 31.Nxc8 the kingside, as well as
and Black instantly Rxd3 23.Qxd3 Rd8 Bxc4+ 32.Ke1 Bxa2 vacating the f5-square for
attacks them – sound 24.Qe2 axb5 25.axb5 and the endgame is the g8-knight. 9.Bg3
opening play. 6.Bxc4 Bd5 26.Nxd5 Rxd5 hopeless. ] [ If 9.Ng3 Nf8! ]
cxd4 7.0-0?! This pawn 27.Rd1 and, though 29...b5 White has been [ or 9.g4 h5! and in both
sacrifice does not look Black has a more under pressure for some cases the white pieces
sound. comfortable position, time. Now he blunders the are not really
[ It was better to play White should be able to exchange and loses the coordinated. ]
7.Nxd4 , albeit with no make a half point. ] game without further 9...Nh6! This is a creative
advantage at all. ] 20...Ba7 21.Bc1 resistance. 30.Rd4? solution to the
7...Bc5 8.b4 White needs White is now ready to [ If 30.Rg4? Bd7! wins. ] development of the black
to play with great vigour. start his counterplay on [ 30.Rh4 was the only pieces. The knight is of
[ After 8.Nbd2?! the kingside with Bc1-g5-f6. move, but after Bd7 course thinking about
he does not win the But Black is in full control 31.Bd2 Be5 followed by going to f5. 10.Bf2
pawn back, as Black has of the position and he 32...Rc2 Black is very [ 10.Nf4 Nf5 11.Bf2 Bd6
the beautiful a6! 9.Nb3 stops it before it becomes close to a winning 12.Nxg6 hxg6
Ba7 when he keeps the unpleasant. f5!! ending all the same. ] does not do White any
advantage because of ... With this paradoxical 30...Be5! 31.Be3 good, as h2 is suddenly
b7-b5. ] move Black solves all his [ 31.Rd3 Be4! weak. ]
8...Bd6 9.e5 problems at a stroke: he was the point behind the 10...Bd6! Aimed against
[ 9.Bb2 e5 is just good either clears the ranks for last move – not a 11 Nf4. Now White has no
for Black. ] his pieces to defend the difficult tactic, but easy way to develop and
[ The same goes for king or, as in the game, sometimes it is hard to this provokes him into
9.Nxd4 Qc7! and Black he gives up a pawn and see what the opponent trying to solve the problem
is a little better. Note that gets open files for his is threatening. ] once and for all. 11.g4?!
10.Nb5? does not work rooks. 22.exf6 31...Bxd4 32.Bxd4 Rd8 [ 11.Ng3 0-0 12.Bd3
because of Bxh2+ [ If 22.Rg3 f4 33.Be5 Rd2 34.f3 Rb2 was better, but Black's
11.Kh1 Qxc4! . ] ( 22...Qe8!? and 23... 35.Bd6 Rc2 36.Be5 a5! position is easier to
9...Bb8 10.Bb2 Ne7 Rd8 is also very 0-1 play. ]
11.Bxd4 White has attractive ) 23.Rg4 f3 11...f5! Black does not
regained his pawn and it 24.g3 Kh8 and Black hesitate to provoke White
looks as if he has a space has the advantage. ] D31 to create further
advantage. But in reality 22...Qxf6 23.Rf4 Qg6 Lautier,J weaknesses in his camp.
his pawns are weak and 24.Qxe6+ Qxe6 25.Rxe6 Vaganian,R 12.h3
he will need to play Rfe8! The f4-rook is badly Aeroflot Open, Moscow [ If 12.gxf5 Nxf5
accurately in order just to placed while the one on e6 [Jacob Aagaard] and Black is clearly
keep the position level. 0-0 is excellent. It is therefore better. ]
12.Re1! White needs to easy to see which rook 1.d4 d5 2.c4 e6 3.Nc3 12...0-0 13.Bg2 Qe7
reinforce e5. Black should exchange. Be7 4.cxd5 exd5 5.Bf4 14.0-0 fxg4 15.hxg4
[ After something like 26.Rxe8+ Rxe8 27.Kf1 c6 6.e3 Bf5 7.Nge2 [ After 15.fxg4 Rae8
12.Qb3 Nc6 13.Rd1 [ The alternatives were: [ 7.g4!? is the most the weaknesses at e4
Qc7 he is in a bad way. ] 27.Be3?! Bxe3 aggressive option here. and e3 will count
Excelling at Chess Calculation− Jacob Aagard 37

eventually, e.g. following 21...Qxh5+ 22.Kg1 Qh3! of the black kingside the black king. ]
...Nh6-f7-g5 and Nd7-f6- The double threat to g2 provided by the first 10.Bg5 b5!? It is hard to
e4. ] and g3 is decisive. 23.Rf2 move. ] understand why a
15...Qg5 16.Qd2 [ If 23.Qxe3 Rxe3 5.exf5 Bxf5 grandmaster would play
[ White cannot free 24.Bxd6 Qg4+ 25.Kh1 [ 5...Nf6 would be no such a move, as it seems
himself. If 16.e4? dxe4 Rh3+ 26.Bh2 Qh4 better as White has the that after the bishop
17.Nxe4 Bxe4 18.fxe4 and Black wins. ] advantage after 6.d4! . ] retreats, Black will just
Nxg4 19.Qb3+ Kh8 23...Bxg3 24.Nxg3 6.d4! After this central have weakened his
20.Bg3 Black wins with Qxg3+ 25.Rg2 thrust I have not managed position. But when we see
Nc5! . ] [ Or 25.Kh1 Re6 26.Rh2 to find anything serious people making
16...Rae8 17.Rae1 Qf3+ 27.Rg2 Nxg2 resembling equality for seemingly unserious
White is trying to get his 28.Rxe6 Nf4+ Black. Nxd4 There is moves, we need to ask
pieces into the game, but and wins. ] nothing better. ourselves why they do so,
Black is already ready to 25...Nxg2 26.Rxe8+ Kf7 [ After 6...Nf6 7.Bb5! e4 in order to fully
deliver a lethal blow. 27.Rc8 Ne3+ 28.Kh1 Nf6 8.d5 a6 9.Nd4 axb5 understand the game in
[ If 17.Nf4 Rxf4!? There is no defence 10.Nxf5 Ne7 11.Ng3 front of us. Here Black
18.exf4 Bxf4 19.Qd1 against 29...Nfg4. Black suffers from the wants simply to survive
Nf7 20.Re1 Rf8 21.Qe2 0-1 weak light squares on the opening and castle
Qh6 22.Qe7 Qh2+ his kingside and queenside, but after ...
23.Kf1 Bd3+ 24.Ne2 problems concerning 0-0-0 the a-pawn would
Nf6 and Black has a A04 the development of Bf8. ] be hanging. Therefore he
lethal attack against Carlsen,M [ Even worse is 6...e4 tries to win a tempo now
the poor white king. ] Dolmatov,S , when White needs to when White cannot ignore
17...Rxf3! With all his Aeroflot Open, Moscow find 7.d5! exf3 8.dxc6 the threat to the bishop.
pieces ready, Black has [Jacob Aagaard] fxg2 9.Bxg2 b6 10.0-0 [ Anyway, after 10...d5
no serious alternative to with a wonderful position. 11.0-0-0 Be7 12.Rhe1
beginning the attack. The 1.Nf3 f5 2.d3!? The lead in Black has an equally
exchange sacrifice is Carlsen is very well development combined slim chance of survival,
thematic: by ruining the prepared for his games, with the open centre may due to 0-0 13.Qe5 . ]
white structure Black is so when I heard he had very well seal Black's 11.Bb3 Be7 12.0-0-0
able to use all the won against Dolmatov in fate. ] This position is all about
remaining pieces, which 19 moves, I was sure that 7.Nxd4 exd4 8.Qxd4 Nf6 swift development. The
are all well placed. it was a matter of opening 9.Bc4 c6 Black no doubt main reason for White to
18.Bxf3 Nxg4 19.Bg3?! preparation. However, it hoped to be able to play ... castle queenside instead
[ It seems that White was should be clear from d6-d5, and he needs to do of kingside is simply that
forced into the very going through the game so in order to get the king he gets a rook to the open
uncomfortable 19.Bxg4 that this was not the case. out of harms way. As this file more quickly. Black is
Qxg4+ 20.Bg3 The idea of 2 d3!? in this turns out to be too slow, one move too late to
( 20.Ng3?! gives Black position was mentioned in his position must be castle. Qd7
extra possibilities such the book 'SOS – Opening considered as close to [ Black cannot escape.
as Qh3! to strengthen Surprises' by Jeroen lost already. After 12...h6 13.Bxf6
the attack ) 20...Bxg3 Bosch, but how well [ Most experienced Bxf6 14.Rhe1+ Kf8
21.Nxg3 Qxg3+ 22.Qg2 acquainted Carlsen was players would disregard 15.Qxd6+ Qxd6
Qxg2+ ( 22...Rxe3 with this is hard to guess. a move like 9...Bxc2 16.Rxd6 Rc8 17.Be6
23.Qxg3 Rxg3+ 24.Kh1 The point here is that on general grounds. But Bxe6 18.Rexe6
is also possible, but Carlsen first plays sometimes it is nice to the endgame holds no
then White would have something creative but analyse it, to refresh chances for him. ]
some counterplay with sound, and then simply and test our intuition, as 13.Rhe1 Kd8 This move
Re7 ) 23.Kxg2 Nf6 plays his own game, well as to present looks silly,
and Black has a better something he does better evidence for less [ but Dolmatov was
endgame, though White than Dolmatov in this experienced players. probably saddened by
can still fight. ] game. d6 White gains a very prospect of 13...0-0-0
19...Nxe3 20.Kh1 [ 2...Nf6 would allow strong attack with 10.0-0 14.g4! ( 14.Qf4 Bf8
[ The endgame after 3.e4!? fxe4 4.dxe4 c6 11.Re1+ Be7 12.Bg5 15.Bxf6 gxf6 16.Ne4
20.Bg2 Bxg3 21.Nxg3 Nxe4 5.Bd3 Nf6 Bg6 13.Re3 (there may Bxe4 17.Rxe4 d5
Qxg3 22.Rf3 Qxg2+ with a reversed From's be other ways, but this is less clear ) 14...Bxg4
23.Qxg2 Nxg2 Gambit a tempo up for seems the most logical 15.Rxe7 Qxe7
24.Rxe8+ Bxe8 25.Kxg2 White. How dangerous to me: all White's pieces 16.Qxg4+ and Black is
gives White no serious this is is hard to say, are introduced to the near to losing. Now the
chances. But now but Black certainly black king) Kd7 14.Rae1 game takes a similar
another piece is played a good move in Re8 15.Bf4 Kc8 turn. ]
introduced into the the game. ] 16.Be6+ Kc7 17.Qc5! 14.Rxe7! The annihilation
attack. ] 3.e4 e5 4.Nc3 Nc6?! (threatening Nb5+) b6 of the defence on the dark
20...Bh5! 21.Bxh5 I really dislike this move, ( if 17...a6 18.Na4! squares is obvious in this
[ White has no hope of and in the game Black and Black is lost ) position where the pawns
survival in the endgame gets into trouble. 18.Qc4! and it is very mainly cover the light
after 21.Nf4 Nxf1 [ The most natural move difficult to believe that squares. Now White can
22.Rxf1 Qxg3 23.Nxh5 is 4...Nf6! Black will hold on for do as he pleases. One
Qh3+ 24.Kg1 Ne5!! to keep control over e4. long. The weakness of instructive point is that the
25.Bd1 Nc4 26.Qg2 Now White correctly the light squares has individual value of the
Qxg2+ 27.Kxg2 Ne3+ seizes the initiative and been transferred to the pieces does not matter
etc. ] exploits the weakening queenside along with much here, as we will not
Excelling at Chess Calculation− Jacob Aagard 38

face an endgame. And no database, the score is immediately by 14.Nf7 very famous for this kind
piece on the board, no about 56-44% in White's Rh7 15.Ng5 Rh8 of chess. 15.exd5 Nf5
matter how powerful, can favour, which is very close , though he might hope This was the idea. 16.Bf2
attack a square more than to the 55/45 average for to exploit the fact that Rd8 Black continues
once. In the attack, where any opening. Such Black can now no being violently active,
battle is often confined to statistics are of course longer castle, but will though this is no longer a
a limited number of always rather superficial, have to play slowly with surprise. Black carries out
squares, the main thing is as they can indicate a lot ...d6-d5, ...Ng8-f6-e4, ... his plan and then both
to have as many pieces of things and at the same Bf6-g7 and Ke8-f8-g7 players have to make their
participating as possible. time say very little about before he can get his choices. Here White has
Therefore is hard to the position. A modern rook into the game. ] the first real choice.
consider the exchange hazard is how many of the 12.Be2?! 17.dxc6 It is hard for
sacrifice as a real sacrifice. games were played by [ Usually in these kind of White to decline the
It is too basic. Qxe7 youngsters with artificially positions it is useful to challenge.
15.Qf4 Bd7 Necessary boosted ratings on the restrict the black knights [ If 17.axb5 cxb5
otherwise 16 Rxd6+ Internet. 5.Qd2 Nd7 as much as possible, followed by ...b5-b4
decides. 16.Ne4 d5 6.h4!? This is not the which could have been and Black wins his pawn
[ The natural 16...Rf8 standard way of done here with the back with good play. ]
loses to 17.Nxd6 h6 proceeding here, but stronger 12.e5! [ An interesting and
18.Qb4 a5 19.Qc5 Rb8 Sokolov was probably . The main threat is to possibly better move
20.Qa7 and the knight's thinking that if Black could play e5-e6 to weaken was 17.Be4!?
dominance becomes be creative, why not him the light squares, while when the position
clear. ] too? h5 7.Nh3 after e6 remains very
17.Nxf6 h6 18.Bh4 g5 The idea behind the last (which seemingly grants complicated. As all the
[ If 18...Rf8 19.Nxd5! move was to locate the Black has good play – action is on the dark
and wins. ] knight permanently on g5. since White has given squares, it makes sense
19.Qd4 Indicating a threat b5 8.Ng5 Bb7 9.a4 c6! both f5 and d5 away with for White at least to
to the rook on h8. Black [ Tiger has a lot of his last move and his retain the possibility of
had had enough. experience with this bishop on e3 is out of exchanging the f5-knight
1-0 system and correctly play), White can strike rather than the g4-knight
avoids 9...b4?! 10.Bc4! immediately with (which would open the h-
e6 11.Ne2 when White 13.axb5 axb5 14.Bxb5! file for the rook after ...
B06 has managed to benefit and it looks as if White hxg4). Now Black
Sokolov,I from his development is better, although the should probably play b4
Hillarp Persson,T over the last few position is still hugely ( as after 17...cxd5
Koge 1997 moves. ] complicated. ] 18.Bxf5 gxf5 19.axb5
[Jacob Aagaard] 10.Rd1 This may just be a 12...Nf6! Now Black has axb5 20.Nxb5 Qb6
waste of time. Qc7 no problems. 13.Bf3 21.Nc3 he might risk
1.d4 g6 2.e4 Bg7 3.Nc3 11.f4!? This is a very [ Here 13.e5 Nfg4 14.e6 ending up worse ) 18.Na2
d6 4.Be3 a6!? aggressive decision which f6 15.Nf3 Qc8 a5! with complications. ]
This system is considered has good sides and bad would only be good for 17...Rxd4! Of course
somewhat offbeat even sides. On the good side it Black. ] Black had foreseen this
though it has been played allows the f1-bishop to 13...Nfg4 14.0-0 move long ago, and it was
by such players as Lautier, find a good square on f3. Here the standard move no secret to White either.
Radjabov, Inarkiev, On the bad side it clearly would be to castle short Strong players have few
Khalifman, Gurevich, weakens the control over and then celebrate a problems foreseeing
Bacrot, Piket, the light squares in the decent outcome from the exchange sacrifices since
Azmaiparashvili, Svidler centre, which could have opening. But Tiger had a nothing is taboo to them.
and so on. There is no been maintained by deep think and identified 18.Nd5! The strongest
particular reason why it playing f2-f3 at some point. the reality of the position: reaction, but still a move
should be bad, though it is Tiger immediately realizes both black knights are which was easy to
hard to imagine that Black the downside to his aiming for the same overlook if you do not
can achieve equality in opponent's last move and square (g4), so with his have the awareness of the
the theoretical sense of uses it to his advantage. next move Tiger opens up different possibilities in
the word. But on the other Nh6!? From here the for the h6-knight and a this kind of muddy
hand – he is not looking knight can go to both g4 general onslaught on the position.
for it! Instead he wishes and f5 and, more dark squares. d5!! [ Instead 18.Qe2? Nxf2
for a game based on importantly, it allows the With this pawn sacrifice 19.Nxb5!? ( after
individual ability and d7-knight to come to f6 (and the coming further 19.Qxf2 Bxc6
creativity, rather than and g4. sacrifices) Black manages Black is just better –
defend a theoretical line [ However, because of to open the position and Hillarp Persson )
all the way to a draw. For White's option on the bring his pieces to life. 19...axb5 20.cxb7 0-0
this reason this system is next move, 11...Ndf6!? This is the idea behind 21.Qxf2 Rxf4
good to play for a win, and seems a better way to being creative: the looks very dangerous for
for exactly this it has been execute the idea. Then position is thrown off White ]
very useful to Hillarp White has nothing balance, and dynamic [ while 18.Bxd4? Bxd4+
Persson. special after 12.e5 Ng4 advantages outbalance wins the queen due to
As regards the correctness 13.e6 f5 and, as it can static advantages. The 19.Kh1? Ng3# . ]
of the system: in games be very hard to find resulting position 18...Qxc6 19.Qa5
between the 200 highest targets in the black becomes hard to evaluate, The only good square for
rated players (who have position, he should even in analysis after the the queen. Now Nc7+ is
played this position) in my consider forcing a draw game. Mikhail Tal was threatened and Black
Excelling at Chess Calculation− Jacob Aagard 39

cannot castle because of (the complicated way gives good initiative. Whatever White
Nxe7+. Hillarp Persson to do it) Bxf3 counterplay against is doing at the queenside
comes up with a ( 21...Nxf2 loses to f4 and h4 ) 25...Ne2 does not interest him.
wonderful queen sacrifice. 22.Rc1! Qxb2 26.Qe1 Qxb7 [ Instead if 24...axb5?
Qd6!! 23.Qc7+ Ke8 27.Qxe2 bxa4 25.Qa8+ Kd7 26.Bc6+
[ Black would be ill 24.Bc6+ Bxc6 and Black is not Kd6 27.Qb7
advised to hand over 25.Qxc6+ Rd7 worse; and the black king is in
the initiative to White 26.Nc7+ Kf8 B2) 23.c3 Nxf2 danger (Hillarp
with 19...Qxc2? 27.Qa8+ ) 22.Rc1! 24.Rxd4 Bxd4 Persson). ]
. First let's analyse a Rxa4 23.Qxb5+ Bc6 25.cxd4 Ng4 25.c3 This is given "?" by
standard candidate and 24.Nc5+ Kc7 and the position is Hillarp Persson, but in fact
see if it works. 25.Nce6+!! Kd6 still very difficult to White could no longer
A) 20.Rc1?! Qxb2 26.Qb6 and Black is assess. Probably it save the game.
( 20...Qd2 21.Nc7+ toast.; is balanced in some [ He was concerned
Kf8 22.Qxd2 Rxd2 B2) 21.Rc1 Qd2 weird way. ] about the complications
23.Bxb7 Nd6 24.Be1 ( or 21...Qxa4 ) [ Instead 20.Bxd4 Bxd4+ after 25.bxa6
Rxb2 25.Bxa6 bxa4 22.Qb6! Bxf3 21.Rxd4 Nxd4 , and now the Danish
is only somewhat 23.Nd5 would have been grandmaster Lars Bo
worse for White, but with a winning preferable to Black. One Hansen gave some long
why involve himself attack.; main point is that after and interesting analysis
in something this B3) , and now White the greedy 22.Nc7+? with Rd7!?
dodgy? ) 21.Nc7+ Kf8 has two ways to win Kf8 23.Bxb7 White is , which leaves White
and now we have: the game: 21.--; mated by Ne2+ 24.Kh1 hanging on the ropes,
A1) 22.Nce6+ fxe6 C) 20.-- ] Ng3+ 25.Kg1 Qd4+ but still breathing:
23.Bxb7 Bf6 20.Ne4? White accepts 26.Rf2 Qxf2# . ] A) 26.a7 Bd4+
24.Rc8+ Kg7 the queen sacrifice, 20...Qxd5 The idea behind 27.Kh2 Bxa7 28.Qxa7
25.Nxe6+ Kf7 probably because he was Black's move, and really Nde3 and White loses
26.Nd8+ Rhxd8 unable to find his way in the only move in the by force, e.g. 29.Rf3
27.Rxd8 Nxf2 all the tactics. It is very position, so it should not ( or 29.Re1 Ng4+
28.Rxf2 Qc1+ likely that he overlooked be any surprise. 21.Nf6+ 30.Kg1 Nd6
29.Kh2 Rxf4 the difference the knight Sokolov had to take an and Black has a
with a winning sacrifice would make in all intuitive decision here. decisive material
attack.; the main lines, simply [ 21.Rxd4 Nxd4 22.Nf6+ advantage ) 29...Ng4+
A2) 22.Bxb7 Bf6 because he never thought Bxf6 23.Bxd5 Bxd5 30.Kh3 ( if 30.Kg1
23.Qxa6 Nxf2 in that direction. This is looks very dangerous Nfe3!! wins ) 30...Rd1
24.Qxb5 Qxb5 another example of how it for White as well. ] 31.g3 Rfd8 32.Rf2
25.Nxb5 Ng4 is possible to end up in a 21...Bxf6 22.Bxd5 Nxf2! R8d2 33.Rxd2 Rxd2
26.Rc8+ Kg7 position where only very Not the most obvious 34.Qg1 Nfe3 35.b4
with a clear non-standard moves are move. In these kind of Rd1 and Black should
advantage to Black.; playable, and where we desperado positions it is win the endgame
A3) 22.Nxb5 Bf6! need to open our minds to important to look at pretty easily a piece
( not 22...axb5?? a completely different way everything, since virtually up.;
23.Qa8+!! Bxa8 of understanding chess. any move can prove to be B) 26.c3! (preventing
24.Rc8+ and mates ) [ Here 20.Nc7+! the killer. the black bishop from
23.Nxd4 Bxd4 was much stronger, [ 22...Rxd5 is weaker entering the game
24.Qd8+ Kg7 when Black has only because of 23.Qc7! directly) Nde3 27.a7
25.Ne6+ fxe6 one playable move: ( 23.Rxd5 Bxd5 Rxb7 28.a8Q Rxb2!
26.Bxd4+ Qxd4+ A) 20...Kd7? 24.Qxa6 0-0 25.Qxb5 29.Qf3 Rxg2+
27.Qxd4+ Nxd4 is foolhardy after e6 is also less clear: 30.Qxg2 Nxg2
28.Bxb7 Ne2+ 21.Nxb5! Black obviously has a 31.Kxg2 Ne3+ 32.Kg1
29.Kh1 Nxc1 ( the key move in all very dangerous attack, Nxf1 33.Kxf1 Rc8!
30.Rxc1 a5 lines; not 21.Nxf7?? but maybe White will be 34.f5! and Black will
with a better Qxf4 and Black wins ) able to create a little have some
endgame for Black.; 21...axb5 22.Nxf7 counterplay with his difficulties realizing
A4) 22.--; and White wins. The pawns ) 23...Nd6 his advantage as
B) . The right move is difference can be seen (the only move) 24.Rxd5 White will have a lot
also the most tempting after Qxf4 23.Qxb5+ Bxd5 25.Rd1 Nxf2 ( or of checks.;
one: 20.Nc7+! Ke6 24.Ng5+ Kf6 25...Ba8 26.Bd4! C) 26.-- ]
(White needs to bring 25.Bxd4+ Nxd4 with roughly even [ But all this analysis
the black king into the 26.Bxb7 and so on; chances ) 26.Kxf2 Be4 could have been omitted
middle, so intuitively B) 20...Kf8 21.Nxb5!! 27.Rxd6! exd6 28.Qxd6 if we had simply looked
this is the best decision, axb5 22.Bxb7 Bg7 29.Qxa6 0-0 after candidates to
but in such a complex and now White has 30.Qxb5 Bxc2 31.a5 begin with. After 25.bxa6
position you also have Qa8+ which prevents and White has good Nxb2! followed by 26...
to back it up with 22...Qxf4, so Black can counterplay. ] Ra4 Black has nothing to
variations) Kd7 only keep his position [ 22...Bxd5? is just bad, fear and will win without
( 20...Kf8 together with Qb8! since after 23.Bxd4 too much trouble. ]
loses instantly to a B1) if 23.Bxd4 Bxd4+ 24.Rxd4 Nxd4 25...Nde3!! Black treats
very nice mate: Bxd4+ 24.Rxd4 25.Qd2! White wins the threat to his rook as
21.Nce6+!! fxe6 Nxd4 25.Kh1 ( while (Hillarp Persson). ] an illusion. Now the attack
22.Qd8+ etc ) 25.Bf3 Nxf3+ 23.Bxb7 Nxd1 24.axb5 becomes truly irresistible.
B1) 21.Nxa6 26.gxf3 Nf6 0-0! Black clings on to the 26.bxa6
Excelling at Chess Calculation− Jacob Aagard 40

[ 26.cxd4 Bxd4 34.Qe5 f6 35.Qe6+ Kg7 investigate it. Had he . In the starting
leaves White with a 36.g3 Ne3 37.Qxe7+ done so, he would have position Black has no
heavy material Rf7 followed by ...Rh8+. ] spotted the follow-up. advantage if he
advantage... and no 32...Kg7! 33.Qe5+ f6 25.Kxg2 Bxh3+! doesn't sacrifice so,
chance of saving the 0-1 If you calculate this move in order to play it, all
game. After 27.Rf2 you will most likely reach you need to know is
Black wins by Ng4 the conclusion that it is that Black will not be
28.Kf1 Bxf2 29.Ke2 Hellsten,J very dangerous for White. worse afterwards.;
axb5 and now Black is Lopez Silva,H For the practical player it D) . But once you get
actually a piece up. It Pinamar 2004 would have been enough here, of course, you
is not really possible to [Jacob Aagaard] to calculate a few lines can stop and calculate
see any other moves and realize that the attack the consequences of
which are playable for XABCDEFGHY holds only chances for 28.Qd3 Rh2+ 29.Kg1
White. The threats are 8r+-+n+k+( Black, and then refrain gxf4!? , when White
simply too strong. ] 7+-+-wqr+p' 57 from 24 f4? in favour of has two ways of
26...Rd7!? This gives 6p+-+-+-+& something more cautious continuing:
White some extra options, 5+-zpPzpp+-% like 24 Re2. D1) 30.Rf1 Rh3 ( or
though they hardly matter. 4-+-+-+-+$ Now White panicked and 30...g5 31.gxf4 gxf4
[ 26...Rd2 would have 3tR-+-+-+-# lost like a stone. 26.Kg1?! 32.Rb3 Qg6 33.e5
been more natural, but 2-zPP+-+PzP" [ In order to decide on Qg5 34.Qf3 dxe5
then chess is difficult 1+-+QtRLmK-![ the double sacrifice, 35.Qxg4 Qxg4+
and this has not been an xabcdefghy however, Black would 36.Kxh2 Qe2+
easy game for Black to 22.d6! Very thematic; have also had to 37.Kg1 e4 38.Rh3
play either. ] White sees the possible calculate 26.Kxh3! Rh8+ f3 and Black wins )
27.Bc6?! pin on the c4-g8 diagonal 27.Kg2 Ng4 . This is the 31.Rbd1 Ne5
[ The best chance was and pushes the pawn to critical position. On 32.Rxf4 ( if 32.Qc3
27.a7! Rxb7 28.a8Q open up for the bishop. Of calculating the sacrifice f3 wins ) 32...Qxf4
Rxb2 29.Qf3 course Black had seen this, you would quickly 33.gxf4 Rxd3
transposing to the 26 c3 but he had not taken all realize this was the place 34.Rxd3 Nxd3
line in the 25 bxa6 note facets of the move into to stop and create your and Black should be
above. ] account. Then again, his stepping-stone position. able to win the
[ Instead 27.Qa4 loses to position was not that hot (26 Kg1 could probably endgame.;
Rd2 28.a7 Bxh4 anyway. Nxd6 be calculated first – and D2) 30.e5 Qg5!
29.a8Q Nxf1 because of [ If 22...Qxd6 23.Bc4 quickly, since it is not and if 31.Qf3 ( while
the brilliant tactic 30.Kxf1 and White wins the too difficult to see that 31.Qe4
Bf2!! and White is endgame easily. ] White is in a crisis; the is met strongly by f3!!
helpless to prevent the 23.Qd5 Rd8 exact details are not 32.Qxf3 Qd2
knight mate despite his [ 23...Re8 24.Rxa6 Ne4 important for the and White is finished;
two queens vs. two 25.Bc4 also wins for decision making and e.g. 33.Rbd1 Rh1+!!
rooks. ] White. ] could be disregarded.) 34.Qxh1 Qf2# )
27...Rd2! Black now plays 24.Rxe5 White now has: 31...Rf2 32.Qxg4
flawlessly in the finishing [ Black resigned because A) 28.Qa3 Rh2+ Qxg4 33.Kxf2
phase. 28.Rb1 of 24.Rxe5 Qc7 29.Kf1 Qd4 Qxg3+ 34.Kf1 dxe5
[ 28.a7 loses to Bxh4! 25.Qxc5 when he has no and White has no and Black has a
29.a8Q ( if 29.g4 hxg4 real chances of saving defence.; winning endgame.;
30.a8Q Bg3 31.Bg2 the endgame. Of course B) 28.Qf3 Rh2+ D3) 30.--;
Nxf1 and Black has a it was a bit early to 29.Kf1 Ne5! 30.Qe3 E) 28.-- ]
winning advantage, resign, but Black did ( if 30.Qc3 gxf4 26...gxf4 27.gxf4
mainly because of 32... not feel lucky. ] 31.Nd7 Qe6 32.Re2 [ 27.Qxf4 Nf3+
Bf2+! again ) 1-0 Rh1+ 33.Kf2 fxg3+ would leave Black with
A) though Black also 34.Qxg3 Rxb1 an extra pawn in the
wins after 29...Bf2+ and Black is winning ) endgame and White has
30.Kh2 ( or 30.Rxf2 Kirov,N 30...gxf4 very few chances of
Rd1+ 31.Kh2 Ng4+ Hillarp Persson,T B1) if 31.gxf4 Ng4 survival. ]
32.Kh3 Rh1# ) Saragossa 1995 32.Qg3 ( or 32.Qf3 27...Qh4 White resigned
30...Nxf1+ 31.Kh3 [Jacob Aagaard] Qh4 ) 32...Qd4 wins; because the attack is
Rxa8 32.Qxa8+ Kg7; B2) 31.Qxf4 Qd8!! lethal. He could perhaps
B) 29...Nxf1 (simplest) XABCDEFGHY 32.Kg1 Qh8 33.Nd7 have played on a few
30.Q8a7 N5e3 31.Bf3 8-+l+r+-+( Rh1+ 34.Kf2 Nd3+ moves, but the outcome
Bf2+ 32.Kh1 Ng3+ 7+p+-+pmk-' 58 35.Ke3 Nxf4 would have been the
33.Kh2 Nef1+ 34.Kh3 6-sNpzp-wqp+& 36.Rxh1 Nh5 same.
Bxa7 35.Qxa7 e5 5+-+-sn-zp-% and the endgame is 0-1
and White is 4-zPP+P+-+$ very close to winning
completely tangoed. ] 3+-+-wQ-zPP# for Black, if not
28...Bxh4 29.Bf3 Bf2+ 2r+-+-zPL+" simply winning.; Sturua,Z
30.Kh1 Ng4! Threatening 1+R+-tR-mK-![ C) 28.Qd3 Rh2+ Xu Jun
mate in one and thus xabcdefghy 29.Kg1 and here it is Istanbul Olympiad 2000
forcing White to open the 24.f4 Rxg2+!! It is hard to actually enough to see [Jacob Aagaard]
h-file. 31.Bxg4 hxg4 believe that Kirov did not that you have a way
32.c4? consider this move, but out, that is an 24.Bc3?!
[ But 32.a7 loses to somehow he did not make immediate draw with [ The fastest way to win
Rdd8 33.Ra1 Ra8 the effort to really Rh1+ 30.Kg2 Rh2+ the game was with
Excelling at Chess Calculation− Jacob Aagard 41

XABCDEFGHY 32.Bb4+ and so on.; on a pawn down. Afek,Y


8-+q+rvl-+( C) 25...-- ] [ White's ideas were: Kogan,A
7trpzp-+pmkL' 59 25.Nf6? 19...f5 20.Rxc8 Bxc8 Israel 1999
6-+-+-+p+& [ White again needed to 21.Qh5 h6 22.Ngxe4 [Jacob Aagaard]
5+P+-zp-+-% find 25.Bg8! winning a pawn. ]
4p+nzPN+P+$ , which would have won [ 19...Rxc1+ 20.Rxc1 f5 XABCDEFGHY
3zPl+-zP-+Q# fairly easily after Nxe4 21.Qh5 h6 22.Ngxe4 8r+-+-trk+(
2-vL-+-zP-zP" ( or 25...Kxg8 26.Nf6+ Qd5 23.Qg6! fxe4? 7+p+lvlpzpp' 62
1tR-+R+-mK-![ Kg7 and wins 27.dxe5 ) 24.Rc7 and wins. ] 6p+nzpp+-+&
xabcdefghy 26.Qh7+ Kf6 27.dxe5+ 20.Ngxe4 Bxe4 21.Nxe4 5wq-+-+-zPQ%
24.Bg8!! . After Kxg8 Ke7 ( or 27...Rxe5 and White won in 58 4-+-sNP+-+$
25.Nf6+ Kg7 26.dxe5 28.Qh8+ and wins ) moves with his extra pawn. 3+PsN-vL-+-#
Nxe5 ( if 26...Rxe5 27.f4 28.Bb4+ Nd6 29.exd6+ This looks pretty 2-zPP+-zP-zP"
Bd6 28.fxe5 Nxe5 cxd6 30.Rd4 convincing, but actually it 1+K+R+-tR-![
29.Rf1 wins ) 27.f4 and the attack is is less so. xabcdefghy
White is totally winning; decisive. Black could have saved 1.Bd2!! This is creative
e.g. Bd6 ( or 27...Kxf6 Reading Ftacnik's her position with some thinking. White sees the
28.Bxe5+ Rxe5 29.g5+! annotations it is easy combinational magic of her possibility for a
and the black queen to believe that the text own. combination and uses it to
hangs ) 28.Nxe8+ Qxe8 move should also win . create this little
29.b6!? ( 29.Rd2 against the best defence, (press F10 to continue) masterpiece. Black now
is also strong; , or 29.fxe5 but that is not the case. ] 1-0 has the choice of taking
Bxe5 30.Rd8 Qxd8 25...Nxb5 26.Bb2!? the knight or not. Maybe
31.Bxe5+ f6 32.Bd4 [ If 26.Bxg6 fxg6 Kogan thought White's
followed by 33 g5 or 33 27.Qh7+ Kxf6 28.dxe5+ analysis Beliavsky-Houska move was a blunder, and if
Rf1 with a winning Rxe5 29.g5+ Ke6 [Jacob Aagaard] he believed there was a
attack ) 29...Ra6 30.fxe5 and the position is piece going for free, he
Bxa3 31.Rxa3 Bxd1 quite unclear. Instead XABCDEFGHY should of course take it.
32.e6+ f6 33.Rd3 White played a quiet 8-+r+-trk+({ Nxd4? Black had to say
and White wins. move. ] 7+l+-+pzpp' 61 no to White's generous
Not an easy combination 26...Kxf6? Black fails to 6p+-wqp+-+& offer.
to work out, and in the find his way in this very 5sn-+-+-sN-% [ 1...Qc7 2.Nf5! exf5
game Sturua missed it, complicated position. 4Pzp-zPp+-+$ 3.Nd5 Qd8 4.Bc3
not once, but twice! ] [ The miraculous defence 3+-+-zP-+-# with a strong attack.
24...Nd6? was 26...Bxd1! 27.dxe5 2-zP-sNQzPPzP" After Ne5 5.Bxe5 dxe5
[ Black could have Bxa3!! 28.Rxa3 Rd8 1tR-tR-+-mK-! White continues with
offered more resistance 29.Bg8 Qe6! xabcdefghy another combination:
with 24...Bxd1!? and White's attack 19...Qd5! This move can 6.Nf6+ Bxf6 7.Rxd7!
25.Rxd1 seems to have been be thematically filed under Qb6! (the only move)
A) 25...Bxa3! 26.dxe5 repelled. This is far X-ray threats. It is not 8.gxf6 Qxf6 9.Qxf5!
Bb2 , trying to from being the only really a combination, but it (securing the rook on
neutralize the terrible variation, but I have requires a good feeling for the seventh rank) Qxf5
dark-squared bishop, not found any obvious the soft spots in the 10.exf5 Rfd8 11.Rgd1
when White has to way to improve the opponent's position in Rxd7 12.Rxd7 Rb8
play accurately in order attack. order to see such a move. 13.Re7 and White has
to win: 27.Rd7!! Now White wins without Now White has no way to the advantage in the
( but not 27.Qh4? too much effort. ] gain an advantage: 20.-- rook endgame. Black
Bxc3 28.Qf6+ Kh6 27.dxe5+ Ke6 28.g5+ [ 20.Ngxe4?? f5! ] now has the
29.Qh4+ Kg7 Ke7 29.e6 fxe6 30.Bf6+ [ and 20.Ndxe4?? f6! uncomfortable choice
with a draw – Ftacnik ) 1-0 both drop a piece, between f6 and losing
27...Qxd7 ( or 27...Re6 because of the X-ray the e-pawn. In both
28.g5 Kf8 29.Nf6 threat against g2. ] cases his chances of
and wins ) 28.e6+ Beliavsky,A [ 20.Qg4 Rxc1+ 21.Rxc1 survival are meagre. ]
Bxc3 29.exd7 Rd8 Houska,J Nb3 22.Nxb3 Qxb3 [ 1...Qd8! was probably
30.Nxc3 and White is Copenhagen 2003 and Black is not worse at the strongest reaction,
winning as there is [Jacob Aagaard] all; in fact it is more defending the key
no defence against 31 likely that she is better. ] squares on the kingside
g5.; XABCDEFGHY [ 20.Rxc8 Rxc8 21.Qh5 and preventing all
B) . Ftacnik also gave 8-+r+-trk+( Qf5 22.g4 Qg6 23.Qxg6 White's tricks with Nf5
25...Rd8 , but this does 7+l+-+pzpp' 60 hxg6 24.Ndxe4 f6!? ( or and Nd5. ]
not work either 6p+-wqp+-+& 24...Nb3 25.Rd1 Bxe4 2.Nd5 Qd8
because of 26.Bxg6!! 5sn-+p+-+-% 26.Nxe4 Rc2 27.d5 [ Objectively stronger was
( the idea was 26.Qh4 4Pzp-zPn+-+$ exd5 28.Rxd5 f6 2...Qb5 , but of course
Be7! 27.g5 Kf8 3+-+LzPN+-# with approximate this is a computer move,
28.Bxg6 fxg6 29.Qh8+ 2-zP-sNQzPPzP" equality ) 25.Nd6 fxg5 and White still wins after
and a draw by 1tR-tR-+-mK-![ 26.Nxc8 Bxc8 27.Rc1 3.g6! fxg6 4.Nxe7+ Kf7
perpetual check ) xabcdefghy Bb7 and the endgame is 5.Qxh7! with a decisive
26...fxg6 27.Ng5 White won in convincing truly complicated. ] attack. ]
with a winning attack; e. style after 18.Bxe4!? 3.Ba5!! This is the point of
g. Be7 28.Qh7+ Kf6 dxe4 19.Ng5 h6!? White's play.
29.Ne4+ Ke6 30.d5+ Black tries to find some [ Now 3.Ba5 Qe8
Kd7 31.Nf6+ Kd6 kind of defence and plays is met by 4.Nf6+!
Excelling at Chess Calculation− Jacob Aagard 42

and the black king 7...Kd5!! 8.Kb5 XABCDEFGHY with 26.f7+! Kxf7 27.0-0
cannot be defended. with a draw. 8-tr-+-+-mk( and he ends a piece up.
Therefore Black will have ½-½ 7+-+-wq-+P' 65 .
to lose the queen in one 6-+-+p+-+& (press F10 for the next
way or the other; in the 5+-+-zp-zp-% exercise)
game he allowed White Leko,P 4p+-+P+-+$
to carry out his main Khalifman,A 3+-+-wQ-+-#
idea. ] FIDE World Ch., New Delhi 2PzPr+-+-+" exercise 002
3...Qxa5 4.g6! With the [Jacob Aagaard] 1mK-+-+RtR-![ [Jacob Aagaard]
obvious threat of mate. xabcdefghy
fxg6 5.Nxe7+ Kf7 XABCDEFGHY [ 45.Rf8+? Qxf8 XABCDEFGHY
6.Qxa5 8-tr-+-+-mk( 46.Rg8+ Qxg8 8r+-+k+-+(
1-0 7+-+-wq-+P' 64 47.hxg8Q+ Kxg8 7zp-+-+pzpp' 68
6-+-+p+-+& gives White a draw at 6-+l+-+-+&
5+-+-zp-zp-% best. ] 5tR-+Lsn-+-%
Laveryd,P 4p+-+P+-+$ 45...Kxh7 46.Rb8!! 4-vL-+-+-+$
Wikstrom,P 3+-+-wQ-+-# A fabulous resource. 3+-+-+-+-#
Umea 1997 2PzPr+-+-+" Rxa2+ 2-zP-+-zPPzP"
[Jacob Aagaard] 1mK-+-+RtR-![ [ If 46...Rxb8 47.Qh3+ 1+-+-+-mK-![
xabcdefghy Kg7 48.Rg1+ xabcdefghy
XABCDEFGHY 44.Rb1? a3! 45.Qxa3 and White wins. ] .
8-+-+-+-+({ Qxa3 46.bxa3 Rxb1+! 47.Kb1 Rh2 48.Rf3!! White to play and win.
7+-+-+pzpp' 63 47.Kxb1 Rc4 48.Rxg5 Now Black has no (Note that the black king
6-+-+p+-+& Kxh7! defence against the white has already moved.)
5+-mk-zP-+-% [ 48...Rxe4 49.Rh5 attack. .
4-zpP+p+-+$ would give White very [ Not 48.Rff8?? Rhb2+ (press F10 for the solution)
3+K+-zP-+-# serious winning and Black is OK. ]
2-+-+-zPPzP" chances. ] 48...Rad2 49.Rh3+ Rxh3
1+-+-+-+-! 49.Rg4?! [ If 49...Kg7 50.Qh6+ solution 002
xabcdefghy [ 49.Rxe5 Kg6 50.Kb2! and mates. ] [Jacob Aagaard]
1...h5! This is the right ( if 50.Rxe6+ Kf7 50.Qxh3+ Kg7 51.Qh8+
move. 51.Re5 Kf6 and White Kf7 52.Qg8+ Kf6 XABCDEFGHY
[ Instead the game will lose his e-pawn and 53.Rf8+ and White wins. 8r+-+k+-+(
concluded: 1...h6? all chances of winning 7zp-+-+pzpp' 69
2.h3?? ( 2.g4! the game ) 50...Kf6 6-+l+-+-+&
would have won ) 2...h5 51.Kb3 Rd4 52.Rc5 exercise 001 5tR-+Lsn-+-%
3.h4 g5! 4.g3 ( if 4.hxg5 Rxe4 53.a4 was White's [Jacob Aagaard] 4-vL-+-+-+$
h4 and it is White who last chance to play for a 3+-+-+-+-#
has run out of moves ) win. I think Black XABCDEFGHY 2-zP-+-zPPzP"
4...g4 0-1. should draw this 8-+r+r+k+( 1+-+-+-mK-![
Now White has a lot of endgame, but it is hard 7zp-+-+-zp-' 66 xabcdefghy
moves, but only one that to be certain. ] 6Qzpq+-zP-zp& E.Mednis-H.Westerinen,
works! ] 49...Kh6 50.Kb2 Kh5 5+-+p+-+-% Szolnok 1975 (variation).
2.h3! 51.Kb3 Rd4 52.Rg2 4-zP-+-sn-vL$ .
[ 2.h4? g5! and Black Rxe4 53.a4 Rd4 54.a5 3zP-+-+-+-# In his analysis Mednis
wins. ] e4 55.a3 e5 Now it is just 2-+-+N+PzP" disregarded this position
[ 2.f4? exf3 3.gxf3 h4 a draw. 56.a4 e3 57.Re2 1+-+RmK-+R![ because he missed 1.Be4!
4.h3 f5 wins. ] Kg4 58.Rxe3 Kf4 59.Rh3 xabcdefghy winning a piece and the
[ 2.g3? f5 3.exf6 gxf6 e4 60.Rh8 Rd6 61.Re8 . game. It is important that
4.h3 f5 5.h4 e5 e3 62.Kb4 Kf3 63.Kb5 White to play and win. the black king had
and Black wins. ] Rd5+ 64.Kb6 Rd4 . . previously moved (to e7
2...f6! In his analysis Leko came (press F10 for the solution) and back), otherwise 1
[ Not 2...h4? 3.g3 g5 up with this marvellous Be4 could be answered by
4.g4! and White wins. ] finish: 1...0-0-0!.
3.h4! . solution 001 .
[ Not 3.exf6? gxf6 (press F10 to continue) [Jacob Aagaard] (press F10 for the next
and Black wins. ] ½-½ exercise)
3...fxe5! XABCDEFGHY
[ If 3...f5? 4.f4 ( but not 8-+r+r+k+(
4.f3?? f4 and Black analysis Leko-Khalifman 7zp-+-+-zp-' 67 exercise 003
wins ) 4...exf3 5.gxf3 f4 [Jacob Aagaard] 6Qzpq+-zP-zp& [Jacob Aagaard]
6.exf4 g6 7.f5! gxf5 5+-+p+-+-%
8.f4 and White wins. ] 4-zP-+-sn-vL$
4.g4! g6 (Diagram 65) 3zP-+-+-+-# (Diagram 70)
[ Not 4...hxg4? 5.h5 2-+-+N+PzP"
and White wins. ] 44.Rxg5!! Rbxb2 1+-+RmK-+R![ .
5.g5 Kb6!! 6.Kxb4 Kc6 [ 44...a3 45.Rg8+ Rxg8 xabcdefghy White to play and win.
7.c5 46.hxg8Q+ Kxg8 K.Sasikiran-M.Venkatesh, .
[ If 7.Kb3 Kc5 8.Kc3 47.Qg3+ Qg7 48.Qxa3 Commonwealth Ch., (press F10 for the solution)
Kb6 9.Kb4 Kc6 also leaves White with Mumbai 2004 (variation).
and White makes no a winning position. ] .
progress. ] 45.Rg8+ White wins immediately
Excelling at Chess Calculation− Jacob Aagard 43

XABCDEFGHY XABCDEFGHY XABCDEFGHY exercise 007


8-trlwq-trk+( 8-+rwq-tr-+({ 8-+r+-+-+( [Jacob Aagaard]
7zp-+-+p+-' 70 7+-+l+P+k' 73 7zp-tr-+lmkp' 76
6-+-+-vlp+& 6-+-zp-snNvl& 6-+-+-zp-+& XABCDEFGHY
5+-zp-+-+p% 5+p+-+-+-% 5+-+p+-zp-% 8-+n+-+-+({
4-+PzpN+-+$ 4p+-zpP+-+$ 4R+-sN-+-+$ 7+P+p+-+-' 78
3+-vL-+-zPQ# 3+L+-+P+-# 3+P+-+-+-# 6-+-zP-+-+&
2P+-+-zPLzP" 2PzPPwQ-+P+" 2P+-mK-zPPzP" 5+-+-zP-+-%
1tR-+R+-mK-![ 1+-mK-+-+R! 1+-+-tR-+-![ 4-+-zP-+-+$
xabcdefghy xabcdefghy xabcdefghy 3vL-+-+-zp-#
do so is to create an . 2pzp-+-+P+"
escape hatch by (press F10 for the solution) 1sNk+-+-+K!
solution 003 21...Rxc2+! 22.Bxc2 xabcdefghy
[Jacob Aagaard] [ or 22.Kxc2 axb3+ .
and Black can again solution 006 Black to play and draw.
XABCDEFGHY escape via f7 ] [Jacob Aagaard] .
8-trlwq-trk+( 22...Kxg6 23.Rxh6+ Kxf7 (press F10 for the solution)
7zp-+-+p+-' 71 and the white attack does XABCDEFGHY
6-+-+-vlp+& not seem that dangerous. 8-+r+-+-+(
5+-zp-+-+p% . 7zp-tr-+lmkp' 77 solution 007
4-+PzpN+-+$ (press F10 for the next 6-+-+-zp-+& [Jacob Aagaard]
3+-vL-+-zPQ# exercise) 5+-+p+-zp-%
2P+-+-zPLzP" 4R+-sN-+-+$ XABCDEFGHY
1tR-+R+-mK-![ 3+P+-+-+-# 8-+n+-+-+({
xabcdefghy exercise 005 2P+-mK-zPPzP" 7+P+p+-+-' 79
S.Ovsejevitsch-J.Aagaard, [Jacob Aagaard] 1+-+-tR-+-![ 6-+-zP-+-+&
Budapest 2003 (variation). xabcdefghy 5+-+-zP-+-%
. XABCDEFGHY L.Polugaevsky-Z. 4-+-zP-+-+$
I had considered entering 8r+-tr-+k+( Azmaiparashvili, 3vL-+-+-zp-#
this position but, as 7zpQ+-zppvl-' 74 Reykjavik 1990. 2pzp-+-+P+"
Ovsejevitsch pointed out, 6-+-+-wq-zp& . 1sNk+-+-+K!
White has the strong 5zP-+n+-zp-% White needs to penetrate xabcdefghy
desperado 22.Ba5!! Qe7 4-+-+-+L+$ the black position without End of study by G.De Rossi,
[ 22...Bxh3 23.Bxd8 ] 3+-+-+-+P# allowing ...Rcc2+. This is 2nd honorary mention, La
23.Nxf6+ Qxf6 24.Qh4 2-zP-+-zPPvL" achieved by simple tactics. Strategie 1912.
and White keeps the extra 1+-tR-tR-mK-![ 27.Re6! Probably the most .
piece. xabcdefghy difficult thing is to see the 1...bxa1R! threatens ...
. . strength of this move. Kc2+ and mates. So White
(press F10 for the next White to play and win. Bxe6 is forced to play 2.bxc8Q
exercise) . [ Black cannot defend. If and Black is stalemated
(press F10 for the solution) 27...Kg6 28.Rd6! (showing why 1...bxa1Q
followed by 29 Raa6 with was no good).
exercise 004 a winning advantage ] .
[Jacob Aagaard] solution 005 [ or 27...Rc2+ 28.Nxc2 (press F10 for the next
[Jacob Aagaard] Bxe6 29.Rxa7+ exercise)
XABCDEFGHY and White wins. ]
8-+rwq-tr-+({ XABCDEFGHY 28.Nxe6+ Kg6 29.Nxc7
7+-+l+P+k' 72 8r+-tr-+k+( Rxc7 30.Ra5 Rd7 31.g4! exercise 008
6-+-zp-snNvl& 7zpQ+-zppvl-' 75 Curtailing Black's kingside [Jacob Aagaard]
5+p+-+-+-% 6-+-+-wq-zp& counterplay, thus enabling
4p+-zpP+-+$ 5zP-+n+-zp-% White to win on the XABCDEFGHY
3+L+-+P+-# 4-+-+-+L+$ queenside. The game 8r+-+-trk+(
2PzPPwQ-+P+" 3+-+-+-+P# concluded: h5 32.h3 h4 7zpp+-zppvlp' 80
1+-mK-+-+R! 2-zP-+-zPPvL" 33.b4 Kf7 34.Ra6 Rc7 6-+nzp-+p+&
xabcdefghy 1+-tR-tR-mK-![ 35.b5 d4 36.a4 Ke7 5wq-zp-+l+-%
. xabcdefghy 37.a5 Rb7 38.b6 axb6 4-+PzPn+-+$
Black to play and win. J.Aagaard-J.Østergaard, 39.Rxb6 Rc7 40.Rb3 3+PsN-zPN+-#
. Helsingør 2003 (variation). [ 40.Rb4! was even 2PvL-+LzPPzP"
(press F10 for the solution) . stronger according to 1+-tRQ+RmK-![
White should play 26.Bc8! Polugaevsky. ] xabcdefghy
and Black will lose the 40...f5 41.f3 Ke6 42.a6 .
solution 004 exchange. Ra7 43.Ra3 Ke5 44.Kd3 White to play and win.
[Jacob Aagaard] . Kf4 45.Kxd4 Kg3 .
(press F10 for the next 46.gxf5 Kxh3 47.Kc5! g4 (press F10 for the solution)
exercise) 48.Kb6 Ra8 49.Kb7 Rf8
(Diagram 73) 50.a7 g3 51.a8Q Rxa8
52.Rxa8 g2 53.Rg8 solution 008
Y.Berthelot-E.Prie, Nantes exercise 006 and Black resigned. [Jacob Aagaard]
2003 (variation). [Jacob Aagaard] .
. (press F10 for the next V.Epishin-A.Volokitin,
The black king needs to . exercise) Copenhagen 2002.
decamp. The only way to White to play and win. .
Excelling at Chess Calculation− Jacob Aagard 44

XABCDEFGHY . Unfortunately, this line XABCDEFGHY solution 012


8r+-+-trk+( was refuted by Murray 8-+-+-+-vl( [Jacob Aagaard]
7zpp+-zppvlp' 81 Chandler, whose Fritz 7+-trNwqp+k' 85
6-+nzp-+p+& computer found Bd1!! 6p+-+-+-zp& XABCDEFGHY
5wq-zp-+l+-% [ Here Adams had 5+p+-+-+-% 8-+r+rvlk+(
4-+PzPn+-+$ analysed 8...Qg5? 4-+-+nsN-zP$ 7zp-zp-+pzp-' 88
3+PsN-zPN+-# 9.Nxf3! Qxg2 10.Ke2!! 3zP-+QzP-+-# 6-zp-+p+-zp&
2PvL-+LzPPzP" Qxh1 11.d5 2-zP-+-+R+" 5sn-+-sNq+-%
1+-tRQ+RmK-![ with a dangerous attack 1+K+-+-+-![ 4l+PzPQvL-zP$
xabcdefghy for White. ] xabcdefghy 3+-zPL+-zP-#
Here the two 9.Qxd1 Qxg2 and White . 2P+-+-zP-+"
grandmasters overlooked has no compensation for (press F10 for the solution) 1+R+-tR-mK-![
a simple winning move. the pawn. xabcdefghy
11.Bd3?! This is also quite . R.Filguth-A.De la Garza,
strong, so if this was your (press F10 for the next solution 011 World Student Team Ch.,
choice, you can take 3 exercise) [Jacob Aagaard] Mexico 1980.
points as consolation. .
[ But much better was XABCDEFGHY White wins with the
11.g4! winning a piece exercise 010 8-+-+-+-vl( surprising 1.Qh1!!
after Bxg4 ( or 11...Nxc3 [Jacob Aagaard] 7+-trNwqp+k' 86 [ and Black resigned due
12.Bxc3; , or if 11...Nxf2 6p+-+-+-zp& to 1.Qh1 Qf6 2.Bg5!
12.Rxf2 Bxg4 13.d5 XABCDEFGHY 5+p+-+-+-% hxg5 3.hxg5 winning the
Nb4 14.Bf1 and Black 8r+-wqr+k+({ 4-+-+nsN-zP$ queen or mating.
has nothing ) 12.Nxe4 . ] 7zppzp-+pvlp' 83 3zP-+QzP-+-# .
11...Nxc3 12.Bxc3 Qa3? 6-+-zp-+p+& 2-zP-+-+R+" (press F10 for the next
[ Preferable was 12...Qd8 5+-+-+-+-% 1+K+-+-+-![ exercise) ]
with the better game for 4-+Psnl+-+$ xabcdefghy
White. ] 3+PsN-sN-zP-# J.Timman-R.Hübner, Wijk
13.Bxf5 gxf5 14.Rc2! 2PvL-wQPzP-zP" aan Zee 1982. exercise 013
Rad8 15.d5 Nb4 1tR-+-+RmK-! . [Jacob Aagaard]
16.Bxg7 Kxg7 17.Rd2 xabcdefghy Although White has many
Qa5 18.e4 fxe4 19.Ng5 . strong moves, he has only XABCDEFGHY
f5 20.Ne6+ Kh8 21.f3 Black to play and win. one which wins on the 8r+r+-+k+(
Rf6 22.Nxd8 Qxd8 . spot: 40.Ne6! and Black 7+p+lzppvlp' 89
23.fxe4 fxe4 24.Rxf6 (press F10 for the solution) must resign, 6pvLq+-+p+&
exf6 25.Qg4 and White [ since if 40.Ne6 Qxe6 5sn-+-+-+-%
won. 41.Nf8# . ] 4-+-+PzP-+$
. solution 010 [ Also possible was 3+-zPL+-+-#
(press F10 for the next [Jacob Aagaard] 40.Nf6+ Bxf6 41.Nd5 2P+-+N+PzP"
exercise) winning the queen after 1+R+Q+RmK-![
XABCDEFGHY Qe6 42.Nxf6+ Qxf6 xabcdefghy
8r+-wqr+k+({ 43.Qxe4+ Qg6 ( since .
exercise 009 7zppzp-+pvlp' 84 43...Kh8? 44.Qe8+ White to play and win.
[Jacob Aagaard] 6-+-zp-+p+& mates. If this was your .
5+-+-+-+-% choice take three (press F10 for the solution)
XABCDEFGHY 4-+Psnl+-+$ points.
8r+-+kvl-tr({ 3+PsN-sN-zP-# .
7zpp+-zppzpp' 82 2PvL-wQPzP-zP" (press F10 for the next solution 013
6-+n+-sn-+& 1tR-+-+RmK-! exercise) )] [Jacob Aagaard]
5+-zpq+-+-% xabcdefghy
4Q+LzP-+-+$ Cu.Hansen-V.Akopian, XABCDEFGHY
3+-zP-+l+-# Groningen 1991 exercise 012 8r+r+-+k+(
2PzP-sN-zPPzP" (variation). [Jacob Aagaard] 7+p+lzppvlp' 90
1tR-vL-mK-+R! . 6pvLq+-+p+&
xabcdefghy Here Black wins with a XABCDEFGHY 5sn-+-+-+-%
. simple tactic: 14...Bf3!! 8-+r+rvlk+( 4-+-+PzP-+$
Black to play and win. 15.Qd3 7zp-zp-+pzp-' 87 3+-zPL+-+-#
. [ 15.exf3?? Nxf3+ 6-zp-+p+-zp& 2P+-+N+PzP"
(press F10 for the solution) wins the queen. ] 5sn-+-sNq+-% 1+R+Q+RmK-![
15...Nxe2+ 16.Nxe2 Bxb2 4l+PzPQvL-zP$ xabcdefghy
and Black is a pawn up for 3+-zPL+-zP-# P.H.Nielsen-V.Ivanchuk,
solution 009 nothing. 2P+-+-zP-+" Malmö 2003 (variation).
[Jacob Aagaard] . 1+R+-tR-mK-![ .
(press F10 for the next xabcdefghy This position (after ...Rd8-
J.M.Degraeve-M.Palac, exercise) . c8) did not arise in the
Montpellier 1997. White to play and win. game as White would then
. . have a very strong move:
The position arose after exercise 011 (press F10 for the solution) 17.Rf2! Threatening 18
the moves 1.e4 c5 2.c3 [Jacob Aagaard] Bxa5 and the black knight
d5 3.exd5 Qxd5 4.d4 cannot go to c4 because
Nf6 5.Nf3 Bg4 6.Nbd2 . the bishop on d7 is
Nc6 7.Bc4 Bxf3 8.Qa4 White to play and win. hanging.
Excelling at Chess Calculation− Jacob Aagard 45

[ White cannot win a 32.Nb6 Be2! 33.Rc5 move exercises I have resigned.
piece immediately ( not 33.Na8? Rc8 ever seen: 34.Rd6!! .
because of 17.Bxa5 34.c7 Bc4 35.Rd1 Bd5 threatening 35 Qa8 mate, (press F10 for the next
Qc5+ ] and Black wins ) 33...b4! [ and Black resigned exercise)
[ while after 17.Kh1 Nc4! 34.axb4 Bb5 since if 34.Rd6 cxd6
18.Bxc4 Qxc4 with a likely draw. But 35.Qc6# .
the knight on e2 would White can first improve . exercise 017
also be hanging. ] his position. ] (press F10 for the next [Jacob Aagaard]
17...Bxc3 The only move. [ In the game White exercise) ]
18.Nxc3 Qxc3 19.Bf1 played 30.Rxc7 Rxc7 XABCDEFGHY
Be8 20.Rf3 Qc6 21.Rb4! 31.Nd6 Be2 32.Nb7 Kf8 8-+-+Q+-+(
when the knight is trapped 33.f4 Bc4 34.Na5 exercise 016 7+-vl-+-+r' 97
out on the rim to be with the advantage, but [Jacob Aagaard] 6-+-+-+-vL&
picked off by 22 Ra3 (for might not have won had 5+-+-+-tR-%
example). Black continued Bd5 XABCDEFGHY 4-+-+-mk-+$
This is computer analysis, 35.Kf2 Ke8 36.Ke3 Kd8 8-+r+-+r+( 3+-+-+-+-#
but we can be certain that 37.Kd4 Rc8 38.Kc5 Kc7 7+p+nmkp+-' 95 2-+-+-+-mK"
White knew and .] 6-+l+p+p+& 1+-+-+q+-![
remembered it all. The 30...Bxg4 5zp-+pzP-+p% xabcdefghy
practical player would [ If 30...Rxd7 31.cxd7 4P+-sN-zP-zP$ .
need to work out the lines Rxc1+ 32.Kg2 Rd1 3+PtR-+-+R# White to play and draw.
himself. Of course 33.Nd6 ] 2-mKP+L+P+" .
Ivanchuk knew all this as [ or 30...Bg6 31.Nd6! 1+-+-+-+-![ (press F10 for the solution)
well and, instead of 16... and White wins. ] xabcdefghy
Rdc8, played 16...Bg4! 31.Nd6 Kf8 .
eventually obtaining a [ If 31...Rxd7 32.cxd7 White to play and win. solution 017
draw from an inferior Rd8 33.Rc8 and White . [Jacob Aagaard]
position. wins. ] (press F10 for the solution)
. 32.Nxc8 Rxc8 33.Ra7 XABCDEFGHY
(press F10 for the next with a clearly won position. 8-+-+Q+-+(
exercise) . solution 016 7+-vl-+-+r' 98
(press F10 for the next [Jacob Aagaard] 6-+-+-+-vL&
exercise) 5+-+-+-tR-%
exercise 014 XABCDEFGHY 4-+-+-mk-+$
[Jacob Aagaard] 8-+r+-+r+( 3+-+-+-+-#
exercise 015 7+p+nmkp+-' 96 2-+-+-+-mK"
XABCDEFGHY [Jacob Aagaard] 6-+l+p+p+& 1+-+-+q+-![
8-+r+-+k+( 5zp-+pzP-+p% xabcdefghy
7+-trR+pzpp' 91 XABCDEFGHY 4P+-sN-zP-zP$ Study by B.Gurgenidze,
6p+P+p+-+& 8-+ktr-+-+( 3+PtR-+-+R# 1st prize, Polish Chess
5+p+-zP-+l% 7zp-zpN+p+p' 93 2-mKP+L+P+" Federation TY 1985.
4-+-+N+-+$ 6-zpR+-+-+& 1+-+-+-+-![ .
3zP-+-+-+-# 5+-+-+-+-% xabcdefghy This brilliant study is
2-+-+-zPPzP" 4-+-zPQ+-+$ P.Leko-D.Ruzele, about stalemate: 1.Qe3+!!
1+-tR-+-mK-![ 3+-+-+-zP-# European Team Ch., Kxe3+ 2.Rg3+!
xabcdefghy 2l+-wq-+PmK" Debrecen 1992. and no matter what, White
. 1+-+-+-+-![ . is stalemated on the next
White to play and win. xabcdefghy White wins a pawn with a move.
. . stunning move: 31.Ba6!! .
(press F10 for the solution) White to play and win. After this White still needs (press F10 for the next
. to show some accuracy in exercise)
(press F10 for the solution) order to gain the full point,
solution 014 but the main work has
[Jacob Aagaard] been done. f6 exercise 018
solution 015 [ If 31...Nb8 32.Nxc6+ [Jacob Aagaard]
XABCDEFGHY [Jacob Aagaard] Rxc6 33.Bxb7 Rxc3
8-+r+-+k+( 34.Rxc3 Kd7 35.Ka3 XABCDEFGHY
7+-trR+pzpp' 92 XABCDEFGHY followed by b3-b4 and 8-+-+-+-mk(
6p+P+p+-+& 8-+ktr-+-+( White wins – Leko. ] 7+psn-+-+p' 99
5+p+-zP-+l% 7zp-zpN+p+p' 94 32.exf6+ Nxf6 33.Nxc6+ 6p+q+-vlp+&
4-+-+N+-+$ 6-zpR+-+-+& Rxc6 34.Bxb7 Rb6 5+-+-+-+-%
3zP-+-+-+-# 5+-+-+-+-% 35.Rc7+ Kd6 36.Rhc3 4P+-vL-+-+$
2-+-+-zPPzP" 4-+-zPQ+-+$ Ne4 37.R3c6+ Rxc6 3+L+-+-+-#
1+-tR-+-mK-![ 3+-+-+-zP-# 38.Rxc6+ Kd7 39.Ra6! 2-+-+QzPPzP"
xabcdefghy 2l+-wq-+PmK" Kc7 40.Ba8 Rf8 41.c4! 1+-+-+-mK-![
A.Abreu Delgado-M. 1+-+-+-+-![ Rxf4 42.cxd5 e5 xabcdefghy
Podgaets, Havana 1999. xabcdefghy 43.Rxa5 Rxh4 44.d6+! .
. J.Van der Wiel-N.Short, Nxd6 45.Rxe5 Kb8 White to play and win.
30.g4!! Biel Interzonal 1985. 46.Bd5 Rg4 47.a5 Rd4 .
[ White would love to play . 48.Bf3 Nf5 49.Kc3 Rf4 (press F10 for the solution)
30.Nd6 , but Black has This legendary finish is 50.Re4 Rh4 51.Rxh4
Rxd7! 31.Nxc8 Rc7 one of the greatest one Nxh4 52.Be4 and Black
Excelling at Chess Calculation− Jacob Aagard 46

solution 018 [ If instead 22...Bf5 45.g3 Rxd1 46.Qd8+ .


[Jacob Aagaard] 23.Bc1 and White is Rxd8 and White resigned. (press F10 for the next
fine. ] Thanks to Mark Dvoretsky exercise) ]
XABCDEFGHY 23.Nxd5 for showing me this
8-+-+-+-mk( [ Or 23.b4 Ba6 24.Qc6 exercise.
7+psn-+-+p' 100 Rc8 25.Qxd5 Bxf1 . exercise 022
6p+q+-vlp+& 26.Rxf1 Ne6 and Black (press F10 for next [Jacob Aagaard]
5+-+-+-+-% wins. ] exercise)
4P+-vL-+-+$ 23...Ba6 24.Qxb6 Bxf1 XABCDEFGHY
3+L+-+-+-# 25.Rxf1 Rxa2 26.Qxd8 8-+-+rtr-mk({
2-+-+QzPPzP" Rxd8 and Black won. exercise 021 7zp-+l+-+p' 107
1+-+-+-mK-![ . [Jacob Aagaard] 6-+-+-+pwq&
xabcdefghy (press F10 for the next 5+-+-sN-+-%
Riga vs. Orel, Intercity exercise) XABCDEFGHY 4-+-zPp+-+$
correspondence game 8-+-+rtr-mk({ 3+-+-zP-zPn#
1896-98. 7+-zp-+-zpp' 105 2P+Q+-+KzP"
. exercise 020 6-zp-zp-sn-wq& 1+R+-tR-+L!
1.Bb2!! intends 2 Qe7 [Jacob Aagaard] 5+N+-+-vl-% xabcdefghy
and wins. The city of Orel 4-+PzPpzp-+$ .
decided to resign since XABCDEFGHY 3+-vL-+PzP-# Black to play and win.
there is no defence; 8-wq-+lmk-+({ 2PzPQ+P+KzP" .
[ for instance if 1.Bb2 h6 7+-+-+pzp-' 103 1+-+RtR-+-! (press F10 for the solution)
( 1...Ne8 2.Qxe8+! Qxe8 6R+-+p+-zp& xabcdefghy
3.Bxf6# is mate ) 2.Qc4! 5+-+-+-+-% .
(threatening Qg8 mate) 4-+-+-+-+$ Black to play and win. solution 022
Qxc4 3.Bxf6+ Kh7 3+-+-+-+P# . [Jacob Aagaard]
4.Bxc4 wins a piece. 2P+-wQ-zPP+" (press F10 for the solution)
. 1+r+N+-mK-! XABCDEFGHY
(press F10 for the next xabcdefghy 8-+-+rtr-mk({
exercise) ] . solution 021 7zp-+l+-+p' 108
Black to play and win. [Jacob Aagaard] 6-+-+-+pwq&
. 5+-+-sN-+-%
exercise 019 (press F10 for the solution) XABCDEFGHY 4-+-zPp+-+$
[Jacob Aagaard] 8-+-+rtr-mk({ 3+-+-zP-zPn#
7+-zp-+-zpp' 106 2P+Q+-+KzP"
XABCDEFGHY solution 020 6-zp-zp-sn-wq& 1+R+-tR-+L!
8r+-wqr+k+({ [Jacob Aagaard] 5+N+-+-vl-% xabcdefghy
7+-+-+pzp-' 101 4-+PzPpzp-+$ L.Koerholz-L.H.Hansen,
6-zp-vll+-zp& XABCDEFGHY 3+-vL-+PzP-# Taastrup 2002.
5+Qsnp+-+-% 8-wq-+lmk-+({ 2PzPQ+P+KzP" .
4-+-+-+-+$ 7+-+-+pzp-' 104 1+-+RtR-+-! The winning combination
3+-sN-zPN+-# 6R+-+p+-zp& xabcdefghy was 26...Rxe5!
2PzP-vL-+PzP" 5+-+-+-+-% K.Bolding-J.Aagaard, [ In a winning position
1+-+R+R+K! 4-+-+-+-+$ Danish Junior Black blundered with
xabcdefghy 3+-+-+-+P# Championship 1992. 26...Rf2+?? losing the
. 2P+-wQ-zPP+" . game in one move to
Black to play and win. 1+r+N+-mK-! Black has two different 27.Qxf2! (2 points). ]
. xabcdefghy ways of executing the 27.dxe5 Bg4 (4 points)
(press F10 for the solution) P.Leko-V.Bologan, combination, exploiting his 28.Rf1 Bf3+ 29.Rxf3
Dortmund 2003. great advantage in exf3+ 30.Kf1 Qxe3
. mobility on the kingside. (4 points) and White has
solution 019 This brilliant finish by 24...Ng4! no defence.
[Jacob Aagaard] Bologan shows his [ 24...fxg3!? 25.hxg3 Maximum score: 8 points.
enormous strength. A Ng4! (8 points) 26.fxg4 .
XABCDEFGHY computer will produce the Rf2+ etc (2 points) is (press F10 for the next
8r+-wqr+k+({ solution almost another way of executing exercise)
7+-+-+pzp-' 102 immediately but it is a the same combination.
6-zp-vll+-zp& hard line for a human to Neither variation is
5+Qsnp+-+-% find. 41...Qe5!! better than the other. ] exercise 023
4-+-+-+-+$ (5 points) With the threat 25.fxg4 fxg3 (8 points) [Jacob Aagaard]
3+-sN-zPN+-# of 42...Qd5. White cannot was the finish in the game.
2PzP-vL-+PzP" allow this and therefore [ White resigned because
1+-+R+R+K! plays: 42.Ra3 of 25...fxg3 26.hxg3 (Diagram 109)
xabcdefghy Now the second half of the Rf2+! (very thematic)
M.Illescas Cordoba-A. exercise was to find: Qg5! 27.Kxf2 Qh2+ 28.Kf1 .
Morozevich, Pamplona (5 points) White loses a Rf8# (2 points). White to play and win.
1998. piece no matter what. Maximum score for this .
. 43.Re3 exercise: 10 points. (press F10 for the solution)
22...Bc8!! After this White [ If 43.Qxg5 Rxd1+ ] You cannot have 20
loses the exchange more [ or 43.f4 Qc5+ 44.Re3 points for seeing both
or less for nothing and Rxd1+ . ] routes to the same
therefore also the game. 43...Ba4 44.Kh2 Qf4+ solution.
Excelling at Chess Calculation− Jacob Aagard 47

XABCDEFGHY XABCDEFGHY XABCDEFGHY 42.Rxh1 Qxh1


8-+r+kvl-tr( 8-+l+r+k+( 8r+l+kvl-+( is very unsound; White
7zp-+p+pzpp' 109 7+-tr-+-+p' 111 7+-+-+p+-' 114 should count himself
6-+-+p+n+& 6pzp-wq-zppwQ& 6pwqpzpp+-+& lucky that after 43.b4
5+-zp-tR-vL-% 5+-+p+-+-% 5+-+nzP-vL-% cxb4 44.Qxb4 Qa1+
4-+P+-+-+$ 4-+-zP-+-+$ 4-+-+N+-+$ 45.Kb3 Qd1+
3+-wqL+-wQ-# 3+-zP-sN-+P# 3+-+L+-+-# Black only has perpetual
2P+-+-zPPzP" 2P+-+-zPP+" 2PzPP+-+Ptr" check. ]
1+-+R+-mK-![ 1+-tR-tR-mK-![ 1tR-+QmK-+R![ 41...h1Q 42.Ne8+!
xabcdefghy xabcdefghy xabcdefghy This is the point of the last
but once you set this up as move.
a combination people can [ 42.Rxh1 Qxh1
solution 023 solution 024 turn crazy. Here the point 43.Ne8+! transposes. ]
[Jacob Aagaard] [Jacob Aagaard] is of course to see the 42...Kh8 43.Rxh1 Qxh1
White's 17th move. 44.Nxf6 This position is
XABCDEFGHY XABCDEFGHY Clearly Black failed to do absolutely winning for
8-+r+kvl-tr( 8-+l+r+k+( so when playing 14... White. Rd8 45.Qb2
7zp-+p+pzpp' 110 7+-tr-+-+p' 112 Rxh2??. Qg1+ 16.Bf1 Many other moves wins as
6-+-+p+n+& 6pzp-wq-zppwQ& Qxh2 17.Qxd5! well here, but 45 Qb2 is
5+-zp-tR-vL-% 5+-+p+-+-% (5 points) Winning a piece. the most human solution.
4-+P+-+-+$ 4-+-zP-+-+$ exd5 18.Nf6+ Ke7 Kg7 46.Qe5
3+-wqL+-wQ-# 3+-zP-sN-+P# 19.Ng4+ Ke6 20.Nxh2 [ Here 46.Ne8+! Kf8
2P+-+-zPPzP" 2P+-+-zPP+" White has a winning 47.Qg7+ Ke7 48.Qf6+
1+-+R+-mK-![ 1+-tR-tR-mK-![ endgame. The game Kf8 49.Qxd8
xabcdefghy xabcdefghy concluded: dxe5 21.Be2 was simplest. ]
F.Gomez-Jos.Alvarez, C.Høi-C.Rossen, f6 22.Bd2 Bd6 23.0-0-0 46...Qd1 47.Ne4+ Kf8
Cuba 1998. Copenhagen 2003. e4 24.Rh1 d4 25.Ng4 e3 48.Qxc5+ Kg7 49.Qe5+
. . 26.Ba5 Bf4 27.Nxf6 Kf8 50.Nd6 Qd2+
White has invested a pawn This is basic pattern and Black resigned. 51.Ka3 Ra8 52.Qe8+
to gain a lead in training. 26.Nxd5! Maximum score: 10 points. Kg7 53.Qxa8 Qxd6+
development and can now (4 points) . 54.Kb2 Qd2+ 55.Ka3
win the pawn back. [ Black resigned because (press F10 for the next Qd6+ 56.b4 Qd3+
However, there is a much of 26.Nxd5 Rxe1+ exercise) 57.Ka4 Qxd7+ 58.b5
stronger continuation 27.Rxe1 Qxd5 28.Re8+ Qd1+ 59.Kb4 Qd2+
based on the black king Kf7 29.Qf8# (4 points). 60.Kc5 Qe3+ 61.Kd6
trapped in the centre, the Maximum score: 8 exercise 026 Qxf4+ 62.Kd7 Qxc4
unprotected queen and a points. [Jacob Aagaard] 63.Qc6 and White won.
change of move order. . Maximum score: 10 points.
1.Rxe6+!! (10 points) (press F10 for the next XABCDEFGHY .
[ 1.Bxg6 Qxg3 2.Bxf7+ exercise) ] 8r+-+-+-+( (press F10 for the next
would just regain the 7+-sN-+pmk-' 115 exercise)
pawn. ] 6-+-zP-vlp+&
1...dxe6 exercise 025 5zP-zp-+-+q%
[ Black cannot play [Jacob Aagaard] 4-+P+-zP-+$ exercise 027
1...fxe6 because of 3+P+-+P+-# [Jacob Aagaard]
2.Bxg6+ and White wins XABCDEFGHY 2K+-wQ-+-zp"
the queen (3 points). ] 8r+l+kvl-+( 1+-+-+R+-![ XABCDEFGHY
2.Be4! (3 points) 7+-+-+p+-' 113 xabcdefghy 8-+-+nmk-+(
[ Black resigned because 6pwqpzpp+-+& . 7wq-+-+p+-' 117
of 2.Be4 Qxg3 ( 2...Qa5 5+-+nzP-vL-% White to play and win. 6p+Qzp-+P+&
3.Bc6+ Rxc6 4.Qb8+ 4-+-+N+-+$ . 5+-+Psn-zPp%
also leads to mate ) 3+-+L+-+-# (press F10 for the solution) 4-zPPzp-+-+$
3.Bc6+! Rxc6 4.Rd8# . 2PzPP+-+Ptr" 3+-+L+-+P#
Maximum score: 16 1tR-+QmK-+R![ 2-+-vL-+K+"
points. xabcdefghy solution 026 1+-+-+-+-![
. . [Jacob Aagaard] xabcdefghy
(press F10 for the next White to play and win. .
exercise) ] . XABCDEFGHY White to play and win.
(press F10 for the solution) 8r+-+-+-+( .
7+-sN-+pmk-' 116 (press F10 for the solution)
exercise 024 6-+-zP-vlp+&
[Jacob Aagaard] solution 025 5zP-zp-+-+q%
[Jacob Aagaard] 4-+P+-zP-+$ solution 027
3+P+-+P+-# [Jacob Aagaard]
(Diagram 111) 2K+-wQ-+-zp"
(Diagram 114) 1+-+-+R+-![
. xabcdefghy (Diagram 118)
White to play and win. N.Castaneda-W.Browne, J.Egger-R.Dive, Istanbul
. Philadelphia 1997. Olympiad 2000. E.Lund-F.Frink, Budapest
(press F10 for the solution) . . 2002 (variation).
15.Rxh2! (5 points) 41.d7!! .
Actually the only move, [ 41.Nxa8? h1Q From an analysis to the
Excelling at Chess Calculation− Jacob Aagard 48

XABCDEFGHY of course. Qxh5 5.Ne7+ exercise 030 exercise 031


8-+-+nmk-+( Kh7 6.Rxh5# . [Jacob Aagaard] [Jacob Aagaard]
7wq-+-+p+-' 118 Taken from the excellent
6p+Qzp-+P+& 'Art of Attack in Chess' by XABCDEFGHY XABCDEFGHY
5+-+Psn-zPp% Vukovic. 8r+lwq-trk+({ 8-+ktr-+-tr({
4-zPPzp-+-+$ Maximum score: 12 points. 7zpp+-+pzp-' 123 7zpp+-+pzpp' 125
3+-+L+-+P# . 6-+-zp-sn-zp& 6-+-+l+-+&
2-+-vL-+K+" (press F10 for the next 5+-vlPsn-+-% 5+p+-+-+-%
1+-+-+-+-![ exercise) 4-+p+P+-vL$ 4-wQP+-vL-+$
xabcdefghy 3+-sN-+P+-# 3+-+-+-+P#
game. White avoided this 2PzPL+N+PzP" 2PzP-+LzPqsn"
position, overlooking that exercise 029 1+R+QmK-+R! 1tR-+-mKR+-!
he wins instantly with: [Jacob Aagaard] xabcdefghy xabcdefghy
1.Qxe8+!! Kxe8 2.g7 . .
Nxd3 3.g8Q+ (8 points) XABCDEFGHY Black to play and win. Black to play and win.
Ke7 4.g6! (4 points) This 8-+l+-+k+({ . .
move should have been 7zpp+-+-+-' 121 (press F10 for the solution) (press F10 for the solution)
found in advance. Kf6 6-+-zp-wQ-zp&
(2 points) 5+-+Psn-+-%
[ The point is 4...fxg6 4-+-+-sNP+$ solution 030 solution 031
5.Qg7+ and White wins. ] 3+-wq-+-+P# [Jacob Aagaard] [Jacob Aagaard]
5.gxf7 Qxf7 does not 2P+-+K+-+"
defend because of 6.Qg5# 1+-+-+L+-! XABCDEFGHY XABCDEFGHY
(2 points). xabcdefghy 8r+lwq-trk+({ 8-+ktr-+-tr({
Maximum score: 16 points. . 7zpp+-+pzp-' 124 7zpp+-+pzpp' 126
. Black to play and win as 6-+-zp-sn-zp& 6-+-+l+-+&
(press F10 for the next clearly as possible. 5+-vlPsn-+-% 5+p+-+-+-%
exercise) . 4-+p+P+-vL$ 4-wQP+-vL-+$
(press F10 for the solution) 3+-sN-+P+-# 3+-+-+-+P#
2PzPL+N+PzP" 2PzP-+LzPqsn"
exercise 028 1+R+QmK-+R! 1tR-+-mKR+-!
[Jacob Aagaard] solution 029 xabcdefghy xabcdefghy
[Jacob Aagaard] S.Mohandesi-A.Barsov, R.Hübner-T.Luther,
XABCDEFGHY Leuven 2002. German Ch., Saarbrücken
8-+-+-trk+( XABCDEFGHY . 2002.
7zpltrn+pzpp' 119 8-+l+-+k+({ The theme is similar to the .
6-zppsN-wq-+& 7zpp+-+-+-' 122 classic Legall's mate. Black wins with a nice
5+-zP-zp-+-% 6-+-zp-wQ-zp& Here the consequences of tactical twist. 17...Qxf1+!
4-+-zP-+-+$ 5+-+Psn-+-% the combination are less (5 points) 18.Bxf1 Nf3+
3+PtRL+-+-# 4-+-+-sNP+$ severe, but still enough to 19.Ke2 Nd4+ 20.Kd2
2P+-+QzPPzP" 3+-wq-+-+P# decide the game in [ 20.Ke1 Nc2+
1+-+-+RmK-![ 2P+-+K+-+" Black's favour. 13...Nxe4!! is even worse. ]
xabcdefghy 1+-+-+L+-! (2 points) This kind of 20...Nc6+ (5 points)
. xabcdefghy sacrifice is thematic and 21.Qd6 Rxd6+ 22.Bxd6
White to play and win as P.E.Nielsen-J.Aagaard, should be calculated by Bxc4 (2 points) Black is
clearly as possible. Danish League 2003. both White and Black. two pawns up and won
. . 14.Bxd8 the endgame.
(press F10 for the solution) In time trouble Black [ Or 14.Nxe4 Qxh4+ Maximum score: 12 points.
repeated moves by with a winning position. ] .
37...Qb2+?! 38.Kd1 Qa1+ 14...Nxf3+ 15.Kf1 (press F10 for the next
solution 028 39.Ke2 Qb2+ 40.Kd1 [ If 15.gxf3 Bf2+ 16.Kf1 exercise)
[Jacob Aagaard] Qb1+ 41.Ke2 Qc2+ Bh3# (8 points). ]
42.Ke3 Qc3+ 43.Ke2 15...Ned2+
XABCDEFGHY (the diagram position has [ It does not matter which exercise 032
8-+-+-trk+( arisen again), before knight Black uses, as [Jacob Aagaard]
7zpltrn+pzpp' 120 falling into deep thought. after 15...Nfd2+ 16.Ke1?
6-zppsN-wq-+& The theme is the Black has Bf2# . ] XABCDEFGHY
5+-zP-zp-+-% unprotected queen on f6. 16.Qxd2 Nxd2+ 17.Ke1 8r+-+rvl-mk(
4-+-zP-+-+$ Bxg4+!! (8 points) Nxb1 18.Ba5 Nxc3 7+l+-+-zpp' 127
3+PtRL+-+-# 44.hxg4 19.Bxc3 Re8 Black has 6-wq-zp-+n+&
2P+-+QzPPzP" [ If 44.Kf2 Nd3+ won the exchange and 5+-zpP+-sN-%
1+-+-+RmK-![ and wins the queen (1 two pawns. The rest is 4-+-+-tR-+$
xabcdefghy point). ] easy. 20.h3 b5 21.Kd1 3+PsN-+-wQ-#
Toth-Szigeti, Budapest 44...Qc2+ 45.Ke1 b4 22.Bd4 Rxe2 23.Bxc5 2-+-+P+LzP"
1946. [ Now if 45.Ke3 Nxg4+ Rxc2 and White resigned. 1+-+-+R+K![
. wins the queen (1 Maximum score: 10 points. xabcdefghy
White wins with a nice point). ] . .
variation on the classical 45...Nf3# (2 points). (press F10 for the next White to play and win.
bishop sacrifice. 1.Bxh7+! Maximum score: 12 points. exercise) .
Kxh7 2.Rh3+ Kg8 3.Qh5 . (press F10 for the solution)
(8 points) Qh6 4.Nf5! (press F10 for the next
(4 points) This is the point exercise)
Excelling at Chess Calculation− Jacob Aagard 49

solution 032 solution 033 solution 034 XABCDEFGHY


[Jacob Aagaard] [Jacob Aagaard] [Jacob Aagaard] 8r+l+r+k+(
7zpp+p+pzp-' 134
XABCDEFGHY XABCDEFGHY XABCDEFGHY 6-+p+-sn-+&
8r+-+rvl-mk( 8r+-+k+-tr( 8r+-+-+-tr( 5+L+-+-vL-%
7+l+-+-zpp' 128 7zp-wq-+pzpp' 130 7zpp+-mk-zpp' 132 4-+-zpP+-+$
6-wq-zp-+n+& 6l+p+p+-+& 6-+p+-+-+& 3+-+P+Q+P#
5+-zpP+-sN-% 5+-vL-+n+Q% 5+-+-sNl+-% 2PwqP+-zP-+"
4-+-+-tR-+$ 4-zPP+Nvl-+$ 4-+-+n+-+$ 1tR-+K+-tR-![
3+PsN-+-wQ-# 3zP-+L+-+-# 3+-+-zP-+-# xabcdefghy
2-+-+P+LzP" 2-+-+-+PzP" 2PzPL+-+PzP" [ 16.Qxf6!! (6 points)
1+-+-+R+K![ 1tR-+-+RmK-![ 1tR-+R+-mK-![ A) , and if 16...gxf6
xabcdefghy xabcdefghy xabcdefghy 17.Bc1+ Kh7 18.Bxb2
A.Karpov-V.Salov, Linares S.Rublevsky-K.Asrian, T.Luther-R.Dausch, cxb5 19.Bxd4 Re6
1993. Aeroflot Open, Moscow German Ch., Höckendorf 20.Kd2 b6 21.Rg4
. 2004. 2004. (6 points) followed by
It is always interesting . . Rag1 and Rh4 mate.;
when a new White wins with a double 22.g4! (4 points) The B) , or 16...Qxa1+
combinational theme is exchange sacrifice. beginning of a not too 17.Bc1 g6 18.Rg4
found in a game. This 19.Rxf4! (3 points) Wiping difficult, but still elegant, (3 points) and Black
example is very nice. out Black's dark squared manoeuvre. Ke6 has no defence; e.g.
32.Nf7+ (4 points) defence. Qxf4 20.Rf1 This is of course the only Re6 19.Qd8+ Kg7
[ White has more difficult Qh6 Black has nothing defence. 23.Nf3! 20.Rh4 with mate to
wins including knight better. (2 points) Other moves come. ( 20.-- );
sacrifices on h7; e.g. [ If 20...Qc7 21.Rxf5! are also strong, but this C) 16...-- ]
32.Nxh7 Nxf4 (2 points) exf5 22.Nd6+ wins the most easily. Bg6 16...Qxb5! (3 points)
( 32...Kxh7 33.Qd3! Be7 and White wins; e.g. Kd8 24.Bxe4 Bxe4 25.Ng5+ [ 16...Qxa1? 17.Qxf6 g6
34.Rg4 ) 33.Ng5 g6 23.Bxf5 Rf8 24.Qh4+ f6 Ke5 26.Nf7+ (2 points) 18.Rg4 transposes to the
( 33...Nh5 34.Qd3 Nf6 25.Qd4 . ] followed by Nxh8 and previous note. ]
35.Rxf6! ) 34.Qxf4 Qc7 [ Also after 20...Qe5 Black had no 17.Qxf6 Qh5+
35.Qh4+ Kg8 36.Nce4 21.Nd6+! it is all over (2 compensation for the [ Not 17...g6 18.Rg5
. But the simplest way points). ] exchange. Qb4 19.Re5! Rxe5
should be appreciated, 21.Rxf5 (3 points) Qxh5 Maximum score: 8 points. 20.Bh6 and White wins. ]
so other wins give just [ If 21...exf5 22.Nd6+ . 18.f3 g6 19.Rg5 Qh7?
6 points. ] Kd7 and now White wins (press F10 for the next [ Black could have made
32...Kg8 33.Qxg6! in many ways, one of exercise) a draw here with
(2 points) them being 23.Qxf7+ 19...Qxh3 20.Rxg6+
[ Black resigned as after Kd8 24.Nxf5 Qc1+ fxg6 21.Qxg6+ Kf8
33.Qxg6 hxg6 34.Rh4 25.Bf1 Qd1 26.Bd6 exercise 035 22.Ba3+ Re7 23.Qf6+
there is no defence and Black must part with [Jacob Aagaard] Kg8 24.Bxe7 Qf1+
against 35 Rh8 mate (6 his queen as well as any 25.Kd2 Qf2+ 26.Kc1
points). hope of saving the XABCDEFGHY Qe1+ 27.Kb2 Qc3+
Maximum score: 12 game. ] 8r+l+r+k+( with perpetual check. ]
points. 22.Nd6+ Kd7 23.Rxh5 7zpp+p+pzp-' 133 20.Qxd4 White now again
. and White won 6-+p+-sn-+& has a very strong attack.
(press F10 for the next convincingly. 5+L+-+-vL-% Qg7
exercise) ] Maximum score: 10 points. 4-+-zpP+-+$ [ Or 20...d5 21.Bb2
. 3+-+P+Q+P# Bxh3 22.Kd2 Qh6
(press F10 for the next 2PwqP+-zP-+" 23.Rag1 dxe4 24.fxe4
exercise 033 exercise) 1tR-+K+-tR-![ with a probably
[Jacob Aagaard] xabcdefghy irresistible attack. ]
. 21.Bb2 Qxd4 22.Bxd4
XABCDEFGHY exercise 034 White to play and win. d6
8r+-+k+-tr( [Jacob Aagaard] . [ Objectively better was
7zp-wq-+pzpp' 129 (press F10 for the solution) 22...b6 23.h4 c5 24.Bb2
6l+p+p+-+& XABCDEFGHY Bb7 25.h5 Re6 26.Kd2
5+-vL-+n+Q% 8r+-+-+-tr( c4 27.Rag1 cxd3
4-zPP+Nvl-+$ 7zpp+-mk-zpp' 131 solution 035 28.cxd3 Rc8 29.hxg6
3zP-+L+-+-# 6-+p+-+-+& [Jacob Aagaard] Rxg6 30.Rxg6+ fxg6
2-+-+-+PzP" 5+-+-sNl+-% 31.Rxg6+ , though the
1tR-+-+RmK-![ 4-+-+n+-+$ endgame greatly favours
xabcdefghy 3+-+-zP-+-# (Diagram 134) White. ]
. 2PzPL+-+PzP" 23.h4 Kf8 24.Kd2 Ke7
White to play and win. 1tR-+R+-mK-![ V.Yemelin-A.Kharlov, 25.f4 c5 26.Bb2 Kd7
. xabcdefghy Aeroflot Open, Moscow [ If 26...b5 27.f5 gxf5
(press F10 for the solution) . 2002. 28.exf5 Bb7 29.h5 Bf3
White to play and win. . 30.Re1+ Kd7 31.h6 d5
. 16.Bc1? (2 points) This 32.Bf6 and White wins. ]
(press F10 for the solution) move looks a good 27.f5 gxf5 28.h5!
candidate, but here White The h-pawn wins the
misses a really strong game. fxe4 29.h6 Kc6
combination: [ Or 29...e3+ 30.Ke2 b5
Excelling at Chess Calculation− Jacob Aagard 50

31.h7 Bb7 32.Rh5 (press F10 for the next solution 038 solution 039
and wins. ] exercise) ] [Jacob Aagaard] [Jacob Aagaard]
30.h7 e3+ 31.Ke2 f6
[ Or 31...Be6 32.Bf6 XABCDEFGHY XABCDEFGHY
Rh8 33.Rh1 Rae8 exercise 037 8-+-trksn-tr( 8-+-+r+k+({
34.Bxh8 Rxh8 35.Rg8 [Jacob Aagaard] 7+l+-wq-+p' 140 7tR-+-+-+-' 142
and it is all over. ] 6psnp+-zppvL& 6-+-+q+p+&
32.Bxf6 Black cannot offer XABCDEFGHY 5+pvl-zp-+-% 5+-+-+n+-%
any more resistance. Be6 8-+-tr-+k+({ 4P+-+P+-wQ$ 4PzP-zp-+-zp$
33.Rh1 Rh8 34.Bxh8 7+p+-+pzpp' 137 3+-zP-+N+P# 3+-+Q+PvL-#
Rxh8 35.Kxe3 b5 36.a3 6-+p+p+-+& 2LzP-+NzPP+" 2-+-+-zPKzP"
a5 37.Rh6 a4 38.Rxe6 5zp-+-zP-+l% 1tR-+R+-mK-![ 1+-+-+-+-!
Rxh7 39.Rgg6 4Pvl-+PzP-+$ xabcdefghy xabcdefghy
and Black resigned. 3+QsN-+-zPq# S.B.Hansen-T.Hillarp T.Nyback-M.Matthiesen,
Maximum score: 15 points. 2-zP-+L+-zP" Persson, Malmö 2003. Copenhagen 2003.
. 1+-+R+-+K! . .
(press F10 for the next xabcdefghy Here White saw the Here Matthiesen surprised
exercise) . winning combination, but his strong opponent with:
Black to play and win. for some reason believed 31...h3+!! (10 points)
. that the alternative was Black is after the king.
exercise 036 (press F10 for the solution) just as strong. 25.Ng5? 32.Kg1
[Jacob Aagaard] [ 25.Rxd8+ Kxd8 26.b4 [ 32.Kxh3 loses to Ne3+
Bd6 27.Ng5 Nc4 33.Kh4 Ng2+! 34.Kg5
XABCDEFGHY solution 037 (4 points) is also Qd5+ 35.Kxg6 Re6#
8rsn-mk-+ntr( [Jacob Aagaard] tempting, but still not (5 points). ]
7zpl+p+p+p' 135 conclusive! ] 32...Qe1+ 33.Qf1 Nxg3!
6-+-+-+-+& XABCDEFGHY [ The right way to (3 points) The cleanest.
5+pvLN+P+P% 8-+-tr-+k+({ proceed was 25.Nxe5! [ Also possible was
4-+-+-+P+$ 7+p+-+pzpp' 138 (6 points) Rxd1+ 33...d3 34.Rd7 d2
3+-+P+Q+-# 6-+p+p+-+& 26.Rxd1 and Black wins (2
2PwqPmK-+-+" 5zp-+-zP-+l% A) , and if 26...fxe5 points). ]
1+-+-tR-+-![ 4Pvl-+PzP-+$ 27.Bg5 [ But not 33...Qd2??
xabcdefghy 3+QsN-+-zPq# wins (3 points); when White has a
. 2-zP-+L+-zP" B) , or 26...Qxe5 counter-attack with
White to play and win. 1+-+R+-+K! 27.Bg7 and Black 34.Qc4+ Kh8 35.Be5+!
. xabcdefghy loses the rook and/or (1 point) and it is Black
(press F10 for the solution) R.Janssen-I.Sokolov, his self-respect (3 who is mated. ]
Dutch Ch., Leeuwarden points). ( 27.-- ); 34.fxg3
2002. C) 26...-- ] [ If 34.hxg3 h2+ wins. ]
solution 036 . 25...Rxd1+ 26.Rxd1 Nc4! 34...Qe3+
[Jacob Aagaard] Black decided the game (8 points) 27.b4 Bb6 [ White resigned because
with a killer blow. 28.Bxc4 bxc4 29.Nf3 of 34...Qe3+ 35.Kh1
XABCDEFGHY 23...Rd2!! (12 points) and White offered a draw Qxf3+! 36.Kg1 Re1
8rsn-mk-+ntr( [ White resigned due to in this advantageous (4 points).
7zpl+p+p+p' 136 23...Rd2 24.Rxd2 Bf3+ position simply to avoid Maximum score: 20
6-+-+-+-+& 25.Bxf3 ( or 25.Kg1 messing things up further. points.
5+pvLN+P+P% Qg2# ) 25...Qf1# . Maximum score: 20 points. .
4-+-+-+P+$ Maximum score: 12 . (press F10 for the next
3+-+P+Q+-# points. (press F10 for the next exercise) ]
2PwqPmK-+-+" . exercise)
1+-+-tR-+-![ (press F10 for the next
xabcdefghy exercise) ] exercise 040
A.Anderssen-L.Kieseritzky, exercise 039 [Jacob Aagaard]
London 1851 (variation). [Jacob Aagaard]
. exercise 038 XABCDEFGHY
In his analysis of this [Jacob Aagaard] XABCDEFGHY 8R+-+-+-+(
classic game Kasparov 8-+-+r+k+({ 7+-+-+k+p' 143
discovered: 24.Bb6+!! XABCDEFGHY 7tR-+-+-+-' 141 6l+p+-+p+&
[ 24.Qe3 does not work 8-+-trksn-tr( 6-+-+q+p+& 5+-+p+p+-%
because of Bxd5 7+l+-wq-+p' 139 5+-+-+n+-% 4r+-zP-zP-+$
25.Bb6+ Kc8! 6psnp+-zppvL& 4PzP-zp-+-zp$ 3+-+-+-zP-#
(3 points). ] 5+pvl-zp-+-% 3+-+Q+PvL-# 2-tR-+-mKLzP"
24...axb6 25.Qe3 4P+-+P+-wQ$ 2-+-+-zPKzP" 1+-+r+-+-![
and Black has no way to 3+-zP-+N+P# 1+-+-+-+-! xabcdefghy
save the game; 2LzP-+NzPP+" xabcdefghy .
[ e.g. 25.Qe3 Bxd5 1tR-+R+-mK-![ . White to play and win.
26.Qxb6+ Kc8 27.Re8# xabcdefghy Black to play and win. .
. . . (press F10 for the solution)
Maximum score: 13 White to play and win. (press F10 for the solution)
points. .
. (press F10 for the solution)
Excelling at Chess Calculation− Jacob Aagard 51

solution 040 solution 041 XABCDEFGHY [ If 31...Ke7 32.Qf6# ]


[Jacob Aagaard] [Jacob Aagaard] 8-trl+-trk+({ [ or 31...Kg5 32.Bf6# . ]
7zp-+pvl-zpp' 148 32.Qxe4+! (6 points) The
XABCDEFGHY XABCDEFGHY 6-+p+pzp-+& simplest and cleanest win.
8R+-+-+-+( 8-+-+-tr-+({ 5+-+-zP-+-% [ 32.Qxf7 is also decisive
7+-+-+k+p' 144 7+-+-+pmkp' 146 4-+P+NzP-+$ after Rd8 33.f4+! Qxf4
6l+p+-+p+& 6-+-zp-+p+& 3zP-+-+-+-# 34.Re1+ Qe4 35.Qc7+!
5+-+p+p+-% 5+pwqN+n+-% 2-zP-tRL+PzP" Bd6 36.Qg7+ Kf5
4r+-zP-zP-+$ 4-+-vl-+-+$ 1+-+-mK-+R! 37.g4+ Qxg4+ 38.hxg4+
3+-+-+-zP-# 3+LzP-+-+P# xabcdefghy Kg5 39.Bd3 Bh2+
2-tR-+-mKLzP" 2RzP-+-zPP+" 17.fxe5 Rf5! (3 points) 40.Kxh2 Rxd3 41.Re5+
1+-+r+-+-![ 1+-+-wQ-mK-! 18.Nd6 Rxe5 19.Nxc8 Nxe5 42.Qxe5+
xabcdefghy xabcdefghy Bg5! (4 points) This was etc. Give yourself a
R.Vaganian-T.L.Petrosian, Zhao Jun-Art.Minasian, the point. 20.Rxd7? bonus 4 points if you
Aeroflot Open, Moscow Aeroflot Open, Moscow [ White should have calculated it all the way
2004. 2004. played 20.Rc2 to here. But it would be
. . (3 points) with a lost unnecessarily risky to
White would be worse 26...Re8!! (10 points) endgame after Rxc8 venture such a
were it not for a surprising Let us see why White ; instead he self- complicated variation in
mate. 35.Bxd5+!! resigned. 27.-- destructed. ] an actual game, when
A wonderful shot which [ 27.Qxe8 Bxf2+ 28.Kh1 20...Rxb2 (2 points) there is a simple win
completely destroys ( or 28.Kh2 Bg1+ 21.Rf1? Rexe2+ 22.Kd1 available. ]
Black's position. Kg7 29.Kh1 Ng3# ) 28...Ng3+ Rxg2 and White resigned. 32...Kxe4 33.Re1+ Kf5
[ If 35...cxd5 36.Ra7+ 29.Kh2 Bg1+ 30.Kxg3 Maximum score: 10 points. 34.Rxe8 White is winning.
Kf6 37.Rb6# . It is rare Qf2+ 31.Kg4 f5+ . Full points only if you saw
that we see mating 32.Kg5 h6# (6 points). ] (press F10 for the next this position in your
combinations in the [ 27.Qf1 Bxf2+! exercise) analysis. Be6 35.Rxf8
endgame. ] (4 points) 28.Qxf2 ( or Bxa2 36.Rc8 (2 points)
36.Bb3 Rb4 28.Kh2 Re1 ) 28...Re1+ Black resigned as his
[ There is no escape. If and Black wins. ] exercise 043 position is hopeless after
36...Rdxd4 37.Bxa4 [ 27.Ne3 Bxe3 (2 points) [Jacob Aagaard] 37 Rc7.
Rxa4 38.Rb6 Bb5 28.Ra1 Re5 Maximum score: 18 points.
39.Rb7+ Kh6 40.Rh8 (other moves win as XABCDEFGHY .
Ra2+ 41.Ke3 well) 29.fxe3 Nxe3 8-+l+rvl-+( (press F10 for the next
and White wins. ] with a winning attack. 7zpp+-+pmk-' 149 exercise)
37.Rxa6 Rd3 38.Ra3 Maximum score: 18 6-+n+-+pzp&
Rb7 39.Ke2 Rxd4 points. 5+-zpN+-+-%
[ Or 39...Rc3 40.Kd2 . 4-+-+qvL-+$ exercise 044
Rf3 41.Kc2 and White (press F10 for the next 3+-zP-+Q+P# [Jacob Aagaard]
wins with his extra exercise) ] 2PzP-+-zPP+"
piece. ] 1+-+R+LmK-![ XABCDEFGHY
40.Rba2 and Black xabcdefghy 8r+-wq-snrmk(
resigned. exercise 042 . 7+-+-vl-zpp' 151
Maximum score: 10 points. [Jacob Aagaard] White to play and win. 6p+-+-zp-+&
. . 5+pzp-zp-+R%
(press F10 for the next XABCDEFGHY (press F10 for the solution) 4P+N+P+Q+$
exercise) 8-trl+-trk+({ 3+P+PvLR+-#
7zp-+pvl-zpp' 147 2K+P+-+PzP"
6-+p+pzp-+& solution 043 1+-+-+-+-![
exercise 041 5+-+-zP-+-% [Jacob Aagaard] xabcdefghy
[Jacob Aagaard] 4-+P+NzP-+$ .
3zP-+-+-+-# XABCDEFGHY White to play and win.
XABCDEFGHY 2-zP-tRL+PzP" 8-+l+rvl-+( .
8-+-+-tr-+({ 1+-+-mK-+R! 7zpp+-+pmk-' 150 (press F10 for the solution)
7+-+-+pmkp' 145 xabcdefghy 6-+n+-+pzp&
6-+-zp-+p+& . 5+-zpN+-+-%
5+pwqN+n+-% Black to play and win. 4-+-+qvL-+$ solution 044
4-+-vl-+-+$ . 3+-zP-+Q+P# [Jacob Aagaard]
3+LzP-+-+P# (press F10 for the solution) 2PzP-+-zPP+"
2RzP-+-zPP+" 1+-+R+LmK-![
1+-+-wQ-mK-! xabcdefghy (Diagram 152)
xabcdefghy solution 042 A.Karpov-V.Topalov, Dos
. [Jacob Aagaard] Hermanas 1994. J.C.Perez-J.M.Bravo, Cuba
Black to play and win. . 1997.
. Karpov decides the game .
(press F10 for the solution) (Diagram 148) with a nice magnet This combination is
sacrifice (drawing the illustrative of several nice
R.Lilja-J.Aagaard, black king out). 30.Nf6!! themes. 1.Nxe5!!
Copenhagen 1992. (8 points) Kxf6 (8 points) This prepares
. [ If 30...Qxf3? 31.Nxe8+ the removal of the
Black won a pawn with a wins. ] important defender on f8.
nice manoeuvre. 16...fxe5 31.Be5+! (2 points) Kxe5 [ 1.Rfh3? achieves
Excelling at Chess Calculation− Jacob Aagard 52

XABCDEFGHY XABCDEFGHY XABCDEFGHY hxg6 17.hxg6+ Kg8


8r+-wq-snrmk( 8l+-tr-+-mk( 8-mkr+-+-tr( 18.Qc1! Qd8 19.Bc7!!
7+-+-vl-zpp' 152 7zp-+-sNpzpp' 154 7+p+-+p+p' 156 (10 points) This is the
6p+-+-zp-+& 6-+-tr-+-+& 6p+-zP-+-+& point.
5+pzp-zp-+R% 5+pwq-+-+-% 5wQ-+N+q+-% [ 19.Rh5? Bh4 20.Qh1
4P+N+P+Q+$ 4-+p+-wQ-+$ 4-+P+-+-+$ Qf6 21.Rxh4 Qxg6
3+P+PvLR+-# 3+-+-tR-+-# 3+-+-vLn+-# is less clear. ]
2K+P+-+PzP" 2PzP-tR-zPPzP" 2P+-+-zP-+" [ However, 19.Rh8+
1+-+-+-+-![ 1+-+-+-mK-![ 1+-mKR+-+-![ (15 points) works well.
xabcdefghy xabcdefghy xabcdefghy I just find my win more
nothing after g6! 2004. [ Perhaps even simpler aesthetic. ]
(2 points). ] . was 32.Rd5!? Qg6 19...Rxf2+? (2 points) This
1...fxe5 30.Ng6+!! (8 points) The 33.Rd8 (10 points) and makes things easy for
[ Declining the knight is beginning of a wonderful the counterplay from the White.
no better: combination. hxg6 next note is prevented. [ 19...Bh4! (5 points)
1...Qe8 2.Rxh7+! [ 30...fxg6 31.Qxd6! But this is a computer 20.Bxd8 Rxf2+ 21.Kd1
(2 points) Nxh7 ( or Qxd6 32.Rxd6 variation of course. ] Bxd8 offered more
2...Kxh7 3.Rh3+ ) wins the exchange (5 32...Qa5 resistance, but then
3.Ng6+ etc. ] points) ] [ Black also loses after 22.Ke1 Rf3 23.Ke2
[ 1...h6 2.Bxh6! [ or if 30...Kg8? 31.Rxd6 the more complex A) 23...Bg5 24.Rh7
(2 points) gxh6 3.Nf7+ wins (2 points). ] 32...Nd4 (3 points) Bh6 ( 24...e5 25.Qh1 )
Kh7 4.Rxh6# . ] 31.Qh4+ Qh5 33.Rxd4 Rg8 34.Rd8 25.Rxh6! gxh6 26.Qh1
[ 1...g6 2.Nf7+ [ If 31...Kg8 32.Qxd8+ Rg1+ 35.Kb2 Qe5+ Kg7 27.g5! wins;
wins the queen. ] Rxd8 33.Rxd8+ Kh7 36.Kc2 Qf5+ 37.Kb3 B) , or if 23...e5
2.Rxf8! (6 points) Rxf8 34.Rh3+! wins (10 Qb1+ 38.Ka3 24.Qg1 (threatening
The main defence. points). ] etc (7 points). ] Qh2) Bg5 25.Qh2
[ If 2...Qxf8 3.Rxh7+! 32.Qxd8+! Rxd8 33.Ra8+!! A nice finish. Rxe3+ 26.Kf2 Rf3+
Kxh7 4.Qh5# ] 33.Rxd8+ Kh7 34.Ree8 Rxa8 34.Nc8# (10 points). 27.Kg2 Bh6 28.Qxe5
[ while 2...Qd6 and White wins the queen Maximum score: 30 points. Bxg4 29.Qe7 Raf8
loses to many different and bishop to finish a rook . 30.Rxh6 wins.;
moves, the strongest up. (press F10 for the next C) 23...-- ]
again being 3.Rxh7+! Maximum score: 30 points. exercise) 20.Kxf2 Bh4+
( otherwise 3.Qf5; . [ If 20...Qxc7 21.Rh8+!
3.Qh3; 3.Rf7; , or 3.Rff5 ) (press F10 for the next Kxh8 22.Qh1+ Bh4+
3...Kxh7 4.Rf7 exercise) exercise 047 23.Qxh4+ Kg8 24.Qh7+
(2 points) with forced [Jacob Aagaard] Kf8 25.Qh8+ Ke7
mate. ] 26.Qxg7+ Kd8 ( or
3.Qg6! (2 points) Kg8 exercise 046 XABCDEFGHY 26...Kd6 27.c5# )
[ If 3...h6 4.Bxh6! [Jacob Aagaard] 8r+l+-tr-mk( 27.Qf8+ Kd7 28.Rh1
and mates. ] 7zpp+-vl-zpp' 157 and mates. ]
4.Rxh7 Rf7 5.Qh5 XABCDEFGHY 6-+p+p+-+& 21.Ke2 Qg5 22.Qg1!
and Black is mated. 8-mkr+-+-tr( 5wq-+psN-+P% (3 points) e5 23.Qh2!
Maximum score: 24 points. 7+p+-+p+p' 155 4-+PzPpvLP+$ Trapping the bishop.
. 6p+-zP-+-+& 3+-zP-zP-+-# Bxg4+ 24.Kd2 exd4
(press F10 for the next 5wQ-+N+q+-% 2P+Q+KzP-+" 25.cxd4 dxc4 and Black
exercise) 4-+P+-+-+$ 1tR-+-+-+R![ resigned on noticing that
3+-+-vLn+-# xabcdefghy the c7-bishop prevents
2P+-+-zP-+" . 26...Qa5+.
exercise 045 1+-mKR+-+-![ White to play and win. Maximum score: 25 points.
[Jacob Aagaard] xabcdefghy . .
. (press F10 for the solution) (press F10 for the next
XABCDEFGHY White to play and win. exercise)
8l+-tr-+-mk( .
7zp-+-sNpzpp' 153 (press F10 for the solution) solution 047
6-+-tr-+-+& [Jacob Aagaard] exercise 048
5+pwq-+-+-% [Jacob Aagaard]
4-+p+-wQ-+$ solution 046 XABCDEFGHY
3+-+-tR-+-# [Jacob Aagaard] 8r+l+-tr-mk( XABCDEFGHY
2PzP-tR-zPPzP" 7zpp+-vl-zpp' 158 8r+-wq-trk+({
1+-+-+-mK-![ 6-+p+p+-+& 7zpp+-vl-zpp' 159
xabcdefghy (Diagram 156) 5wq-+psN-+P% 6-+n+-+-+&
. 4-+PzPpvLP+$ 5+-zpn+-vL-%
White to play and win. E.Sedina-S.Tkeshelashvili, 3+-zP-zP-+-# 4-+-zp-+-+$
. European Women's Ch., 2P+Q+KzP-+" 3+-+-+N+-#
(press F10 for the solution) Istanbul 2003. 1tR-+-+-+R![ 2PzPPwQNzPPzP"
. xabcdefghy 1tR-+-+RmK-!
White decided the game J.Aagaard-S.K.Williams, xabcdefghy
solution 045 with a spectacular queen Hampstead 1998. .
[Jacob Aagaard] sacrifice. 29.Qc7+!! . Black to play and win as
(4 points) Rxc7 30.dxc7+ A classic theme in a new clearly as possible.
S.Mamedyarov-A.Timofeev, Ka8 31.Nb6+ Ka7 and complex wrapping. .
Aeroflot Open, Moscow 32.Rd8 (6 points) 16.Ng6+!! (10 points) (press F10 for the solution)
Excelling at Chess Calculation− Jacob Aagard 53

solution 048 XABCDEFGHY XABCDEFGHY XABCDEFGHY


[Jacob Aagaard] 8r+-+-+k+( 8-+-+-+-+({ 8-+-trrsnk+(
7+-+-+-+p' 161 7+-+-+pzpk' 163 7zp-wq-vl-+p' 165
XABCDEFGHY 6p+P+p+-+& 6-+-sN-sn-zp& 6-+-+p+p+&
8r+-wq-trk+({ 5+p+-sNp+-% 5+-zP-+P+-% 5+-zp-+psN-%
7zpp+-vl-zpp' 160 4-+-+-zP-+$ 4-zP-+-+-+$ 4-+P+-zPn+$
6-+n+-+-+& 3+-+-+-zP-# 3+-tR-+-mKP# 3+P+-+-tR-#
5+-zpn+-vL-% 2-+n+-+-+" 2-+-+Q+P+" 2PvLL+Q+PzP"
4-+-zp-+-+$ 1+R+-+-mK-![ 1+-+-+rwq-! 1+-+-+RmK-![
3+-+-+N+-# xabcdefghy xabcdefghy xabcdefghy
2PzPPwQNzPPzP"
1tR-+-+RmK-!
xabcdefghy solution 049 solution 050 solution 051
N.N.-L.Nilsson, Denmark [Jacob Aagaard] [Jacob Aagaard] [Jacob Aagaard]
2002.
. XABCDEFGHY XABCDEFGHY XABCDEFGHY
Black wins with a 8r+-+-+k+( 8-+-+-+-+({ 8-+-trrsnk+(
standard sacrifice, though 7+-+-+-+p' 162 7+-+-+pzpk' 164 7zp-wq-vl-+p' 166
it requires some accurate 6p+P+p+-+& 6-+-sN-sn-zp& 6-+-+p+p+&
calculation. 1...Rxf3! 5+p+-sNp+-% 5+-zP-+P+-% 5+-zp-+psN-%
(5 points) 2.Bxe7 Qxe7 4-+-+-zP-+$ 4-zP-+-+-+$ 4-+P+-zPn+$
3.gxf3 Ne5! (5 points) 3+-+-+-zP-# 3+-tR-+-mKP# 3+P+-+-tR-#
4.Kg2 2-+n+-+-+" 2-+-+Q+P+" 2PvLL+Q+PzP"
[ Against 4.Kh1!? 1+R+-+-mK-![ 1+-+-+rwq-! 1+-+-+RmK-![
all you need to see is xabcdefghy xabcdefghy xabcdefghy
Qh4! (2 points) and F.Egeland-L.Eckhardt, F.Fabri-P.Hardicsay, N.De Firmian-A.Holst,
White has no defence; e. Oslo 2003 (variation). Hungary 1981. Copenhagen 2003.
g. 5.Ng1 ( or 5.Kg1 Rf8 . . .
wins ) 5...Nf4 (intending Here White wins with a 1...Nh5+!! (8 points) A A position that requires
6...Qg5) 6.Ne2 Rf8 nice twist. 41.Rd1! fantastic move which clarity of thought. De
wins. ] (3 points) drags the white queen Firmian did not fully
4...Nc4! [ If 41.Rc1 Nb4 42.Nd7 away from the defence of concentrate and played
[ Instead Black continued Nd5 and Black is doing f2. 2.Qxh5 White has no 22.Nxh7?? De Firmian
with 4...Rf8? better. ] choice. must have missed Black's
and the game got very 41...Ne3 42.c7!! [ If 2.Kh4 Qd4+ 3.Qg4 24th move.
complicated. ] (7 points) With the threat Qf6+ 4.Kxh5 g6+ [ Instead 22.Rxg4!? fxg4
[ 4...Qf7!? (2 points) was of 43 Rd8+. Nxd1 and mates (4 points). ] 23.Qxg4 (5 points) gives
better; e.g. 5.f4 Qg6+ [ 42...Nd5 43.Rxd5 exd5 2...Qf2+ White more than
6.Ng3 Nc4 7.Qc1 ( if 44.Nc6 is similar. ] [ 2...Rf2? does not work sufficient compensation
7.Qd1 Nde3+ wins ) 43.Nc6 (2 points) White is because of 3.Qf3! Rxf3+ for the exchange, but it
7...Rf8 8.b3 Nxf4+ winning because of the 4.Rxf3 and Black should is not strongest. ]
9.Kg1 Qc6 10.f3 Ne3 threat of 44 Nb8! and the not win against the [ The computer suggests
and Black has a c-pawn queens. -- best defence (4 points). ] another variation that
decisive advantage (2 [ It does not help to play 3.Kh2 wins on the spot:
points). 43...Re8 because we [ If 3.Kg4 Qf4# 22.Nxe6! (1 point)
But the text is even have the same theme (4 points). ] A) , and then if 22...Bf6
stronger. ] with 44.Nd8! ] 3...Qe1!! (10 points) The 23.Bxf5 ( 23.Qxg4
5.Qd3 Qxe2! (10 points) [ while 43...Rc8 point! Black has switched also works ) 23...Bxb2
6.Qxe2 Nf4+ 7.Kh1 Nxe2 of course loses to his rook and queen round 24.Qxg4 wins.;
when Black has two huge 44.Ne7+ etc. on the back rank and mate B) . Black's best try is
knights for a passive white Maximum score: 12 is now inevitable. 22...Bh4 (3 points),
rook. points. Thanks to Mark Dvoretsky when the true
Maximum score: 20 points. . for showing me this desperado 23.Qxg4!!
. (press F10 for the next exercise. (15 points) would have
(press F10 for the next exercise) ] Maximum score: 22 points. won the game
exercise) . ( instead standard
(press F10 for the next thinking leads to
exercise 050 exercise) something like
exercise 049 [Jacob Aagaard] 23.Nxc7? Rxe2
[Jacob Aagaard] 24.Rc3 Rdd2
exercise 051 (2 points) and Black
(Diagram 163) [Jacob Aagaard] wins ) 23...fxg4
(Diagram 161) 24.Nxc7 Re2 25.Rxg4
. and White emerges
. Black to play and win. (Diagram 165) with a decisive
White to play and win. . material advantage.;
. (press F10 for the solution) . C) 22...-- ]
(press F10 for the solution) White to play and win. 22...Kxh7
. [ After 22...Nxh7?
(press F10 for the solution) White wins in a nice
fashion: 23.Rxg4! fxg4
24.Qxe6+ Kf8 25.Bg7+!
Excelling at Chess Calculation− Jacob Aagard 54

(5 points) Kxg7 XABCDEFGHY 1...Nh7 38.Rg1 gives Black the


26.Qxg6+ Kf8 27.Qh6+ 8r+qsnkvl-tr( [ If 1...fxg6 2.Bxe6+ extra option of Rh8!?
Kf7 28.Qxh7+ Ke6 7+-+n+-zpp' 168 winning the queen. ] (2 points) which White
( 28...Kf8 29.Qh6+ Kf7 6pzp-+-+-+& 2.Rxe6! fxe6 3.Qxd8+! may as well prevent. ]
30.Bg6+ Ke6 31.Bxe8+ 5+Q+Rzp-sN-% [ Black resigned due to 37...Ne4 38.Rg1
Bf6 32.Re1+ also leads 4-+p+PzP-+$ 3.Qxd8+ Qxd8 4.Bxe6# (7 points) White has a
to mate ) 29.Qf5+ Kd6 3zP-sN-+-+-# (20 points). winning endgame, but it
30.Qd5# . ] 2-zP-+-+PzP" Maximum score: 28 should have been seen
23.Rxg4 fxg4 24.Qxg4 1+-mK-+R+-![ points. that Black does not have
Threatening 25 Qh5+ Kg8 xabcdefghy . a lot of counterplay here.
26 Qh8+ and mates. But Defence I found the (press F10 for the next The rooks are exchanged
Black has a resource to following nice drawing line. exercise) ] so that the h-pawn
turn the tables. Rd4!! 18.Rxe5+! (4 points) Be7 becomes as strong as
(10 points) Blocking off 19.Nd5!! (10 points) The possible. Rxg1
the bishop on b2 and idea starts to unfold. axb5 exercise 054 [ 38...Rh8 would now be
saving the black king from 20.Rxe7+ Kf8 21.Rf7+!! [Jacob Aagaard] met simply by 39.Kh3
harassment. 25.h4 (10 points) Not so improving the position. ]
[ Now 25.Qh5+ surprising now there is XABCDEFGHY 39.Kxg1 b4 40.h5 bxa3
does nothing after Kg7 nothing else. But earlier it 8-+-+k+ntr( 41.bxa3 Rb7 42.Qg2!
.] could have been hard to 7tr-+-+p+-' 171 Both shielding the king
25...Bf6 26.h5 Kh6? see. Nxf7 22.Ne6+ Ke8 6-+-+p+-+& and assisting the passed
[ Stronger was 26...Qf7! [ Not 22...Kg8? 23.Ne7# 5zpp+pzP-+q% pawn. Kf8 43.h6
, after which White's (4 points). ] 4-+pzP-tRN+$ and White is winning.
attack should be over. ] 23.Nxg7+ Kd8 24.Ne6+ 3zP-zP-+QzP-# Maximum score: 27 points.
27.hxg6? This loses all Ke8 with perpetual check 2-zP-+-+-+" .
control White had over g6, (2 points). 1+-+-+RmK-![ (press F10 for the next
the most dangerous Maximum score: 30 points. xabcdefghy exercise)
square near the black king. . .
[ 27.Bc1! gave more (press F10 for the next White to play and win.
counterplay, when White exercise) Make sure you calculate exercise 055
would still be able to all the way to a clearly [Jacob Aagaard]
cast doubt on the winning position.
result. ] exercise 053 . XABCDEFGHY
27...Kg7 Now the attack is [Jacob Aagaard] (press F10 for the solution) 8-mk-trr+-+({
finished. 28.Kh2 Nd7 7+pzp-+p+-' 173
29.Bxd4 Bxd4 30.Kg3 XABCDEFGHY 6-zpn+n+-+&
Nf6 31.Qh3 e5 32.Kf3 8r+qtr-+k+( solution 054 5+q+pzp-+-%
exf4 33.Be4 Rxe4 7+p+-+pzp-' 169 [Jacob Aagaard] 4-+-zP-+-+$
and White resigned. 6-+pvllsn-+& 3zPN+PzP-+-#
Maximum score: 27 points. 5zp-+-sN-+-% XABCDEFGHY 2-zPQ+-zPP+"
. 4-+-zP-+-wQ$ 8-+-+k+ntr( 1+-tRN+RmK-!
(press F10 for the next 3zP-+-+-zPL# 7tr-+-+p+-' 172 xabcdefghy
exercise) 2-vL-+-zP-+" 6-+-+p+-+& .
1tR-+-tR-mK-![ 5zpp+pzP-+q% Black to play and win.
xabcdefghy 4-+pzP-tRN+$ .
exercise 052 . 3zP-zP-+QzP-# (press F10 for the solution)
[Jacob Aagaard] White to play and win. 2-zP-+-+-+"
. 1+-+-+RmK-![
XABCDEFGHY (press F10 for the solution) xabcdefghy solution 055
8r+qsnkvl-tr( N.Short-A.Dreev, [Jacob Aagaard]
7+-+n+-zpp' 167 Reykjavik (rapid) 2004.
6pzp-+-+-+& solution 053 . XABCDEFGHY
5+Q+Rzp-sN-% [Jacob Aagaard] White wins because of the 8-mk-trr+-+({
4-+p+PzP-+$ theme I called 'big pieces 7+pzp-+p+-' 174
3zP-sN-+-+-# XABCDEFGHY in trouble' in 'Excelling at 6-zpn+n+-+&
2-zP-+-+PzP" 8r+qtr-+k+( Combinational Play', a 5+q+pzp-+-%
1+-mK-+R+-![ 7+p+-+pzp-' 170 theme we see more often 4-+-zP-+-+$
xabcdefghy 6-+pvllsn-+& than we might expect. 3zPN+PzP-+-#
. 5zp-+-sN-+-% 34.Nf6+! (2 points) 2-zPQ+-zPP+"
White to play and draw. 4-+-zP-+-wQ$ [ In the game Short 1+-tRN+RmK-!
. 3zP-+-+-zPL# actually played 34.Qg2 xabcdefghy
(press F10 for the solution) 2-vL-+-zP-+" , which was rather B.Martinez-F.De la Paz,
1tR-+-tR-mK-![ unclear. He later won in Manikaragua 1998
xabcdefghy a rook endgame. ] (manipulated: the author
solution 052 M.Taimanov-Kuzmin, 34...Nxf6 35.Rh4! has added a pawn at a3).
[Jacob Aagaard] USSR 1950. (8 points) Qxh4 36.gxh4 .
. Rg8+ 37.Kh2! (10 points) A standard but pretty
J.Aagaard, Opening This classic combination [ 37.Kf2? Ng4+ 38.Ke2 mating combination.
analysis 1998. is one of my favourites. Nh2 (2 points) would 1...Qxb3!! (10 points)
. 1.Ng6! (8 points) make the win less 2.Qxb3 Ncxd4 3.exd4
Analysing a variation of [ 1.Bxe6? fxe6 obvious ] Nxd4 With the idea of 4...
the Queen's Indian is not clear. ] [ while 37.Kh1 Ne4 Ne2+ and 5...Rh8 mate (5
Excelling at Chess Calculation− Jacob Aagard 55

points). 4.Qc2 Nf3+!! 35.Nh5+!! (10 points) The Now the black queen pawns, thus giving rise
(5 points) point of the combination. cannot interpose, so the to an unusual
[ White resigned because gxh5 white queen can roam stalemate. ]
of 4...Nf3+ 5.gxf3 Rg8+ [ If 35...Kh8 36.Nxf6 freely. Kf4 1...e2
6.Kh2 Rh8+ 7.Kg3 Bxf6 37.Rxf6 Qxf6 [ If 6...Kh5 7.Qxg4# . ] [ White also draws after
Rdg8# . 38.Bxg5 Ra1+ 39.Kh2 7.Qxg4+ Ke5 8.Qg7+ 1...Kxf7 2.h6 e2 3.h7
Maximum score: 20 and Black will have to (3 points) and White wins. e1Q 4.h8Q , although
points. part with his queen (5 Maximum score: 12 points. here it is more important
. points). ] . that Black can draw.
(press F10 for the next 36.Qxg5+! Now it is no (press F10 for the next The quickest way is
exercise) ] longer so difficult. fxg5 exercise) Qg3+ 5.hxg3
37.Rf7+ Kxh6 38.Rxh7# stalemate. ]
(5 points). 2.h6 e1Q 3.h7 Qe3+
exercise 056 Maximum score: 35 points. exercise 058 With 3...Qh4+ prevented
[Jacob Aagaard] . [Jacob Aagaard] by the white king, Black
(press F10 for the next can only stop the h7-pawn
XABCDEFGHY exercise) XABCDEFGHY by capturing the h3-pawn.
8r+-+-+-+( 8-+-+-mk-+( [ Other checks do not
7+-+lvl-mkp' 175 7+-+-+P+-' 179 improve Black's position:
6q+-zp-zppsN& exercise 057 6-+-+-zP-mK& If 3...Qd2+ 4.Kh5 Qe2+
5+psnP+-sn-% [Jacob Aagaard] 5+-+-+P+P% 5.Kg5 Qg2+ 6.Kh4!
4-+p+P+-wQ$ 4-+-+-+-+$ Qe4+ 7.Kg5 Qe3+
3+-zP-+-sN-# XABCDEFGHY 3+-+-zp-+P# 8.Kg6 and Black has
2-zPL+-+P+" 8-+-+-+-mk( 2-+-+-+-zP" made no progress (2
1+-vL-+RmK-![ 7+-+-+-+p' 177 1+-+-+-+-![ points) ]
xabcdefghy 6-+-+pzP-zp& xabcdefghy [ while 3...Qg1+!?
. 5+-+-mK-+-% . is best met by 4.Kh4!
White to play and win. 4-+-+-+p+$ White to play and draw. (2 points) ( but not
. 3zp-+-zp-+P# . 4.Kh6?? Kxf7! 5.h8Q
(press F10 for the solution) 2-+-+-+-+" (press F10 for the solution) Qe3+! and Black wins. )]
1+-+-+-+-![ 4.Kg6 Qxh3 5.h8Q+
xabcdefghy Qxh8 6.h4! (2 points)
solution 056 . solution 058 Qxh4 stalemate.
[Jacob Aagaard] White to play and win. [Jacob Aagaard] [ If instead 6...Qxf6+
. 7.Kxf6 and it is Black
XABCDEFGHY (press F10 for the solution) XABCDEFGHY who is stalemated!
8r+-+-+-+( 8-+-+-mk-+( Maximum score: 10
7+-+lvl-mkp' 176 7+-+-+P+-' 180 points.
6q+-zp-zppsN& solution 057 6-+-+-zP-mK& .
5+psnP+-sn-% [Jacob Aagaard] 5+-+-+P+P% (press F10 for the next
4-+p+P+-wQ$ 4-+-+-+-+$ exercise) ]
3+-zP-+-sN-# XABCDEFGHY 3+-+-zp-+P#
2-zPL+-+P+" 8-+-+-+-mk( 2-+-+-+-zP"
1+-vL-+RmK-![ 7+-+-+-+p' 178 1+-+-+-+-![ exercise 059
xabcdefghy 6-+-+pzP-zp& xabcdefghy [Jacob Aagaard]
M.Carlsen-H.Harestad, 5+-+-mK-+-% V.Kovalenko, Sachové
Copenhagen 2003. 4-+-+-+p+$ Umenie 1979. XABCDEFGHY
. 3zp-+-zp-+P# . 8-+-+-+-+(
To see this combination 2-+-+-+-+" This is the most surreal 7+-+-+-+p' 181
all the way to the end is 1+-+-+-+-![ exercise in the book, but I 6-+-+-+-+&
not so simple. Hats off to xabcdefghy think it is good fun, so I 5+-+-+p+-%
anyone who has achieved J.Urban, Sachové Umenie stepped away from my 4-+-+-+-+$
this! 34.e5!? (15 points) 1972. usual criterion that 3+-+-+-+-#
Opening the diagonal for . positions should at least 2p+K+-zPPzP"
the bishop on c2. 1.Kd6! (3 points) White have the possibility of 1mk-+-+-+-![
[ Despite the obvious has no other way to arising in an actual game. xabcdefghy
attractive nature of the create counterplay. 1.Kg5! (4 points) White's .
combination, White [ 1.Kxe6? e2! only chance of a White to play and win.
actually had a simpler would be rather successful defence is tied .
winning move: 34.Be3! embarrassing ] to the advance of the h- (press F10 for the solution)
(35 points) intending 35 [ and 1.f7? Kg7 2.Kxe6 pawns. The only question
Bd4 and 36 Qxg5. Kf8 is no better. ] is where to put the queen.
Seeing that Black has 1...a2 [ The alternative 1.Kg6 solution 059
no satisfactory defence [ If 1...Kg8 2.Ke7 wins. ] does not work out as [Jacob Aagaard]
is sufficient to obtain 2.f7 Kg7 3.Ke7 a1Q after e2 2.h6 e1Q 3.h7
full points here. ] 4.f8Q+ Kg6 5.Qf7+!! Qh4 4.h8Q+ Qxh8 5.h4
34...dxe5 There is no (6 points) This sneaky Qxh4 the remaining h- (Diagram 182)
alternative; move is the core of the pawn can still move.
[ if 34...Rf8 35.exf6+ solution. White is therefore Z.Jelinek, Ceskoslovensky
Bxf6 36.Bxg5 [ If instead 5.Qg8+? required to lose a tempo, Republika 1932.
and White wins (5 Qg7+ and Black wins. ] so that Black is forced .
points). ] 5...Kg5 6.Qg8+ to take one of the h- White needs to keep the
Excelling at Chess Calculation− Jacob Aagard 56

XABCDEFGHY XABCDEFGHY exercise 061 solution 062


8-+-+-+-+( 8-+-+-+-+( [Jacob Aagaard] [Jacob Aagaard]
7+-+-+-+p' 182 7+-+-zp-zp-' 183
6-+-+-+-+& 6-+-+p+P+& XABCDEFGHY XABCDEFGHY
5+-+-+p+-% 5+-+-+-+-% 8-+k+-+-+( 8-+-+-+-+(
4-+-+-+-+$ 4P+-+-+-zp$ 7+-+p+-zp-' 185 7+-+-+-+-' 188
3+-+-+-+-# 3zp-+-+-+p# 6-+p+-+-+& 6-+-+K+-+&
2p+K+-zPPzP" 2P+-+-+-zp" 5+-mK-+-+-% 5+-+-+-+-%
1mk-+-+-+-![ 1+-+-+K+k![ 4-+-+-zP-+$ 4-+-+-+-+$
xabcdefghy xabcdefghy 3+-+-+-+-# 3+-+-+p+k#
black king in a stalemate 2-+P+-+-+" 2-+-+-zP-zP"
position and then promote 1+-+-+-+-![ 1+-+-+-+-![
his own h-pawn with solution 060 xabcdefghy xabcdefghy
checkmate. 1.Kc1! [Jacob Aagaard] . J.Moravec, 28 Rijen 1925.
(5 points) White to play and draw. .
[ Black would draw after XABCDEFGHY . This exercise can be
all other moves; e.g. 8-+-+-+-+( (press F10 for the solution) solved either by
1.g3 h5 ( not 1...f4? 7+-+-zp-zp-' 184 calculating all lines
2.g4 f3 3.g5 h5 4.g6 6-+-+p+P+& through to the end or
h4 5.g7 h3 6.Kb3! Kb1 5+-+-+-+-% solution 061 simply by comparison.
7.g8Q a1Q 8.Qg1# ) 4P+-+-+-zp$ [Jacob Aagaard] 1.Kd5!! (10 points)
2.h4 ( or 2.h3 h4 ) 2...f4 3zp-+-+-+p# This is the only square.
3.g4 f3! and Black is 2P+-+-+-zp" XABCDEFGHY White needs to approach
stalemated. ] 1+-+-+K+k![ 8-+k+-+-+( the f-pawn to prevent
1...f4 xabcdefghy 7+-+p+-zp-' 186 Black from taking the h2-
[ 1...h6 2.h3! ] V.Kovalenko, Schakend 6-+p+-+-+& pawn first. So there are
[ and 1...h5 2.h4! Nederland 1995. 5+-mK-+-+-% only three candidates. 1
merely transpose and do . 4-+-+-zP-+$ Kd5 is the only one which
not change anything. ] Another improbable 3+-+-+-+-# allows White to queen his
2.Kc2! White has now position leading to 2-+P+-+-+" pawn and to keep it
gained time so Black interesting play. White has 1+-+-+-+-![ afterwards.
cannot avoid the only one way to proceed, xabcdefghy [ The lines are rather
inevitable. f3 so this exercise is all F.Lazard, American Chess simple:
[ Or 2...h5 3.h4 about visualization and Bulletin 1916. 1.Ke5 Kg2! 2.h4 Kxf2
transposing again. ] awareness of the . 3.h5 Kg2 4.h6 f2 5.h7
3.g4! (3 points) Only this opponent's counter- 1.Kd6! (3 points) There f1Q 6.h8Q Qa1+!
move wins. h6 chances. 1.a5 e5 2.a6 are no credible and Black wins. ]
[ Or 3...h5 4.h4! e4 3.a7 e5! Setting up a alternatives. Kd8 2.f5! [ 1.Kf5 Kg2 2.h4 Kxf2
and White wins as in the stalemate trap. 4.a8R! (3 points) It is important to 3.h5 Kg2 4.h6 f2 5.h7
main line. ] (3 points) keep the g-pawn under f1Q+ 6.Kg6 Qf8
4.h3! Zugzwang. [ Promotion to queen (or control. Ke8 3.c3!! and Black wins. ]
[ Not 4.h4? h5 bishop) creates a (4 points) This amazing 1...Kg2 2.h4 Kxf2 3.h5
and White cannot win – stalemate, and 4.a8N time of loss creates the Kg2 4.h6 f2 5.h7 f1Q
and 5.Kc1?? or king moves lose. So situation White needs in 6.h8Q with a draw.
would even lose. ] this is the only way to order to save the game. Maximum score: 10 points.
4...h5 5.h4! (4 points) play for a win. ] Kd8 4.c4 Ke8 5.c5 Kd8 .
This was the zugzwang 4...e3 Intending 5...e2+. 6.f6! gxf6 stalemate. (press F10 for the next
position White was aiming 5.Ra4!! (3 points) One of Maximum score: 10 points. exercise)
for. hxg4 6.h5 g3 7.h6 the really difficult moves in .
g2 8.h7 g1Q 9.h8Q+ this exercise. e2+ 6.Kxe2 (press F10 for the next
and mate next move. e4 exercise) exercise 063
Maximum score: 12 points. [ Or 6...Kg2 7.Rg4+ Kh1 [Jacob Aagaard]
. 8.Kf3! (the fastest win)
(press F10 for the next e4+ 9.Rxe4 Kg1 exercise 062 XABCDEFGHY
exercise) 10.Re1# . ] [Jacob Aagaard] 8-+-+-+-+(
7.Kf1!! (5 points) It is 7zp-zp-+-+-' 189
amazing that only this XABCDEFGHY 6-+p+-+-+&
exercise 060 move wins here. e3 8-+-+-+-+( 5+-zP-+-+-%
[Jacob Aagaard] 8.Re4! (1 point) 7+-+-+-+-' 187 4-zP-+-+-+$
[ Not 8.Rb4 e2+ 9.Kxe2 6-+-+K+-+& 3zPK+-+-+-#
Kg2 10.Rg4+ Kh1 5+-+-+-+-% 2-+-+-+-+"
(Diagram 183) and White cannot make 4-+-+-+-+$ 1+-+-mk-+-![
progress. ] 3+-+-+p+k# xabcdefghy
. 8...e2+ 9.Kf2! e1Q+ 2-+-+-zP-zP" .
White to play and win. 10.Rxe1# . 1+-+-+-+-![ White to play and win.
. Maximum score: 12 points. xabcdefghy .
(press F10 for the solution) . . (press F10 for the solution)
(press F10 for the next White to play and draw.
exercise) .
(press F10 for the solution)
Excelling at Chess Calculation− Jacob Aagard 57

solution 063 [ If instead 5.a4? Kb3! Kc5 5.g6 Kb5 6.Kb2 XABCDEFGHY
[Jacob Aagaard] and White would be in Kb4 7.e6 a3+ 8.Kb1 8-+-+-+-+(
zugzwang. The point is Kc3 9.e7 a2+ 10.Ka1 7+-zp-+-+-' 194
XABCDEFGHY that 6.b5 axb5 7.axb5 Kc2 11.e8Q b2+ 6-+-+-+-+&
8-+-+-+-+( cxb5 8.Kxb5 Kc3! 12.Kxa2 b1Q+ 13.Ka3 5+-+-+-+-%
7zp-zp-+-+-' 190 no longer loses for Black, Qb3# . ] 4-+-+-mK-+$
6-+p+-+-+& and after 9.c6 Kd4 1...b2! This is forced. 3+-+-+-+-#
5+-zP-+-+-% White should be careful Black cannot allow White 2-mk-+-+-zP"
4-zP-+-+-+$ since if 10.Ka6?? Kc5! to queen freely, so he is 1+-+-+-+-![
3zPK+-+-+-# 11.Kb7 Kd6 and Black playing for stalemate. xabcdefghy
2-+-+-+-+" wins. ] 2.Kxb2 a3+ 3.Ka2! .
1+-+-mk-+-![ 5...Kxa3 6.b5 cxb5 (4 points) This simple position is
xabcdefghy 7.Kxb5 Kb3 8.c6 [ Not 3.Kxa3? Kh5 4.e7 winning only through a
E.Vlasak, Sachové Umenie (3 points) Now the black g6 with stalemate next very shrewd manoeuvre.
1974. king cannot make it to d6, move. ] But even after the
. and White wins easily with: 3...Kh5! Black still needs manoeuvre has been
1.Kc4! (6 points) Kc3 9.Ka6 Kd4 10.Kb7 to play for this defence. found White must find the
[ 1.Ka4? allows the black and so on. 4.e7 g6 5.Kb3! correct way to execute it.
king to approach: Kd2 Maximum score: 20 points. (4 points) All other moves 1.Ke4! (6 points)
2.Ka5 ( if 2.b5 Kc3! . leads to stalemate again. [ Planning to meet 1.Ke4
and White needs to (press F10 for the next a2 6.e8N! (5 points) Only c5 with 2.Kd5! . ]
think about making a exercise) this wins. [ The pawn race with
draw, which he can do [ After 6.e8Q a1Q 1.h4? c5 2.h5 c4 3.h6
most simply by 3.b6! White does not have a c3 4.h7 c2 5.h8Q+ Kb1
axb6 4.cxb6 cxb6 exercise 064 winning check, and is a book draw. ]
stalemate ) 2...Kc3 [Jacob Aagaard] Black will follow with [ 1.Ke5? looks similar,
3.Ka6 Kb3 4.Kb7 Kxa3 something like 7...Qa2+! but Black can use the
5.Kxc7 Kxb4 6.Kxc6 a5 XABCDEFGHY to make the draw Réti trick and move his
and Black makes a 8-+-+-+-+( imminent. ] king to the kingside with
draw. ] 7+-+-+-zp-' 191 6...a1Q 7.Nf6+ Qxf6 Kc3!!
1...a6 6-+-+-+k+& 8.gxf6 Kh6 9.Kc4 A) , and then 2.Kd5
[ If now 1...Kd2 5+-+-zPpzP-% Black cannot stop the f- c6+ 3.Kxc6 Kd4
White wins with 2.b5 4p+-+-zPp+$ pawn now. White wins 4.Kd6 Ke4 and White
cxb5+ ( or 2...Ke3 3mKp+-+-zP-# after: Kh7 10.Kd5 Kg8 cannot win. ( 4...-- );
3.bxc6 Ke4 4.Kb5 Kd5 2-+-+-+-+" 11.Ke6 Kf8 12.f7 g5 B) , while if 2.h4 c5
5.a4 and Black is in 1+-+-+-+-![ [ Or 12...Kg7 13.Ke7 . ] 3.h5 c4 4.h6 Kb2
zugzwang ) 3.Kxb5 Kc3 xabcdefghy 13.fxg5 f4 14.Kf6 f3 5.h7 c3 6.h8Q
4.a4 ( 4.c6 also wins ) . 15.g6 f2 16.g7# . , because of the
4...a5 ( or 4...Kd4 5.a5 White to play and win. Maximum score: 15 points. unhappy position of the
Kd5 6.Ka6 Kxc5 7.Kxa7 . . white king on the long
and White wins ) 5.Kxa5 (press F10 for the solution) (press F10 for the next diagonal, Black can
( 5.c6 still wins as well ) exercise) again reach the book
5...Kc4 6.c6! Kc5 7.Ka6 draw after c2 7.Kd5+
Kxc6 8.a5 and White solution 064 Kb1;
wins (3 points). [Jacob Aagaard] exercise 065 C) 2.-- ]
After Black has [Jacob Aagaard] 1...Kb3
weakened his queenside XABCDEFGHY [ 1...Kc3 now loses to
with his last move, 8-+-+-+-+( XABCDEFGHY 2.h4! c5 3.h5! c4 4.h6
White changes course 7+-+-+-zp-' 192 8-+-+-+-+( Kb2 ( if 4...Kb3 5.h7 c3
and attacks via the a- 6-+-+-+k+& 7+-zp-+-+-' 193 6.h8Q c2 7.Qa1 wins; or
file. ] 5+-+-zPpzP-% 6-+-+-+-+& 4...Kd2 5.h7 c3 6.h8Q
2.Kb3!! (4 points) 4p+-+-zPp+$ 5+-+-+-+-% c2 7.Qb2 Kd1 8.Kd3
[ Black is in time after 3mKp+-+-zP-# 4-+-+-mK-+$ and Black is soon mated,
2.Kd4 Kd2 3.Ke5 Kc3 2-+-+-+-+" 3+-+-+-+-# c1Q 9.Qe2# is the most
4.Ke6 Kb3 5.Kd7 Kxa3 1+-+-+-+-![ 2-mk-+-+-zP" obvious ) 5.h7 c3 6.h8Q
6.Kxc7 Kxb4 7.Kxc6 a5 xabcdefghy 1+-+-+-+-![ and this time the black
with a draw. ] J.Aagaard, Original 2004. xabcdefghy pawn is pinned. ]
2...Kd2 3.Ka4 Kc3 4.Ka5 . . [ 1...c5 2.Kd5 Kc3
Kb2! An interesting Based on two incorrect White to play and win. 3.Kxc5! ( not 3.h4? c4
defensive try. studies (I will not mention . and Black makes a
[ After 4...Kb3 5.a4 Ka3 the composers), this is a (press F10 for the solution) draw ) 3...Kd3 4.Kd5!
( 5...Kc4 6.Kxa6 Kxb4 small composition of my (the white king is cutting
7.a5 Ka4 8.Kb7 Kxa5 own. It should not be so off the black king) Ke3
9.Kxc7 Kb5 10.Kd6 difficult to solve. The main solution 065 5.Ke5 Kf3 6.Kf5
wins as well ) 6.b5 axb5 agenda here is [Jacob Aagaard] and White wins (5
7.axb5 cxb5 8.Kxb5 visualization and White's points). ]
Kb3 9.c6! White wins as 6th move. 1.e6! 2.Kd4!! (5 points) The only
in the main line. ] (2 points) This is (Diagram 194) winning move.
5.Kxa6! (4 points) The obviously the only move. [ After 2.Kd5?
only winning move. It is all [ After 1.Kb2? Kf7 N.Grigoriev, Shakmatnoe Black draws with Kb4!
about comparison and the Black wins; e.g. 2.Ka3 Tvorchestvo Grigorieva 3.h4 ( or 3.Kc6 Kc4
location of the black king. Ke6 3.Kb2 Kd5 4.Ka3 1954. with a draw ) 3...c5
Excelling at Chess Calculation− Jacob Aagard 58

and he is in time. ] 1...Kb8 2.g3! (5 points) solution 067 XABCDEFGHY


2...Kb4 3.h4 c5+ 4.Ke3!! White needs to play this. [Jacob Aagaard] 8-+-+-+-mk(
(8 points) This way White [ After 2.b6? Black draws 7+-+-+-zp-' 200
gains a tempo. by Kc8!! (2 points) 3.b7+ XABCDEFGHY 6-+-+-+K+&
[ If instead 4.Kd3? Kb3 Kb8 4.g3 c5 5.Kb5 8-+-+-+-+( 5zp-+-+-zP-%
5.Kd2 Kb2 and White Kxb7 6.Kxc5 Kc7 7.Kd5 7+-+-+-+-' 198 4-+-+-sN-+$
cannot win. ] f4! 8.gxf4 Kd7 6-+kzP-+K+& 3+-+-+-+-#
4...Kb3 5.h5 c4 6.h6 c3 and Black has the 5+-+-tr-+-% 2-+-+-+-+"
7.h7 c2 8.Kd2! opposition. ] 4-+-+-+-+$ 1+-+-+-+-![
Forcing Black into the 2...Ka8! 3+-+p+-+-# xabcdefghy
check, otherwise he loses [ 2...Kc8 loses to 3.Ka7 2-+-+-+-+" game White must first
his pawn. Kb2 9.h8Q+ Kd8 4.Kb8 Kd7 5.Kb7 1+-+-tR-+-![ force the black king out of
and wins. Kd8 ( or 5...Kd6 6.Kc8! ) xabcdefghy the corner. He has only
Maximum score: 24 points. 6.Kc6 Kc8 7.Kd5 Kb7 G.Lolli, Osservazioni one way to do this: 1.Kf7!
. 8.Ke5 Kb6 9.Kxf5 Kxb5 Teorico-Practiche sopra il (6 points) a4 The only
(press F10 for the next 10.g4 c5 11.g5 c4 Giuoco degli Scacchi move.
exercise) 12.Ke4! ( not 12.Kf4? c3 1792. [ 1...Kh7 probably loses
13.Ke3 Kb4 14.g6 . to all legal moves. The
Ka3! ) 12...Kb4 13.g6 c3 1.Rxe5! (2 points) easiest way for White to
exercise 066 14.Kd3 Kb3 15.g7 c2 [ Not 1.d7?? Rd5 2.Rd1 win is by 2.Ne6 a4
[Jacob Aagaard] 16.g8Q+ etc. ] Kxd7 and Black wins 3.Nxg7 a3 4.Nf5 a2
3.b6! (3 points) Kb8! easily. ] 5.g6+ Kh8 6.g7+ Kh7
XABCDEFGHY [ 3...cxb6 loses without a 1...d2 2.Rd5!! (6 points) 7.g8Q# . ]
8k+-+-+-+( fight. ] Without this move White 2.Ng6+! (4 points) Kh7
7+-zp-+-+-' 195 4.Kb5!! (5 points) This would lose. Kxd5 3.d7 3.Ne5 a3 4.g6+ Kh6
6-+-+-+-+& time-wasting move is d1Q [ If 4...Kh8 5.Kf8 a2
5mKP+-+p+-% fantastic. [ 3...Kd4 4.d8Q+ Kc3 6.Nf7# . ]
4-+-+-+-+$ [ Instead 4.b7? c5 5.Kb5 does not work either. 5.Ng4+ Kg5 6.Ne3
3+-+-+-+-# Kxb7 6.Kxc5 Kc7 7.Kd5 Only a lone c-pawn and and White wins (3 points).
2-+-+-+P+" f4! is a draw again. ] the a-pawn can Maximum score: 13 points.
1+-+-+-+-![ 4...Kb7 5.bxc7! (3 points) occasionally draw (1 .
xabcdefghy Kxc7 6.Kc5 Kd8! point). ] (press F10 for the next
. Black tries to get the 4.d8Q+ and wins. exercise)
White to play and win. opposition, but... 7.Kd6!! Maximum score: 9 points.
. (2 points) White will not .
(press F10 for the solution) allow it. (press F10 for the next exercise 069
[ Not 7.Kd4? f4 8.gxf4 exercise) [Jacob Aagaard]
Ke8 9.Ke4 Kf8!
solution 066 and White cannot retain XABCDEFGHY
[Jacob Aagaard] the distant opposition. exercise 068 8-+q+-+-+(
Now White wins easily. ] [Jacob Aagaard] 7+-zp-+-+-' 201
XABCDEFGHY 7...Ke8 8.Ke5 f4 9.Kxf4 6N+-+p+-+&
8k+-+-+-+( Kf7 10.Kg5 Kg7 11.g4 XABCDEFGHY 5zp-+-zP-+-%
7+-zp-+-+-' 196 Kf7 12.Kh6 and so on. 8-+-+-+-mk( 4kzpLzp-+-+$
6-+-+-+-+& Maximum score: 25 points. 7+-+-+-zp-' 199 3+-+-+-+-#
5mKP+-+p+-% . 6-+-+-+K+& 2-+P+-+-+"
4-+-+-+-+$ (press F10 for the next 5zp-+-+-zP-% 1+K+-+-+-![
3+-+-+-+-# exercise) 4-+-+-sN-+$ xabcdefghy
2-+-+-+P+" 3+-+-+-+-# .
1+-+-+-+-![ 2-+-+-+-+" White to play and win.
xabcdefghy exercise 067 1+-+-+-+-![ .
N.Grigoriev, Unknown first [Jacob Aagaard] xabcdefghy (press F10 for the solution)
publication 1933. .
. XABCDEFGHY White to play and win.
This is one of my all time 8-+-+-+-+( . solution 069
favourite studies. I like the 7+-+-+-+-' 197 (press F10 for the solution) [Jacob Aagaard]
simplicity of pawn endings 6-+kzP-+K+&
and the feeling of 5+-+-tr-+-% XABCDEFGHY
domination in general. 4-+-+-+-+$ solution 068 8-+q+-+-+(
1.Ka6! (5 points) 3+-+p+-+-# [Jacob Aagaard] 7+-zp-+-+-' 202
Domination and opposition 2-+-+-+-+" 6N+-+p+-+&
are the two themes of this 1+-+-tR-+-![ 5zp-+-zP-+-%
endgame. White wants to xabcdefghy (Diagram 200) 4kzpLzp-+-+$
take the black f-pawn with . 3+-+-+-+-#
Black having the White to play and win. J.Vesely-Antos, USSR 2-+P+-+-+"
opposition. This is . 1968. 1+K+-+-+-![
achieved through (press F10 for the solution) . xabcdefghy
domination of the black The point of the exercise V.Platov & P.Bobrov,
king. is that the white knight will Shakmatno Obozrenji
[ Not 1.b6? Kb7! have to go to the 1909.
and Black makes an queenside to stop the .
immediate draw. ] pawn. In order to win the 1.Kb2 With the threat of 2
Excelling at Chess Calculation− Jacob Aagard 59

Nc5 mate (4 points); this exercise 071 XABCDEFGHY solution 073


is the only serious move. [Jacob Aagaard] 8-+-+-+-+( [Jacob Aagaard]
c5 7+-+-+-+-' 207
[ If 1...Qf8 2.Nxc7 b3 XABCDEFGHY 6-+-+-+-+& XABCDEFGHY
3.cxb3+ Kb4 4.Na6# . ] 8-+-+-tr-+( 5vl-mK-+-+p% 8-+k+-sN-+(
2.Bxe6! Qxa6 7+RzP-+-+-' 205 4-+-+k+P+$ 7+-+-+-+-' 210
[ Or 2...Qc6 3.Bd7! 6-zP-+-+-+& 3+-+-+-+-# 6-+-+PmK-+&
and White wins ( but not 5mK-mk-+-+-% 2-+N+-+-+" 5+-+-+-+-%
3.Nxc5+?? when Black 4-+-+-+-+$ 1vL-+-+-+-![ 4-+-tr-+-+$
escapes with Kb5 4.Bd7 3zp-+-+-+-# xabcdefghy 3+-+-+-+-#
Kxc5 and wins. )] 2-+-+-+-+" 2-+-+-+-+"
3.Bd7+ Qb5 4.e6! 1+-+-+-+-![ 1+-+-+-+-![
(3 points) and White wins. xabcdefghy solution 072 xabcdefghy
Black cannot reach the . [Jacob Aagaard] M.Palmer, Iowa News
passed pawn. White to play and win. 1917.
Maximum score: 9 points. . XABCDEFGHY .
. (press F10 for the solution) 8-+-+-+-+( White of course wants to
(press F10 for the next 7+-+-+-+-' 208 queen his pawn without
exercise) 6-+-+-+-+& allowing Black to sacrifice
solution 071 5vl-mK-+-+p% his rook for it. This is
[Jacob Aagaard] 4-+-+k+P+$ achieved through a nice
exercise 070 3+-+-+-+-# mutual zugzwang. 1.e7!
[Jacob Aagaard] XABCDEFGHY 2-+N+-+-+" (5 points) Rd6+!
8-+-+-tr-+( 1vL-+-+-+-![ It was necessary to
XABCDEFGHY 7+RzP-+-+-' 206 xabcdefghy foresee this check in order
8-+-+-+-+( 6-zP-+-+-+& A.Troitzky, Eskilstuna to solve the exercise, but
7+-+-+-+-' 203 5mK-mk-+-+-% Kuriren 1917. maybe not in order to play
6-tr-+-+-zp& 4-+-+-+-+$ . 1 e7 in a practical game.
5+-+-+-zp-% 3zp-+-+-+-# 1.gxh5 (2 points) This is This is the problem with
4-+-+R+P+$ 2-+-+-+-+" both the natural move and solving exercises in a
3+-+-+-+-# 1+-+-+-+-![ a pleasure to play. Kf5 book rather than working
2p+K+-+-+" xabcdefghy [ If 1...Bd2 2.Bg7 Kf5 with a trainer or an
1mk-+-+-+-![ P.Stamma, Essai sur le 3.Nd4+! wins the black interactive computer
xabcdefghy Jeu des Echecs 1792. bishop, either for the program.
. . pawn or with a knight [ Other moves loses
White to play and win. 1.c8Q+! (4 points) Only in fork on f3. ] without a fight:
. this way can White win; 2.Nd4+ (2 points) Kg5 1...Re4 2.Ne6 Kd7
(press F10 for the solution) [ although 1.c8R+! [ If 2...Kf6 3.Nc6+ . ] 3.Nc5+ and White wins ]
is just as good of 3.Kb5!! (7 points) The [ or 1...Rd8 2.Kf7!
course. ] black bishop is trapped! and Black is in
solution 070 [ The exercise is all This is the point of the zugzwang (3 points). ]
[Jacob Aagaard] about seeing the study. Bd8 2.Kg7!! (8 points) Only
opponent's opportunities. [ Similarly if 3...Bc7 this triangulation wins.
XABCDEFGHY Thus 1.Rb8? 4.Ne6+ ] [ After 2.Kf7 Rd8!
8-+-+-+-+( would lose to the brilliant [ 3...Bd2 4.Nf3+ ] White is in zugzwang
7+-+-+-+-' 204 Rc8!! 2.Rxc8 a2 [ or 3...Be1 4.Nf3+ . ] and cannot prevent both
6-tr-+-+-zp& and Black wins (4 4.Ne6+ (1 point) and White ...Re8 and ...Rd7 on the
5+-+-+-zp-% points). ] wins. next move, after which
4-+-+R+P+$ 1...Rxc8 2.Rc7+ Maximum score: 12 points. Black makes a draw. ]
3+-+-+-+-# Now it is a walk in the . 2...Rd8 3.Kf7! Now Black
2p+K+-+-+" park. (press F10 for the next is in zugzwang. Kc7
1mk-+-+-+-![ Maximum score: 8 points. exercise) 4.Ne6+ and White wins.
xabcdefghy . Maximum score: 16 points.
P.Stamma, Essai sur le (press F10 for the next .
Jeu des Echecs 1792. exercise) exercise 073 (press F10 for the next
. [Jacob Aagaard] exercise)
1.Re1+ Rb1 2.Rc1!!
(6 points) This move must exercise 072 XABCDEFGHY
be found to solve the [Jacob Aagaard] 8-+k+-sN-+( exercise 074
exercise. Black is now in 7+-+-+-+-' 209 [Jacob Aagaard]
zugzwang. Rxc1+ 6-+-+PmK-+&
[ 2...h5 3.gxh5 (Diagram 207) 5+-+-+-+-%
just transposes. ] 4-+-tr-+-+$ (Diagram 211)
3.Kxc1 The position now . 3+-+-+-+-#
resembles Exercise 59. h5 White to play and win. 2-+-+-+-+" .
4.gxh5 g4 5.h6 g3 6.h7 . 1+-+-+-+-![ White to play and win.
g2 7.h8Q# (2 points). (press F10 for the solution) xabcdefghy .
Maximum score: 8 points. . (press F10 for the solution)
. White to play and win.
(press F10 for the next .
exercise) (press F10 for the solution)
Excelling at Chess Calculation− Jacob Aagard 60

XABCDEFGHY exercise 075 12.Qh3# . ] Maximum score: 20 points.


8-+-wqk+-+( [Jacob Aagaard] 7.Ke5! (5 points) and wins. .
7+-zpp+-tr-' 211 [ White could still throw it (press F10 for the next
6-+P+-+-mK& XABCDEFGHY all away with 7.Ke6?? exercise)
5+-+P+P+-% 8-+-+-+-+( c5! and we have a
4-+-+-wQ-+$ 7+-+-+-zPP' 213 draw.
3+-+-+-+-# 6-+p+-mk-mK& Maximum score: 20 exercise 077
2-+-+-+-+" 5+p+-+-+-% points. [Jacob Aagaard]
1+-+-+-+-![ 4-zP-+r+-+$ .
xabcdefghy 3+-+-+-+-# (press F10 for the next XABCDEFGHY
2-+-+-+-+" exercise) ] 8k+-+-+-+(
1+-+-+-+-![ 7+-+-+-+-' 217
solution 074 xabcdefghy 6L+-+P+-+&
[Jacob Aagaard] . exercise 076 5+-+-+-+-%
White to play and win. [Jacob Aagaard] 4-mK-+-+-vl$
XABCDEFGHY . 3+-+-+-+-#
8-+-wqk+-+( (press F10 for the solution) XABCDEFGHY 2-+-+N+-zp"
7+-zpp+-tr-' 212 8-+-+-+-wq( 1+-+-+-+-![
6-+P+-+-mK& 7+-vl-+-+-' 215 xabcdefghy
5+-+P+P+-% solution 075 6-+-+-+-+& .
4-+-+-wQ-+$ [Jacob Aagaard] 5+-+-+-+-% White to play and draw.
3+-+-+-+-# 4-+-zp-+-+$ .
2-+-+-+-+" XABCDEFGHY 3+-+-+-+p# (press F10 for the solution)
1+-+-+-+-![ 8-+-+-+-+( 2-+-+-tR-+"
xabcdefghy 7+-+-+-zPP' 214 1mk-+-+K+R![
E.Pogosiants, Shakhmaty 6-+p+-mk-mK& xabcdefghy solution 077
v SSSR 1964. 5+p+-+-+-% . [Jacob Aagaard]
. 4-zP-+r+-+$ White to play and draw.
1.Qe5+! (2 points) 3+-+-+-+-# . XABCDEFGHY
[ Not 1.Kxg7? Qe7+ 2-+-+-+-+" (press F10 for the solution) 8k+-+-+-+(
2.Kg6 Qf7+ and Black 1+-+-+-+-![ 7+-+-+-+-' 218
makes a draw. ] xabcdefghy 6L+-+P+-+&
1...Qe7 2.Qxe7+! E.Antohi, Revista de solution 076 5+-+-+-+-%
(4 points) Only this Romana de Sah 1954 [Jacob Aagaard] 4-mK-+-+-vl$
surprising exchange wins (from move 4 in a study 3+-+-+-+-#
the game. which had a minor flaw in XABCDEFGHY 2-+-+N+-zp"
[ If instead 2.Qxg7? the beginning). 8-+-+-+-wq( 1+-+-+-+-![
Qh4+ 3.Kg6 Qg4+ . 7+-vl-+-+-' 216 xabcdefghy
4.Kf6 Qd4+ 1.g8N+! (3 points) Any 6-+-+-+-+& V.Platov & M.Platov,
with perpetual check ] other promotion and Black 5+-+-+-+-% Belgorodskaja Pravda
[ or 2.cxd7+ Kd8!! has 1...Rh4 mate. Kf7 4-+-zp-+-+$ 1911.
3.Qxg7 Qh4+ 4.Kg6 [ If 1...Kf5? 2.Kh5 Re1 3+-+-+-+p# .
Qg4+ when Black either 3.Nh6+ wins. ] 2-+-+-tR-+" White has a big problem
has perpetual again or 2.Nf6! (4 points) Rh4+ 1mk-+-+K+R![ in that he cannot stop the
else wins the f-pawn and [ 2...Kxf6 3.h8Q+ wins. ] xabcdefghy black h-pawn, whereas
can safely exchange into 3.Nh5 Rxh5+ 4.Kxh5 H.Rinck, 150 Fins de the black bishop prevents
a drawn pawn ending. ] Kg7 5.Kg5! (6 points) The Partie 1909. his e-pawn from advancing.
2...Rxe7 3.d6!! white king shoulders off . Therefore White needs to
(6 points) This magical the black king, Though the computer is set up a spectacular
move exploits the [ and prevents the book able to find the best stalemate in order to save
unhappy placement of the draw which follows moves for White here, it is the game. 1.Ng3
black pieces. This is what 5.h8Q+? Kxh8 6.Kg6 not able to see that these (5 points) This should not
studies are all about – that Kg8 7.Kf6 Kf8 8.Ke6 draw. The reason is that be too difficult to find as
and tactics... Rf7!? Ke8 9.Kd6 Kd8 10.Kxc6 the computer cannot work all other candidate moves
(6 points) Did you see this Kc8 11.Kxb5 Kb7 . ] with concepts such as the are without ideas. Bxg3
defensive try? You should 5...Kxh7 wrong colour bishop for 2.e7! Bd6+ 3.Ka4!!
have. Otherwise it was [ On 5...c5 the composer the corner. White draws A fantastic move that
pure luck that you would wanted White to play the by means of a stalemate secures the draw. Bxe7
win rather than lose a flashy 6.h8Q+ trap. 1.Rd2!! (8 points) 4.Bc4! h1Q 5.Bd5+! Qxd5
seemingly drawn position. ( but even simpler is Threatening a discovered stalemate (15 points).
[ If instead 3...cxd6 4.c7 6.bxc5 b4 7.c6 b3 8.c7 check after Kf2 or Ke2. Maximum score: 20 points.
wins easily ] b2 9.c8Q b1Q 10.h8Q+ Qa8! (2 points) Answering .
[ or 3...dxc6 4.dxe7 Kf7 11.Qce8# the threat by attacking the (press F10 for the next
Kf7!? 5.e8Q+ Kxe8 , as White already rook in the corner. But exercise)
6.Kg7 and White wins. ] queens first and White is prepared for this.
4.dxc7 Rf6+ 5.Kg7 Rxc6 therefore does not need 2.Kf2+! Qxh1 3.Rd1+!
6.f6 and one of the white the check. )] (10 points) Kb2 exercise 078
pawns queens. 6.Kf6! (2 points) Kh6 [ 3...Qxd1 is stalemate. ] [Jacob Aagaard]
Maximum score: 18 points. [ After 6...c5 7.bxc5 b4 4.Rxh1 h2 5.Kg2
. 8.c6 b3 9.c7 b2 10.c8Q and White sacrifices the .
(press F10 for the next b1Q White mates by, for rook for the d-pawn to White to play and win.
exercise) example, 11.Qd7+ Kh6 leave a book draw.
Excelling at Chess Calculation− Jacob Aagard 61

XABCDEFGHY remember is that the XABCDEFGHY 2.Ng7!! (12 points) Only


8-+-+-+-mK( knight cannot survive on 8-+-+L+-+( this fantastic move wins.
7+-+-+-zp-' 219 its own. When White is 7+-+-+p+p' 222 [ 2.Nf6 looks convincing;
6-+-+-+-+& able to force it away from 6-+-+l+k+& A) ; e.g. 2...Qf8
5+-+-+-+p% the black king, the knight 5+-+-+-wq-% 3.Re8;
4-+-+-+-+$ will eventually be lost. 4-+-+-+-+$ B) , or 2...Qg6 3.Re8+
3+-+-+-+-# Here is some analysis to 3+-+N+R+-# Kg7 4.Rg8+ Kxf6
2R+-+-+-+" back up this claim. Nd3 2-+-+-+-+" 5.Rxg6+ hxg6 6.a5
1+-+-+-mk-![ 9.Ra4 Not the only winning 1+-+-+K+-![ and wins.;
xabcdefghy move, but probably the xabcdefghy C) . But there is a trap:
. fastest. Kh2 White makes a draw 2...Qg1! 3.Re8+?
(press F10 for the solution) [ If 9...Ne5+ 10.Kg3 Kf1 through a nice stalemate. ( 3.Nh5! Qg8 4.Ng7!
11.Rf4+ wins ] 1.Bxf7+! (10 points) Bxf7 would still win of
[ or 9...Nc5 10.Rc4 Nb3 Otherwise Black can never course, but you need
solution 078 11.Ke2 Na5 12.Rc7 win. 2.Nf4+ Kh6 to see the trap to
[Jacob Aagaard] Kh2 13.Kd2 Nb3+ [ 2...Kg7 3.Rg3 understand it ) 3...Kg7
14.Kc3 Nc1 15.Re7 transposes. ] 4.Rg8+ Kh6!! 5.Rxg1
XABCDEFGHY Kg3 16.Re1 Na2+ 3.Rh3+ Kg7 4.Rg3! stalemate (5 points).;
8-+-+-+-mK( 17.Kc4 and White wins. ] Qxg3 Black cannot win D) 2...-- ]
7+-+-+-zp-' 220 10.Re4 Kg1 with bishop and pawn vs. 2...h5
6-+-+-+-+& [ Or 10...Nc5 11.Re5 knight here, so why bother. [ If 2...Qxg7 3.Re8+ Qg8
5+-+-+-+p% Nd3 12.Rd5 Nb4 5.Nh5+! (10 points) Bxh5 4.Rxg8+ Kxg8 5.a5
4-+-+-+-+$ 13.Rd2+ Kh3 ( or stalemate. wins ]
3+-+-+-+-# 13...Kg1 14.Rg2+ Kf1 Maximum score: 20 points. [ or 2...Kxg7 3.Rg2+ Kf8
2R+-+-+-+" 15.Rb2 ) 14.Rd6 Kh2 . 4.Rxg8+ etc. ]
1+-+-+-mk-![ 15.Ke3 and the black (press F10 for the next 3.Re8 Kxg7 4.Rxg8+
xabcdefghy knight is cornered. ] exercise) Kxg8 5.a5 and White wins
J.Moravec, 10. prize, La 11.Rd4 Nc5 12.Rd5 by one tempo (5 points).
Strategie 1912. The black knight is Maximum score: 30 points.
. trapped no matter where it exercise 080 .
1.Kh7! (10 points) goes. Nb7 [Jacob Aagaard] (press F10 for the next
[ White needs to ignore [ 12...Ne6 13.Kg3 Kf1 exercise)
the black g-pawn as after 14.Rf5+ Kg1 15.Re5 XABCDEFGHY
1.Kxg7? h4 wins. ] 8-+-+-+-mk(
White cannot stop the h- 13.Ke3 Kg2 14.Kd3 Kf3 7+-+-+p+p' 223 exercise 081
pawn from promoting. It 15.Kc3! The fastest way to 6-+-+-+-+& [Jacob Aagaard]
seems that 1 Kh7 hardly win the knight. The threat 5+-+-+-+N%
makes any difference, is 16 Kb4. Ke4 16.Kc4 4P+P+-+-+$ XABCDEFGHY
but there is a twist. ] Zugzwang. Kf4 17.Kb4 3+-+-+-+-# 8-+-+-+-+(
1...h4 Ke4 18.Rd7 and White 2K+R+-+-+" 7+-+-+-vL-' 225
[ Black cannot save wins. 1+-+-+-wq-![ 6-tr-zP-+-+&
himself with 1...g5 2.Kg6 Maximum score: 27 points. xabcdefghy 5+-+-+-+-%
g4 3.Kg5! g3 4.Kh4 . . 4-+-+K+-+$
( 4.Kf4 also wins ) 4...g2 (press F10 for the next White to play and win. 3+-+-+-+-#
5.Kh3 Kh1 6.Rxg2 exercise) . 2-+-+-+-vl"
as there is no (press F10 for the solution) 1+-mk-+-+-![
stalemate. ] xabcdefghy
2.Kg6 h3 3.Kg5 h2 exercise 079 .
4.Kg4 g5! The only try for [Jacob Aagaard] solution 080 White to play and draw.
a defence. [Jacob Aagaard] .
[ The point of 1 Kh7 is XABCDEFGHY (press F10 for the solution)
seen after 4...h1Q?! 8-+-+L+-+( XABCDEFGHY
5.Kg3! (5 points) and the 7+-+-+p+p' 221 8-+-+-+-mk(
threat of 6 Ra1+ is 6-+-+l+k+& 7+-+-+p+p' 224 solution 081
decisive. Had White 5+-+-+-wq-% 6-+-+-+-+& [Jacob Aagaard]
played 1 Kxg7? then 5 4-+-+-+-+$ 5+-+-+-+N%
Kg3? would fail to 5... 3+-+N+R+-# 4P+P+-+-+$ XABCDEFGHY
Qh8! covering a1, but 2-+-+-+-+" 3+-+-+-+-# 8-+-+-+-+(
here the g7-pawn blocks 1+-+-+K+-![ 2K+R+-+-+" 7+-+-+-vL-' 226
the long diagonal. ] xabcdefghy 1+-+-+-wq-![ 6-tr-zP-+-+&
[ 4...h1N 5.Kf3! . xabcdefghy 5+-+-+-+-%
(2 points) holds no White to play and draw. D.Przepiorka, Szachista 4-+-+K+-+$
chances for Black. One . Polski 1920. 3+-+-+-+-#
possible finish is g5 (press F10 for the solution) . 2-+-+-+-vl"
6.Rd2 g4+ 7.Kxg4 Nf2+ White wins the queen, but 1+-mk-+-+-![
8.Kf3 Nh3 9.Kg3 it is necessary to spot xabcdefghy
and Black loses the solution 079 Black's trap. 1.Re2 L.Kubbel, Listok
knight. ] [Jacob Aagaard] (5 points) Qg8 Easily the Shakhmatnogo Kruzhka
5.Kg3 h1N+ 6.Kf3 g4+ only move. Petrogubkomunni 1921.
7.Kxg4 Nf2+ 8.Kf3! G.Ling, Deutsche [ If 1...h6 2.Re8+ Kh7 .
(10 points) White is now Schachzeitung 1912. 3.Nf6+ Kg6 4.Rg8+ White makes a draw by
winning. The main thing to . wins (3 points). ] playing on different horses.
Excelling at Chess Calculation− Jacob Aagard 62

Here pawn vs rook is a 6.Ke7 Rxe5+ 7.Kd7 [ Or 5...Kg6 6.Qg2+ a3 ( and 3...b2+!?
draw, and the same goes Rd5+ 8.Kc7 Rc5+ 9.Kb7 and 7 Qxb2 wins. ] 4.Rxb2 Rxf6 5.Bd4
for queen vs rook and Rb5+ 10.Kc6! (8 points) 6.Qa7+ Kg6 7.Qg7# . Rf1+ 6.Kc2 a3!! 7.Rb1+
bishop. So, with these two Coming at last to sixth Maximum score: 20 points. Ka2 8.Rxf1 stalemate. )]
options ruled out for Black, rank, now that the black . 3...Rxg5
White is able to rook cannot go to the (press F10 for the next [ 3...a3 loses to 4.Rd1!!
manoeuvre himself into back. Rb6+ 11.Kc5! Ra6 exercise) (5 points) when Black
the corner. 1.Be5! [ If 11...Rb5+ 12.Kc4 cannot do anything; e.g.
(5 points) Rb4+ 2.Kd5 and the pawn Rd6 5.f8Q b2+ 6.Kc2+
Rb5+ 3.Kc6!! (10 points) promotes. ] exercise 084 Rxd1 7.Qxa3# . ]
Giving up the bishop is 12.axb4# (10 points) A [Jacob Aagaard] 4.f8Q Rg1+ 5.Rd1 Rg2!
the only way to succeed. very beautiful end. This amazing counterplay
Of course this had to be Maximum score: 30 points. XABCDEFGHY should have been
foreseen. Rxe5 4.d7 . 8-+-+-+-+( foreseen. Now White can
Re6+ 5.Kb7 Rd6 6.Kc8! (press F10 for the next 7+-+-+-+-' 231 win in only one way.
(10 points) Threatening to exercise) 6-+-+-zPr+& 6.Qa3+ Ra2 7.Rd2!
queen the pawn. Rc6+ 5+-+R+-+-% (10 points) The only way
7.Kb7 Rc7+ 8.Ka8! Rxd7 4pzp-+-+-+$ to win.
stalemate (5 points). exercise 083 3+-+-+-+-# [ 7.Qc5 does not achieve
Maximum score: 30 points. [Jacob Aagaard] 2k+-vL-+-+" anything after Rh2!
. 1+-mK-+-+-![ ( but not 7...Rg2?
(press F10 for the next XABCDEFGHY xabcdefghy 8.Rd2!; , or 7...b2+?
exercise) 8-+-+-mK-mk( . 8.Kd2+ b1Q+ 9.Ke1!
7zp-+-+-+-' 229 White to play and win. and White wins. )]
6P+-+-+pzp& . 7...Rxa3
exercise 082 5+-+-+-+p% (press F10 for the solution) [ Or 7...b2+ 8.Qxb2+
[Jacob Aagaard] 4p+-+-zP-+$ and White wins after
3+p+R+-+-# Rxb2 9.Rxb2 a3
XABCDEFGHY 2-+-+-+-+" solution 084 10.Rb1+! Ka2 11.Rb8
8-+-+-+-+( 1+-+-+-+-![ [Jacob Aagaard] Ka1 12.Kc2 a2 13.Kb3!
7+k+-+-+K' 227 xabcdefghy Kb1 14.Rh8 a1N+
6p+-+-+-+& . XABCDEFGHY 15.Kc3! ]
5tRp+-zP-tr-% White to play and win. 8-+-+-+-+( 8.Rb2! A nice zugzwang.
4-zpP+-+-+$ . 7+-+-+-+-' 232 Ra2 9.Rb1# .
3zPP+-+-+-# (press F10 for the solution) 6-+-+-zPr+& Maximum score: 30 points.
2-+-+-+-+" 5+-+R+-+-% .
1+-+-+-+-![ 4pzp-+-+-+$ (press F10 for the next
xabcdefghy solution 083 3+-+-+-+-# exercise)
. [Jacob Aagaard] 2k+-vL-+-+"
White to play and win. 1+-mK-+-+-![
. XABCDEFGHY xabcdefghy exercise 085
(press F10 for the solution) 8-+-+-mK-mk( G.Kasparian, 1st prize, [Jacob Aagaard]
7zp-+-+-+-' 230 Shakhmaty v SSSR 1939.
6P+-+-+pzp& . XABCDEFGHY
solution 082 5+-+-+-+p% 1.Bg5! (5 points) This 8-+-+-+-+(
[Jacob Aagaard] 4p+-+-zP-+$ move queens the pawn by 7+-+R+P+-' 233
3+p+R+-+-# force. 6-zppzP-zp-+&
XABCDEFGHY 2-+-+-+-+" [ If instead 1.f7 Rf6 5+-+r+-+-%
8-+-+-+-+( 1+-+-+-+-![ and Black makes a 4-+-+-+-+$
7+k+-+-+K' 228 xabcdefghy draw. ] 3mK-+-+-+-#
6p+-+-+-+& V.Kitsigin, Magyar [ Or 1.Rf5? Rg1+ 2.Kc2 2p+-+-mk-+"
5tRp+-zP-tr-% Sakkelet 1978. b3+ 3.Kc3 b2 4.f7 1+-+-+-+-![
4-zpP+-+-+$ . Rc1+ 5.Kd4 Rc8 xabcdefghy
3zPP+-+-+-# White wins by a nice rook and it will be White who .
2-+-+-+-+" manoeuvre and sacrifice. has to make a draw. ] White to play and win.
1+-+-+-+-![ 1.f5! (5 points) gxf5 1...b3! .
xabcdefghy [ 1...b2?! 2.fxg6 b1Q [ On 1...Rg8 2.Rd2+!! (press F10 for the solution)
A.Selesniev, Tidskrift för 3.g7+ Kh7 4.g8Q# . ] (5 points) ( but not 2.f7?
Schack 1921. 2.Rd6 Kh7 It seems as if Rc8+ 3.Kd2 Rf8 4.Rd7
. White has achieved very b3 and Black makes a solution 085
1.cxb5! (3 points) Kb6! little, but he wins with a draw ) 2...Kb3 [Jacob Aagaard]
On all other moves the fantastic move. 3.Rb6!! (the black king is now
rook endgame is lost. (15 points) axb6 worse placed) 3.f7 Rf8
2.bxa6! (7 points) A [ Ignoring the threat of 4 ( or 3...Rc8+ 4.Kb1 a3 (Diagram 234)
common trick in rook Rb7+ does not work for 5.Rd3+ and 6 Rd8 ) 4.Rf2
endings. Black cannot Black either: 3...f4 a3 5.Kb1 and White A.Herbstmann,
catch the pawn. Kxa5 4.Rb7+ Kg6 5.Rxa7 b2 wins. ] Achalgazdra Kommunisti
3.a7 Rh5+ 4.Kg7! ( 5...f3 6.Rg7+ ) 6.Rb7 2.Rd2+ Ka1 3.f7! 1954.
(2 points) a3 7.a7 a2 8.a8Q b1Q (5 points) The only way to .
[ Not 4.Kg6? Rh8 9.Rg7+ Kf6 10.Qc6+ win. This exercise is about
and draws. ] with a mating attack. ] [ 3.Be3? allows Black to awareness of the
4...Rg5+ 5.Kf7 Rf5+ 4.a7 b2 5.a8Q b1Q make a draw with both opponent's counter-
Excelling at Chess Calculation− Jacob Aagard 63

XABCDEFGHY solution 086 solution 087 Maximum score: 27 points.


8-+-+-+-+( [Jacob Aagaard] [Jacob Aagaard] .
7+-+R+P+-' 234 (press F10 for the next
6-zppzP-zp-+& XABCDEFGHY XABCDEFGHY exercise)
5+-+r+-+-% 8-+-sN-+-+( 8-vl-+-+-+(
4-+-+-+-+$ 7+-+-+-+-' 236 7+p+-+-zp-' 238
3mK-+-+-+-# 6-+-+-+-+& 6-+-zP-+-+& exercise 088
2p+-+-mk-+" 5+-+-mkp+-% 5zP-zP-+-+p% [Jacob Aagaard]
1+-+-+-+-![ 4-+-+-+-+$ 4-+-zP-+-+$
xabcdefghy 3+KzP-+-+-# 3+-+-+-+-# XABCDEFGHY
chances. 1.Kb2!! 2-vL-+-+p+" 2-+-+-+k+" 8-+-vl-+-vL(
(10 points) Only this move 1+-+-+-+-![ 1+-+-mK-+-![ 7+-+P+-+-' 239
wins. xabcdefghy xabcdefghy 6-+-+Pzp-+&
[ If 1.Kxa2? f5!! H.Rinck, E.Barca, Tidskrift för 5+-zP-zp-+-%
(5 points) and White Mecklenburgisches Schack 1916. 4-mK-+-+-zp$
cannot prevent perpetual. Neues Wochenblatt 1911. . 3+-+-+-+-#
The black rook checks . 1.Ke2!! (10 points) White 2-+k+-+n+"
continually either on the 1.Nf7+ (2 points) Clearly needs to step out of the 1+-+-+-+-![
e-file 2.Ra7 the only move. checks. xabcdefghy
( or on the fifth rank [ 1.c4+? Ke4! ] [ 1.d7? Bc7 2.c6? .
2.f8Q Ra5+ etc ) 2...Re5! [ and 1.Nc6+ Ke4 Bxa5+ wins. ] White to play and draw.
3.f8Q Re2+ etc. ] both win immediately for [ 1.c6? Bxd6 2.cxb7 h4 .
1...a1Q+ Black. ] 3.a6 Bb8 4.d5 h3 5.d6 (press F10 for the solution)
[ If 1...Rb5+ 2.Ka1! 1...Kd5 h2 and Black wins. ]
and the perpetual is [ If 1...Kf4 2.Bc1+ Kf3 [ 1.d5? h4 2.d7 Bc7
gone ] 3.Ng5+ Kg4 4.Be3 f4 3.a6 h3! 4.a7 h2 5.a8Q solution 088
[ or 1...Ra5 2.Ka1 Ra8 5.Bg1 Kxg5 6.Kc4 Kf5 h1Q+ 6.Kd2 ( or 6.Ke2 [Jacob Aagaard]
3.Re7 and Black cannot 7.Kd3 and White wins ] Qh5+ ) 6...Bf4+ 7.Kc2
prevent both pawns [ 1...Ke4 2.Ng5+ Kf4 Qc1+ 8.Kd3 Qe3+ XABCDEFGHY
from queening. ] 3.Bc1+ Kg4 9.Kc4 Qe2+ 10.Kb3 8-+-vl-+-vL(
2.Kxa1 Ra5+ transposes. ] Qb5+ wins. ] 7+-+P+-+-' 240
[ If now 2...f5 3.Ra7 Re5 2.c4+ Kc5! (3 points) The [ 1.a6? bxa6 2.d5 h4 6-+-+Pzp-+&
4.Ra2+! and White best try. If the king moves 3.d7 Bc7 4.d6 Ba5+ 5+-zP-zp-+-%
wins. ] to a light square the white 5.Ke2 h3 6.c6 h2 7.c7 4-mK-+-+-zp$
3.Kb2 Rb5+ knight gives a check and h1Q 8.d8Q 3+-+-+-+-#
[ Or if 3...f5 4.Ra7!! makes it back to cover g1 is more complicated, but 2-+k+-+n+"
Rxa7 5.f8Q Ra5 6.d7 in time. 3.Ng5 g1Q certainly White is not 1+-+-+-+-![
Rb5+ 7.Kc3 Rc5+ 4.Bd4+!! (10 points) going to win. ] xabcdefghy
8.Qxc5+ and wins (2 Without this White would 1...h4 As this is not the F.Lazard, La Strategie
points). ] be lost. Now he is winning; most useful move Black 1921.
4.Kc3 Rc5+ 5.Kd4 f5 [ i.e. 4.Bd4+ Qxd4 ( or can play, White has now .
6.Ra7! (5 points) Still the 4...Kxd4 5.Nf3+ ) 5.Ne6+ improved his chances on White escapes from this
only winning move. Rd5+ etc. the queenside and can endgame by means of an
7.Kc3 Rc5+ 8.Kb2 Rb5+ Maximum score: 15 make a swift breakthrough. original stalemate. 1.Bxf6!
9.Ka1 Re5 10.Ra2+! points. 2.d7 Bc7 3.c6! (5 points) [ 1.c6!? Nf4 2.Bxf6!
(6 points) and wins. The . bxc6 works in the same way. ]
perpetual check net is (press F10 for the next [ If 3...h3 4.cxb7 h2 1...Bxf6 2.c6 Nf4
gone. exercise) ] 5.b8Q Bxb8 6.d8Q h1Q [ 2...Ne3 3.c7 Nd5+
Maximum score: 28 points. 7.Qd5+ Kg1 8.Qxh1+ also transposes. ]
. Kxh1 9.d5 3.c7 Nd5+ 4.Kc4!!
(press F10 for the next exercise 087 and the bishop cannot Here and only here. This
exercise) [Jacob Aagaard] hold both pawns ] is the beauty of
[ or 3...b5 4.a6 h3 5.a7 stalemates. Nxc7 5.d8Q
XABCDEFGHY h2 6.d8Q! Bxd8 Bxd8 6.e7! Bxe7
exercise 086 8-vl-+-+-+( ( 6...h1Q 7.Qd5+ ) 7.c7 stalemate (25 points).
[Jacob Aagaard] 7+p+-+-zp-' 237 and White wins. ] Maximum score: 25 points.
6-+-zP-+-+& 4.a6 h3 5.a7 h2 6.a8Q .
XABCDEFGHY 5zP-zP-+-+p% h1Q 7.Qxc6+ Kg1 (press F10 for the next
8-+-sN-+-+( 4-+-zP-+-+$ 8.Qc1+! (10 points) exercise)
7+-+-+-+-' 235 3+-+-+-+-# [ After 8.Qxc7 Qe4+
6-+-+-+-+& 2-+-+-+k+" it is not easy to escape
5+-+-mkp+-% 1+-+-mK-+-![ the checks. ] exercise 089
4-+-+-+-+$ xabcdefghy 8...Kg2 9.Qg5+ Kh3 [Jacob Aagaard]
3+KzP-+-+-# . [ 9...Kh2 10.Kf2!
2-vL-+-+p+" White to play and win. wins as below. ]
1+-+-+-+-![ . 10.Qh5+! Kg2 11.Qg4+ (Diagram 241)
xabcdefghy (press F10 for the solution) Kh2
. [ 11...Bg3 12.Qf3+ .
White to play and win. wins. ] White to play and win.
. 12.Kf2 (2 points) Black .
(press F10 for the solution) cannot prevent mate for (press F10 for the solution)
long.
Excelling at Chess Calculation− Jacob Aagard 64

XABCDEFGHY White is able to dance wins because the black easiest of the positions in
8K+-+-+-+( around the black knight. rook cannot remain on the set of calculation
7+-+-+-+-' 241 [ Instead 6.Kc7? Nd4! the rank ) 3.Kf7+ Bd8 exercises. It is from one of
6-+-+-zP-+& 7.Kd6 Nf5+ 8.Kc5 Ng7! 4.h6 Rd7+ 5.Ke8 wins. ] my own games – in which
5+-+-+-+-% draws. ] 2.g5!! (10 points) This I displayed bad calculation
4-sn-+-+-+$ 6...Nd4 7.Ke7 Nf5+ 8.Kf6 brilliant idea changes technique, by not looking
3+-+-+-mkl# Nd6 9.f8Q and White wins everything. for opportunities at the
2-+-+-+-+" (8 points). The endgame of [ Not 2.Rc8? Rc1! 3.Rd8 end of the variation.
1+-+-+-+-![ Q v B+N is virtually Rc7 and White is lost. ] 68...Bh4+?
xabcdefghy always won for the queen. 2...Bxg5 3.h6! (5 points) [ The solution is 68...Rd2!
Maximum score: 35 points. The idea is to open the h- (5 points). White only
. file, after which the white has 69.Rxe3 Bxe3
solution 089 (press F10 for the next rook can threaten mate 70.h7 Rd8 71.Bg8
[Jacob Aagaard] exercise) from two directions. Bxh6 , but now it looks as if
The only move, otherwise Black has been swindled.
XABCDEFGHY 4 Kf7 or 4 hxg7 wins. 4.f4!! This is where I stopped
8K+-+-+-+( exercise 090 (10 points) The point. Now anyway. However, Black
7+-+-+-+-' 242 [Jacob Aagaard] Black has no way to wins with Kf3!
6-+-+-zP-+& defend himself. Note that (15 points) 72.h8Q Bf2+
5+-+-+-+-% XABCDEFGHY this move prevents 4... 73.Kf1 Rd1#
4-sn-+-+-+$ 8-+-+-mK-mk( Rc1 and therefore . The lesson: always look
3+-+-+-mkl# 7+p+-+-zp-' 243 threatens 5 Kf7 strongly. for candidates at the end
2-+-+-+-+" 6p+-+-+Pvl& Bxf4 There is nothing of a forced line. There
1+-+-+-+-![ 5zP-+-zp-+P% better. may be a final twist you
xabcdefghy 4-+-+-+P+$ [ If 4...Rd1 5.Ke7 need to attend to! ]
I.Vandecasteele, Eclaireur 3tR-+-+P+-# mates. ] 69.Ke2 Bf2! (5 points)
de Nice 1999. 2-+-+-+-tr" 5.Kf7! Rc1 6.Rh3+ Bh6 Though I have obviously
. 1+-+-+-+-![ 7.Rxh6+ gxh6 8.g7+ been stupid, I am still
White wins by queening xabcdefghy Kh7 9.g8Q# . playing with an idea. The
his pawn, but this by no . Maximum score: 30 points. point is that the black rook
means as easy as it looks. White to play and win. . now can come to the h-file.
The first move is obvious, . (press F10 for the next 70.Rxe3! Otherwise 70...
but then it becomes (press F10 for the solution) exercise) Rd2 mate. Bxe3 71.h7?!
complicated. 1.f7 Bg2+ Surprisingly this loses a
2.Kb8! (8 points) Only this tempo. White can still draw,
wins. solution 090 exercise 091 but now it becomes rather
[ Black is better after [Jacob Aagaard] [Jacob Aagaard] more challenging. At first
2.Ka7? Nc6+ 3.Kb6 ( or it even seems as if 71 h7
3.Ka6 Bf1+! 4.Kb6 XABCDEFGHY XABCDEFGHY gains a tempo which
Ne5 ) 3...Ne5! 4.f8N 8-+-+-mK-mk( 8-+-+-+-+({ allows White to eliminate
. This endgame with B+N 7+p+-+-zp-' 244 7+-+-+-+-' 245 the black a-pawn. I
vs. N arises all the time 6p+-+-+Pvl& 6-+-+Rzp-zP& considered this pawn as
in the analysis. The 5zP-+-zp-+P% 5zp-+-+-vl-% something of a meal ticket
main point is not whether 4-+-+-+P+$ 4PzP-tr-+-+$ during the game. However,
it is winning for Black 3tR-+-+P+-# 3+L+-zp-+-# Jonathan Tait has
or drawn, but that it is 2-+-+-+-tr" 2-+-+-+k+" improved my analysis in
clearly undesirable for 1+-+-+-+-![ 1+-+-mK-+-! the preparation of this
White. ] xabcdefghy xabcdefghy book, and he shows that
2...Nc6+ 3.Kc7! (4 points) Y.Afek, Pula Congress . the contrary is actually the
[ Not 3.Kc8? Ne7+! 1997. Black to play and win. case. The theme of the
and 4...Ng6. ] . . best way for White to
3...Nd4 4.Kd6! White wins in a most (press F10 for the solution) defend is domination.
(10 points) Domination extraordinary fashion with [ White should have
again. a series of pawn sacrifices. played 71.Kxe3!
[ If 4.Kd7? Bh3+ 1.Rc3! (5 points) solution 091 A) , and if 71...Rxb4
and 5...Ne6 draws. ] Threatening 2 Rc8 and [Jacob Aagaard] he can defend his
4...Nf5+ 5.Kd7!! mates. advanced h-pawn by
(10 points) Only this move [ Instead 1.Rd3? Rd2 XABCDEFGHY 72.Bc2 Kg3 73.h7
wins. wins for Black ] 8-+-+-+-+({ , which gives him
[ If 5.Kc5? Ng7! ] [ while 1.Rb3? Rf2 7+-+-+-+-' 246 sufficient counterplay.
[ or 5.Ke6? Bd5+!! also gets White 6-+-+Rzp-zP& My line continued Rh4
6.Kxd5 Ne7+ nowhere. ] 5zp-+-+-vl-% 74.Bf5 Rh2 75.Ke4
and 7...Ng6 draws ] 1...Rh1 The best defence, 4PzP-tr-+-+$ Rh1 , but here
[ while if 5.Ke5? Ne7 which should have been 3+L+-zp-+-# Jonathan came up with
6.Kf6 ( 6.Ke6 Bd5+ anticipated. Now White 2-+-+-+k+" 76.Kd5! Kf4 77.Ke6
7.Kxe7 Bxf7 ) 6...Nd5+ wins by opening a second 1+-+-mK-+-! Rh6 ( or 77...Kg5
7.Kg6 ( 7.Kg7 Nc7! ) line of attack. xabcdefghy 78.Bc2 ) 78.Bg6 Kg5
7...Nf4+ 8.Kf5 Bh3+ [ Instead after 1...Rf2 E.Lobron-J.Aagaard, 79.Kf7 f5 80.Bxf5
and 9...Ne6 draws. ] 2.Rc8 Rxf3+ 3.Ke7+ Stockholm 2003. Kxf5 81.Kg7
5...Bc6+! Black is mated ] . with a draw.;
Did you anticipate this? [ or if 1...Rd2 2.Rc8 Bg5 Though this position is not B) . If Black tries to
6.Kd8!! (5 points) Now ( 2...Rd7 3.Ke8! too easy, I think it is the transpose to the game
Excelling at Chess Calculation− Jacob Aagard 65

with 71...Rh4 exercise 092 g5 18.Re4 f6 19.Nbc7 exercise 093


then White plays [Jacob Aagaard] Rb8 20.Bb5 Qa3 [Jacob Aagaard]
instead 72.bxa5! Rh3+ 21.Re3 Qd6 22.Bxa7
( or 72...Rxh6 73.Bd5+ XABCDEFGHY Ng4 23.Rh3 b6 XABCDEFGHY
and 74 a6 etc ) 73.Ke4 8r+l+-trk+({ 24.Qd3 Qc5+ 25.Ne3 8-+-+rtrk+(
Rxb3 74.Kf5 Rh3 7zpp+-zppvlp' 247 f5 26.Bxb8 Nxe3 7+-+-+pzpp' 248
75.Kg6 and White 6-wqnzp-snp+& 27.Rxe3 f4 6p+l+-+-+&
draws after Rg3+ ( not 5+-+-zP-+-% and Black might be 5+p+-vL-+n%
75...f5 76.h7 f4?? 4-+-sN-zP-+$ better, though it looked 4-+-+P+-+$
77.a6 and White wins ) 3+-sN-vL-+-# very suspicious on the 3vl-sNL+R+-#
76.Kxf6 Rh3 77.Kg7 2PzPP+L+PzP" way. No matter what, 2-zPP+-+PzP"
Rg3+ etc.; 1tR-+Q+RmK-! White should not go for 1+-+-tR-+K![
C) 71...-- ] xabcdefghy this, but for the simple xabcdefghy
71...Rh4 72.Kxe3 Rxh7 . line with 15 Ned5 .
[ 72...f5 Black to play and win. which offers good White to play and win.
is no improvement: . play. ( 27...-- ); .
73.Bd5+ Kg3 74.bxa5 (press F10 for the solution) C) 15.-- ] (press F10 for the solution)
f4+ 75.Kd3 Rxh7 15.Bxe5
76.Kc4! reaches a [ 15.Ncd5? Nxd5
similar position to that solution 092 16.Nxd5 is met by solution 093
after 74 bxa5 below but [Jacob Aagaard] Qxd4+! 17.Qxd4 Nf3+ [Jacob Aagaard]
with an extra – and and Black wins. ]
more advanced – a- L.Hansen-J.Aagaard, [ One of the critical lines XABCDEFGHY
pawn for White. ] Taastrup 1999. to see was 15.Nxc8 8-+-+rtrk+(
73.Bd5+! I had overlooked . and then: 7+-+-+pzpp' 249
this simple move. I was The position arose in the A) 15...Raxc8 16.Rb1 6p+l+-+-+&
looking forward to 73... Sicilian Defence after 1.e4 Qe7 17.Rb5! Ne8 5+p+-vL-+n%
Rh3+. Kg3 c5 2.Nf3 d6 3.d4 cxd4 18.Nd5 Qd7 19.Bxa7 4-+-+P+-+$
Here my experienced 4.Nxd4 Nf6 5.Nc3 g6 with equality.; 3vl-sNL+R+-#
opponent throws the draw 6.Be2 Bg7 7.Be3 Nc6 B) 15...Rfxc8 16.Rb1 2-zPP+-+PzP"
away. 74.b5?? 8.0-0 0-0 9.f4 Qb6 Qe7 17.Rb5 Nc6 1+-+-tR-+K![
[ He should have played 10.e5!? . This gambit was 18.Bxf6 Bxf6 19.Nd5 xabcdefghy
74.bxa5 Ra7 75.Kd4 unknown to me and Bd4+ 20.Kh1 Qd7 A.Shirov-C.Lutz, German
Rxa5 76.Bc6 f5 77.Kc4 hopefully also to you. I 21.Nf6 Qe7 22.Nd5 League 2001.
f4 78.Kb4 Ra6 and now managed to calculate the repeating moves.; .
79.Bh1 Rh6 ( or if 79...f3 theoretical refutation at C) 15...Rd8!! Black has entered a
80.Bxf3! with a book the board. Did you? dxe5 (5 points) 16.Nb5 desperado. Fortunately
draw ) 80.Be4! 11.fxe5 Nxe5 12.Nf5 Raxc8 and White has you have read that
(only here) and Black Qxb2! (5 points) Greedy little for his pawn. chapter and should be
cannot make progress. ] and necessary. ( 16...-- ); able to solve this position
74...Rh4 [ 12...Qe6 13.Nxg7 D) 15...-- ] without too much effort.
[ Not 74...f5? 75.b6 f4+ Kxg7 14.Qd2 Kg8 15...Qxe7 16.Qd4 Nh5 Right? 21.Bxg7!
76.Kd4 f3 77.b7! 15.Rae1 gives White [ 16...Ne8!? is equally (5 points) This is the only
when Black has to be good play for the pawn ] good. ] move.
careful: Rh4+ ( not [ while after 12...Qc7 17.Bxg7+ Nxg7 [ 21.Bd4? Bxb2 22.Nxb5
77...Rh8? 78.Kc5 f2 13.Nb5! Qb8 14.Nxe7+ Black has won a pawn axb5 23.Bxb2 f5
79.Bc4 Kg2 80.Kb6 f1Q Kh8 15.Qd6! White is and stands much better. gives Black a strong
81.Bxf1+ Kxf1 82.Kxa5! even better. ] 18.Rae1 Be6 19.Nd5 initiative. ]
and White wins ) 78.Kc5 13.Nxe7+ Kh8 14.Bd4 Bxd5 20.Qxd5 Rad8 21...Bxb2
Rb4 79.Bc6! f2 80.Bb5 This was what White was 21.Qb3 b6 22.Kh1 Qc7 [ After 21...Kxg7 22.bxa3
f1Q 81.b8Q+ Qf4 preparing. 23.Rf3 Rde8 24.Rf2 f5 f5 23.Ref1! fxe4
82.Qxf4+ Kxf4 83.Kb6 [ 14.Nb5 Qb4! 25.Rd1 Qc5 26.Rdf1 Re3 24.Rxf8 Rxf8 25.Rxf8
with a draw. ] does not help White. ] 27.Bd3 Rfe8 28.Qf7 Kxf8 26.Bxe4
75.Be4 f5 76.Bxf5 Rxa4 14...Qb4!! (10 points) This R3e7 29.Qb3 Nh5 White keeps his extra
77.Kd2 Rb4 78.Bd3 Kf4 move had to be foreseen 30.Bb5 Nf6 31.Qb2 Rf8 pawn for the endgame. ]
[ 78...a4! 79.Kc3 a3 in order to capture on b2. 32.Rd2 Qe5 33.Rd4 Kg7 22.Bxf8 Bxc3 23.Ref1!
80.Kxb4 a2 would have [ After 14...Ng8 34.Qb4 a5 35.Qa3 Rc7 (10 points) As so often
been slightly faster. ] A) , White plays 36.Rc4 Rxc4 37.Bxc4 before, the desperado is
79.Kc3 Ke5 80.Bc4 Kd6 15.Ned5! Rd8 16.Nb5 Ng4 38.Qh3 Rc8 39.Bd3 decided by a surprisingly
81.Kd4 a4 82.Kc3 Kc5 Nf3+ 17.Bxf3 Qxb5 Rxc2! 40.Qh4 quiet move. White does
83.Bf7 a3 84.Bb3 Rxb5 18.Bxg7+ Kxg7 [ If 40.Bxc2 Nf2+! not lose material because
85.Kc2 Kd4 and White 19.Qd4+ f6 20.c4 41.Rxf2 Qe1+ mates. ] all Black's minor pieces
resigned. Qa6 21.Rae1 40...Nf2+ 41.Rxf2 Qe1+ are hanging.
Maximum score: 20 points. with strong 42.Bf1 Rxf2 43.Qd4+ [ Not 23.Rd1? Kxf8!
. compensation for the Kf7 44.Qd7+ Qe7 24.Rf5 Re5 and Black
(press F10 for the next pawn.; 45.Bc4+ Kf8 46.Qd1 wins. ]
exercise) B) . The alternative Qe4 47.Qd8+ Kg7 23...Rxf8
15.Nb5 and White resigned. [ 23...Kxf8 loses to
is very complicated. I Maximum score: 20 points. 24.Rxf7+ Kg8 25.R7f5
have analysed one line . Ng3+ ( or 25...Re5
which goes Qb4 (press F10 for the next 26.Rxe5 Bxe5 27.Rf5
16.Nd5 Qa4 17.Rf4 exercise) and wins ) 26.hxg3 Bxe4
Excelling at Chess Calculation− Jacob Aagard 66

27.Rg5+ Bg7 28.Bxe4 Unfortunately it is not very Kxf7 38.h3 Qxf2+ not be too difficult.
Rxe4 29.Rf6! good – or perhaps 39.Qxf2 Bxf2 40.Bxe6+ 36...axb3!
and Black can soon fortunately, if you Rxe6 41.Kxf2 c3 [ Ljubojevic actually
resign. ] managed to calculate it to and Black wins the played 36...Kg7?
24.Rf5! (5 points) White the end. endgame easily ) 36...Rf7 and Tal won after
regains the piece and ends [ Instead 31.Bxc6! Rxc6 37.Rxf7 Kxf7 38.Qxc4 37.Bxe6 Qxb2+ 38.Kh3
an exchange up. Nf6 32.Be3 with an extra Qxf2 39.Qc7+ Kf6 Kxg6 39.Bxf7+ Kh6
[ 24...Re8 25.Rc5 Bxe4 pawn would have been 40.Qd8+ Kf7 ( or 40.Rd6+ Kg7 41.Rd7
26.Rxc3 also wins for better. ] 40...Ke5 41.Qh8+ ) b5 42.Be6+ Kg6
White. ] 31...Bxb7 (1 point) Quite 41.Qd7+ Kf6 43.Rd8 c5? 44.Rg8+
25.Rg5+ Kh8 26.Rc5 clearly the only move. with a draw. The end of Kh7 45.Ng5+ Kh6
White is winning. The Now White has two lanes this line is really 46.Nf7+ Kh7 47.Rg5
black pawns on the he can go down: 32.-- difficult to calculate (threatening 48 Bf5
queenside are not passed [ 32.Bh7+ Kg7! accurately. The main mate) 1-0. ]
pawns, but weaknesses. (2 points) 33.Bh6+ ( if thing is not to analyse 37.Rxf7 Qd4! (7 points)
Bxe4 27.Rxc3 Re8 33.Bg8 f5! and Black everything completely, The only defence. 38.axb3
28.Kg1 Bxd3 29.cxd3 wins – 1 point ) 33...Kh8 but to be able to [ 38.Nf6+? Qxf6 39.Rxf6
Re6 30.Ra3 Rd6 31.Rfa1 34.Bxf8 ( if 34.Bg8 Kxg8 calculate what can be bxa2 was Black's main
Kg7 32.Rxa6 Rxd3 35.Bxf8 Bh4! calculated and to defensive resource. ]
33.Rb6 Rb3 34.Rf1 Ng4 and there is no attack – evaluate correctly that 38...Qxb2+ 39.Kh3 Qh8+!
35.Rb7 Ne5 36.h3 Kg6 3 points ) 34...Rxa3!! which remains. (3 points) This check can
37.Rb6+ Kg7 38.Rf5 f6 (5 points) 35.Rxa3 ( if So the conclusion of also be difficult to spot.
39.Rbxf6 and Black 35.f3 Rxf3 and Black is the exercise is that 31 40.Kg4 Qd4! (3 points)
resigned. better ) 35...Bxa3 Bxg6, though very Another important move.
Maximum score: 20 points. 36.Bxa3 Qh4! tempting is a mistake, This is all you need to see
. ( threatening ...Qxg4+; if and the clear advantage in order to play 36...axb3,
(press F10 for the next instead 36...Qd5 a pawn up with 31 Bxc6 perhaps not even this
exercise) 37.Be4! Qxe4 38.Qxe4 should be preferred. much. 41.Kf4!?
Bxe4 39.Bb2 Maximum score: 30 The only winning attempt.
and Black would have points. Now Black still needs to
exercise 094 struggle a little bit for . play accurately. b5
[Jacob Aagaard] the draw ) 37.Kf1 ( if (press F10 for the next 42.Re7 Qb2! This move
37.Bc1 Qh3! since the exercise) ] keeps the balance.
XABCDEFGHY h7-bishop isn't going [ It was still possible to
8-+-wqltrk+({ anywhere; , or 37.Be7!? go wrong by 42...c5?
7+R+-vlp+-' 250 Qxe7! 38.Be4 Bxe4 exercise 095 43.Rxe6 c4 44.Rd6
6r+-+p+p+& 39.Qxe4 Qc5 [Jacob Aagaard] Qa1 45.Kf5! and Black
5+-+-zP-+-% with a drawn queen has no defence; e.g.
4-+p+L+P+$ endgame ) 37...Qh3+ XABCDEFGHY Qb1 46.Rd8+ Kg7
3zP-+-+-+R# 38.Ke1 Qxa3 (6 points) 8-+-+-+k+({ 47.Rd7+ Kg8 48.b4!
2-+Q+-zP-zP" 39.Be4 Bxe4 40.Qxe4 7+p+R+p+-' 252 Qc2 49.Rf7 Qd3
1+-vL-+-mK-! c3 and despite his extra 6-+p+p+P+& 50.Ke6 Qe3 51.Kd5
xabcdefghy pawn White needs to 5+-+-+-+-% Qd3+ 52.Kc6
. secure a draw quickly, 4pwq-+N+-+$ and 53 Nf6+ wins. ]
Calculate the although this should not 3+L+-+P+-# 43.Rxe6 Qxb3 44.Rxc6
consequences of 30... be a major problem after 2PzP-+-+K+" Kg7 and only Black can
Bc6!? 31 Bxg6 from 41.Qc2 . ] 1+-+-+-+-! possibly win, so White
White's perspective. [ 32.Rh7!? (2 points) is xabcdefghy needs to make his draw
. an interesting and . soon.
(press F10 for the solution) surprising idea. Black Black to play and draw. Maximum score: 15 points.
must reply f5! 33.exf6! . .
( 33.Bh6!? (press F10 for the solution) (press F10 for the next
solution 094 is less dangerous; e.g. exercise)
[Jacob Aagaard] Be4 34.Qc1 Bxa3
35.Qf1 Qh4 solution 095
XABCDEFGHY and the threats of ... [Jacob Aagaard] exercise 096
8-+-wqltrk+({ Qxg4+ or ...Ra7 force [Jacob Aagaard]
7+R+-vlp+-' 251 White to take the XABCDEFGHY
6r+-+p+p+& perpetual; if 36.h3?? 8-+-+-+k+({
5+-+-zP-+-% Black also has Qg3+!! 7+p+R+p+-' 253 (Diagram 254)
4-+p+L+P+$ 37.fxg3 Bc5+ 38.Kh2 6-+p+p+P+&
3zP-+-+-+R# Ra2+ and wins ) 33...Bxf6 5+-+-+-+-% .
2-+Q+-zP-zP" 34.Rxb7 Bd4! 4pwq-+N+-+$ Calculate the
1+-vL-+-mK-! (5 points). This position 3+L+-+P+-# consequences of 8 Nxd4?!
xabcdefghy could be foreseen with 2PzP-+-+K+" from Black's perspective.
J.Aagaard-Th.Johansson, some anticipation. Now 1+-+-+-+-! .
Stockholm 2003 White needs to play xabcdefghy (press F10 for the solution)
(variation). 35.Bf5! to keep his head M.Tal-L.Ljubojevic, Wijk
. above water. Black can aan Zee 1973.
30...Bc6! 31.Bxg6? only respond with Qh4! .
This was the move you (2 points) 36.Kh1 ( not Black has only one move,
had to calculate. 36.Kg2? Rf7! 37.Rxf7 so this exercise should
Excelling at Chess Calculation− Jacob Aagard 67

XABCDEFGHY check ) 14...cxd4 Kxd7 15.Rxb2 ( this should be played


8r+-wqkvl-tr( 15.0-0 Ke5 16.Nd3+ White is probably slightly now; not 24...Rxf7?
7zpp+-+pzpp' 254 Ke6 ( not 16...Kd5? better, but the players 25.Rd8+ Rf8 26.Rdxf8+
6-+-zppsn-+& 17.c4+! dxc3 18.b4! showed their true spirit by Qxf8 27.Rxf8+ Kg7
5+-zp-+-vL-% and mates ) 17.Nf4+ agreeing on a draw. 28.Rxc8 and White
4-+LsnP+l+$ ( or if 17.Rfe1+ Ne4 Maximum score: 25 points. wins ) 25.Ka1 Rxf7
3+-sNP+N+-# 18.Bxd8 Rxd8 19.f3 . 26.Rd8+ Rf8 27.Rdxf8+
2PzPP+-zPPzP" d5! 20.fxe4 dxe4 (press F10 for the next ( not 27.Rfxf8+?? Kg7
1tR-+QmK-+R![ with three pawns for exercise) and Black wins )
xabcdefghy the piece ) 17...Ke5 27...Qxf8 28.Rxf8+ Kg7
18.Nd3+ with a draw 29.Rg8+! ( 5 points – this
as well.; exercise 097 manoeuvre needed to
solution 096 B) 13.d4+ Kxd4 [Jacob Aagaard] be anticipated in order
[Jacob Aagaard] 14.Rxd1+ Ke5 to find a clear advantage
15.Nd3+ XABCDEFGHY for White in this line;
XABCDEFGHY and now Black can 8-+l+-trk+( instead 29.Rxc8 Nxe4
8r+-wqkvl-tr( even lose with Kxe4? 7+-+-+pzpp' 256 30.Rc7 Kh8 is not so
7zpp+-+pzpp' 255 ( instead 15...Ke6 6ptrN+p+-wq& clear ) 29...Kf7 30.Bxg6+
6-+-zppsn-+& 16.Nf4+ Ke5 17.Nd3+ 5+p+-zP-+-% (the point! – White wins
5+-zp-+-vL-% is also a draw ) 16.f3+ 4n+-+L+-zP$ a pawn) hxg6 ( if
4-+LsnP+l+$ Kd5 ( or 16...Kd4 3zP-zP-+-+-# 30...Kxe7 31.Rxc8 hxg6
3+-sNP+N+-# 17.c3+ ) 17.c4+! Ke6 2-+P+-wQP+" 32.Rc7+ Kf8 33.Rxc3
2PzPP+-zPPzP" 18.Nf4+ Ke7 19.Nd5+ 1+K+RtR-+-![ and White has
1tR-+QmK-+R![ Ke6 20.0-0 f4 xabcdefghy excellent chances in the
xabcdefghy 21.Nxf4+ Kf5 22.h4! . rook endgame ) 31.Rxc8
R.Lilja-S.Holm, d5 ( 22...h5 23.Ba4! White to play and win. (10 points) Nd5 ( if
Copenhagen 2002. d5 24.Rfe1 . 31...b4 32.Rc7 Nb5
. and 25 Bc2+ ) 23.g4+ (press F10 for the solution) 33.Nc8+ Nxc7 34.Nxb6
You were asked to Nxg4 24.fxg4+ Kxg4 bxa3 35.c4
calculate the 25.Bxd8 Rxd8 and the knight ending is
consequences Richard 26.Nxd5 and White solution 097 good for White with
Lilja's imaginative but wins.; [Jacob Aagaard] passed c- and h-pawns )
incorrect sacrifice. C) 13.d4+ cxd4! 32.Nc6! and White has
8.Nxd4?! Bxd1 (8 points) 14.Nd3+ XABCDEFGHY an extra pawn in the
This is of course the test. Kxe4! (5 points) and 8-+l+-trk+( endgame, as well as the
[ Not 8...cxd4? 9.Bb5+ Black wins because he 7+-+-+pzpp' 257 more active pieces. So
Ke7 10.Qxg4 dxc3 has ...f5-f4 giving the 6ptrN+p+-wq& the endgame after 24
11.0-0 with a clear king an escape route, 5+p+-zP-+-% Rf1 favours White, but it
advantage to White. ] although it is still not 4n+-+L+-zP$ is not as strong as 24
9.Bb5+ Nd7? that easy after 3zP-zP-+-+-# Rd8!. ]
[ Black decided he did not 15.Kxd1! ( if 15.0-0 2-+P+-wQP+" 24...Rxd8 25.Qf6+ Qg7
like the look of 9...Ke7! Bxc2 16.Bc4 f4 1+K+RtR-+-![ 26.Nxg6+! (8 points) hxg6
accepting the challenge, and it is all over ) 15...f4 xabcdefghy 27.Qxd8+ Qg8 28.Qc7!
which is a shame. If for ( the threat was 15...-- M.Paolozzi-M.Chandler, (10 points) Without this
no other reason, then out 16.Re1+ Kd5 17.b4 World Student Team Ch., move White would have
of principle he should and 18 Nf4 mate ) Chicago 1983 (variation). nothing more than a draw.
have attempted the king 16.Nxf4 Kf5 17.h4 d5 . Now the threats are 29
march to the centre: 18.Re1! ( if 18.Bd3+ 22.Ne7+ The only move. Rd1 and 29 h5, and it is
10.Nf5+ ( 3 points – Ke5! and ...Kd6 ) [ If 22.Ka1 Rb7! very hard to find a
obviously the idea; after 18...Qa5 19.Bd3+ Ne4 and it is not easy to defence for Black. It is a
10.Nd5+ exd5 11.Nf5+ 20.Nxd5 Qxd5 21.f3 suggest a way for White matter of complete
Ke6 White has no way to Ke6 22.Bxe4 Qa5 to proceed. ] domination. A very
continue the attack ) 23.Bxb7+ Kd7 22...Kh8 23.Qxf7 original position indeed.
10...exf5 11.Nd5+ Ke6 24.Bxa8 Bb4 (3 points) So far so good. Bb7! (5 points)
12.Nf4+ Ke5 (5 points) (only this tactic finally g6! The right move order. [ If 28...Qf8!? 29.Rd1
13.d4+! ( 4 points – finishes White off) [ After 23...Nxc3+ 24.Ka1 Bd7! 30.Rxd7 Qf1+
again this seems to be 25.Re4 Rxa8 g6 White wins with 31.Ka2 Qc4+ 32.Qxc4
the only way to continue and Black is a piece 25.Rd8! Rxd8 26.Qf6+ bxc4 33.Rb7 and White
the attack; if instead up.; Qg7 27.Nxg6+ hxg6 wins the endgame
13.Kxd1 fxe4 14.dxe4 D) 13.d4+! Black has 28.Qxd8+ Qg8 29.Qxb6! without too many
Qb6 15.Nd3+ Kxe4 the following choices. (2 points), the troubles. ]
16.Bc4 Kf5 . )] Of course these could difference being that [ 28...Bd7 29.Qxd7
[ So after 9...Ke7 have been examined the rook is not hanging Nxc3+ 30.Kb2 Nxe4
10.Nf5+ exf5 11.Nd5+ on arriving at this when the knight is still at ( 30...Na4+ 31.Ka2
Ke6 12.Nf4+ Ke5 position, but here in a4. ] Nc3+ 32.Kb3
A) 13.d4+ Kxe4 the test the 24.Rd8!! (5 points) This is only improves the
and then either requirements are the best way of freeing the white king's position )
14.Rxd1 ( or 14.Kxd1 higher if you want to queen. 31.Rxe4 Qf8 ( if
Kxd4 15.c3+ Ke5 achieve full points! ] [ White can also play 31...Rb8 32.Rd4!
16.Re1+ Ne4 17.Nd3+ 10.Bxd8 cxd4 11.Kxd1 24.Rf1!? (a very original with ideas of 33 Rd6;
Ke6 18.Nf4+ dxc3 12.Ba5 cxb2 way of getting the queen Black is under heavy
gives White perpetual 13.Rb1 a6 14.Bxd7+ out) Nxc3+! pressure with no
Excelling at Chess Calculation− Jacob Aagard 68

counterplay and has should be very careful 16.Qxa1 c6 17.Bc4 Bf5 XABCDEFGHY
already lost a pawn ) regarding the following and Black wins ) 8-+-+-trk+(
32.Qc7 Rb8 33.Qd6 lines as they contain a 15...Rxf7 16.Nxf7 Be6! 7+-+-+pzpp' 259
and White will win trap! Nxd4 12.Nb3 17.Ne5 Bxa2 18.Qxa1 6-+-zp-+-+&
without too much effort; (2 points) White's idea. Be6 and it looks as if 5+-+L+-wq-%
e.g. Qe8 34.Rf4 Rd8 Nxb3 13.Bxd5 Black has just won a 4R+-+P+n+$
35.Qxa6 Rd5 36.Qa7 [ After 13.Qxb3 c6 pawn. ] 3+-tr-+-+-#
Kg8 37.Rf6 Rxe5 14.Rad1 Qb6 15.Bxd5 14.Bxf7+ Kh8 15.Qh5 2P+-+Q+PzP"
38.Rxg6+ Qxg6 cxd5 16.Qxd5 Be6 Bf5? This loses quite 1+-+-+R+K![
39.Qb8+ Kh7 40.Qxe5 Black has slightly quickly. xabcdefghy
is one possible better prospects. ] [ Other options were:
continuation. ] 13...Nxa1? This might 15...g6 16.Qh6 Bf5
29.Bxb7 Qf8 With the seem forced, but again we 17.Bxg6 and White was solution 099
idea of 30...b4!. have a kind of desperado a pawn up in E. [Jacob Aagaard]
[ e.g. 29...Qf8 30.h5 b4 scenario, similar to the Oudshoorn-A.Hendriks,
31.axb4 Rxb4+ 32.Kc1 Shirov-Lutz game above, Dutch Junior XABCDEFGHY
Qh6+ 33.Kd1 Rb1+ where Black can save Championship 1994. ] 8-+-+-trk+(
34.Ke2 Rxe1+ 35.Kxe1 himself from trouble by [ 15...g5 16.Qh6 Bf5 7+-+-+pzpp' 260
Qc1+ 36.Kf2 Qxc2+ stepping outside the 17.Bg6 Bxg6 ( or 6-+-zp-+-+&
37.Kg3 Qxc3+ 38.Qxc3 absolutely forcing lines. 17...Qe7 18.Bxf5 Rf6 5+-+L+-wq-%
Nxc3 39.Bxa6 gxh5 [ 13...Qf6! 19.Ng6+ Rxg6 20.Bxg6 4R+-+P+n+$
and Black will hold the was the correct move and White will win the 3+-tr-+-+-#
endgame. ] (10 points). The forced knight on a1 ) 18.Nxg6+ 2P+-+Q+PzP"
30.Qc8! (5 points) Qxc8 line would now continue Kg8 19.Nxf8 Qxf8 1+-+-+R+K![
31.Bxc8 Nxc3+ 32.Kc1 14.Bxf7+ ( or 14.Qxb3 20.Qxg5+ Qg7 21.Qd2 xabcdefghy
with a clearly better, Qxe5 15.Bxf7+ Rxf7 Qf6 22.Rxa1 and White R.J.Fischer-J.Sherwin, US
probably winning endgame transposing; , but not should have won in K. Championship, New York
for White. 14.Nxf7? Nxa1! Pilgaard-M.Stoinev, 1957.
Maximum score: 30 points. when White has no Copenhagen 1989. ] .
. compensation ) 14...Rxf7 [ 15...h6! 16.Rxa1 Qd6 30.Rxf7! (5 points)
(press F10 for the next 15.Qxb3 ( not 15.Nxf7? 17.Ng6+ ( if 17.Rd1 Without this move there is
exercise) Nxa1! and Black wins ) Bg4!! 18.Ng6+ Qxg6 no advantage for White.
15...Qxe5 ( 4 points; 19.Qxg6 Bxd1 [ Hopefully it did not take
weaker is 15...Be6?! with a level position ) you long time to see that
exercise 098 16.Qxb7 Rff8 17.Qxc7 17...Kh7 18.Nxf8+ Qxf8 after 30.Bxf7+?! Kh8
[Jacob Aagaard] and Black does not 19.Qf3! and White keeps White has won a pawn,
have enough for the the initiative, though but Black has all the
XABCDEFGHY pawn ) 16.Rae1 ( after nothing has been play: none of the white
8r+lwq-trk+({ 16.Rfe1 Be6 17.Rxe5 decided. ] pieces serve any
7zppzp-+pzpp' 258 Bxb3 18.axb3 a6 16.Qxf5 Qf6? purpose anymore. White
6-+n+-+-+& White is left with a [ 16...g6 17.Qe4 Qf6 would have to be content
5+-+nsN-+-% worse pawn structure 18.Bxg6 Nc2 19.Bxh7 with even chances after
4-+LzP-+-+$ and pawn endings could Rfe8 20.f4 is very good 31.h3 or something
3+-+-+-+-# be impossible for him to for White but still not similar.
2PzP-sN-zPPzP" defend ) 16...Be6! 100% conclusive, Now after 30 Rxf7! it is
1tR-+Q+RmK-! (4 points; without this whereas now not likely that you will
xabcdefghy move Black was toast, Capablanca is just a be able to see all
. but now the position is piece down. ] variations; therefore the
Calculate the even) 17.Qxe6 Qxe6 17.Ng6+!! Qxg6 18.Bxg6 sum of points available
consequences of 11... ( perhaps Black could Rxf5 19.Bxf5 g6 20.Be4! in this exercise is
Nxd4 from Black's even try 17...Qxb2!? and Black resigned. greater than the
perspective. 18.Rb1 Qf6 19.Qxf6 Maximum score: 20 points. maximum allowed score
. gxf6! 20.Rxb7 c5 . of 35 points. ( 31.Bd5?
(press F10 for the solution) with an active position (press F10 for the next Qf4! and wins is just one
– still, White should not exercise) example of the dangers
lose with accurate play ) in the position. )]
solution 098 18.Rxe6 is known to be 30...Rc1+?! This is not the
[Jacob Aagaard] a draw. ] exercise 099 best defence. White did
[ 13...Qe7!? (8 points). [Jacob Aagaard] not need to work out lines
W.Kluxen-J.R.Capablanca, White has the extra of play against all
Hamburg (simul) 1911. option of 14.Re1?! defences in order to
. ( otherwise 14.Qxb3 (Diagram 259) decide upon 30 Rxf7!, but
This is an important Qxe5 15.Bxf7+ Rxf7 as this is a bit of an
theoretical position, transposes to the . artificial situation, in
arising after 1.e4 e5 previous line ) White to play. Calculate which ruminating over the
2.Nf3 Nc6 3.Bc4 Bc5 , but this does not all variations after 30 position is encouraged,
4.c3 Nf6 5.d4 exd4 appear to be anything Rxf7!. Refute as many as there are points available
6.cxd4 Bb4+ 7.Bd2 special: 14...Nxa1! you can. for seeing various
Bxd2+ 8.Nbxd2 d5 15.Bxf7+ ( after 15.Nxf7 . refutations.
9.exd5 Nxd5 10.0-0 0-0 Qf6! White cannot (press F10 for the solution) [ 30...Qxd5? 31.Rxf8+
11.Ne5 . Black can indeed continue his attack in Kxf8 32.Qf1+!
take the d-pawn, but he any meaningful way; e.g. and White wins. ]
Excelling at Chess Calculation− Jacob Aagard 69

[ 30...Qc1+? 31.Qf1! 33.Qxc4 Qe5! solution 100 [ White's strongest


wins (5 points). ] 34.Qf1+ Ke7 35.Qg1 [Jacob Aagaard] defence is 14.d6!?
[ 30...Rfc8? 31.Rc4! Qf4! when White has (6 points), contemplating
( simplest; 31.Qf1 nothing better than XABCDEFGHY Bb3-d5.
also wins ) 31...Rc1+ perpetual check ) 8r+l+k+-tr({ Now we have:
32.Qf1! and White wins. ] 32...Rxf7 33.Bxf7+ 7+-zp-+pzpp' 262 A) 14...0-0-0
[ 30...Rcc8?! 31.Ra8!! Kh7 ( if 33...Kf8 6p+-+-+-+& (with the idea of ...
(8 points) Rxa8 32.Rf5+ 34.Be6 Qe5 5+pvlPzp-+-% Rxd6) looks tempting,
Kh8 33.Rxg5 Rac8 ( if 35.Qc8+ Ke7 4-+-snN+nwq$ but White can defend:
33...Rab8 34.Bb3 36.Qd7+ Kf6 3+L+-+-+P# 15.c3! ( not 15.Bd5?
is simplest ) , and now 37.Qf7+ Kg5 2PzPPzP-zPP+" Rxd6 16.Nxh3 Qg3
the only clear win is 38.Qf5+ Qxf5 1tRNvLQ+RmK-! 17.Kh1 Rh6
34.Bc4! (4 points) Ne5 39.exf5 xabcdefghy and Black wins )
35.Rxe5 dxe5 36.h3 and the passed a- Scemama-Baudry, New 15...Nxb3 ( if 15...Bg4
with a decisive material pawn gives White Delhi 2000 (variation). 16.Qxg4+ Qxg4
advantage for White. ] close to a winning . 17.Nxg4 Nxb3+ 18.d4
[ 30...h5! (10 points) is position ) 34.Qf1! 12...Nxf2! (8 points) Nxa1 19.dxc5
the toughest defence. with a clear Immediate action is and White is probably
White now has the advantage to White. required. better! ) 16.axb3 Qg3
following options: ( 34.Qc8 [ If 12...Bb6? 13.d3 17.Qf3 Qxf3 18.gxf3
A) 31.Qf1 is met by is less strong (threatening 14 Bg5) h6 Bxf1 19.Kxf1
the cool Kh7! because of Nh6! 14.Qe1 and it is hard to and the endgame is
( 5 points; not 35.Bd5 Qd2 36.h3 see that Black is better for Black, but
31...Rxf7? 32.Qxf7+ Qe1+ 37.Kh2 Ng4+! getting better ] still enormously
Kh7 33.Qg8+ Kh6 38.hxg4 Qh4+ [ while 12...Nh2? messy.;
34.Qh8+ Kg6 35.Bf7+! with perpetual is fancy but bad: B) 14...f5! (with the
Kxf7 36.Ra7+ Kf6 check. ); A) not 13.Nxc5? idea of ...Bg4) and
37.Qd8+ Kg6 D) 31.-- ] Bxh3! ( or 13...Nhf3+ then:
38.Rxg7+ and White 31.Qf1!! (10 points) This 14.gxf3 Qxh3 B1) 15.Bd5? Bg4!
wins ) 32.Rxf8 Rc1 was obviously the plan. and wins ); and Black wins.;
and the position is [ Not 31.Rf1+?? Kh8 B) 13.Kxh2! Qxe4 B2) 15.Re1!? Bg4!
unclear. If 33.Be6? and Black wins. ] 14.d3 Qg6 15.f4 ( if 15...0-0-0 16.Re3
Black wins with Ne3! 31...h5!? A last try. and White is even Bg4 17.Qe1
34.Bf5+ Nxf5 35.Rxf5 [ If 31...Rxf1+ 32.Rxf1+ preferable here. ] with a mess; , or
Rxf1+ 36.Rxf1 Qb5! Qxd5 33.Rxf8+ Kxf8 13.Nxf2 15...cxd6 16.Re3
and White loses one of 34.exd5 and White [ On 13.Rxf2 , simplest is Nxb3 17.axb3 Bg4
the rooks.; wins. ] Qxe4! (2 points) and 18.Nxg4 Qxg4
B) 31.Rxf8+ Kxf8 32.Qxc1! (5 points) The White loses material. 19.Qxg4 fxg4
32.Qf1+ Qf6 ( if only winning move. Candidates! ] and Black is better,
32...Nf6 33.Rc4 [ Not 32.Rxf8+? Kh7 13...Bxh3! (4 points) This but not enough to
and White has the with an unclear shot is absolutely impress the referee )
advantage ) position. ] necessary for the attack. 16.Rxe5+ Kd7
B1) or 33.Ra8+ Ke7 32...Qh4 33.Rxf8+ Kh7 [ Black does not achieve 17.Qe1 cxd6 18.Re3
34.Ra7+ Kd8 35.Rf7 34.h3 Qg3 35.hxg4 h4 anything special with f4 19.Re4 Rae8
Qxf1+ ( 35...Nf2+ 36.Be6 and Black 13...Qg3? 14.Kh1 Nxb3 and White cannot
36.Kg1 Qh6! resigned. ( 14...Bxh3 15.Nxh3! survive against the
is not completely Maximum score: 35 points. would obviously be entire black army.;
clear either ) 36.Rxf1 . wrong now; so with this B3) 15.Nxh3!
Ne3 and Black is no (press F10 for the next bad move order Black (the only move)
worse; exercise) loses the option of Nxb3+ 16.Kh1 Nxa1
B2) 33.Qxf6+ Nxf6 bringing the bishop into (5 points). This
34.h3 Nxd5 35.exd5 the attack ) 15.axb3 position should not
Rd3 36.Ra5 Kf7 exercise 100 ( also interesting is be deeply analysed,
and Black should [Jacob Aagaard] 15.Ne4!? Qg6 16.axb3 but simply evaluated,
draw this endgame Qxe4 17.Nc3 Qh4 as there are more
without any XABCDEFGHY 18.Qe1 Qh5 19.d4!? complicated and
significant effort.; 8r+l+k+-tr({ Bxd4 20.Nxb5 important lines to
C) 31.Rc4! (5 points) 7+-zp-+pzpp' 261 with unclear play ) calculate elsewhere.
C1) forced; after 6p+-+-+-+& 15...Bxf2 16.Qe2 Bb6 The position is not
31...Rxf7 32.Bxf7+ 5+pvlPzp-+-% 17.Rf3 Qg5 18.Nc3 100% clear, but I can
Kxf7 ( or if 32...Kh7!? 4-+-snN+nwq$ and White is not worse. ] still analyse further
33.Rxc3 Qe5 34.g3 3+L+-+-+P# 14.Nxh3 of course: 17.dxc7
Qxc3 35.Bxh5 2PzPPzP-zPP+" [ Not 14.gxh3? Qg3+ 0-0! ( 17...Qg4!?
and White wins ) 1tRNvLQ+RmK-! 15.Kh1 Nf3 and Black 18.b4! Qxd1
33.Rxc3 Qe5 xabcdefghy wins ] 19.Rxd1 Bd6 20.Bb2
34.Rf3+ Ke7 35.g3 . [ while 14.Re1?! Nxc2 21.Rc1 Kd7
and Black's Black to play and win. is easily refuted by Qg3! 22.Rxc2 Rhc8
counterplay has . ( 14...Nxb3 15.Rxe5+ is a very good
been repulsed; (press F10 for the solution) Kf8 16.Re3 Nxa1 endgame for Black,
C2) 31...Rxc4 should also win ) though not enough to
32.Qxc4 ( not 15.Rxe5+ Qxe5 satisfy me
32.Rxf8+?! Kxf8 and Black wins. ] completely ) 18.b3
Excelling at Chess Calculation− Jacob Aagard 70

and then one idea is


e4 19.Bb2 f4
20.Qe1! ( if 20.Bxa1
f3! and the white king
is in trouble )
20...Qxe1 21.Rxe1
Nxc2 22.Rc1 Rac8
23.Rxc2 Rxc7
24.Be5 Rcc8
25.Bxf4 g5!
and Black ends the
exchange up.;
B4) 15.--;
C) 14...-- ]
14...Nxb3+ 15.Kh1 Nxa1
(5 points) and Black is
winning.
As said before, a position
such as this cannot be
analysed to a conclusion
since White no longer has
any forcing options.
Rather you should make
the evaluation that Black
has a sea of pawns and a
rook vs. two knights
(maybe just one knight if
the one on a1 escapes).
Looking at the position in
this way Black is
obviously winning.
Maximum score: 35 points.
.
And that's it.

You might also like